PM2020 I+Endocrinology

Download as pdf or txt
Download as pdf or txt
You are on page 1of 676

01

A patient with type 2 diabetes mellitus is started on sitagliptin. What is


the mechanism of action of sitag liptin?

lncretin inhibitor

Dipeptidyl peptidase-4 (DPP-4) inhibitor

Alpha-glucosidase inhibitor

Glucagon inhibitor

Glucagon-like peptide-1 (GLP-1) mimetic

Submit answer

Reference ranges v
D1pept 1dyl pept idase-4 (DPP-4) 1nht61tor

Alpha-glucosidase inhibitor

Glucagon inhibitor

Glucagon-like peptide-1 (GLP-1) mimetic

Gliptins = Dipeptidyl peptidase-4 (DPP-4) inhibitors


Importance: 50

•• •• tt Discuss (1) Improve

Next question )

Diabetes mellitus: GLP-1 and the new drugs

A number of new drugs to treat diabetes mellitus have become


available in recent years. Much research has focused around the role of
glucagon-like peptide-1 (GLP-1), a hormone released by the small
intestine in response to an oral glucose load

Whilst it is well known that insulin resistance and insufficient 8-cell


compensation occur other effects are also seen in type 2 diabetes
mellitus (T2DM). In normal physiology an oral glucose load results in a
greater release of insulin than if the same load is given intravenously -
this known as the incretin effect. This effect is largely mediated by GLP-
1 and is known to be decreased in T2DM.

Increasing GLP-1 levels, either by the administration of an analogue


(glucagon-like peptide-1, GLP-1 mimetics, e.g. exenatide) or inhibiting
its breakdown (dipeptidyl peptidase-4 ,DPP-4 inhibitors - the gliptins), is
therefore the target of two recent classes of drug.
Glucagon-like peptide-1 (GLP-1) mimetics (e.g. exenatide)

Exenatide is an example of a glucagon-like peptide-1 (GLP-1) mimetic.


These drugs increase insulin secretion and inhibit glucagon secretion.
One of the major advances of GLP-1 mlmetics is that they typically
result in weight loss, in contrast to many medications such as insulin,
sulfonylureas and thiazolid inediones. They are sometimes used in
combination with insulin in T2DM to minimise weight gain.

Exenatide must be given by subcutaneous injection within 60 minutes


before the morning and evening meals. It should not be given after a
meal.

Liraglutide is the other GLP-1 mimetic currently available. One the main
advantages of liraglutide over exenatide is that it only needs to be given
once a day.

Both exenatide and liraglutide may be combined w ith metformin and a


sulfonylurea. Standard release exenatide is also licensed to be used
with basal insulin alone or with metformin. Please see the BN F for a
more complete list of licensed indications.

NICE state the following:

Consider adding exenatide to metformin and a sulfonylurea if:


• BMI >= 35 kg/ m 2 in people of European descent and there are
problems associated with high weight, or
• BMI < 35 kg/ m 2 and insulin is unacceptable because of
occupational implications or weight Joss would benefit other
comorbidities.
I

NICE like patients to have achieved a 11 mmol/mol (1 %) reduction in


HbA1 c and 3% weight loss after 6 months to justify the ongoing
prescription of GLP-1 mimetics.

The major adverse effect of GLP-1 mimetics is nausea and vomiting.


The Medicines and Healthcare products Regulatory Agency has issued
specific warnings on the use of exenatide, reporting that is has been
linked to severe pancreatitis in some patients.

Dipeptidyl peptidase-4 {DPP-4) inhibitors (e.g. Vildagliptin, sitagliptin)

Key points
• oral preparation
• trials to date show that the drugs are relatively well tolerated with
no increased incidence of hypoglycaemia
• do not cause weight gain

NICE guidelines on DPP-4 inhibitors


• NICE suggest that a DPP-4 inhibitor might be preferable to a
thiazolidinedione if further weight gain would cause significant
problems, a thiazolidinedione is contraindicated or the person has
had a poor response to a thiazolidinedione

Next question >

Save my notes
Q2

A 33-year-old man with tingling in his t humb, index and middle finger
has been referred for endocrine consultation. He also complains of
waking up incredibly tired and says his wife comp lains that he snores.
On examination, he is noted to have a prominent brow ridge. Looking at
old photos, it becomes clear that his facia l appearance has drastically
changed over time. After some blood tests and an MRI scan, he is
prescribed octreotide. What is the mechanism of action of this drug?

Somatostatin analogue

Growth hormone receptor antagonist

Dopamine agonist

Dopamine antagonist

Insulin-growth factor 1 antagonist

Submit answer

Reference ranges v
prescribed octreotide. What is the mechanism of action of this drug?

I Somatostatln analogue

Growth hormone receptor antagonist

Dopamine agonist

Dopamine antagonist

Insulin-growth factor 1 antagonist

Acromegaly is caused by excessive growth hormone.


Somatostatin directly inhibits the release of growth hormone, and
hence somatostatin analogues are used to treat acromegaly
Importance: 50

This question is two-fold. First, one has to recognise the symptoms of


acromegaly. Carpal tunnel syndrome and sleep apnoea are classic
complications of acromegaly. The changing nature of his face over time
is another clue. The second part of the question was to acknowledge
that octreotide, a useful treatment for acromegaly is a somatostatin
analogue.

Dopamine agonists were initially used to treat acromegaly but have


fallen out of favour due to superior treatments. Dopamine antagonists
have never been a treatment for acromegaly. An example of a growth
hormone antagonist is pegvlsomant. Growth hormone stimulates
insulin growth factor-1 release from the liver, but antagonists have not
been developed yet.

••
11
f • Discuss (1) Improve

Next question )
Next question >

Acromegaly: management

Trans-sphenoidal surgery is the first-line treatment for acromegaly in


the majority of patients

Dopamine agonists
• for example bromocriptine
• the first effective medical treatment for acromegaly, however now
superseded by somatostatin analogues
• effective only in a minority of patients

Somatostatin analogue
• directly inhibits the release of growth hormone
• for example octreotide
• effective in 50-70% of patients
• may be used as an adjunct to surgery

Pegvisomant
• GH receptor antagonist - prevents dimerization of the GH receptor
• once daily s/c administration
• very effective - decreases IGF-1 levels in 90% of patients to normal
• doesn't reduce tumour volume therefore surgery still needed if
mass effect

External irradiation is sometimes used for older patients or following


failed surgical/medical treatment

Next question )
Q3

A 75-year-old woman presents with weight gain, lethargy and


constipation. She says that she has not changed her diet but has
started 'piling on the pounds'. She also feels 'bunged up' and is only
opening her bowels twice a week, and she used to go daily. She has
doubled her dose of senna but this hasn't made much difference. She
has a past medical history of hypothyroidism and has recently been
diagnosed with angina and iron deficiency anaemia. She takes
nifedipine, atorvastatin, aspirin, senna, levothyroxine and ferrous
sulphate.

Which of her medications is most likely to be the cause of this


presentation?

Nifedipine

Atorvastatin

Aspirin

Senna

Ferrous sulphate

Submit answer

Reference ranges v
Atorvastatin

Aspirin

Senna

Ferrous sulphate

Iron / calcium carbonate tablets can reduce the absorption of


levothyroxine - should be given 4 hours apart
Importance: 50

Ferrous sulphate is correct. This woman is presenting with symptoms


of hypothyroidism - weight gain, lethargy and constipation. This is
despite treatment with levothyroxine, indicating that the hypothyroidism
is effectively undertreated.

Tablets containing iron can reduce the absorption of levothyroxine, and


as these have recently been started, this is the likely cause of the
symptoms. Tablets containing iron and levothyroxine should be taken at
least 2 and preferably over 4 hours apart to prevent this effect.

There is no interaction between levothyroxine and any of the other


medications.

116 it • Discuss Improve

CamScanne, -? Y.¥-> i.b . ~I


Hypothyroidism: management

Key points
• initial starting dose of levothyroxine should be lower in elderly
patients and those with ischaemic heart disease. The BNF
recommends that for patients with cardiac disease, severe
hypothyroidism or patients over 50 years the initial starting dose
should be 25mcg od with dose slowly titrated. Other patients
should be started on a dose of 50-1 00mcg od
• following a change in thyroxine dose thyroid function tests should
be checked after 8-12 weeks
• the therapeutic goal is 'normalisation' of the thyroid stimulating
hormone (TSH) level. As the majority of unaffected people have a
TSH value 0.5-2.5 mU/I it is now thought preferable to aim for a
TSH in this range
• women with established hypothyroidism who become pregnant
should have their dose increased 'by at least 25-50 micrograms
levothyroxine'* due to the increased demands of pregnancy. The
TSH should be monitored carefully, aiming for a low-normal value
• there is no evidence to support combination therapy with
levothyroxine and liothyronine

Side-effects of thyroxine therapy


• hyperthyroidism: due to over treatment
• reduced bone mineral density
• worsening of angina
• atrial fibrillation

Interactions
• iron, calcium carbonate
o absorption of levothyroxine reduced, give at least 4 hours
apart
Q4

A 46-year-old man with suspected diabetes mellitus has an oral glucose


tolerance test, following the standard WHO protocol. The following
resu lts are obtained:

Time (hours) Blood glucose (mmol/1)

0 5.7

2 7.6

How should these results be interpreted?

Normal

Impaired fasting glucose and impaired glucose tolerance

Diabetes mellltus

Impaired glucose tolerance

Impaired fasting glucose

Subm it answer

Reference ranges v
Normal

Impaired fasting glucose and impaired glucose tolerance

Diabetes mellitus

Impaired glucose tolerance

Impaired fasting glucose

Both the fasting and two-hour glucose are within normal limits.

9' 1f • Discuss (1) Improve

Next question )

Diabetes mellitus (type 2): diagnosis

The diagnosis of type 2 diabetes mellitus can be made by either a


plasma glucose or a HbA1 c sample. Diagnostic criteria vary according
to whether the patient is symptomatic (polyuria, polydipsia etc) or not.

If the patient is symptomatic:


• fasting glucose greater than or equal to 7.0 mmol/1
• random glucose greater than or equal to 11.1 mmol/1 (or after 75g
oral glucose tolerance test)

If the patient is asymptomatic the above criteria apply but must be


demonstrated on two separate occasions.

CamSc.an,ner ~ '....,.i;lo-" ~ ~ I
Fasting glucose: a Ommoll >• 7,0 mmol,I

Horwalwu-iccontnll
M:IA It rs ~:'..'7 ITTnohnel II 0-e 4 l
Cl
~ I • "'G 'II 6 Hl 9 IM'IO'.I

Hb.A1c· <a 41 mmOllmo, - .. mmol.'mo,


1! ) I • )

I • values equal :o 1h11


celow • t0nsidcred norrno!
°' i e- l:!e 011Vnc:1$tC
U,tcshold far bet.es
·---
Diagram showing the spectrum of diabetes diagnosis

In 2011 WHO released supplementary guidance on the use of HbA1c on


the diagnosis of diabetes:
• a HbA1c of greater than or equal to 48 mmol/mol (6.5%) is
diagnostic of diabetes mellitus
• a HbAlc value of less than 48 mmol/mol (6.5%) does not exclude
diabetes (i.e. it is not as sensitive as fasting samples for detecting
diabetes)
• in patients without symptoms, the test must be repeated to
confirm the diagnosis
• it should be remembered that misleading HbA1 c results can be
caused by increased red cell turnover (see below)

Conditions where HbA1 c may not be used for diagnosis:


• haemoglobinopathies
• haemolytic anaemia
• untreated iron deficiency anaemia
• suspected gestational diabetes
• children
• HIV
• chronic kidney disease
• people taking medication that may cause hyperglycaemia (for
example corticosteroids)
Impaired fasting glucose and impaired glucose tolerance

A fasting glucose greater than or equal to 6.1 but less than 7.0 mmol/I
implies impaired fasting glucose (IFG)

Impaired glucose tolerance (IGT) is defined as fasting plasma glucose


less than 7.0 mmol/I and OGTT 2-hour value greater than or equal to 7.8
mmol/ 1but less than 11.1 mmol/1

Diabetes UK suggests:
• 'People with IFG should then be offered an ora l glucose tolerance
test to rule out a diagnosis of diabetes. A resu lt below 11 .1
mmol/I but above 7.8 mmol/1 indicates that the person doesn't
have diabetes but does have IGT.'

Next question )

B I ~ A ..... ·-·- 1-
i: =
- ... T l ..., lffi ..... ~ c-:,

-
Save my notes

Search


Search textbook...
---- ·
Q Google search on "Diabetes mellitus (type 2): diagnosis"

Links
QS

You are reviewing a 24-year-old man who has recently been diagnosed
with type 1 diabetes mellitus. He has no comorbidities and works as an
accountant. What HbA1 c target should he aim for initially?

42 mm ol/mol

45 mmol/mol

48 mmol/mol

50 mmol/ mol

52 mmol/mol

Submit answer

Reference ranges v
45 mmol/mol

48 mmol/ mol

50 mmol/mol

52 mmol/ mol

In type 1 diabetics, a general HbA 1c target of 48 mmol/mol (6.5%)


should be used
Importance: 50

9' •• tit Discuss Improve

Next question )

Diabetes mellitus: management of type 1

The long-term management of type 1 diabetics is an important and


complex process requiring the input of many different clin ical
specialties and members of the healthcare team. A diagnosis of type 1
diabetes can still reduce the life expectancy of patients by 13 years and
the micro and macrovascular complications are well documented.

NICE released guidelines on the diagnosis and management of type 1


diabetes in 2015. We've only highlighted a very select amount of the
guidance here which will be useful for any clinician looking after a
patient with type 1 diabetes.
HbA1c
• should be monitored every 3-6 months
• adults should have a target of HbA1c level of 48 mmol/mol (6.5%)
or lower. NICE do however recommend taking Into account factors
such as the person's daily activities, aspirations, likelihood of
complications, comorbidities, occupation and history of
hypoglycaemia

Self-monitoring of blood glucose


• recommend testing at least 4 times a day, including before each
meal and before bed
• more frequent monitoring is recommended if frequency of
hypoglycaemic episodes increases; during periods of illness;
before, during and after sport; when planning pregnancy, during
pregnancy and while breastfeeding

Blood glucose targets


• 5-7 mmol/I on waking and
• 4-7 mmol/I before meals at other times of the day

Type of insulin
• offer multiple daily injection basal-bolus insulin regimens, rather
than twice-daily mixed insulin regimens, as the insulin injection
regimen of choice for all adults
• twice-daily insulin detemir is the regime of choice. Once-daily
insulin glargine or insulin detemir is an alternative
• offer rapid-acting insulin analogues injected before meals, rather
than rapid-acting soluble human or animal insulins, for mealtime
insulin replacement for adults with type 1 diabetes

Metformin
• NICE recommend considering adding metformin if the BMI >= 25
kg/m 2
Q6

You are called to see a 34 year-old man in the late afternoon while you
are on-call. He suffers with type 1 diabetes mellitus and was admitted
after being diagnosed with diabetic ketoacidosis. He has been treated
with a fixed-rate insulin infusion with potassium replacement. He
usually takes Lantus glargine and Novorapid insulins, but the nurses
have not been administering these while he has been on his insulin
infusion. His latest arterial blood gas is shown:

pH 7.37

pC02 4.3 kPa

p02 11 .9kPa

Bicarbonate 26 mmol/L

Glucose 5.2 mmol/ L

What is the best course of action?

Stop insulin infusion now and restart normal insulin regimen

Give Novorapid insulin, then stop insulin Infusion with next meal

Continue insulin infusion overnight

Give 1Og oral glucose

Give Lantus glargine, then stop insulin infusion with next meal

Submit answer

Reference ranges v
I Give Lantus glarglne, then stop Insulin Infusion with next meal

Stopping an insulin infusion in the context of an insulin-dependent


diabetic needs to be done with care. Long-acting insulins should be
continued throughout the duration of any insulin infusion, but in this
case this has not occurred - a not-uncommon problem on the wards.

The key focus here is that an Insulin-dependent diabetic should never


be without insulin as they risk precipitating diabetic ketoacidosis (DKA).
Option A will leave a gap in insulin therapy (the patient's next insulin
dose will be novorapid due with dinner) and risk recurrence of DKA.
Option B gives only a short acting insulin, which will have worn off by
the time the patient has his next meal. Option C is a safe choice, but
this patient no longer requires an insulin infusion as evidenced by his
normalised pH and blood glucose and continuing the infusion (with the
attendant hourly blood glucose checking) overnight is not in the
patient's best interests. Option D is inappropriate as the patient's blood
glucose is in the normal range. Option E is therefore the correct option -
it allows the patient's long-acting insulin to take effect before stopping
the insulin infusion. He should also restart his Novorapid insulin with
his next meal.

9' •t • Discuss (6) Improve

Next question )

Diabetic ketoacidosis

Diabetic ketoacidosis (DKA) may be a complication existing type 1


diabetes mellitus or be the first presentation, accounting for around 6%
of cases. Rarely, under conditions of extreme stress, patients with type
2 diabetes mellitus may also develop DKA.

Whilst OKA remains a serious condition mortality rates have decreased


from 8% to under 1% in the past 20 years .
Pathophysiology
• DKA is caused by uncontrolled lipolysis (not proteolysis) which
results in an excess of free fatty acids that are ultimately
converted to ketone bodies

The most common precipitating factors of DKA are infection, missed


insulin doses and myocardial infarction.

Features
• abdominal pain
• polyuria, polydipsia, dehydration
• Kussmaul respiration (deep hyperventilation)
• Acetone-smelling breath ('pear drops' smell)

Diagnostic criteria

American Diabetes Joint British Diabetes Societies


Association (2009) (2013)

Key points Key points


• glucose > 13.8 • glucose > 11 mmol/1 or known
mmol/1 diabetes mellitus
• pH< 7.30 • pH< 7.3
• serum bicarbonate • bicarbonate < 1 5 m mol/1
<18 mmol/1 • ketones > 3 mmol/ 1or urine
• anion gap > 10 ketones ++ on dipstick
• ketonaemia
Management
• fluid replacement: most patients with OKA are deplete around 5-8
litres. Isotonic saline is used initially. Please see an example fluid
regime below.
• insulin: an intravenous infusion should be started at 0.1
unit/kg/hour. Once blood glucose is < 15 mmol/I an infusion of 5%
dextrose should be started
• correction of hypokalaemia
• long-acting insulin should be continued, short-acting insulin
should be stopped

JBDS example of fluid replacement regime for patient with a systolic


BP on admission 90mmHg and over

Fluid Volume

0.9% sodium chloride 1 L 1000ml over 1st hour

0.9% sodium chloride 1 L with potassium 1000ml over next 2


chloride hours

0.9% sodium chloride 1 L with potassium 1000ml over next 2


chloride hours

0.9% sodium chloride 1 L with potassium 1000ml over next 4


chloride hours

0.9% sodium chloride 1 L with potassium 1000ml over next 4


chloride hours

0.9% sodium chloride 1 L with potassium 1000ml over next 6


ch loride hours
Please note that slower infusion may be indicated in young adults (aged
18-25 years) as they are at greater risk of cerebral oedema.

JBDS potassium guidelines

Potassium level in first 24 Potassium replacement in mmol/L


hours (mmol/L} of infusion solution

Over 5.5 Nil

3.5-5.5 40

Below 3.5 Senior review as additional


potassium needs to be ,given

Complications of OKA and its treatment


• gastric stasis
• thromboembolism
• arrhythmi'as secondary to hyperkalaemia/iatrogenic hypokalaemia
• iatrogenic due to incorrect fluid therapy: cerebral oedema*,
hypokalaemia, hypoglycaemia
• acute respiratory distress syndrome
• acute kidney injury

* children/ young adults are particularly vulnerable to cerebral oedema


following fluid resuscitation in OKA and often need 1 :1 nursing to
monitor neuro-observations, headache, irritability, visual disturbance,
focal neurology etc. It usually occurs 4-12 hours following
commencement of treatment but can present at any time. If there is any
suspicion a CT head and senior review should be sought
Q7

Which one of the following is least characteristic of Addison's disease?

Hypoglycaemia

Metabolic alkalosis

Hyponatraemia

Hyperkalaemia

Positive short ACTH test

Submit answer

Reference ranges v
Metabolic alkalosis

Hyponatraemia

Hyperkalaemia

Positive short ACTH test

Addison's disease is associated with a metabolic acidosis


Importance: 50

,6 •• t9 Discuss (8) Improve ]

Next question )
Next question )

Addison's disease: investigations

In a patient with suspected Addison's disease the definite investigation


is an ACTH stimulation test (short Synacthen test). Plasma cortisol ls
measured before and 30 minutes after giving Synacthen 250ug IM .
Adrenal autoantibodies such as anti-21-hydroxylase may also be
demonstrated.

If an ACTH stimulation test is not readily available (e.g. in primary care)


then sending a 9 am serum cortisol can be useful:
• > 500 nmol/I makes Addison's very unlikely
• < 100 nmol/1 is definitely abnormal
• 100-500 nmol/I shou ld prompt a ACTH stimulation test to be
performed

Associated electrolyte abnormalities are seen in around one-third of


undiagnosed patients:
• hyperkalaemla
• hyponatraemla
• hypoglycaemia
• metabolic acidosis

Next question )

B I l!!J A ... ·- 2
·- =
1- - - ..... C-:)

Save my notes l
Q8 fc:::J

Which of t he followi ng results est ablishes a diagnosis of diabetes


mellitus?

Asymptomatic patient with fast ing glucose 7. 9 mmol/L on one


occasion

Symptomatic patient with fasting glucose 6.8 mmol/L on two


occasions

Glycosuria +++

Asymptomatic patient with random glucose 22.0 mmol/ L on one


occasion

Symptomatic patient with random glucose 12.0 mmol/L on one


occasion

Submit answer

Reference ranges v
Symptomatic patient with fasting glucose 6.8 mmol/L on two
occasions

Glycosuria +++

Asymptomatic patient with random glucose 22.0 mmol/L on one


occasion

Symptomatic patient with random glucose 12.0 mmol/ L on one


occasion

Diabetes diagnosis: fasting > 7.0, random > 11.1 - if


asymptomatic need two readings
Importance: 50

[ ,6 • ilt Discuss (4) Improve

Next question )
Q9 Jc::l

A 43-year-old woman is referred to the endocrine clinic w ith symptoms


of weight gain, fatigue and headache. She was also recently diagnosed
with type two diabetes.

On examination, you note truncal obesity with proximal wasting of the


arms and legs. Hirsutism is present, and the skin appears thin with
multiple striae and bruises.

You suspect Cushing's syndrome and perform routine blood tests as


part of your investigations.

What biochemical abnormality would you expect to ftnd in this


condition?

Hypercalcaemia

Hyperkalaemic metabolic acidosis

Hyperkalaemic metabolic alkalosis

Hypokalaemic metabolic acidosis

Hypokalaemic metabolic alkalosis

Submit answer

Reference ranges v
I Hypokalaemic metabolic alkalosis

Cushing's syndrome - hypokalaemic metabolic alkalosis


Importance: 50

This patient has presented with several characteristic features of


Cushing's syndrome.

In terms of biochemical abnormalities seen on routine testing,


Cushing's syndrome often results in impaired glucose tolerance and
hyperglycaemia, as seen in this patient. A hypokalaemic metabolic
alkalosis can also be seen, as a result of increased renal
mineralocorticoid action from the excess cortisol. This results in an
increased exchange of sodium and water for potassium and H+, leading
to a hypokalaemic metabolic acidosis.

This picture is also seen in Conn's syndrome.

Hypercalcaemia is not usually associated with Cushing's syndrome.

116 1f • Discuss (1) Improve

Next question )

Cushing's syndrome: investigations

Investigations are divided into confirming Cushing's syndrome and then


localising the lesion. A hypokalaemic metabolic alkalosis may be seen,
along with impaired glucose tolerance. Ectopic ACTH secretion (e.g.
secondary to small cell lung cancer) is characteristically associated
with very low potassium levels. An insulin stress test is used to
differentiate between true Cushing's and pseudo-Cushing's

Tests to confirm Cushing's syndrome


The two most commonly used tests are:
• overnight dexamethasone suppression test (most sensitive)
• 24 hr urinary free cortisol

Localisation tests

The first-line localisation is 9am and midnight plasma ACTH (and


cortisol) levels. If ACTH is suppressed then a non-ACTH dependent
cause is likely such as an adrenal adenoma

Both low- and high-dose dexamethasone suppression tests may be


used to localise the pathology resulting in Cushing's syndrome. These
tests may be interpreted as follows:

Cortisol result Interpretation

Not suppressed by low-dose Cushing's syndrome not due to


dexamethasone primary causes, i.e. likely secondary
to corticosteroid therapy

Not suppressed by low- Cushing's disease


dose, but suppressed by
high-dose dexamethasone

Not suppressed by low- or Ectopic ACTH syndrome likely


high-dose dexamethasone

CRH stimulation
• if pituitary source then cortisol rises
• if ectopic/ adrenal then no change in cortisol

Petrosal sinus sampling of ACTH may be needed to differentiate


between pituitary and ectopic ACTH secretion
0 Q10

A 41-year-old man presents with recurrent headaches. These typica lly


occur 2-3 times a day and are associated with sweating and
palpitations. As he was concerned that it may be due to blood pressure
he borrowed his fathers home monitor. During these episodes his blood
pressure is around 210/11 O mmHg. Given the likely diagnosis, what is
the most appropriate next test?

MRI adrenals

Phenoxybenzamine suppression test

24 hour urinary collection of vanillylmandelic acid

24 hour urinary collection of metanephrines

24 hour urinary collection of catecholamines

Submit answer

Reference ranges v
Phenoxybenzamine suppression test

24 hour urinary collection of vanillylmandelic acid

24 hour urinary collection of metanephrines

24 hour urinary collection of catecholamines

Phaeochromocytoma: do 24 hr urinary metanephrines, not


catecholam ines
Importance: 50

Three 24 hour collections are needed as some patients have


intermittently raised levels.

" If t9 Discuss (3) Improve ]


Phaeochromocytoma

Phaeochromocytoma is a rare catecholamine secreting tumour. About


10% are familial and may be associated with MEN type II,
neurofibromatosis and van Hippel-Lindau syndrome

Basics
• bilateral in 10%
• malignant in 10%
• extra-adrenal in 10% (most common site = organ of Zuckerkandl,
adjacent to the bifurcation of the aorta)

Features are typically episodic


• hypertension (around 90% of cases, may be sustained)
• headaches
• palpitations
• sweating
• anxiety

Tests
• 24 hr urinary collection of metanephrines (sensitivity 97%*)
• this has replaced a 24 hr urinary collection of catecholamines
(sensitivity 86%)

Surgery is the definitive management. The patient must first however be


stabilized with medical management:
• alpha-blocker (e.g. phenoxybenzamine), given before a
• beta-blocker (e.g. propranolol)

*BMJ 2012; 344 doi: http://dx.doi.org/ 10.1136/ bmj.e1042 (Published


20 February 2012)

Next uestion )
Q11

A 25-year-old man with a family history of multiple endocrine neoplasia


type 1 is reviewed in clinic. What is the single most useful investigation
to monitor such patients?

Short synacthen test

Urinary catecholamines

Serum calcium

Thyroid function tests

Serum prolactin

Submit answer

Reference ranges v
Urinary catecholamines

Serum calcium

Thyroid function tests

Serum prolactin

Peptic ulceration, galactorrhoea, hypercalcaemia - multiple


endocrine neoplasia type I
Importance: 50

The high incidence of parathyroid tumours and hypercalcaemia make


serum calcium a useful indicator of MEN type 1 in suspected
individuals

9' at - Discuss Improve

Next question )
Multiple endocrine neoplasia

The table below summarises the three main types of multiple endocrine
neoplasia (MEN). MEN is inherited as an autosomal dominant disorder.

MEN type I MEN type Ila MEN type llb

Medullary thyroid Medull ary thyroid


3 P's
cancer {70%) cancer

Parathyroid (95%):
hyperpa rathyro id ism 2 P's 1p

due to parathyroid
hyperplasia Parathyroid (60%) Phaeochromocytom,
Pituitary (70%) Phaeochromocytoma
Pancreas (50%): e.g. Marfanoid body
insulinoma, habitus
gastrinoma (leading Neuromas
to recurrent peptic
ulceration)

Also: adrenal and


thyroid

MEN1 gene RET oncogene RET oncogene

Most common
presentation =
hypercalcaemia
MEN type 1

Pancreatic tumours
(e.g. gastrinoma,
insulinoma)

Pituitary tumours
(e .g. prolactinoma)

Primary
hyperparathyroidism

MEN type 2a MEN type 2b

Phaeochromocytoma
Marfanoid body habitus
Medullary thyroid
cancer
Neuromas

RET oncogene

C Passmed'C.ne com
Q12

You review a 47-year-old man one year after he was diagnosed with
prediabetes. Last year he had a HbA 1 c taken after being diagnosed as
having hypertension. This was recorded as being 43 mmol/ mol (6.1%).
His most recent blood test is recorded as being 45 mmol/mol (6.3%)
despite the patient reporting that he has changed his diet as instructed
and exercising three times a week. His body mass index (BMI) today is
26.5 kg/m 2 • Last year it was 27.5kg/m 2 . What is the most appropriate
course of action?

Start metformin

Start pioglitazone

Review again in 12 months

Start orlitstat

Do a oral glucose tolerance test

Submit answer

Reference ranges v
Start metformin

Start pioglitazone

Review again in 12 months

Start orlitstat

Do a oral glucose tolerance test

NICE recommend metformin for adults at high risk 'whose blood


glucose measure (fasting plasma glucose or HbA 1c) shows they are still
progressing towards type 2 diabetes, despite their participation in an
intensive lifestyle-change programme'.

116 11
f • Discuss (7) Improve

Next question )

Prediabetes and impaired glucose regulation

Prediabetes is a term which is increasingly used where there is


impaired glucose levels which are above the normal range but not high
enough for a diagnosis of diabetes mellitus. The term includes patients
who have been labelled as having either impaired fasting glucose (IFG)
or impaired glucose tolerance (IGT). Diabetes UK estimate that around
1 in 7 adults in the UK have prediabetes. Many individuals with
prediabetes will progress on to developing type 2 diabetes mellitus
(T2DM) and they are therefore at greater risk of microvascular and
macrovascular complications.

Terminology
• Diabetes UK currently recommend using the term prediabetes
when talking to patients and impaired glucose regulation when
talkina to other healthcare orofessionals
• research has shown that the term 'prediabetes' has the most
impact and is most easily understood

Identification of patients with prediabetes


• NICE recommend using a validated computer based risk
assessment tool for all adults aged 40 and over, people of South
Asian and Chinese descent aged 25-39, and adults with
conditions that increase the risk of type 2 diabetes
• patients identified at high risk should have a blood sample taken
• a fasting plasma glucose of 6.1-6.9 mmol/1 or an HbA 1 c level of
42-47 mmol/mol (6.0-6 .4 %) indicates high risk

Fasting glucose: <s C.O mmol !I >• 7,0mmol l

~,c is ~l-AT n-rnol'mel Oo..e ~ l

" ~~
°'
gV.OM 6 1~ l> l'M'<CL1

HbA'1e;

1e
ea 41
.~
\;Jiu..~
mmotnnot
)

~:othilor
:>-= d mmoL'mot
(6~ )

="lcw e comiocred nomwil

Diagram showing the spectrum of diabetes diagnosis

Management
• lifestyle modification: weight loss, increased exercise, change in
diet
• at least yearly follow-up with blood tests is recommended
• NICE recommend metformin for adults at high risk 'whose blood
glucose measure (fasting plasma glucose or HbA 1c) shows they
are still progressing towards type 2 diabetes, despite their
participation in an intensive lifestyle-change programme'
Impaired fasting glucose and impaired glucose tolerance

There are two main types of IGR:


• impaired fasting glucose (IFG) - due to hepatic insulin resistance
• impaired glucose tolerance (IGT) - due to muscle insulin
resistance
• patients with IGT are more likely to develop T2DM and
cardiovascular disease than patients with IFG

Definitions
• a fasting glucose greater than or equal to 6.1 but less than 7.0
mmol/ 1implies impaired fasting glucose (IFG)
• impaired glucose tolerance (IGT) is defined as fasting plasma
glucose less than 7.0 mmol/1 and OGTT 2-hour value greater than
or equal to 7.8 mmol/ 1but less than 11 .1 mmol/ 1
• people with IFG should then be offered an oral glucose tolerance
test to rule out a diagnosis of diabetes. A result below 11 .1
mmol/ 1but above 7.8 mmol/1 indicates that the person doesn't
have diabetes but does have IGT

Next question )

B /

Save my notes

Search
Q13

An 18-year-old male is reviewed due to concerns about delayed pubertal


development, despite being 1.77m tall. On examination he has scant
pubic hair and reduced testicular volume. The following blood results
are obtained:

Testosterone 6.7 nmol/I {9 - 30)

LH 3 .1 mu/I (3-10)

FSH 5.7 mu/I (3 -10)

What is the most likely diagnosis?

Klinefelter's syndrome

Acute lymphoblastic leukaemia

Testicular feminisation syndrome

Primary testicular failure

Kallman's syndrome

Submit answer

Reference ranges v
Acute lymphoblastic leukaemia

Testicular feminisation syndrome

Primary testicular failure

Kallman's syndrome

Klinefelter's - LH & FSH raised


Kallman's - LH & FSH low-normal
Importance: 50

The LH and FSH levels are inappropriately low-normal given the low
testosterone concentration, which points towards a diagnosis of
hypogonadotrophic hypogonadism. In Klinefelter's syndrome the LH
and FSH levels are raised

8' if • Discuss (6) Improve

Next question )
I

Next question )

Kallmann's syndrome

Kallmann's syndrome is a recognised cause of delayed puberty


secondary to hypogonadotrophic hypogonadism. It is usually inherited
as an X-linked recessive trait. Kallmann's syndrome is thought to be
caused by failure of GnRH-secreting neurons to migrate to the
hypothalamus.

The clue given in many questions is lack of smell (anosmia) in a boy


with delayed puberty

Features
• 'delayed puberty'
• hypogonadism, cryptorchidism
• anosmia
• sex hormone levels are low
• LH, FSH levels are inappropriately low/normal
• patients are typically of normal or above average height

Cleft lip/palate and visual/hearing defects are also seen in some


patients

Next question)

B I ~

Save my notes
0 Q14

A 64-year-old man with a history of type 2 diabetes comes to the clin ic


for review. His HbA1c is elevated at 64 mmol/ mol despite taking 1g of
metformin 8D. On examination his blood pressure is 142/88 mmHg, his
pulse is 82 beats per minute and regular. His body mass index is
elevated at 33 kg/m 2 • A decision is made to start him on dapagliflozin.
Which of the fo llowing would you expect on starting therapy?

Hypoglycaemia

Increased blood pressure

Increased serum urate

Increased total cholesterol

Weight gain

Submit answer

Reference ranges v
Increased blood pressure

Increased serum urate

Increased total cholesterol

Weight gain

SGLT-2 inhibitors like dapagliflozin promote increased glucose excretion


because they inhibit glucose reabsorption in the kidney. This
corresponds to a calorie load of 200-400 kcal per day. In some patients,
this results in dramatic weight loss, although on average this equates to
1-2% reduction in weight over 6 months. SGLT-2 inhibitors are
recognised to increased total cholesterol, (both HDL and LDL), although
cardiovascular outcome studies as yet do not suggest this translates
into increased risk of MACE events. In fact, the EM PA-reg study with
empagliflozin demonstrated a reduction in overall mortality.

Hypoglycaemia is not a feature of SGLT2 inhibitor use and SGLT-2


inhibitors are associated with increased urate excretion rather than an
increase in serum uric acid.

https://www.evidence.nhs.uk/ formulary/ bnf/current/6-endocrine-


system/61-drugs-used-in-diabetes/612-antidiabetic-drugs/6123-other-
antidiabetic-drugs/ dapagliflozin

http://www.nejm.org/ doi/ full/ 10.1056/ NEJMoa1504720#t=article

,6 •• t9 Discuss (3) Improve ]

Next question )
9' •• • Discuss (3) Improve

Next question >

SGLT-2 inhibitors

SGLT-2 inhibitors reversibly inhibit sodium-glucose co-transporter 2


(SGLT-2) in the renal proximal convolut ed tubule to reduce glucose
reabsorption and increase urinary glucose excretion.

Examples include canagliflozin, dapagliflozin and empagliflozin.

Important adverse effects include


• urinary and genital infection (secondary to glycosuria). Fournier's
gangrene has also been reported
• normoglycaemic ketoacidosis
• increased risk of lower-limb amputat ion: feet shou ld be closely
monitored

Patients taking SGLT-2 drugs often lose weight, which can be beneficial
in type 2 diabetes mellitus.

Next question >

B I A ... ·-
·- 1-
~
- - ...

Save my notes
Q15

A 18-year-old man with a background of Marfan syndrome presents to


the emergency department with palpitations and sweating. He was
hypertensive on admission at 198/ 101 mmHg. He Is also complaining
of the development of nodules on his torso and cheek which provide a
pins and needle like sensation. A 24 hour urinary catecholamine has
been sent and is currently pending.

What is the most likely underlying diagnosis?

Thyroid carcinoma

MEN type 2A

MEN type 1

MEN type 2B

Pheochromocytoma

Submit answer

Reference ranges v
MEN type 2A

MEN type 1

MEN type 2B

Pheochromocytoma

Medullary thyroid cancer, phaeochromocytoma, marfanoid body


habitus - multiple endocrine neoplasia type lib
Importance: 50

When differentiating between MEN 2A and 28, it is worth remembering


that MEN 28 has similar characteristics as MEN 2A (Thyroid
carcinoma's, Adrenal tumours, Parathyroid hyperplasia) but in addition
typically have a Marfanoid appearance and mucosa! neuromas, as well
as the absence of hyperparathyroidism.

MEN type 1 is characterised by pancreatic neuroendocrine tumours,


pituitary adenoma and parathyroid hyperplasia.

8' •• tt Discuss (5) Improve

Next question )
Q16

A 48-year-old lady is seen in the diabetes clinic with uncontrolled blood


sugars ranging from 14 mmol/L to 22 mmol/L. She has a past medical
history of type 2 diabetes, ischaemic heart disease, rheumatoid arthritis
and recurrent episodes of thrush alongside chronic obstructive
pulmonary disease. Her body mass index is 30. Which medical co-
morbidity is the strongest contraindication to starting an SGLT2
(sodium glucose transport protein 2) inhibitor class of drugs?

lschaemic heart disease

Chronic obstructive pulmonary disease

Type 2 diabetes

Rheumatoid arthritis

Recurrent thrush

Submit answer

Reference ranges v
Chronic obstructive pulmonary disease

Type 2 diabetes

Rheumatoid arthritis

Recurrent thrush

Dapagliflozin is a newer drug for the treatment of diabetes. It is a


member of the sodium-glucose transport protein 2 (SGLT2) inhibitor
class of drugs.

SGLT2 inhibitors prevent the resorption of glucose from the proximal


renal tubule, resulting in more glucose being secreted in the urine. Due
to an increased amount of glucose being secreted in the urine, these
medications are contra-indicated in patients with recurrent thrush. The
increased amount of glucose in the urine is thought to predispose to
bacterial growth. It should also be noted that urine dip sticks will test
positive for glucose.

The other medications in this class include: canagliflozin &


empagliflozin

The other answers are distractors with no known contraindication to


SGLT2 inhibitor use in ischaemic heart disease, chronic obstructive
pulmonary disease or rheumatoid arthritis. SGLT2's are indicated in
patients with type 2 diabetes. Note that although trials are ongoing,
SGLT2 inhibitors are not currently licensed as an adjunct in patients
with type 1 diabetes.
Q17

A 51-year-old woman who is known to have poorly controlled type 1


diabetes mellitus is reviewed. Her main presenting complaint is bloating
and vom iting after eating. She also notes that her blood glucose
readings have become more erratic recently. Which one of the following
medications is most likely to be beneficial?

He/icobacter pylori eradication therapy

Lansoprazole

Amitriptyline

Metoclopramide

Cyclizine

Submit answer

Reference ranges v
Lansoprazole

Amitriptyline

Metoclopramide

Cyclizine

The true prevalence of gastroparesis in patients with type 1 diabetes


mellitus is not known but most modern studies estimate around 5% are
affected. This leads to both upper gastrointestinal symptoms and
erratic glucose control due to gastric emptying dysfunction.
Metoclopramide is a pro-kinetic drug that can improve gastric emptying.

" •• _, Discuss (1) Improve ]

Next question )
Diabetic neuropathy

Diabetes typically leads to sensory loss and not motor loss in peripheral
neuropathy. Painful diabetic neuropathy is a common problem in
clinical practice.

NICE updated it's guidance on the management of neuropathic pain in


2013. Diabetic neuropathy is now managed in the same way as other
forms of neuropathic pain:
• first-line treatment: amitriptyline, duloxetine, gabapentin or
pregabalin
• if the first-line drug treatment does not work try one of the other 3
drugs
• tramadol may be used as 'rescue therapy' for exacerbations of
neuropathic pain
• topical capsaicin may be used for localised neuropathic pain (e.g.
post-herpetic neuralgia)
• pain management clinics may be useful in patients with resistant
problems

Gastrointestinal autonomic neuropathy

Gastroparesis
• symptoms include erratic blood glucose control, bloating and
vomiting
• management options include metoclopramide, domperidone or
erythromycin (prokinetic agents)

Chronic diarrhoea
• often occurs at night

Gastro-oesophageal reflux disease


• caused by decreased lower esophageal sphincter (LES) pressure
Q18 {CJ

A 53-year-old male presents to the Emergency Department complaining


of extreme fatigue. He has a background of treated Graves disease. On
examination his blood pressure is 103/ 58 mmHg, pulse 64/ min and
temperature 36.3°C. The following results are obtained:

Na+ 135 mmol/ 1

K+ 5.4 mmol/1

Urea 5.2 mmol/ 1

Creatinine 42 umol/1

TSH 3.5 mu/ I

Free thyroxine (T4) 12 pmol/ 1

You arrange for a random cortisol test however whilst awaiting the
result he becomes unresponsive. In addition to giving intravenous
steroid and fluid, what test is it imperative to check first given the likely
diagnosis?

Serum calcium

ECG

Arterial pH

Prolactin

Glucose

Submit answer
I

CamScanner -! W~ :.i.;,-~\
I Glucose l
's disease. The
This que stio n is alluding to a diag nos is of Addison
low bloo d pressure
auto imm une history, raised potassium, fatig ue and
are all clues to this .

ing hypoglycaemia
Patients with Addison 's disease are prone to develop
ids. Given the
due to loss of the gluc oge nic effe ct of gluc oco rtico
checked.
sudden deterioration in GCS, a glucose level mus t be

Addison 's disease:


Addison's is adre noc ortic al insu ffici enc y due to the
cts both
dys func tion /des truc tion of the adrenal cortex. It affe
eral oco rtico id (sal t
gluc oco rtico id (me tabo lism of gluc ose etc.) and rnin
eloped wor ld is
balance) function. The com mon est cause in the dev
tion of the cort ex
auto imm une adrenalitis. Other causes include des truc
by infe ctio ns such as TB, or met asta sis.

Signs/ sym ptom s of Addisonian cris is:

Neurological
• syncope
• con fusi on
• lethargy
• convulsions

Haemodynamic
• hypotension
• hypothermia

Biochemical
• hyponatraemia
• hyperkalaernia
• hypoglycaemia

Cu,,1Scilnr1cf _, L,J;.4 .i.> _........i..t.l'


Management of Addisonian crisis (medical emergency):
• intravenous fluids
• corticosteroids (e.g iv dexamethasone)

Note: iv dexamethasone is often preferred as this will not interfere with


cortisol assays needed for a short synacthen test, unlike
hydrocortisone.

a6 1f - Discuss (5) Improve ]

'

Next question )

Addisonian crisis

Causes
• sepsis or surgery causing an acute exacerbation of chronic
insufficiency (Addison's, Hypopituitarism)
• adrenal haemorrhage eg Waterhouse-Friderichsen syndrome
(fulminant meningococcemia)
• steroid withdrawal

Management
• hydrocortisone 100 mg im or iv
• 1 litre normal saline infused over 30-60 mins or with dextrose if
hypoglycaemic
• continue hydrocortisone 6 hourly until the patient is stable. No
fludrocortisone is required because high cortisol exerts weak
mineralocorticoid action
• oral replacement may begin after 24 hours and be reduced to
maintenance over 3-4 days

Next question )
C Q19 Jr::J

A 23-year-old woman presents for review. She has not had a normal
period for around 8 months now. A recent pregnancy test was negative.
Blood tests are ordered:

FSH 2.2 IU/ L (0-20 IU/ L)

Oestradiol 84 pmol/I (100-500 pmol/ 1)

Thyroid stimulating hormone 3.1 mlU/ L

Prolactin 2 ng/ ml (0-1 0 ng/ ml)

Free androgen index 3(< 7)

What is the most likely cause of her symptoms?

Prolactinoma

Premature ovarian failure

Polycystic ovarian syndrome

Addison's disease

Excessive exercise

Submit answer

Reference ranges v

CamScanne, -? Y.¥-> i.b. ~I


Premature ovarian failure

Polycystic ovarian syndrome

Addison's disease

Excessive exercise

The bloods show a hypothalamic amenorrhoea which may be caused


by stress or excessive exercise. The FSH would be raised in premature
ovarian failure.

[ ,6 'f • Discuss (5) Improve ]

Next question )

Amenorrhoea
Amenorrhoea may be divided into prima ry (failure to start mense s by
the age of 16 years) or secondary (cessation of established, regular
menstruation for 6 month s or longer).

Causes of primary amenorrhoea


• Turner's syndrome
• testicu lar feminisation
• congenital adrenal hyperplasia
• congenital malfo rmatio ns of the genital tract

Secondary amenorrhoea is defined as when menst ruatio n has


previously occurred but has now stopp ed for at least 6 month s.

Causes of secondary amenorrhoea (after excluding pregnancy)


• hypothalamic amenorrhoea (e.g. Stress, excessive exercise)
• polycystic ovarian syndrome (PCOS)
• hyperprolactinaemia
• premature ovarian failure
• thyrot oxicos is*
• Sheehan's syndrome
• Asherman's syndrome (intrauterine adhesions)

Initial investigations
• exclude pregnancy with urinary or serum bHCG
• gonadotrophins: low levels indica te a hypot halam ic cause where
as raised levels sugge st an ovarian problem (e.g. Premature
ovarian failure)
• prolactin
• androgen levels: raised levels may be seen in PCOS
• oestradiol
• thyroid functio n tests

*hypothyroidism may also cause amenorrhoea


Q20

A 56-year-old man is reviewed in the Cardiology outpatient clinic


following a myocardial infarction one year previously. During his
admission he was found to be hypertensive and diabetic. He complains
that he has put on 5kg in weight in the past 6 months. Which of his
medications may be contributing to his weight gain?

Metformin

Losartan

Clopidogrel

Gliclazide

Simvastatin

Submit answer

Reference ranges v
Losartan

Clopidogrel

Gliclazide

Simvastatin

Sulfonylureas often cause weight gain


Importance: 50

[ .. •• ,- Discuss (3) Improve ]

Next question )
Next question )

Sulfonylureas

Sulfonylureas are oral hypoglycaemic drugs used in the management of


type 2 diabetes mellitus. They work by increasing pancreatic insulin
secretion and hence are only effective if functional 8-cells are present.
On a molecular level they bind to an ATP-dependent K+(KArP) channel
on the cell membrane of pancreatic beta cells .

Common adverse effects


• hypoglycaemic episodes (more common with long-acting
preparations such as chlorpropamide)
• weight gain

Rarer adverse effects


• hyponatraemia secondary to syndrome of inappropriate ADH
secretion
• bone marrow suppression
• hepatotoxicity (typically cholestatic)
• peripheral neuropathy

Sulfonylureas shou ld be avoided in breastfeeding and pregnancy.

Next question >

B I l!!J A ...,
Q21

Which one of the following skin disorders is least associated with


hypothyroidism?

Xanthomata

Pruritus

Pretibial myxoedema

Eczema

Dry, coarse hair

Submit answer

Reference ranges v
I _ a_ ~ ~ - ~
Pretibial myxo_e_d_em

Eczema

Dry, coarse hair

is
For the purposes of postg radu ate exam s pretibial myxo edem a
of it
associated with thyrotoxicosis. There are however case repo rts
pittin g
been foun d in hypothyroid patients, especially the diffu se non-
variety

[ ,6 ,. • Discu ss (1) Im prove ]

Next ques tion )

Skin disorders associated with thyroid dis eas e

Skin man ifesta tions of hypo thyro idism


• dry (anhydrosis), cold, yellowish skin
• non-pitting oedema (e.g. hands, face)
• dry, coarse scalp hair, loss of lateral aspe ct of eyebrows
• eczema
• xanth oma ta

Skin man ifesta tions of hype rthyr oidis m


e
• pretibial myxoedema: erythematous, oede mato us lesions abov
the lateral malleoli
• thyroid acropachy: clubbing
• scalp hair thinning
• increased swea ting

Pruritus can occu r in both hyper- and hypo thyro idism


Q22

A 57-year-old man with a known history of type-2 diabetes presents to


clinic for a review. He currently takes metformln only for his diabetes
and reports following the regime as instructed.

His HbA 1c is 63 mmol/mol (target = 53mmol/mol) and the clinician and


patient decide he should start a sulfonylurea in addition to his
metformin.

Which of the following best describes the new treatment's mechanism


of action?

Increases stimulation of insulin secretion by pancreatic 8-cells


and decreases hepatic clearance of insulin

Inhibits sodium-glucose co-transporter-2 in the proximal


convoluted tubule of the nephron to stop glucose reabsorption,
meaning it is excreted in urine

Inhibits the principal enzyme that breaks down GLP-1 - an incretin


hormone that increases insulin secretion and suppresses
glucagon secretion

Reduces hepatic gluconeogenesis, increases peripheral glucose


uptake and also reduces the absorption of carbohydrate in the
gut

Upregulation of transcription of insulin responsive genes, leading


to an increase in glucose transporters and insulin receptors at
the surface of the cell

Submit answer
Increases stimulation of insulin secretion by pancreatic 8-cells and
decreases hepatic clearance of insulin

Inhibits sodium-glucose co-transporter-2 in the proximal convoluted


tubule of the nephron to stop glucose reabsorption, meaning it is
excreted in urine

Inhibits the principal enzyme that breaks down GLP-1 - an incretin


hormone that increases insulin secretion and suppresses glucagon
secretion

Reduces hepatic gluconeogenesis, increases peripheral glucose


uptake and also reduces the absorption of carbohydrate in the gut

Upregulation of transcription of insulin responsive genes, leading to


an increase in glucose transporters and insulin receptors at the
surface of the cell

Sulfonyureas increase stimulation of insulin secretion by


pancreatic 8-cells and decrease hepatic clearance of insulin
Importance: 50

The correct answer is option 1 as sulfonylureas are insulin


secretagogues - one of the classes of oral hypoglycaemic agents. [,]

Option 2 best describes the mechanism of action of SGLT-2 inhibitors


(dapagliflozin, canagliflozin).l2l

Option 3 best describes the mechanism of action of DPP-4 inhibitors


(sitagliptin, vildagliptin).l3l

Option 4 best describes the mechanism of action of metformin, an


insulin sensitiser.l4l

Option 5 best describes the mechanism of action of thiazolidinediones


(pioglitazone), also an insulin sensitiser,[41
di ,u I t:µu1 L~ 1u11owing 1ne regime as instructed.

His HbA 1 c is 63 mmol/mol (target = 53mmol/mol) and the clinician and


patient decide he should start a sulfonylurea in addition to his
metformin.

Which of the following best describes the new treatment's mechanism


of action?

Increases stimulation of insulin secretion by pancreatic B-cells and


decreases hepatic clearance of insulin

Inhibits sodium-glucose co-transporter-2 in the proximal convoluted


tubule of the nephron to stop glucose reabsorption, meaning it is
excreted in urine

Inhibits the principal enzyme that breaks down GLP-1 - an incretin


hormone that increases insulin secretion and suppresses glucagon
secretion

Reduces hepatic gluconeogenesis, increases peripheral glucose


uptake and also reduces the absorption of carbohydrate in the gut

Upregulation of transcription of insulin responsive genes, leading to


an increase in glucose transporters and insulin receptors at the
surface of the cell

Sulfonyureas increase stimulation of insulin secretion by


pancreatic 8-cells and decrease hepatic c learance of insulin
Importance: 50

Ca mSc.an,ner ~ '....,.i;lo-" ~ ~ I
Importance: 50

The corre ct answ er is optio n 1 as sulfonylureas are insulin


1
secre tago gues - one of the class es of oral hypo glyca emic agents. [ 1

Option 2 best desc ribes the mech anism of actio n of SGLT-2 inhibitors
2
(dap aglifl ozin, cana gliflo zin).[ 1

Optio n 3 best desc ribes the mech anism of actio n of DPP-4 inhibitors
3
( sitag liptin , vilda glipti n). [ ]

Optio n 4 best desc ribes the mech anism of actio n of metf ormi n, an
insul in sens itiser.[4]

es
Option 5 best desc ribes the mech anism of actio n of thiazolidinedion
4
(piog litazo ne), also an insul in sensitiser,[ 1

Acco rding to NICE, metf ormi n should be the first drug treat ment of
choic e for thos e peop le who can tolera te it. The second medication, to
be given in comb inatio n with metf ormi n, can be any one of the four
types of oral hypo glyca emic agents listed_[sJ
Q23

A 44-year-old woman presents with a neck lump. She reports tiredness


and fatigue and has put on around 3kilograms of weight recently; she
reports going up 3 belt notches. Her blood results show normocytic
anaemia.

On palpation of her neck, a hard, fixed, painless lump is felt.

Which one of the following complications is associated with her


condition?

Hyperthyroidism

Retroperitoneal fibrosis

Ascites

Atrial fibrillation

Photosensitivity

Submit answer

Reference ranges v
Retroperitoneal fibrosis

Ascites

Atrial fibrillation

Photosensitivity

Riedel's thyroiditis is associated with retroperitoneal fibrosis


Importance: 50

Riedel thyroiditis (RT) is characterized by the replacement of normal


thyroid parenchyma with dense fibrotic tissue and by the extension of
this fibrosis to adjacent structures of the neck. Most patients are
euthyroid, but hypothyroidism is noted in approximately 30% of cases.

Patient's may present with a painless neck lump and symptoms of


hypothyroidism such as weight gain, tiredness, fatigue and intolerance.

Riedel's thyroiditis is not associated with hyperthyroidism, ascites or


photosensitivity and therefore these answers are incorrect.

Atrial fibrillation is a complication of hyperthyroidism and patients with


AF should have their thyroid function assessed. However, there is no
evidence to suggest that Riedel's thyroiditis is associated with atrial
fibrillation .

[ ,6 'f tit Discuss (2) Improve

Next question )
Next question )

Riedel's thyroiditis

Riedel's thyroiditis is a rare cause of hypothyroidism characterised by


dense fibrous tissue replacing the normal thyroid parenchyma. On
examination a hard, fixed, painless goitre is noted. It is usually seen in
middle-aged women. It is associated with retroperitoneal fibrosis.

Hypothyroidism Hyperthyrodlsm

H sh tmo10'$
thyroid tlS Subaculti thyrokf
(d Ou rv n' )

Ri
--
rs lhyroid1t11
Postpartum thYTOkfius
Am:.odarone
Gr ves' dis a

Toxic mu t nodular g01tre

~ .......

Venn diagram showing how different causes of thyroid


dysfunction may manifest. Note how many causes of
hypothyroidism may have an initial thyrotoxic phase.

Next question )

B J I!!! A • ·- 2:
1- - - ....
Q24

A 93-year-old female who lives alone comes to see you regarding


troublesome urge incontinence. Over the past year, you have noted a
steady decline and she is becoming increasingly frail. She has had a
number of falls while rushing to the bathroom, resulting in attendance
at the local emergency department. She previously underwent a course
of bladder retraining with no significant improvement in symptoms.

What would be the most appropriate treatment of her urge


incontinence?

Pelvic floor exercises

Immediate release oxybutynin

Mirabegron

Doxazosin

Surgical repair

Submit answer

Reference ranges v
Mirabegron

Doxazosin

Surgical repair

Oxybutynin should not be used in frail older women with urinary


incontinence due to the risk of impairment of daily functioning,
confusion and acute delirium
Importance: 50

The correct answer here is mirabegron. According to the latest NICE


guidance, first line medical treatment of urge incontinence after bladder
retraining can be with oxybutynin, tolterodine or darifenacin. NICE has
1
issued a do not use' statement on the use of oxybutynin in frail elderly
women due to the risk of cognitive impairment, falls and general
decline. This is particularly the case with immediate release
preparations. There is no similar statement for tolterodine or
darifenacin. A trial or tolterodine or darifenacin, may have been an
appropriate answer if this option was given, although these are also
associated with anticholinergic side effects. Given the patients• history
of general decline and recurrent falls, avoidance of an anticholinergic
and treatment with mirabegron would, therefore, be a more appropriate
choice than oxybutynin.

Oxybutynin is normally the antimuscarinic of choice in urge


incontinence, however, has a high anticholinergic burden. Per NICE
guidance it should be avoided in frail older women due to an increased
risk of delirium, confusion and impaired function. Given the patients'
age, history of gradual decline and recurrent falls, oxybutynin should be
avoided. This is particularly the case, given that she lives alone and so
increased confusion or delirium may not be picked up straight away,
increasing her risk of falls.

Pelvic floor exercises are used in conservative management of stress


incontinence not urge incontinence.
and urinary
Doxazosin is used in the trea tme nt of hypertension
inco ntin enc e.
retention. It is likely to worsen her sym pto ms of urge

ntinence.
Surgery is indicated in stress as opposed to urge inco

[ ,. •• tt Discuss Improve ]

Next que stio n )

Urinary incontinence

ctin g around 4-5 %


Urinary incontinence (UI) is a com mon problem , affe
ales .
of the population. It is mor e com mon in elderly fem

Risk fact ors


• advancing age
• previous pregnancy and chil dbir th
• high body mas s index
• hysterectomy
• fam ily history

Classification
to detr uso r
• overactive bladder (OAB)/urge incontinence: due
overactivity
coughing or
• stress incontinence: leaking sma ll amo unts when
laughing
• mixed incontinence: both urge and stress
tion, e.g. due
• overflow incontinence: due to bladder out let obstruc
to prostate enlargement
Initial investigation
• bladder diaries should be compl eted for a minim um of 3 days
• vaginal examination to exclude pelvic organ prolapse and ability to
initiate voluntary contra ction of pelvic floor muscl es ('Kegel'
exercises)
• urine dipstic k and culture
• urodynamic studies

Management depends on wheth er urge or stress UI is the predo minan t


picture. If urge incontinence is predominant:
• bladder retraining (lasts for a minim um of 6 weeks, the idea is to
gradually increase the intervals between voiding)
• bladder stabilising drugs: antim uscar inics are first-line. NICE
recommend oxybutynin (imme diate release), toltero dine
(imme diate release) or darifenacin (once daily preparation).
Immediate release oxybutynin should, however, be avoided in 'frail
older women'
• mirabegron (a beta-3 agoni st) may be useful if there is concern
about anticholinergic side-e ffects in frail elderly patien ts

If stress incontinence is predominant:


• pelvic floor muscle training: NICE recom mend at least 8
contra ctions performed 3 times per day for a minim um of 3
month s
• surgical procedures: e.g. retropubic mid-urethral tape procedures

Next questi on >


r---------------------------
B I ~ A ...

Save my notes
Q25

th centre with
An 18-year-old man presents to the nurse at the local heal
has vague
a third episode of balanitis over the past 3 mon ths. He also
diag nose d with
sym ptom s of tiredness. His fathe r and gran dfath er were
He is slim with a
type 1 diabetes and take a basal-bolus insulin regimen.
ia on urine
body mas s index of 22 kg/m . He is note d to have glycosur
2

dips tick testi ng.

A fasti ng bloo d sample show s the follo wing :

Na+ 140 mmol/ 1

K+ 3.9 mmo l/ 1

Urea 6.1 mmol/ 1

Creatinine 91 µmol/ 1

Glucose 9.2 mmol/ 1

Which of the follo wing is the mos t likely diagnosis?

Latent auto imm une diabetes of adul ts (LADA)

Matu rity onse t diabetes of the young (MODY)

Renal glycosuria

Type 1 diabetes

Type 2 diabetes

Sub mit answer

Reference ranges v

CamScanne r ➔ •..;~ L - ~1
Maturity onset diabetes of the young (MODY)

Renal glycosuria

Type 1 diabetes

Type 2 diabetes

MODY is autosomal dominant diabetes mellitus which often presents


for the first time in young slim individuals without symptoms of polyuria
and polydipsia. Insidious onset with for instance with recurrent balanitis
as here is usual. It's important to recognise the diagnosis because
many patients with MODY including those with the HNF-7 alpha form of
the disease can be managed with sulphonylureas for many years before
needing to start insulin therapy. Evaluation of family history and testing
for antibodies for type 1 diabetes can help to differentiate MODY from
other forms of diabetes mellitus.

Type 1 diabetes isn't associated with such strong heritability as that


seen here, and given this patient's body habitus, type 2 diabetes is very
unlikely. LADA is associated with a body habitus similar to the
overweight / obese picture seen in type 2 diabetes for many patients,
although progression to insulin therapy occurs more quickly. Renal
glycosuria is ruled out by the elevated fasting glucose seen here.

[ " " - Discuss (6) Improve ]

Next question )
MODY

Maturity-onset diabetes of the young (MODY) is characterised by the


development of type 2 diabetes mellitus in patients< 25 years old. It is
typically inherited as an autosomal dominant cond ition. Over six
different genetic mutations have so far been identified as leading to
MODY.

It is thought that around 1-2% of patients with diabetes mellitus have


MODY, and around 90% are misclassified as having either type 1 or type
2 diabetes mellitus.

MODY3
• 60% of cases
• due to a defect in the HNF-1 alpha gene
• is associated with an increased risk of HCC

MODY2
• 20% of cases
• due to a defect in the glucokinase gene

MODY 5
• rare
• due to a defect in the HNF-1 beta gene
• liver and renal cysts

Features of MODY
• typically develops in patients < 25 years
• a family history of early onset diabetes is often present
• ketosis is not a feature at presentation
• patients with the most common form are very sensitive to
sulfonylureas, insulin is not usually necessary
Q26

A 61-year-old woman is investigated for hoarseness and dyspnoea


which has got progressively worse over the past month. ln the past she
has been diagnosed with toxic multinodular goitre which was
successfully treated with radioiodine. On examination she has a firm,
asymmetrical swelling of the thyroid gland. Laryngoscopy
demonstrates a right vocal cord paralysis and apparent external
compression of the trachea . What is the most likely diagnosis?

Follicular thyroid cancer

Papillary thyroid cancer

Medullary thyroid cancer

Lymphoma of the thyroid gland

Anaplastic thyroid cancer

Submit answer

Reference ranges v
( Anaplastic t hyroid cancer
---------

Anaplastic thyroid cancer - aggressive, difficult to treat and often


causes pressure symptoms
Importance: 50

9' if tt Discuss (3) Improve

Next question )

Thyroid cancer

Features of hyperthyroidism or hypothyroidism are not commonly seen


in patients with thyroid malignancies as they rarely secrete thyroid
hormones

Main points

Type Percentage

Papillary 70% Often young females - excellent


prognosis

Follicular 20%

Medullary 5% Cancer of parafollicular (C) cells,


secrete calcitonin, part of MEN-2

Anaplastic 1% Not responsive to treatment, can


cause pressure symptoms

Lymphoma Rare Associated w ith Hashimoto's


Papillary thyroid cancer

Medullary thyroid cancer

Lymphoma of the thyroid gland

IAnaplastic thyroid cancer

Anaplastic thyroid cancer - aggressive, difficult to treat and often


causes pressure symptoms
Importance: 50

[ ,. .. • Discuss (3) Improve ]

CamScanne, -? Y.¥-> i.b.~I


Thyroid cancer

Features of hyperthyroidism or hypothyroidism are not commonly seen


in patients with thyroid malignancies as they rarely secrete thyroid
hormones

Main points

Type Percentage

Papillary 70% Often young females - excellent


prognosis

Follicular 20%

Medullary 5% Cancer of parafollicular (C) cells,


secrete calcitonin, part of MEN-2

Anaplastic 1% Not responsive to treatment, can


cause pressure symptoms

Lymphoma Rare Associated with Hashimoto's


thyroiditis

Management of papillary and follicular cancer


• total thyroidectomy
• followed by radioiodine (1-131) to kill residual cells
• yearly thyroglobulin levels to detect early recurrent disease
Papillary • Usually contain a mixture of papillary and
carcinoma colloidal filled follicles
• Histologically tumour has papillary projections
and pale empty nuclei
• Seldom encapsulated
• Lymph node metastasis predominate
• Haematogenous metastasis rare

Follicular • Usually present as a solitary thyroid nodule


adenoma • Malignancy can only be excluded on formal
histological assessment

Follicular • May appear macroscopically encapsulated,


carcinoma microscopically capsular invasion is seen.
Without this finding the lesion is a follicular
adenoma.
• Vascular invasion predominates
• Multifocal disease raree

Medullary • C cells derived from neural crest and not thyroid


carcinoma tissue
• Serum calcitonin levels often raised
• Familial genetic disease accounts for up to 20%
cases
• Both lymphatic and haematogenous metastasis
are recognised, nodal disease is associated with
a very poor prognosis.

Anaplastic • Most common in elderly females


carcinoma • Local invasion is a common feature
• Treatment is by resection where possible,
palliation may be achieved through
isthmusectomy and radiotherapy.
Chemotherapy is ineffective.
Q27 JC::,

The fasting glucose for a patient is reported as follows:

Glucose (fasting) 6.3 mmol/1

What is the most likely underlying pathophysiological change?

Beta-cell hyperplasia

Beta-cell atrophy

Muscle insulin resistance

Hepatic insulin resistance

Adipose tissue insulin resistance

Submit answer

Reference ranges v
I Hepatic insulin resistance

Adipose tissue insulin resistance

"' •t • Discuss (1) Improve

Next question )

Prediabetes and impaired glucose regulation

Prediabetes is a term which is increasingly used where there is


impaired glucose levels which are above the normal range but not high
enough for a diagnosis of diabetes mellitus. The term includes patients
who have been labelled as having either impaired fasting glucose (IFG)
or impaired glucose tolerance (IGT). Diabetes UK estimate that around
1 in 7 adults in the UK have prediabetes. Many individuals with
prediabetes will progress on to developing type 2 diabetes mellitus
(T2DM) and they are therefore at greater risk of microvascular and
macrovascular complications.

Terminology
• Diabetes UK currently recommend using the term prediabetes
when talking to patients and impaired glucose regulation when
talking to other healthcare professionals
• research has shown that the term 'prediabetes' has the most
impact and is most easily understood

Identification of patients with prediabetes


• NICE recommend using a validated computer based risk
assessment tool for all adults aged 40 and over, people of South
Asian and Chinese descent aged 25-39, and adults with
conditions that increase the risk of type 2 diabetes
• patients identified at high risk should have a blood sample taken
• a fasting plasma glucose of 6.1-6. 9 mmol/1 or an HbA 1 c level of
42-47 mmol/mol (6.0-6.4%) indicates high risk

Fasting glucose: ea Omrno11 >-7,0mmol.'I

~.!.It 11 ,:...;?rm,ol mcl 16 o:.fl .1"-l


or
" no O'-'«>M ~ ,~ ~ mmo:,

HbA1c· oi 41 mmollmot >• 4' rnmol'mo4


15 ) I )

1 c Inc ~noitc
tnrc:shol:1 IOf b::'es

Diagram showing the spectrum of diabetes diagnosis

Management
• lifestyle modification: weight loss, increased exercise, change in
diet
• at least yearly follow-up with blood tests is recommended
• NICE recommend metformin for adults at high risk 'whose blood
glucose measure (fasting plasma glucose or HbA 1c) shows they
are still progressing towards type 2 diabetes, despite their
participation in an intensive lifestyle-change programme'

Impaired fasting glucose and impaired glucose tolerance

There are two main types of IGR:


• impaired fasting glucose (IFG) - due to hepatic insulin resistance
• impaired glucose tolerance (IGT) - due to muscle insulin
resistance
• patients with IGT are more likely to develop T2DM and
cardiovascular disease than patients with IFG
Definitions
• a fasting glucose greater than or equal to 6.1 but less than 7.0
mmol/1 implies impaired fasting glucose (IFG)
• impaired glucose tolerance (IGT) is defined as fasting plasma
glucose less than 7.0 mmol/ 1and OGTT 2-hour value greater than
or equal to 7.8 mmol/1 but less than 11.1 mmol/ 1
• people with IFG should then be offered an oral glucose tolerance
test to rule out a diagnosis of diabetes. A result below 11.1
mmol/1 but above 7.8 mmol/ 1indicates that the person doesn't
have diabetes but does have IGT

Next question )

B I t, A .... T I""

Save my notes

Search


Search textbook...
----- ·
O. Google search on "Prediabetes and impaired glucose regulation"

Links
Q28

An obese 48-year-old man presents with lethargy and polydipsia. What


is the minimum HbA1c that wou ld be diagnostic of type 2 diabetes
mellitus?

Cannot use HbA1 c for diagnosis

6.0% (42 mmol/mol)

6.3% (45 mmol/mol)

6.5% (48 mmol/mol)

7.0% (53 mmol/mol)

Submit answer

Reference ranges v
Q29

A 42-year-old man presents to his GP feeling generally unwell. For the


past three months he has been experiencing daily frontal headaches
which have not been helped by regular paracetamol. He has also
noticed some unusual symptoms such as his wedding ring no longer
fitting, his shoe size apparently increasing and a small amount of milky
discharge from both nipples. On examination his blood pressure is
168/96 mmHg. What is the most likely diagnosis?

Phaeochromocytoma

Cushing's syndrome

Diabetes insipidus

Macroprolactinoma

Acromegaly

Submit answer

Reference ranges v
I Acrom egaly

[ ,t •• • Discu ss (1) Impro ve ]

Next quest ion )

Acromegaly: features

In acrom egaly there is exces s growt h hormo ne secon dary to a pituita ry


adeno ma in over 95% of cases . A minor ity of cases are cause d by
ectop ic GHRH or GH produ ction by tumou rs e.g. pancr eatic.

Features
• coarse facial appearance, spade-like hands, increa se in shoe size
• large tongue, progn athism , interd ental space s
• excessive sweat ing and oily skin: cause d by sweat gland
hyper trophy
• featur es of pituita ry tumou r: hypop ituitar ism, headaches,
bitemp oral hemia nopia
• raised prolac tin in 1 / 3 of cases -+ galact orrhoe a
• 6% of patien ts have MEN-1

Comp licatio ns
• hyper tensio n
• diabet es (>10%)
• cardio myopa thy
• colore ctal cance r

Next quest ion )


Q30

A 32-year-old male presents with a 10-day history of a worsening rash


on his back and chin . He does not complain of pain or pruritus but is
concerned as the rash is unsightly. His past medical history includes
ulcerative colitis which is being treated with prednisolone due to a
recent flare-up 7 weeks ago and is responding well. He does not smoke,
use tobacco nor alcohol and works as a gardener.

On examination, he has uniform papules with some small pustules with


moderate erythema over his upper trunk, back and jaw. There are no
comedones nor cysts. Vital signs are normal.

Which of the following is the most likely cause of this patient's skin
rash?

Acne vulgaris

Drug-induced adverse effect

Erythema nodosum

Occupational exposure to herbicides

Suppurative folliculitis ( Pseudomonas aeruginosa )

'

Submit answer
Drug-induced adverse effect

Erythema nodosum

Occupational exposure to herbicides

Suppurative folliculitis ( Pseudomonas aeruginosa )

Systemic glucocorticoids can cause drug-induced acne. This is


characterised as monomorphic papular rash without comedones
or cysts. This does not respond to acne treatment but improves
on drug discontinuation
Importance: 50

This patient has drug-induced acne is most commonly caused by


glucocorticoids and androgens but can also be precipitated by
1
azathioprine, phenytoin, antipsychotics, and isoniazid

This typically present s as monomorphic papules and pustules with no


comedones, cysts or nodules within 2 weeks of starting medication.
This subset of patients will not respond to standard acne treatment,
discontinuation of offending agents typical ly clears up the rash.

Acne vulgaris displays lesions in various stages of development and


typically occurs in the face in adolescence.

Dermal manifestations of 180 include erythema nodosum, pyoderma


and pyoderma gangrenosum. Erythema nodosum presents as tender
red nodules, typically on legs and arms.

Herbicides, fungicides, and insecticides contain halogenated aromatic


hydrocarbons. Halogenated compounds can cause occupational acne
(also known as chloracne). Th is rash is characterised by inflammatory
nodules and large comedones.
suppurative folliculitis is most often caused by Staphylococcus aureus,
it can also be caused by pseudomonas.

References
(1) 4. Du-Thanh A, Kluger N, Bensalleh H, Guillot B. Drug-Induced
Acneiform Eruption. American Journal of Clinical Dermatology.
2011 ;12(4):233-245.

a, Rf tt Discuss (1) Improve

Next question >

Corticosteroids: side-effects

Glucocorticoid side-effects
• endocrine: impaired glucose regulation, increased appetite/ weight
gain, hirsutism, hyperlipidaemia
• Cushing's syndrome: moon face, buffalo hump, striae
• musculoskeletal: osteoporosis, proximal myopathy, avascular
necrosis of the femoral head
• immunosuppression: increased susceptibility to severe infection,
reactivation of tuberculosis
• psychiatric: insomnia, mania, depression, psychosis
• gastrointestinal: peptic ulceration, acute pancreatitis
• ophthalmic: glaucoma, cataracts
• dermatological: acne
• suppression of growth in children
• intracranial hypertension
• neutrophilia

Mineralocorticoid side-effects
• fluid retention
• hypertension
Q31

A 54-year-old man w ith type 2 diabetes mellitus is reviewed in clinic. He


is currently taking pioglitazone, metformin, aspirin and simvastatin.
Which one of the following problems is most likely to be caused by
pioglitazone?

Photosensitivity

Thrombocytopaenia

Myalgia

Peripheral oedema

Hyponatraemia

Submit answer

Reference ranges v
I Peripheral oedema

Hyponatraemia

Pioglitazone may cause fluid retention


Important for me Less important

[ ,6 1f tit Discuss (3) Improve ]

Next question >

Thiazolidinediones

Thiazolidinediones are a class of agents used in the treatme nt of type 2


diabetes mellitus . They are agonists to the PPAR-gamma receptor and
reduce peripheral insulin resistance. Rosiglitazone was withdraw n in
201 O following concerns about the ca rdiovasc ular side-effe ct profile.

The PPAR-gamma receptor is an intracellular nuclear receptor. It's


natural ligands are free fatty acids and it is thought to control adipocy te
different iation and function.

Adverse effects
• weight gain
• liver impairment: monitor LFTs
• fluid retention - therefore contrain dicated in heart failure. The risk
of fluid retention is increased if the patient also takes insulin
• recent studies have indicated an increased risk of fracture s
• bladder cancer: recent studies have shown an increased risk of
bladder cancer in patients taking pioglitaz one (hazard ratio 2.64)

Next question >


0 Q32

A 55-year-old taxi driver with type 2 diabetes mellitus comes for review.
When he was diagnosed 12 months ago he was started on metformin
and the dose was titrated up. His IFCC-HbA 1 cone year ago was 75
mmol/mol (DCCT-HbA 1 c 9%) and is now 69 mmol/mol (8.5%). His body
mass index is 33 kg/m 2 . What is the most appropriate next step in
management?

Add exenatide

Add sitagliptin

Add glipizide

Make no changes t o his medication

Add insulin

Submit answer

Reference ranges v
Add sitagliptin

Add glipizide

Make no changes to his medication

Add insulin

His HbA1c is still significantly above target so some change to the


medication is indicated.

The NICE type 2 diabetes mellitus guidelines would generally advocate


the use of a sulfonylurea in this situation.

However. the patient is a taxi driver and overweight. A DPP-4 inhibitor


such as sitagliptin would be ideal in this situation. There is no risk of
hypoglycaemia and they DPP-4 inhibitors are weight neutral.

8' •• • Discuss (12) Improve

Next question )

Diabetes mellitus: management of type 2

NICE updated its guidance on the management of type 2 diabetes


mellitus (T2DM) in 2015. Key points are listed below:
• HbA 1c targets have changed. They are now dependent on what
antidiabetic drugs a patient is receiving and other factors such as
frailty
• there is more flexibility in the second stage of treating patients
(i.e. after metformin has been started) - you now have a choice of
4 oral antidiabetic aoents
It's worthwhile thinking of the average patient who is taking
metformin for T2DM, you can titrate up metformin and encourage
lifestyle changes to aim for a HbA1c of 48 mmol/mol (6.5%), but
should only add a second drug if the HbA 1c rises to 58 mmol/mol
(7.5%)

Dietary advice
• encourage high fibre, low glycaemic index sources of
carbohydrates
• include low-fat dairy products and oily fish
• control the intake of foods containing saturated fats and trans
fatty acids
• limited substitution of sucrose-containing foods for other
carbohydrates is allowable, but care should be taken to avoid
excess energy intake
• discourage use of foods marketed specifically at people with
diabetes
• initial target weight loss in an overweight person is 5-10%

HbA1c targets

This is area which has changed in 2015


• individual targets should be agreed with patients to encourage
motivation
• HbA 1c should be checked every 3-6 months until stable, then 6
monthly
• NICE encourage us to consider relaxing targets on 'a case-by-case
basis, with particular consideration for people who are older or frail,
for adults with type 2 diabetes'
• in 2015 the guidelines changed so HbA1c targets are now
dependent on treatment:
Lifestyle or single drug treatment

HbA1c
Management of T2DM target

Lifestyle 48
mmol/mol
(6.5%)

Lifestyle+ metformin 48
mmol/mol
(6.5%)

Includes any drug which may cause hypoglycaemia 53


(e.g. lifestyle+ sulfonylurea) mmol/mol
(7.0%)

Practical examples
• a patient is newly diagnosed with HbA 1c and wants to try lifestyle
treatment first. You agree a target of 48 mmol/mol (6.5%)
• you review a patient 6 months after starting metformin. His
HbA 1c is 51 mmol/mol (6.8%). You increase his metformin from
500mg bd to 500mg tds and reinforce lifestyle factors

Patient already on treatment

Management of T2DM HbA1 c target

Already on one drug, but HbA1c has risen to 58 53 mmol/mol


mmol/mol (7.5%) (7.0%)
Metformin Metformin not tolerated or Cl

Mettorrmn Glitpln
OR
Sulfonylurea
OR
HbA1c > 58 mmol/mol (7.5%) PloglltaZone

Metformfn
i 1- gltpun
HbA1c > 58 mmolimol (7 5%)
OR I
Metrormln .., su1rony1urea
OR
Metformin • pioglrtazone Ghplln + p1oghtazone
OR OR
Metformln + SGLT-2 Inhibitor Gllptm + sulfonylurea
OR
Plogotazone + sulfonylUrea
HbA1c > 58 mmoL1mol (7 5~o)

Metformin 1- gllptm 1- sul1on\ lurea


OR
r.tetformm + p1091nazone + sulfonylurea HbA1c > 58 mmoVmol (7.5%}
OR
Metformin + sulfon~ lurea + SGLT -2 anh1b1tor
OR
Metform1n + p109lttazone + SGLT-2 inh1b1tor
lnsuun
Triple therapy ·nor effecwe, nor ro'eraced or conua•'ld caced·
AND
Br.tl > 35

i
Metform1n + sulfonylurea -+- GLP-1 muneuc 0 Passmed,cme corn
Drug treatment

The 2015 NICE guidelines introduced some changes into the


management of type 2 diabetes. There are essentially two pathways,
one for patients who can tolerate metformin, and one for those who
can't

Metroonln nO( tolerated or Cl


,-..uormln

Tolerates metformin:
• metformin is still first-line and should be offered if the HbA 1c
rises to 48 mmol/mol (6.5%)* on lifestyle interventions
• if the HbA1c has risen to 58 mmol/mol (7.5%) then a second drug
should be added from the following list:
• - sulfonylurea
• - gliptin
• - pioglitazone
• - SGLT-2 inhibitor
• if despite this the HbA1c rises to, or remains above 58 mmol/mol
(7.5%) then triple therapy with one of the following combinations
should be offered:
• - metformin + gliptin + sulfonylurea
• - metformin + pioglitazone + sulfonylurea
• - metformin + sulfonylurea + SGLT-2 inhibitor
• if despite this the HbA 1c rises to, or remains above 58 mmol/mol
(7.5%) then triple therapy with one of the following combinations
should be offered:
• -. metformin + gliptin + sulfonylurea
• -. metf ormin + pioglitazone + sulfonylurea
• -. metformin + sulfonylurea + SGLT-2 inhibitor
• -. metformin + pioglitazone + SGLT-2 inhibitor
• -. OR insulin therapy should be considered

Criteria for glucagon-like peptide1 (GLP1) mimetic ( e.g. exenatide)


• if triple therapy is not effective, not tolerated or contraindicated
then NICE advise that we consider combination therapy with
metformin, a sulfonylurea and a glucagonlike peptide1 (GLP1)
mimetic if:
• _. BMI >= 35 kg/ m 2 and specific psychological or other medical
problems associated with obesity or
• _. BMI < 35 kg/ m 2 and for whom insulin therapy would have
significant occupational implications or

weight loss would benefit other significant obesityrelated comorbidities


• only continue if there is a reduction of at least 11 rnmol/mol
[1.0%] in HbA1c and a weight loss of at least 3% of initial body
weight in 6 months

Practical examples
• you review an established type 2 diabetic on maximum dose
metformin. Her HbA1 c is 55 mmol/mol (7.2%). You do not add
another drug as she has not reached the threshold of 58
mmol/ mol (7.5%)
• a type 2 diabetic is found to have a HbA 1c of 62 mmol/ mol (7.8%)
at annual review. They are currently on maximum dose metformin.
You elect to add a sulfonylurea
Cannot tolerate metformin or contraindicated
• if the HbA1c rises to 48 mmol/ mol (6.5%)* on lifestyle
interventions, consid er one of the followi ng:
• -- sulfonylurea
• -- gliptin
• _. pioglitazone
• if the HbA1c has risen to 58 mmol/ mol (7.5%) then a one of the
followi ng combin ations should be used:
• -- gliptin + pioglitazone
• -- gliptin + sulfonylurea
• -- pioglitazone + sulfonylurea
• if despite this the HbA 1c rises to, or remains above 58 mmol/ mol
(7.5%) then consid er insulin therapy

Starting insulin
• metfor min should be continued. In terms of other drugs NICE
advice: 'Review the continu ed need for other blood glucos e lowerin g
therapies '
• NICE recommend starting with human NPH insulin (isopha ne,
intermediate acting) taken at bed-tim e or twice daily accord ing to
need

Risk factor modification

Blood pressure
• target is< 140/ 80 mmHg (or < 130/ 80 mmHg if end-organ
damage is present)
• ACE inhibitors are first-line

Anti platelets
• should not be offered unless a patient has existing cardiov ascula r
disease

CamScanner ~ '..J~ ~ j,,-,&a:'I


Lipids
• following the 2014 NICE lipid modification guidelines only patients
with a 10-year cardiovascular risk > 10% (using QRISK2) should be
offered a statin. The first-line statin of choice is atorvastatin 20mg
on

Primary pqvenUon Secondary prevention

10.yur catdiov'aKia risk • 10,. OR IIOWnlldlNmtc!w-,dilffMOR


fflClll t)1le 1 --..CS OR
CKD I eOFII IOdB•ilhl)
...,_.,.,....._
~ia,dleaNOR

Atorvaslatin 20mg od
Ator,rastatin IOmg od
d ron-HDL l'..at noc ,_,, by .,,. ~
CCNidlf n-.ig '4> IOIOmg

Graphic showing choice of statin.

*this is a bit confusing because isn't the diagnostic criteria for T2DM
HbA1c 48 mmol/mol (6.5%)? So shouldn't all patients be offered
metformin at diagnosis? Our interpretation of this is that some patients
upon diagnosis will elect to try lifestyle measures, which may reduce
their HbA1c below this level. If it then rises to the diagnostic threshold
again metformin should be offered

Next question >

Save my notes
Q33

A 34-year-old female presents with fever and lower abdominal pain.


Over the past five days she has noticed deep dyspareunia and some
post-coital bleeding. Her last period began 1 O days ago. She is diffusely
tender in the suprapubic area and vaginal examination reveals cervical
excitation. Endocervical swabs are taken. A diagnosis of pelvic
inflammatory disease Is suspected. What is the most appropriate
management?

Oral doxycycline

Await endocervical swab results

Oral amoxicillin + ciprofloxacin

Oral doxycycline + ciprofloxacin

Oral ofloxacin + metronidazole

Submit answer

Reference ranges v
I Oral ofloxac in + metronidazole

Treatment of pelvic inflammatory disease: oral ofloxacin + oral


metronidazole or intramuscular ceftriaxone + oral doxycycline +
oral metronidazole
Importance: 50

Consensus guidelines recommend treatment once a diagnosis of pelvic


inflammatory disease is suspected, rather than waiting for the results of
swabs

at 22 ,, 30

at f
1 tt Discuss (2) Improve

Next question >

Pelvic inflammatory disease

Pelvic inflammatory disease (PID) is a term used to describe infection


and inflammation of the female pelvic organs including the uterus,
fallopian tubes, ovaries and the surrounding peritoneum. It is usually the
result of ascending infection from the endocervix.

Causative organisms
• Chlamydia trachomatis

+ the most common cause


• Neisseria gonorrhoeae
• Mycoplasma genitalium
• Mycop/asma hominis

CamScanner -! W~ :.i.;,-~\
Peaiures
• lower abdominal pain
• fever
• deep dyspareunia
• dysuria and menstru al irregularities may occur
• vaginal or cervical discharg e
• cervical excitation

Investigation
• a pregnancy test should be done to exclude an ectopic pregnan cy
• high vaginal swab
o these are often negative
• screen for Chlamyd ia and Gonorrhoea

Manage ment
• due to the difficulty in making an accurate diagnosis, and the
potential complica tions of untreate d PIO, consens us guidelin es
recomm end having a low threshol d for treatme nt
• oral ofloxacin + oral metronid azole or intramus cular ceftriaxo ne +
oral doxycycline + oral metronid azole
• RCOG guidelines suggest that in m ild cases of PIO intrauter ine
contraceptive devices may be left in. The more recent BASHH
guidelines suggest that the evidence is limited but that 'Remova l
of the IUD should be considered and may be associat ed with better
short term clinical outcome s'

Complic ations
• perihepatitis (Fitz-Hugh Curtis Syndrome)
0 occurs in around 10% of cases

0 it is characte rised by right upper quadran t pain and may be

confuse d with cholecys titis


• infertility - the risk may be as high as 10-20% after a single
episode
• chronic pelvic pain
• ectopic pregnancy
Q34

A 25-year-old male develops type 2 diabetes mellitus. Which one of the


following genes is most likely to be responsible?

Glucokinase

HNF-1 alpha

HNF-4 alpha

HNF-1 beta

IPF-1

Submit answer

Reference ranges v
Q34 ,c

A 25-year-old male develops type 2 diabetes mellitus. Which one of the


following genes is most likely to be responsible?

Glucokinase

HNF-1 alpha

HNF-4 alpha

HNF-1 beta

IPF-1

116 11
f • Discuss (2) Improve

Next question )
MODY
rised by the
Matu rity-o nset diabetes of the youn g (MODY) is char acte
years old. It is
deve lopm ent of type 2 diab etes mell itus in patie nts < 25
. Over six
typic ally inhe rited as an auto som al dom inan t cond ition
as lead ing to
diffe rent gene tic muta tions have so far been iden tified
MODY.

mell itus have


It is thou ght that around 1-2% of patie nts with diab etes
r type 1 or type
MODY, and around 90% are misc lass ified as having eithe
2 diab etes mell itus.

MOD Y3
• 60% of case s
• due to a defe ct in the HNF-1 alpha gene
• is asso ciate d with an increased risk of HCC

MODY2
• 20% of case s
• due to a defe ct in the gluc okin ase gene

MODY 5
• rare
• due to a defe ct in the HNF-1 beta gene
• liver and rena l cyst s

Features of MODY
• typic ally develops in patie nts < 25 years
• a fami ly histo ry of early onse t diab etes is ofte n pres ent
• keto sis is not a featu re at pres enta tion
to
• patie nts with the mos t com mon form are very sens itive
sulfo nylu reas, insu lin is not usua lly nece ssar y

Nex t ques tion )


CamSca nne r ➔ •..;~ L - ~1
Q35

A 33-year-old female is referred to endocrinology with thyrotoxicosis.


Following a discussion of management options she elects to have
radioiodine therapy. Which one of the following is the most likely
adverse effect?

Hypothyroidism

Thyroid malignancy

Agranulocytosis

Oesophagitis

Precipitation of thyroid eye disease

Submit answer

Reference ranges v
Q35

A 33-year-old female is referred to endocrinology with thyrotoxicosis.


Following a discussion of management options she elects to have
radioiodine therapy. Which one of the following is the most likely
adverse effect?

I Hypothyroidism

Thyroid malignancy

Agranulocytosis

Oesophagitis

Precipitation of thyroid eye disease

It is well documented that radioiodine therapy can precipitate thyroid


eye disease but a majority of patients will eventually require thyroxine
replacement

"' 'f t9 Discuss (3) Improve ]

Next question )
Graves• disease: management

Despite many trials there is no clear guidance on the optimal


management of Graves' disease. Treatment options include titration of
anti-thyroid drugs (ATDs, for example carbimazole), block-and-replace
regimes, radioiodine treatment and surgery. Propranolol is often given
initially to block adrenergic effects

ATD titration
• carbimazole is started at 40mg and reduced gradually to maintain
euthyroidism
• typically continued for 12-18 months
• patients following an ATD titration regime have been shown to
suffer fewer side-effects than those on a block-and-replace
regime

Block-and-replace
• carbimazole is started at 40mg
• thyroxine is added when the patient is euthyroid
• treatment typically lasts for 6-9 months

The major complication of carbimazole therapy is agranulocytosis

Radioiodine treatment
• contraindications include pregnancy (should be avoided for 4-6
months following treatment) and age< 16 years. Thyroid eye
disease is a relative contraindication, as it may worsen the
condition
• the proportion of patients who become hypothyroid depends on
the dose given, but as a rule the majority of patient will require
thyroxine supplementation after 5 years

Next question )
Q36

A 54-year-old man with type 2 diabetes mellitus is started on exenatide.


Which one of the following statements regarding exenatide is
incorrect?

Typically results in weight loss

May be combined with a sulfonylurea

The major adverse effect is flu-like symptoms

May be combined with metformin

Must be given by subcutaneous injection

Submit answer

Reference ranges v
The major adverse effect is flu-like symptoms

May be combined with metformin

Must be given by subcutaneous injection

Exenatide causes vomiting


Importance: 50

The major adverse effect is nausea and vomiting

8' •t tt Discuss (4) Improve

Next question )

Diabetes mellitus: GLP-1 and the new drugs

A number of new drugs to treat diabetes mellitus have become


available in recent years. Much research has focused around the role of
glucagon-like peptide-1 (GLP-1 ), a hormone released by the small
intestine in response to an oral glucose load

Whilst it is well known that insulin resistance and insufficient B-cell


compensation occur other effects are also seen in type 2 diabetes
mellitus (T2DM). In normal physiology an oral glucose load results in a
greater release of insulin than if the same load is given intravenously -
this known as the incretin effect. This effect is largely mediated by GLP-
1 and is known to be decreased in T2DM.

Increasing GLP-1 levels, either by the administration of an analogue


(glucagon-like peptide-1, GLP-1 mimetics, e.g. exenatide) or inhibiting
its breakdown (dipeptidyl peptidase-4 ,DPP-4 inhibitors - the gliptins), is
therefore the target of two recent classes of drug.
Glucagon-like peptide-1 (GLP-1) mim etics (e.g. exenatide)

mim etic.
Exenatide is an example of a gluca gon- like peptide-1 (GLP-1)
tion.
These drugs increase insulin secretion and inhib it gluca gon secre
ally
One of the majo r advances of GLP-1 mim etics is that they typic
insulin,
result in weig ht loss, in cont rast to man y med icatio ns such as
in
sulfonylureas and thiaz olidin edion es. They are some time s used
comb inatio n with insulin in T2DM to minim ise weig ht gain.

minu tes
Exenatide mus t be given by subc utan eous injec tion withi n 60
after a
before the morn ing and evening mea ls. It shou ld not be given
meal.

the main
Liraglutide is the othe r GLP-1 mim etic curre ntly available. One
be given
advantages of liraglutide over exenatide is that it only needs to
once a day.

n and a
Both exenatide and lirag lutide may be comb ined with metf ormi
used
sulfonylurea. Standard release exenatide is also licensed to be
for a
with basal insulin alone or with metf ormi n. Please see the BNF
more comp lete list of licensed indication s.

NICE state the follow ing:

Consider adding exenatide to metf ormi n and a su/fony/urea if•


• BMI >= 35 kg/ m in people of European desc ent and there are
2

prob lems associated with high weight, or


• BMI < 35 kg/ m and insulin is unacceptable because of
2

occu patio nal impli catio ns or weig ht Joss woul d bene fit othe r
comorbidities.
NICE like patients to have achieved a 11 mmol/mol (1 %) reduction in
HbA 1c and 3% weight loss after 6 months to justify the ongoing
prescription of GLP-1 mimetics.

The major adverse effect of GLP-1 mimetics is nausea and vomiting.


The Medicines and Healthcare products Regulatory Agency has issued
specific warnings on the use of exenatide, reporting that is has been
linked to severe pancreatitis in some patients.

Dipeptidyl peptidase-4 (DPP-4) inhibitors (e.g. Vildagliptin, sitagliptin)

Key points
• oral preparation
• trials to date show that the drugs are relatively well tolerated with
no increased incidence of hypoglycaemia
• do not cause weight gain

NICE guidelines on DPP-4 inhibitors


• NICE suggest that a DPP-4 inhibitor might be preferable to a
thiazolidinedione if further weight gain would cause significant
problems, a thiazolidinedione is contraindicated or the person has
had a poor response to a thiazolidinedione

Next question )

Save my notes

Search
Q37

A 36-year-old female with a BMI of 34 kg/m"2 is reviewed after


managing to lose 3 kg in the past month. She asks about the possibility
of starting a drug to help her lose weight. What is the primary mode of
action of orlistat?

Leptin antagonist

Pancreatic lipase inhibitor

Prevents intestinal absorption of low-density lipoproteins

HMG-CoA reductase inhibitor

Centrally-acting appetite suppressant

Submit answer

Reference ranges v
Pancreatic lipase inhibitor

Prevents intestinal absorption of low-density lipoproteins

HMG-CoA reductase inhibitor

Centrally-acting appetite suppressant

Orlistat works by jnhibiting gastric and pancreatic lipase to reduce


the digestion of fat
Importance: 50

The primary mode of action of orlistat is to inhibit pancreatic lipases,


which in turn will decrease the absorption of lipids from the intestine

[ ,. •t - Discuss (1) Improve ]

I
Next question )
Next question )

Obesity: therapeutic options

The management of obesity consists of a step-wise approach:


• conservative: diet, exercise
• medical
• surgical

Orlistat is a pancreatic lipase inhibitor used in the management of


obesity. Adverse effects include faecal urgency/ incontinence and
flatulence. A lower dose version is now available without prescription
('Alli'). NICE have defined criteria for the use of orlistat. It should only be
prescribed as part of an overall plan for managing obesity in adults who
have:
• BMI of 28 kg/ m"2 or more with associated risk factors, or
• BMI of 30 kg/ m"2 or more
• continued weight loss e.g. 5% at 3 months
• orlistat is normally used for< 1 year

Next question )

B / l!j A ,., T l ""

Save my notes
Q38

A 72-year-old man is reviewed in the diabetes clinic. He has a history of


heart failure and type 2 diabetes mellitus. His current medications
include furosemide 40mg od, ramipri l 10mg ad and bisoprolol 5mg ad.
Clinical examination is unremarkable with no evidence of peripheral
oedema, a clear chest and blood pressure of 130/ 76 mm Hg. Recent
renal and liver function tests are normal. Which one of the following
medications is contraindicated?

Sitagliptin

Pioglitazone

Gliclazide

Exenatide

Metformin

Submit answer

Reference ranges v
Pioglitazone

Gliclazide

Exenatide

Metformin

Pioglitazone - contraindicated by: heart failure


Importance: 50

Thiazolidinediones are absolutely contraindicated in heart failure

9' 11
f tt Discuss (2) Improve

Next question >


0 Q39

A 45-year-old woman is investigated for weight gain. She had had been
unwell for around four months and described a combination of
symptoms including depression, facial male-pattern hair growth and
reduced libido. During the work-up she was found to be hypertensive
with a blood pressure of 170/ 100 mmHg. Which one of the following
tests is most likely to be diagnostic?

Renin:aldosterone levels

High-dose dexamethasone suppression test

Pelvic ultrasound

Overnight dexamethasone suppression test

24 hr urinary free cortisol

Submit answer

Reference ranges v
Overnight dexamethasone suppression test

24 hr urinary free cortisol

The overnight dexamethasone suppression test is the best test to


diagnosis Cushing's syndrome
Importance: 50

This patient has Cushing's syndrome as evidenced by the weight gain,


hirsutism, depression and hypertension. Polycystic ovarian syndrome
may give some of these features but would not cause such an elevated
blood pressure.

[ ,t 'f - Discuss (1 D) Improve

'

Next question >

Cushing's syndrome: investigations

Investigations are divided into confirming Cushing's syndrome and then


localising the lesion. A hypokalaemic metabolic alkalosis may be seen,
along with impaired glucose tolerance. Ectopic ACTH secretion (e.g.
secondary to small cell lung cancer) is characteristically associated
with very low potassium levels. An insulin stress test is used to
differentiate between true Cushing's and pseudo-Cushing's

Tests to confirm Cushing's syndrome

The two most commonly used tests are:


• overnight dexamethasone suppression test (most sensitive)
• 24 hr urinary free cortisol

Localisation tests
Localisation tests

The first-line localisation is 9am and midn ight plasma ACTH (and
t
corti sol) levels. If ACTH is suppressed then a non-ACTH dependen
cause is likely such as an adrenal adenoma

be
Both low- and high-dose dexamethasone suppression tests may
. These
used to localise the path olog y resulting in Cushing's syndrome
tests may be interpreted as follow s:

Cortisol result Interpretation

Not suppressed by low-dose Cushing's syndrome not due to


dexamethasone prim ary causes, i.e. likely seco ndar y
to corti coste roid therapy

Not suppressed by low- Cushing's disease


dose, but suppressed by
high-dose dexamethasone

Not suppressed by low- or Ectopic ACTH syndrome likely


high-dose dexamethasone

CRH stimu lation


• if pituitary source then corti sol rises
• if ectopic/adrenal then no change in corti sol

Petrosal sinus sampling of ACTH may be needed to diffe renti ate


between pituit ary and ecto pic ACTH secretion

Next ques tion )


Q40

Which one of the following may be associated wit h galactorrhoea?

Primary hypothyroidism

Addison's disease

Cushing's syndrome

Grave's disease

Bromocriptine

Submit answer

Reference ranges v
Q40

Which one of the following may be associated with galactorrhoea?

Primary hypothyroidism

Addison's disease

Cushing's syndrome

Grave's disease

Bromocriptine

[ ,t •• tt Discuss (3) Improve

Next question)
Prolactin and galactorrhoea

Prolactin is secreted by the anterior pituitary gland with release being


controlled by a wide variety of physiological factors. Dopamine acts as
the primary prolactin releasing inhibitory factor and hence dopamine
agonists such as bromocriptine may be used to control galactorrhoea.
It is important to differentiate the causes of galactorrhoea (due to the
actions of prolactin on breast tissue) from those of gynaecomastia

Features of excess prolactin


• men: impotence, loss of libido, galactorrhoea
• women : amenorrhoea, galactorrhoea

Causes of raised prolactin


• prolactinoma
• pregnancy
• oestrogens
• physiological: stress, exercise, sleep
• acromegaly: 1/ 3 of patients
• polycystic ovarian syndrome
• primary hypothyroidism (due to thyrotrophin releasing hormone
(TRH) stimulating prolactin release)

Drug causes of raised prolactin


• metoclopramide, domperidone
• phenothiazines
• haloperidol
• very rare: SSRls, opioids

Next question >


Q41

Each one of the following is a feature of pseudohypoparathyroidism,


except:

Short fourth and fifth metacarpals

Round face

Normal calcium and phosphate levels

Cognitive impairment

Short stature

Submit answer

Reference ranges v
Round face

Normal calcium and phosphate levels

Cognitive impairment

Short stature

[ •• •• ,_ Discuss Improve ]

Next question )
Pseudohypoparathyroidism

Pseudohypoparathyroidism is caused by target cell insensitivity to


parathyroid hormone (PTH) due to a mutation in a G-protein. In type I
pseudohypoparathyroidism
there is a complete receptor defect whereas in type II the cell receptor is
intact. Pseudohypoparathyroidism is typically inherited in an autosomal
dominant fash ion*

Bloods
• PTH: high
• calcium: low
• phosphate: high

Features
• short fourth and fifth metacarpals
• short stature
• cognitive impairment
• obesity
• round face

Investigation
• infusion of PTH followed by measurement of urinary phosphate
and cAMP measurement - this can help differentiate between type
I (neither phosphate or cAMP levels rise) and II (cAMP rises but
phosphate levels do not change)

*it was previously thought to be an X-linked dominant condition

Next question )
01

Which one of the following is the most common non-iatrogenic cause


of Cushing's syndrome?

Ectopic ACTH production

Adrenal adenoma

Micronodular adrenal dysplasia

Adrenal carcinoma

Pituitary tumour

Submit answer

Reference ranges v
I ~ou_r____
Pituit ary tu_m

Cushing's disease is the most comm on, non-iatrogenic, cause of


Cushing's syndrome

[ ,t ,. t9 Discuss (5) Impro ve ]

Next ques tion )

Cushing's syndrome: causes

It should be noted that exogenous cause s of Cushing's syndr ome ( e.g.


gluco cortic oid therapy) are far more comm on than endo geno us ones.

ACTH dependent causes


• Cushing's disease (80%) : pituit ary tumo ur secre ting ACTH
produ cing adrenal hyperplasia
• ectop ic ACTH produ ction (5-10%): e.g. small cell lung canc er

ACTH indep ende ntcau ses


• iatrogenic: stero ids
• adrenal adenoma (5-10%)
• adrenal carcin oma (rare)
• Carney comp lex: syndr ome includ ing cardi ac myxo ma
• micro nodu lar adrenal dyspl asia (very rare)

Pseudo-Cushing 's
• mimi cs Cushing's
• often due to alcoh ol excess or severe depression
• causes false positi ve dexam ethas one suppr essio n test or 24 hr
urinary free cortis ol
• insulin stress test may be used to differ entia te

Next uestio n )
Q2

A 42-year-old woman presents to the GP with a 6-week history of


constipation, lethargy and cold intolerance. She tells you that these
symptoms are much the same as when was diagnosed with
hypothyroidism five years ago. Since then, she has been established on
a dose of levothyroxine 75 micrograms once daily.

She reports starting a new medication 8 weeks ago, but is unable to


recall what it was called .

Of the following, which is most likely to be the medication that the


patient has recently started?

Cholecalciferol

Ferrous sulphate

Leveti ra ceta m

Metronidazole

Sertraline

Submit answer

Reference ranges v
I Ferrous sulphate

Levetiracetam

Metronidazole

Sertraline

Iron / calcium carbonate tablets can reduce the absorption of


levothyroxine - should be given 4 hours apart
Importance: 50

Iron (ferrous sulphate and ferrous fumarate) and calcium carbonate


tablets can reduce the absorption of levothyroxine.

This interaction is be a common reason why patients do not respond to


increasing doses of levothyroxine (especially in the exam setting!).

The BNF recommends that these medications should be given at least


4 hours apart.

The remaining medications are not known to interact with


levothyroxine.

[ •• •• • Discuss Improve ]

Next question )
Hypothyroidism: management

Key points
• initial starting dose of levothyroxine should be lower in elderly
patients and those with ischaemic heart disease. The BNF
recommends that for patients with cardiac disease, severe
hypothyroidism or patients over 50 years the initial starting dose
should be 25mcg od with dose slowly titrated. Other patients
should be started on a dose of 50-1 00mcg od
• following a change in thyroxine dose thyroid function tests should
be checked after 8-1 2 weeks
• the therapeutic goal is 'normalisation' of the thyroid stimulating
hormone (TSH) level. As the majority of unaffected people have a
TSH value 0.5-2.5 mU/I it is now thought preferable to aim for a
TSH in this range
• women with established hypothyroidism who become pregnant
should have their dose increased 'by at least 25-50 micrograms
levothyroxine'* due to the increased demands of pregnancy. The
TSH should be monitored carefully, aiming for a low-normal value
• there is no evidence to support combination therapy with
levothyroxine and liothyronine

Side-effects of thyroxine therapy


• hyperthyroidism: due to over treatment
• reduced bone mineral density
• worsening of angina
• atrial fibrillation

Interactions
• iron, calcium carbonate
o absorption of levothyroxine reduced, give at least 4 hours
apart

*source: NICE Clinical Knowledge Summaries


Q3

What chromosome abnormality is associated with Klinefelter's


syndrome?

47, XO

47, XXV

46, XXV

47, XYY

47, xxo

Submit answer

Reference ranges v
47, XXV

46, XXV

47, XYY

47, xxo

Klinefelter's - 47, XXY


Importance: 50

,t 'f - Discuss Improve ]

Next question >

Klinefelter's syndrome

Klinefelter's syndrome is associated with karyotype 47, XXY.

Features
• often taller than average
• lack of secondary sexual characteristics
• small, firm testes
• infertile
• gynaecomastia - increased incidence of breast cancer
• elevated gonadotrophin levels but low testosterone

Diagnosis is by karyotype (chromosomal analysis).

Next question )
Q4

A 65-year-old man with a background of chronic obstructive pulmonary


disease, rheumatoid arthritis and polymyalgia rheumatica presents with
difficulty in climbing the stairs, worsening bruising on his arms and
weight gain. These symptoms have been progressively getting worse
and he is concerned whether it could be due to the fact that he has
been taking long term steroids for his underlying conditions. He is
aware of the sick-day rules which he reports adhering to and denies
ever stopping his treatment.

If a venous blood gas was to be performed on this patient you would


most likely find which of the following?

Hyperkalaemic metabolic acidosis

Hyperkalaemic metabolic alkalosis

Hypokalaemic metabolic acidosis

Hypokalaemic metabolic alkalosis

Mixed metabolic acidosis

Submit answer
Hyperkalaemic metabolic alkalosis

Hypokalaemic metabolic acidosis

Hypokalaemic metabolic alkalosis

Mixed metabolic acidosis

Cushing's syndrome - hypokalaemic metabolic alkalosis


Importance: 50

This patient has symptoms suggestive of Cushing's syndrome caused


by prolonged exposure to exogenous corticosteroids such as
Prednisolone for his medical conditions.

Cushing 's syndrome - hypokalaemic metabolic alkalosis. The metabolic


alkalosis is due to excess aldosterone which increases acid and
potassium excretion in the kidney.

Addison's disease - hyperkalaemic metabolic acidosis. The metabolic


acidosis is due to insufficiency of aldosterone which decreases acid
secretion in the kidney and leads to the retention of potassium.

,. ,. • Discuss Improve ]

Next question )
I

CamSc.an,ner ~ '....,.i;lo-" ~ ~ I
Q5

A 54-year-old man is reviewed shortly after being diagnosed with


hypertension. as part of his work-up he had a series of blood tests to
screen for other risk factors:

Na+ 142 mmol/ I

K+ 3.9 mmol/ 1

Urea 6.2 mmol/ 1

Creati nine 91 µmol/1

Fasting glucose 7.7 mmol/1

Total cholesterol 7.2 mmol/1

Based on the fasting glucose result you arrange a HbA1 c:

HbA1 c 31 mmol/ mol (5.0%)

Which one of the following would explain the discrepancy between the
HbA1c and fasting glucose levels?

Vitamin 812 deficiency

Conn's syndrome

Raised cholesterol level

Sickle-cell anaemia

A history of alcohol excess

Submit answer
R81S€0 Cl IOIESLEIOI IEVEI

Sickle-cell anaemia

A history of alcohol excess

"' •t tt Discuss (3) Improve

Next question )

Glycosylated haemoglobin

Glycosylated haemoglobin (HbA1c) is the most widely used measure of


long-term glycaemic control in diabetes mellitus. HbA 1c is produced by
the glycosylation of haemoglobin at a rate proportional to the glucose
concentration. The level of HbA1c therefore is dependant on
• red blood cell lifespan
• average blood glucose concentration

A number of conditions can interfere with accurate HbA1c


interpretation:

Lower-than-expected levels of Higher-than-expected levels of


HbA1c (due to reduced red HbA1c (due to increased red
blood cell lifespan) blood cell lifespan)

Sickle-cell anaemia Vitamin B12/ folic acid


GP6D deficiency deficiency
Hereditary spherocyt osis Iron-deficiency anaemia
Splenectomy

,, • • t
HbA 1c is generally thought to reflect the blood glucose over the
previous '3 months' although there is some evidence it is weighed more
strongly to glucose levels of the past 2-4 weeks. NICE recommend
'HbA 1c should be checked every 3-6 months until stable, then 6 monthly'.

The relationship between HbA1c and average blood glucose is complex


but has been studied by the Diabetes Control and Complications Trial
(DCCT). A new internationally standardised method for reporting HbA 1c
has been developed by the International Federation of Clinical
Chemistry (IFCC). This will report HbA1c in mmol per mol of
haemoglobin without glucose attached.

HBA1c Average plasma glucose


(%) (mmol/1) IFCC-HbA1 c (mmol/mol)

5 5.5

6 7 .5 42

7 9 .5 53

8 11 .5 64

9 13.5 75

10 15.5

11 17.5

12 19.5

From the above we can see that average plasma glucose = (2 * HbA1c)
-4.5
Q6

A 50 year-old woman presents with polyuria and polydipsia. She has


recently been started on cita lopram for depression, but is otherwise fit
and well. She has complained of constipation recently, but has put this
down to her new medication.

Calcium 2.8mmol/l

Phosphate 0.7mmol/l

Parathyroid hormone 5.0pmol/I (1.2-5.Bpmol/ l)

Renal function and ful l blood count are normal.

What is the most likely cause for these blood results?

Parathyroid adenoma

Myeloma

Metastatic cancer

Drug induced

Parathyroid hyperplasia

Submit answer
Parathyroid adenoma

Myeloma

Metastati c cancer

Drug induced

Parathyroid hyperplasia

The PTH level in primary hyperparathyroidism may be normal


Importance: 50

This woman has signs and symptoms of hypercalcaemia. Her


parathyroid hormone should be suppressed in the presence of
hypercalcaemia. Given that it is normal (inappropriately), this indicates
the parathyroid as the cause for the hypercalcaemia. The most
common cause of hyperparathyroidism is an adenoma.

11• 11f ta Discuss (7) Improve

Next question )

Primary hyperparathyroidism

In exams, primary hyperparathyroidism is stereotypically seen in elderly


females with an unquenchable thirst and an inappropriately normal or
raised parathyroid hormone level. It is most commonly due to a solitary
adenoma

Causes of primary hyperparathyroidism


• 80%: solitary adenoma
• 15%: hyperplasia
• 4%: multiple adenoma
• 1%: carcinoma
Features - 'bones, stones, abdominal groans and psychic moans'
• polydipsia, polyuria
• peptic ulceration/constipation/pancreatitis
• bone pain/ fracture
• renal stones
• depression
• hypertension

Associations
• hypertension
• multiple endocrine neoplasia: MEN I and II

Investigations
• raised calcium, low phosphate
• PTH may be raised or (inappropriately, given the raised calcium)
normal
• technetium-MIBI subtraction scan
• pepperpot skull is a characteristic X-ray finding of
hyperparathyroidism

Treatment
• the definitive management is total parathyroidectomy
• conservative management may be offered if the calcium level is
less than 0.25 mmol/L above the upper limit of normal AND the
patient is > 50 years AND there is no evidence of end-organ
damage
• calcimimetic agents such as cinacalcet are sometimes used in
patients who are unsuitable for surgery
Bilateral hand radiographs in a middle-aged woman
demonstrating generalised osteopenia, erosion of the terminal
phalangeal tufts (acro-osteolysis) and subperiosteal resorption
of bone particularly the radial aspects of the 2nd and 3rd middle
phalanges. These changes are consistent with a diagnosis of
hyperparathyroidism.
Q7

A 33-year-old woman presents with weight loss and excessive


sweating. her partner reports that she is 'on edge' all the time and
during the consultation you notice a fine tremor. Her pulse rate is
96/min. A large, non-tender goitre is noted. Examination of her eyes is
unremarkable with no evidence of exophthalmos.

Free T 4 26 pmoVI

Free T3 12.2 pmol/1 (3.0-7.5)

TSH < 0.05 mu/ I

What is the most likely diagnosis?

Toxic multinodular goitre

Hashimoto's thyroiditis

T3-secreting adenoma

De Quervain's thyroiditis

Graves' disease

Submit answer
Graves a1sease

Graves' disease is the most common cause of thyrotoxicosis


Importance: 50

Only around 30% of patients with Graves' disease have eye disease so
the absence of eye signs does not exclude the diagnosis.

16 11
f • Discuss (5) Improve

Next question )

Graves' disease: features

Graves' disease is the most common cause of thyrotoxicosis. It is


typically seen in women aged 30-50 years.

Features
• typical features of thyrotoxicosis
• specific signs limited to Grave's (see below)

Features seen in Graves' but not in other causes of thyrotoxicosis


• eye signs {30% of patients)
o exophthalmos
o ophthalmoplegia
• pretibial myxoedema
• thyroid acropachy, a triad of:
o digital clubbing
o soft tissue swelling of the hands and feet
o periosteal new bone formation

Autoantibodies
• TSH receptor stimulating antibodies {90%)
• anti-thyroid peroxidase antibodies (75%)
Q8

A 25-year-old female has type I diabetes. Her HbA 1c is 58 mmol/L. Her


blood pressure is 126/68 mmHg. Her BMI is 28 kg/m 2 • She is using a
basal-bolus regimen which she finds easy to manage. She is not keen to
increase her total insulin dose. Which of the following adjuncts could
you consider to help improve her glycaemic control?

Add metformin

Switch to a mixed insulin regime

Add sitagliptin

Add exenatide

Enrol in supported weight loss programme

Submit answer

Reference ranges v
A 25-year-old female has type I diabetes. Her HbA 1c is 58 mmol/ L. Her
blood pressure is 126/ 68 mmHg. Her BMI is 28 kg/m 2 • She is using a
basal-bolus regimen which she finds easy to manage. She is not keen to
increase her total insulin dose. Which of the following adjuncts cou ld
you consider to help improve her glycaemic control?

I Add metformin
,---------

Switch to a mixed insulin regime

Add sitagliptin

Add exenatide

Enrol in supported weight loss programme

Patients with type I diabetes and a BMI > 25 should be considered


for metformin in addition to insulin
Importance: 50

The correct answer is to add metformin. NICE recommends that adding


metformin should be considered in type I diabetics with a BMI > 25,
either on patient preference or to avoid the need to increase t heir insulin
therapy. The other oral diabetic medications are not currently
recommended. Weight loss is likely to be beneficial but there is greater
evidence for benefit with the use of metformin. A mixed insulin regime
might be used if a multiple injection basal-bolus regime was not suited
to the patient's lifestyle but is not usually chosen for better glycaemic
control.

NICE: Tvoe I diabetes in adults


Diabetes mellitus: management of type 1

The long-term management of type 1 diabetics is an important and


complex process requiring the input of many different clinical
specialties and members of the healthcare team. A diagnosis of type 1
diabetes can still reduce the life expectancy of patients by 13 years and
the micro and macrovascular complications are well documented.

NICE released guidelines on the diagnosis and management of type 1


diabetes in 201 5. We've only highlighted a very select amount of the
guidance here which will be useful for any clinician looking after a
patient with type 1 diabetes.

HbA1c
• should be monitored every 3-6 months
• adults should have a target of HbA1c level of 48 mmol/mol (6.5%)
or lower. NICE do however recommend taking into account factors
such as the person's daily activities, aspirations, likelihood of
complications, comorbidities, occupation and history of
hypoglycaemia

Self-monitoring of blood glucose


• recommend testing at least 4 times a day, including before each
meal and before bed
• more frequent monitoring is recommended if frequency of
hypoglycaemic episodes increases; during periods of illness;
before, during and after sport; when planning pregnancy, during
pregnancy and while breastfeeding

Blood glucose targets


• 5-7 mmol/I on waking and
• 4-7 mmol/I before meals at other times of the day
Type of insulin
• offer multiple daily injection basal-bolus insulin regimens, rather
than twice-daily mixed insulin regimens, as the insulin injection
regimen of choice for all adults
• twice-daily insulin detemir is the regime of choice. Once-daily
insulin glargine or insulin detemir is an alternative
• offer rapid-acting insulin analogues injected before meals, rather
than rapid-acting soluble human or animal insulins, for mealtime
insulin replacement for adults with type 1 diabetes

Metformin
• NICE recommend considering adding metformin if the BMI >= 25
kg/ m 2

Next question )

-------------------------------
B J !!!J A . . . ·-
· - i::
~ 1- --

Save my notes

Search
Q1

A 27-year-old female develops eye pain and reduced visual acuity


following the initiation of treatment for her recently diagnosed Grave's
disease. Which one of the following treatments is likely to have been
started?

Radioiodine treatment

Thyroidectomy

Propylthiouracil

Carbimazole and thyroxine

Carbimazole

Submit answer

Reference ranges v
Thyroidectomy

Propylthiouracil

Carbimazole and thyroxine

Carbimazole

Radioiodine treatment may lead to the development/ worsening of


thyroid eye disease in up to 15% of patients with Grave's disease

9' •• tt Discuss (2) Improve

Next question )

Thyroid eye disease

Thyroid eye disease affects between 25-50% of patients with Graves'


disease.

Pathophysiology
• it is thought to be caused by an autoimmune response against an
autoantigen, possibly the TSH receptor - retro-orbital
inflammation
• the inflammation results in glycosaminoglycan and collagen
deposition in the muscles

Prevention
• smoking is the most important modifiable risk factor for the
development of thyroid eye disease
• radioiodine treatment may increase the inflammatory symptoms
seen in thyroid eye disease. In a recent study of patients with
Graves' disease around 15% developed, or had worsening of, eye
disease. Prednisolone mav help reduce the risk
Features
• the patient may be eu-, hypo- or hyperthyroid at the time of
presentation
• exophthalmos
• conjunctiva! oedema
• optic disc swelling
• ophthalmoplegia
• inability to close the eyelids may lead to sore, dry eyes. If severe
and untreated patients can be at risk of exposure keratopathy

Management
• topical lubricants may be needed to help prevent corneal
inflammation caused by exposure
• steroids
• radiotherapy
• surgery

Monitoring patients with established thyroid eye disease

For patients with established thyroid eye disease the following


symptoms/signs should indicate the need for urgent review by an
ophthalmologist (see EUGOGO guidelines):
• unexplained deterioration in vision
• awareness of change in intensity or quality of colour vision in one
or both eyes
• history of eye suddenly 'popping out' (globe subluxation)
• obvious corneal opacity
• cornea still visible when the eyelids are closed
• disc swelling

Next question >


Q1

Which one of the following drugs used in the management of type 2


diabetes mellitus has the Medicines and Healthcare products
Regulatory Agency warned is associated with an increased risk of
severe pancreatitis and renal impairment?

Rosiglitazone

Metformin

Acarbose

Exenatide

Sitagliptin

Submit answer

Reference ranges v
I Exenatide -~-- ---- ---'

Sitagliptin

[ ,6 •• • Discuss (4) Improve ]

Next quest ion )

Diabetes mellitus: GLP-1 and the new drugs

A number of new drugs to treat diabetes mellit us have becom e


available in recent years. Much research has focused around the role of
glucagon-like peptide-1 {GLP-1 ), a horm one released by the small
intestine in response to an oral glucose load

Whilst it is well known that insulin resistance and insuff icient B-cell
compensation occur other effect s are also seen in type 2 diabe tes
mellitus (T2DM). In normal physiology an oral glucose load results in a
greater release of insulin than if the same load is given intravenously -
this known as the incretin effect. This effec t is largely mediated by GLP-
1 and is known to be decreased in T2DM .

Increasing GLP-1 levels, either by the administration of an analogue


(glucagon-like peptide-1, GLP-1 mimetics, e.g. exenatide) or inhibi ting
its breakdown (dipeptidyl peptidase-4 ,DPP-4 inhibitors - the gliptins), is
therefore the target of two recent classes of drug.
Glucagon-like peptide-1 (GLP-1) mimetics (e.g. exenatide)

Exenatide is an example of a glucagon-like peptide-1 (GLP-1) mimetic.


These drugs increase insulin secretion and inhibit glucagon secretion.
One of the major advances of GLP-1 mimetics is that they typically
result in weight loss, in contrast to many medications such as insulln,
sulfonylureas and thiazolidinediones. They are sometimes used in
combination with insulin in T2DM to minimise weight gain.

Exenatide must be given by subcutaneous injection within 60 minutes


before the morning and evening meals. It should not be given after a
meal.

Liraglutide is the other GLP-1 mimetic currently available. One the main
advantages of liraglutide over exenatide is that it only needs to be given
once a day.

Both exenatide and liraglutide may be combined with metformin and a


sulfonylurea. Standard release exenatide is also licensed to be used
with basal insulin alone or with metformin. Please see the BNF for a
more complete list of licensed indications.

NICE state the following:

Consider adding exenatide to metformin and a sulfonylurea if


• BMI >= 35 kg/ m 2 in people of European descent and there are
problems associated with high weight, or
• BMI < 35 kg/ m 2 and insulin is unacceptable because of
occupational implications or weight loss would benefit other
comorbidities.
I

NICE like patients to have achieved a 11 mmol/mol (1 %) reduction in


HbA1c and 3% weight loss after 6 months to justify the ongoing
prescription of GLP-1 mimetics.

The major adverse effect of GLP-1 mimetics is nausea and vomiting.


The Medicines and Healthcare products Regulatory Agency has issued
specific warnings on the use of exenatide, reporting that is has been
linked to severe pancreatitis in some patients.

Dipeptidyl peptidase-4 {DPP-4) inhibitors (e.g. Vildagliptin, sitagliptin)

Key points
• oral preparation
• trials to date show that the drugs are relatively well tolerated with
no increased incidence of hypoglycaemia
• do not cause weight gain

NICE guidelines on DPP-4 inhibitors


• NICE suggest that a DPP-4 inhibitor might be preferable to a
thiazolidinedione if further weight gain would cause significant
problems, a thiazolidinedione is contraindicated or the person has
had a poor response to a thiazolidinedione

Next question )

Save my notes
Q2

A 32-year-old woman who is 24 weeks pregnant with her third child


comes to the clinic for review. She has been diagnosed with gestational
diabetes mellitus, and a fasting plasma glucose following 2 weeks of
adherence to lifestyle changes is still elevated at 6.8 mmol/1. Her blood
pressure is 122/82 mmHg, and her body mass index is 25 kg/m 2 • She is
reluctant to start insulin initially because her sister has Type 1 diabetes
and suffers from frequent hypoglycaemia.

Which of the following is the most appropriate next intervention?

Metformin

Glibenclamide

Insulin glargine

Dapagliflozin

Insulin pump therapy

Submit answer

Reference ranges v
Glibenclamide

Insulin glargine

Dapagllflozin

Insulin pump therapy

Metformin is the first line therapy of choice for diabetes in


pregnancy
Importance: 50

Metformin has been evaluated in a large Australasian trial for the


treatment of gestational diabetes mellitus. Versus insulin initiation at
the point of diagnosis, those patients treated with metformin gained
less weight during pregnancy and suffered slightly fewer episodes of
hypoglycaemia. There was no difference in the primary endpoint of
adverse foetal outcomes, and women treated with metformin first
preferred this option, even though most eventually progressed to insulin
in addition to oral therapy. This has precipitated NICE to recommend
metformin as a first line option where fasting glucose is less than 7.0
mmol/I despite dietary modification.

https://www.nejm.org/ doi/ full/10.1056/ NEJMoa0707193


https://www.nice.org.uk/guidance/ng3

Although glibenclamide is safe in pregnancy it does not limit weight


gain and control is inferior to insulin therapy. It's therefore only an
option in patients who refuse metformin and insulin. Out of the insulin
options listed, insulin pump therapy is preferred, although many women
find it more difficult to comply with pump therapy and are therefore
treated with a basal bolus regimen. There is no evidence to support the
use of SGLT-2 inhibitors such as dapagliflozin in pregnancy. One
problem which leads to diabetes is relative insulin resistance, which
SGLT-2 inhibitors will not significantly impact upon.
Next question ~

Pregnancy: diabetes mellitus

Diabetes mellitus may be a pre-existing problem or develop during


pregnancy, gestational diabetes. It complicates up to 1 in 20
pregnancies. NICE estimate the following breakdown:
• 87.5% have gestational diabetes
• 7.5% have type 1 diabetes
• 5% have type 2 diabetes

Risk factors for gestational diabetes


• BMI of> 30 kg/m 2
• previous macrosomic baby weighing 4.5 kg or above
• previous gestational diabetes
• first-degree relative with diabetes
• family origin with a high prevalence of diabetes (South Asian,
black Caribbean and Middle Eastern)

Screening for gestational diabetes


• women who've previously had gestational diabetes: oral glucose
tolerance test (OGTT) should be performed as soon as possible
after booking and at 24-28 weeks if the first test is normal. NICE
also recommend that early self-monitoring of blood glucose is an
alternative to the OGTTs
• women with any of the other risk factors should be offered an
OGTT at 24-28 weeks

Diagnostic thresholds for gestational diabetes


• these have recently been updated by NICE, gestational diabetes is
diagnosed if either:
• fasting glucose is >= 5.6 mmol/1
• 2-hour glucose is >= 7.8 mmol/1
Management of gestational diabetes
• newly diagnosed women should be seen in a Joint diabetes and
antenatal clinic within a week
• women should be taught about selfmonitoring of blood glucose
• advice about diet (including eating foods with a low glycaemic
index) and exercise should be given
• if the fasting plasma glucose level is < 7 mmol//I a trial of diet and
exercise should be offered
• if glucose targets are not met within 1-2 weeks of altering
diet/ exercise metformin shou ld be started
• if glucose targets are still not met insulin should be added to
diet/ exercise/ metformin
• if at the time of diagnosis the fasting glucose level is >= 7 mmol/I
insulin should be started
• if the plasma glucose level is between 6-6.9 mmol/I, and there is
evidence of complications such as macrosomia or hydramnios,
insulin should be offered
• glibenclamide should only be offered for women who cannot
tolerate metformin or those who fail to meet the glucose targets
with metformin but decline insulin treatment

Management of pre-existing diabetes


• weight loss for women with BMI of> 27 kg/m"2
• stop oral hypoglycaemic agents, apart from metformin, and
commence insulin
• folic acid 5 mg/ day from pre-conception to 12 weeks gestation
• aspirin 75mg/day from 12 weeks until the birth of the baby, to
reduce the risk of pre-eclampsia
• detailed anomaly scan at 20 weeks including four-chamber view
of the heart and outflow tracts
• tight glycaemic control reduces complication rates
• treat retinopathy as can worsen during pregnancy

CamScanne, -? Y.¥-> i.b. ~I


Targets for self monitoring of pregnant women (pre-existing and
gestational diabetes)

Time Target

Fasting 5.3 mmol/I

1 hour after meals 7.8 mmol/I, or:

2 hour after meals 6.4 mmol/I

Next question )

Save my notes

Search

Search textbook...
----■
Q Google search on "Pregnancy: diabetes mellitus''

Links

Ca mSc.an,ner ~ '....,.i;lo-" ~ ~ I
Q3

What is the mechanism of action of thiazolidinediones?

PPAR-gamma receptor antagonist

PPAR-alpha receptor antagonist

PPAR-alpha receptor agonist

PPAR-gamma receptor agonist

Increases endogenous insulin secretion

Submit answer

Reference ranges v
PPAR-gamma receptor agonist

Increases endogenous insulin secretion

Glitazones are agonists of PPAR-gamma receptors, reducing


peripheral insulin resistance
Importance: 50

116 •• tit Discuss Improve

Next question )

Thiazolidinediones

Thiazolidinediones are a class of agents used in the treatment of type 2


diabetes mellitus. They are agonists to the PPAR-gamma receptor and
reduce peripheral insulin resistance. Rosiglitazone was withdrawn in
201 O following concerns about the cardiovascular side-effect profile.

The PPAR-gamma receptor is an intracellular nuclear receptor. It's


natural ligands are free fatty acids and it is thought to control adipocyte
differentiation and function.

Adverse effects
• weight gain
• liver impairment: monitor LFTs
• fluid retention - therefore contraindicated in heart failure. The risk
of fluid retention is increased if the patient also takes insulin
• recent studies have indicated an increased risk of fractures
• bladder cancer: recent studies have shown an increased risk of
bladder cancer in patients taking pioglitazone (hazard ratio 2.64)
A 29-year-old female who is 7 weeks into her first pregnancy is
investigated for excessive sweating and tremor. Blood tests reveal the
following:

TSH < 0.05 mu/ I

T4 188 nrnol/1

What is the most appropriate management?

Immediate surgery

Carbimazole

Surgery at start of third trimester

Propylthiouracil

Radioiodine

Submit answer

Reference ranges v
Propylthiouracil

Radioiodine

According to CKS:

Propy/thiouraci/ is used in the first trimester of pregnancy in place


of carbimazole, as the latter drug may be associated with an
increased risk of congenital abnormalities. At the beginning of the
second trimester, the woman should be switched back to
carbimazole, as propylthiouraci/ has been associated with a small
risk of severe hepatic injury

[ ,6 •• tt Discuss Improve ]

Next question )

Pregnancy: thyroid problems

In pregnancy, there is an increase in the levels of thyroxine-binding


globulin (TBG). This causes an increase in the levels of total thyroxine
but does not affect the free thyroxine level.

Thyrotoxicosis

Untreated thyrotoxicosis increases the risk of fetal loss, maternal heart


failure and premature labour

CamScanner -! W~ :.i.;,-~\
Graves' disease is the most common cause of thyrotoxicosis in
pregnancy. It is also recognised that activation of the TSH receptor by
HCG may also occur - often termed transient gestational
hyperthyroidism. HCG levels will fall in the second and third trimester

Management
• propylthiouracil has traditionally been the antithyroid drug of
choice
• however, propylthiouracil is associated with an increased risk of
severe hepatic injury
• therefore NICE Clinical Knowledge Summaries advocate the
following : 'Propylthiouracil is used in the first trimester of pregnancy
in place of carbimazole, as the latter drug may be associated with
an increased risk of congenital abnormalities. At the beginning of
the second trimester, the woman should be switched back to
carbimazole'
• maternal free thyroxine levels should be kept in the upper th ird of
the normal reference range to avoid fetal hypothyroidism
• thyrotrophin receptor stimulating antibodies should be checked at
30-36 weeks gestation - helps to determine the risk of neonatal
thyroid problems
• block-and-replace regimes should not be used in pregnancy
• radioiodine therapy is contraindicated

Hypothyroidism

Key points
• thyroxine is safe during pregnancy
• serum thyroid stimulating hormone measured in each trimester
and 6-8 weeks post-partum
• some women require an increased dose of thyroxine during
pregnancy
• breast feeding is safe whilst on thyroxine

Next question >


QS

Which one of the following statements regarding maturity-onset


diabetes of the young (MODY) is true?

There is usually a strong family history

Body mass index is typically > 30

Doesn't respond to glimepiride

Autosomal recessive inheritance

Frequent episodes of diabetic ketoacidosis are typical

Submit answer

Reference ranges v
QS

Which one of the following statements regarding maturity-onset


diabetes of the young (MODY) is true?

IThere is usually a strong family history

Body mass index is typically > 30

Doesn't respond to glimepiride

Autosomal recessive inheritance

Frequent epi sodes of diabetic ketoacidosis are typical

116 11
f tt Discuss Improve

Next question )
MODY

Maturity-onset diabetes of the young (MODY) is characterised by the


development of type 2 diabetes mellitus in patients< 25 years old. It is
typically inherited as an autosomal dominant condition. Over six
different genetic mutations have so far been identified as leading to
MODY.

It is thought that around 1-2% of patients with diabetes mellitus have


MODY, and around 90% are misclassified as having either type 1 or type
2 diabetes mellitus.

MODY3
• 60% of cases
• due to a defect in the HNF-1 alpha gene
• is associated with an increased risk of HCC

MODY2
• 20% of cases
• due to a defect in the glucokinase gene

MODY 5
• rare
• due to a defect in the HNF-1 beta gene
• liver and renal cysts

Features of MODY
• typically develops in patients < 25 years
• a family history of early onset diabetes is often present
• ketosis is not a feature at presentation
• patients with the most common form are very sensitive to
sulfonylureas, insulin is not usually necessary
Q6

Which one of the following is not associated with primary


hyperparathyroidism?

Hypotension

Multiple endocrine neoplasia type 1

Multiple endocrine neoplasia type 2a

Depression

Pancreatitis

Submit answer

Reference ranges v
r Hypotension

Multiple endocrine neoplasia type 1

Multiple endocrine neoplasia type 2a

Depression

Pancreatitis

Primary hyperparathyroidism is associated with hypertension

" 'f tit Discuss (3) Improve ]

Next question )

Primary hyperparathyroidism

In exams, primary hyperparathyroidism is stereotypically seen in elderly


females with an unquenchable thirst and an inappropriately normal or
raised parathyroid hormone level. It is most commonly due to a solitary
adenoma

Causes of primary hyperparathyroidism


• 80%: solitary adenoma
• 15%: hyperplasia
• 4%: multiple adenoma
• 1%: carcinoma
Features - 'bones, stones, abdominal groans and psychic moans'
• polydipsia, polyuria
• peptic ulceration/constipation/pancreatitis
• bone pain/fracture
• renal stones
• depression
• hypertension

Associations
• hypertension
• multiple endocrine neoplasia: MEN I and II

Investigations
• raised call cium, low phosphate
• PTH may be raised or (inappropriately, given the raised calcium)
normal
• technetium-MIBI subtraction scan
• pepperpot skull is a characteristic X-ray finding of
hyperparathyroidism

Treatment
• the definitive management is total parathyroidectomy
• conservative management may be offered if the calcium level is
less than 0.25 mmol/L above the upper limit of normal AND the
patient is > 50 years AND there is no evidence of end-organ
damage
• calcimimetic agents such as cinacalcet are sometimes used in
patients who are unsuitable for surgery
Q7

A 61-year-old man presents as he developed enlargement of his breast


tissue. He has become very self-conscious and is worried about going
on holiday in the summer. Which one of the following drugs is most
likely to be responsible?

Amitriptyline

lsoniazid

Verapamil

Methyldopa

Spironolactone

Submit answer

Reference ranges v
lsoniazid

Verapamil

Methyl dopa

Spironolactone

All the above drugs may cause gynaecomastia but spironolactone is the
most common cause.

" •• _. Discuss (2) Improve ]

Next question )

Ca mSc.an,ner ~ '....,.i;lo-" ~ ~ I
Gynaecomastia

Gynaecomastia describes an abnormal amount of breast tissue in


males and is usually caused by an increased oestrogen:androgen ratio.
It is important to differentiate the causes of galactorrhoea (due to the
actions of prolactin on breast tissue) from those of gynaecomastia

Causes of gynaecomastia
• physiological: normal in puberty
• syndromes with androgen deficiency: Kallman's, Klinefelter's
• testicular failure: e.g. mumps
• liver disease
• testicular cancer e.g. seminoma secreting hCG
• ectopic tumour secretion
• hyperthyroidism
• haemodialysis
• drugs: see below

Drug causes of gynaecomastia


• spironolactone (most common drug cause)
• cimetidine
• digoxin
• cannabis
• fmasteride
• gonadorelin analogues e.g. Goserelin, buserelin
• oestrogens, anabolic steroids

Very rare drug causes of gynaecomastia


• tricyclics
• isoniazid
• calcium channel blockers
• heroin
• busulfan
• methyldopa
Q8

A 35-year-old gentleman is followed up in general practice after a


routine health check at work has Identified high blood pressure. He has
been started on initial anti-hypertensive therapy whi lst awaiting
investigation. He is otherwise well with no past medical history of note.

He reports that his grandfather had been previously diagnosed with


Conn's syndrome at an early age.

Which of the following can interfere with testing for primary


hyperaldosteronism?

Digoxi n

Amlodipine

lvabradine

Bisoprolol

Ramipril

Submit answer

Reference ranges v
( Kam1prn

The answer here is ramipril. The reason behind this is due to its
interference with the renin-angiotensin-aldosterone system, for which
the other medications do not.

Medications that can cause false negative renin:aldosterone ratio


results are the following:
• Angiotensin-converting enzyme inhibitors (e.g. ramipril or
lisinopril).
• Angiotensin receptor blockers (e.g. losartan).
• Direct renin inhibitors (e.g aliskiren).
• Aldosterone antagonists (e.g. spironolactone or eplerenone).

a6 if t9 Discuss (5) Improve

Next question )

Primary hyperaldosteronism

Primary hyperaldosteronism was previously thought to be most


commonly caused by an adrenal adenoma, termed Conn's syndrome.
However, recent studies have shown that bilateral idiopathic adrenal
hyperplasia is the cause in up to 70% of cases. Differentiating between
the two is important as this determines treatment. Adrenal carcinoma is
an extremely rare cause of primary hyperaldosteronism.

Features
• hypertension
• hypokalaemia
o e.g. muscle weakness
o this is a classical feature in exams but studies suggest this
is seen in only 10-40% of patients
• alkalosis
Investigations
• the 2016 Endocrine Society recommend that a plasma
aldosterone/renin ratio is the first-line investigation in suspected
primary hyperaldosteronism
o should show high aldosterone levels alongside low renin
levels (negative feedback due to sodium retention from
aldosterone)
• following this a high-resolution CT abdomen and adrenal vein
sampling is used to differentiate between unilateral and bilateral
sources of aldosterone excess
• Adrenal Venous Sampling (AVS) can be done to identify the gland
secreting excess hormone in primary hyperaldosteronism

Management
• adrenal adenoma: surgery
• bilateral adrenocortical hyperplasia: aldosterone antagonist e.g.
spironolactone
Q9

A 68-year-old man attends his GP surgery for his chronic disease annual
review following on from recent blood tests.

His past medical history includes hypertension, polymyalgia rheumatica


and type 2 diabetes mellitus.

He regularly takes amlodipine, bendroflumethiazide, prednisolone on a


reducing regimen, sitagliptin and metformin.

M ale: (135-180)
Hb 142 g/ L
Female: (115 - 160)

Plat elets 270 * 109 / L (150 - 400)

WBC 12.8 * 109 / L (4.0 - 11 .0)

Neuts 10.6 * 109 / L (2.0 - 7.0)

Which Is his medications is the most likely cause of his raised


neutrophil count?

Amlodipine

Bendroflumethiazide

Metformin

Prednisolone

Sitagliptin

Submit answer

Cam Sconner ...,, ',&~ .G. ~ I


Bendroflumethiazide

Metformin

Prednisolone

Sitagliptin

Glucocorticoid treatment can induce neutrophilia


Importance: 50

Amlodipine and bendroflumethiazide can cause leucopenia, not


neutrophilia.

Sitagliptin and metformin don't affect leukocytes.

Prednisolone can cause neutrophilia through:


• Demargination of neutrophils via the endovascular lining.
• Delayed migration of neutrophils into tissue.
• Release of immature neutrophils from bone marrow.

During bacterial infection, there is an increased 'left shift' seen on


microscopy. In 'left shift' neutrophils are produced and released from
the bone marrow at an equal rate to their consumption in the
circulation.

With steroid use, 'left shift' generally does not occur due to increased
production and release of neutrophils from the bone marrow but no
neutrophil consumption.

9' 11
f tt Discuss Improve

Next question )
Next question )

Corticosteroids: side-effects

Glucocorticoid side-effects
• endocrine: impaired glucose regulation, increased appetite/ weight
gain, hirsutism, hyperlipidaemia
• Cushing's syndrome: moon face, buffalo hump, striae
• musculoskeletal: osteoporosis, proximal myopathy, avascular
necrosis of the femoral head
• immunosuppression: increased susceptibility to severe infection,
reactivation of tuberculosis
• psychiatric: insomnia, mania, depression, psychosis
• gastrointestinal: peptic ulceration, acute pancreatitis
• ophthalmic: glaucoma, cataracts
• dermatological: acne
• suppression of growth in children
• intracranial hypertension
• neutrophilia

Mineralocorticoid side-effects
• fluid retention
• hypertension

Next question )

B J I!! A • ·-
·- 1-
~
- - ...
Q10

A 25-year-old Asian woman who is 26 weeks pregnant has an oral


glucose tolerance test (OGTT). This was requested due t o a
combination of her et hnicit y and a background of obesity. A recent
ult rasound shows that the fetus is large for dates. The f ollowing results
are obtained:

Time (hours) Blood glucose (mmol/1)

0 9.2

2 14.2

What is the most appropriate management ?

Start insulin

Give advice about a diabetic diet

Give advice about a diabetic diet + repeat OGTT in 4 weeks

Start gliclazide

Start metformin

Submit answer
A L~-year-u1u Asian woman wno 1s zo weeKs pregnan1 nas an oral
glucose tolerance test (OGTT). This was requested due to a
combination of her ethnicity and a background of obesity. A recent
ultrasound shows that the fetus is large for dates. The following results
are obtained:

Time (hours) Blood glucose (mmol/ 1)

0 9.2

2 14.2

What is the most appropriate management?

I Start insulin

Give advice about a diabetic diet

Give advice about a diabetic diet+ repeat OGTT in 4 weeks

Start gliclazide

Start metformin

Insulin should be started straight away given the blood glucose levels
and evidence of macrosomia. Aspirin should also be considered as she
is at increased risk of pre-eclampsia.

9' •• tt Discuss Improve

Next question )
Q11

A 50-year-old female complains that she has put on weight around her
abdomen and th ighs in the last six months. She has a past medical
history of hypertension, high cholesterol, type II diabetes and asthma.
She has not changed her diet or lifestyle during the last six months.
Which of her medications may be contributing to her weight gain?

Atorvastatin

Tolbutamide

Exenatide

Chlortalidone

Saxagliptin

Submit answer

Reference ranges v
Tolbutamide

Exenatide

Chlortalidone

Saxagliptin

Sulfonylureas often cause weight gain


Importance: 50

The correct answer is 2. Tolbutamide is a member of the sulfonylurea


class, which ar,e known to often cause weight gain because they
stimulate the pancreas to release more insulin, therefore allowing for
the utilisation of more glucose. The gliptins are not associated with
weight gain.

BNF
https://bnf.nice.org.uk/ treatment-summary/ type-2-diabetes.html

116 t
11 • Discuss (2) Improve

Next question >


Next question )

Sulfonylureas

Sulfonylureas are oral hypoglycaemic drugs used in the management of


type 2 diabetes mellitus. They work by increasing pancreatic insulin
secretion and hence are only effective if functional 8-cells are present.
On a molecular level they bind to an ATP-dependent K+(KATP) channel
on the cell membrane of pancreatic beta cells .

Common adverse effects


• hypoglycaemic episodes (more common with long-acting
preparations such as chlorpropamide)
• weight gain

Rarer adverse effects


• hyponatraemia secondary to syndrome of inappropriate ADH
secretion
• bone marrow suppression
• hepatotoxicity (typically cholestatic)
• peripheral neuropathy

Sulfonylureas should be avoided in breastfeeding and pregnancy.

Next question )

B I A .. 2.=
1- -_ ....
~ c-:::>
Q12

A 59-year-old woman comes to the diabetes clinic for review. Current


medication incJudes metformin and sitagliptin, but her HbA1 c has
continued to rise and additional therapy is required . Her body mass
index is 35 kg/ m 2 • Blood pressure is 132/ 82 mmHg, her pulse is 74
beats per minute and regular. A decision is made to start her on
degludec liraglutide combination therapy.

Which of the following should you warn her about?

Patients often get skin nodules around the injection site

Pancreatitis is commonly associated with treatment

Risk of hypoglycaemia will increase when she starts treatment

She will lose approximately 5kg in weight when she starts


treatment

She will gain approximately 6kg in the first year of treatment

Submit answer

Reference ranges v
rdlll,;lt:dllll::> I::> l,;UIIIIIIUIIIY d::>::>Ul,;ldlt:U VVILII llt:dllllt:lll

Risk of hypoglycaemia will increase when she starts treatment

She will lose approximately 5kg in weight when she starts treatment

She will gain approximately 6kg in the first year of treatment

insulin and GLP-1 fixed dose combinations are increasingly attractive as


a treatment option for diabetologists because they combine the potent
glucose-lowering effect of GLP-1 agonists and long-acting insulin
analogues in one preparation. They are however associated with
increased risk of hypoglycaemia, Rates of confirmed hypoglycaemia
(<3.1 mmol/I), run at between 1.8 and 3.5 per patient year of exposure.

Skin nodules are most often seen around injection sites when modified
release systems are used for delivering GLP-1, such as those used in
weekly exenatide, (microspheres). Pancreatitis features in case reports
associated with the use of GLP-1 agonists, although no consistent link
has been established. Modest weight loss is seen when patients start
degludec / liraglutide against a background of oral agents, but this is
less than that seen for patients who start liraglutide alone.

https://www.evidence.nhs.uk/formulary/bnf/current/6-endocrine-
system/61-drugs-used-in-diabetes/612-antidiabetic-drug s/6123-other-
antidiabetic-drugs

8' ,., t9 Discuss (8) Improve ]

Next question )
Next question )

Diabetes mellitus: GLP-1 and the new drugs

A number of new drugs to treat diabetes mellitus have become


available in recent years. Much research has focused around the role of
glucagon-like peptide-1 (GLP-1), a hormone released by the small
intestine in response to an oral glucose load

Whilst it is well known that insulin resistance and insufficient B-cell


compensation occur other effects are also seen in type 2 diabetes
mellitus (T2DM). In normal physiology an oral glucose load results in a
greater release of insulin than if the same load is given intravenously -
this known as the incretin effect. This effect is largely mediated by GLP-
1 and is known to be decreased in T2DM.

Increasing GLP-1 levels, either by the administration of an analogue


(glucagon-like peptide-1, GLP-1 mimetics, e.g. exenatide) or inhibiting
its breakdown (dipeptidyl peptidase-4 ,DPP-4 inhibitors - the gliptins), is
therefore the target of two recent classes of drug.

Glucagon-like peptide-1 (GLP-1) mimetics (e.g. exenatide)

Exenatide is an example of a glucagon-like peptide-1 (GLP-1) mimetic.


These drugs increase insulin secretion and inhibit glucagon secretion.
One of the major advances of GLP-1 mimetics is that they typically
result in weight loss, in contrast to many medications such as insulin,
sulfonylureas and thiazolidinediones. They are sometimes used in
combination with insulin in T2DM to minimise weight gain.

Exenatide must be given by subcutaneous injection within 60 minutes


before the morning and evening meals. It should not be given after a
meal.
Liraglutide is the other GLP-1 mimetic currently available. One the main
advantages of liraglutide over exenatide is that it only needs to be given
once a day.

Both exenatide and liraglutide may be combined with metformin and a


sulfonylurea. Standard release exenatide is also licensed to be used
with basal insulin alone or with metformin. Please see the BNF for a
more complete list of licensed indications.

NICE state the following:

Consider adding exenatide to metformin and a sulfonylurea if:


• BMI >= 35 kg/ m 2 in people of European descent and there are
problems associated with high weight, or
• BMI < 35 kg/ m 2 and insulin is unacceptable because of
occupational implications or weight loss would benefit other
comorbidities.

NICE like patients to have achieved a 11 mmol/mol (1 %) reduction in


HbA 1c and 3% weight loss after 6 months to justify the ongoing
prescription of GLP-1 mimetics.

The major adverse effect of GLP-1 mimetics is nausea and vomiting.


The Medicines and Healthcare products Regulatory Agency has issued
specific warnings on the use of exenatide, reporting that is has been
linked to severe pancreatitis in some patients.
Dipeptidyl peptidase-4 {DPP-4) inhibitors ( e.g. Vildagliptin, sitagliptin)

Key points
• oral preparation
• trials to date show that the drugs are relatively well tolerated with
no increased incidence of hypoglycaem ia
• do not cause weight gain

NICE guidelines on DPP-4 inhibitors


• NICE suggest that a DPP-4 inhibitor might be preferable to a
thiazolidinedione if further weight gain wou ld cause significant
problems, a thiazolidinedione is contraindicated or the person has
had a poor response to a thiazolidinedione

Next question )

B I

Save my n ates

Search

Search textbook ...



Q13

An 81-year old female is admitted with a 6-week history dysphagia to


both solids and liquids. She describes odynophagia, weight loss and
night sweats.

On examination there was firm irregular mass in the right side of the
anterior triangle of the neck. It was fixed, cold and painless. The mass
moved with swallowing and you note a faint stridor like sound on
inspiration. There was a further 3 irregular lymph nodes of note on
palpation.

Bloods:

Thyroid stimulating hormone 4.5 mu/I

Free T4 12 pmol/1

Total T4 99 nmol/1

An ultrasound-guided biopsy is likely to reveal which histological


tumour?

Papillary

Follicular

Medullary

Anaplastic

Lymphoma
Ana plastic

Lymphoma

Anaplastic thyroid cancer is a highly aggressive, locally invasive tumour.


It typically presents in older patients with a rapidly increasing mass or
lymph node. Anaplastic tumours invades local surrounding tissues
causing compression symptoms including: pain, shortness of breath
and dysphagia. The aggression of the tumour often leads to
lymphovascular invasion and subsequent bone and lung metastasis.
The cancer originates from follicular cells, which are poorly
differentiated and have a high mitotic rate. The prognosis is poor with a
5-year survival rate quoted between 7% and 14%. Treatment is usually
palliative, with a combination of radiotherapy and chemotherapy.

11• 11 f tt Discuss (5) Improve

Next question )

Thyroid cancer

Features of hyperthyroidism or hypothyroidism are not commonly seen


in patients with thyroid malignancies as they rarely secrete thyroid
hormones

Main points

Type Percentage

Papillary 70% Often young females - excellent


prognosis

Follicular 20%
Medullary 5% Cancer of parafollicular (C) cells,
secrete calcitonin, part of MEN-2

Anaplastic 1% Not responsive to treatment, can


cause pressure symptoms

Lymphoma Rare Associated with Hashimoto's


thyroiditis

Management of papillary and follicular cancer


• total thyroidectomy
• followed by radioiodine (1-131) to kill residual cells
• yearly thyroglobulin levels to detect early recurrent disease

Further information

Type Notes

Papillary • Usually contain a mixture of papillary and


carcinoma colloidal filled follicles
• Histologically tumour has papillary projections
and pale empty nuclei
• Seldom encapsulated
• Lymph node metastasis predominate
• Haematogenous metastasis rare

Follicular • Usually present as a solitary thyroid nodule


adenoma • Malignancy can only be excluded on formal
histological assessment

Follicular • May appear macroscopically encapsulated,


carcinoma microscopically capsular invasion is seen.
Without this finding the lesion is a follicular
adenoma.
Follicular • May appear macroscopically encapsulated,
carcinoma microscopically capsular invasion is seen.
Without this finding the lesion is a follicular
adenoma.
• Vascular invasion predominates
• Multifocal disease raree

Medullary • C cells derived from neural crest and not thyroid


carcinoma tissue
• Serum calcitonin levels often raised
• Familial genetic disease accounts for up to 20%
cases
• Both lymphatic and haematogenous metastasis
are recognised, nodal disease is associated with
a very poor prognosis.

Anaplastic • Most common in elderly females


carcinoma • Local invasion is a common feature
• Treatment is by resection where possible,
palliation may be achieved through
isthmusectomy and radiotherapy.
Chemotherapy is ineffective.

Next question )

Save my notes
Q14

Which of the following is least recognised as a potential complication


of acromegaly?

Colorectal cancer

Hypertension

Cardiomyopathy

Diabetes mellitus

Pulmonary hypertension

Submit answer

Reference ranges v
I Pulmonary hype_r_te_n_s_io_n_ _ _ _ _~ .

Acromegaly is associated with systemic rather than pulmonary


hypertension.

Secondary causes of pulmonary hypertension include COPD, congenital


heart disease (Eisenmenger's syndrome), recurrent pulmonary
embolism, HIV and sarcoidosis.

16 11
f • Discuss (4) Improve

Next question >

Acromegaly: features

In acromegaly there is excess growth hormone secondary to a pituitary


adenoma in over 95% of cases. A minority of cases are caused by
ectopic GHRH or GH production by tumours e.g. pancreatic.

Features
• coarse facial appearance, spade-like hands, increase in shoe size
• large tongue, prognathism, interdental spaces
• excessive sweating and oily skin: caused by sweat gland
hypertrophy
• features of pituitary tumour: hypopituitarism, headaches,
bitemporal hemianopia
• raised prolactin in 1/ 3 of cases _... galactorrhoea
• 6% of patients have MEN-1

Complications
• hypertension
• diabetes (>10%)
• cardiomyopathy
• colorectal cancer
Q15 [CJ

A 58-year-old gentleman with longstanding type 2 diabetes presents to


the acute medical take. Blood tests are demonstrated in the table
below. The blood test results are consistent with diabetic ketoacidosis.
He has no other past medical history other than type 2 diabetes and
obesity. He has not had episodes of diabetic ketoacidosis before and
does not drink alcohol. His medication history includes aspirin, losartan,
metformin, dapagliflozin and glimepiride. He is allergic to penicillin.

pH 7.26

Blood ketones 3.6 mmol/ L

Blood sugar 15 mmol/ L

Which of his medications is most likely to have contributed to


developing diabetic ketoacidosis?

Metformin

Dapagliflozin

Glimepiride

Aspirin

Losartan

Submit answer
Dapagliflozin

Glimepiride

Aspirin

Losartan

Dapagliflozin is a newer drug for the treatment of diabetes. It is a


member of the sodium-glucose transport protein 2 (SGLT2) inhibitor
class of drugs.

SGLT2 inhibitors prevent the resorption of glucose from the proximal


renal tubule, resulting in more glucose being secreted in the urine.

The other medications in this class include: canagliflozin &


empagliflozin

Importantly, whilst these medications represent an effective class of


drugs, there are reports of patients with type 2 diabetes presenting in
diabetic ketoacidosis whilst taking them. It essential that acute medical
teams are vigilant for such presentations as the prevalence of SGLT2
inhibitor prescribing increases.

The other possible answers (metformin / gllmepiride / aspirin /


losartan) are not associated with the development of diabetic
ketoacidosis. It should, of course, be noted that metformin can lead to a
lactic acidosis.

Source: UK Government Drug Safety Bulletin (https://www.gov.uk/drug-


safety-update/ sglt2-inhibitors-updated-advice-on-the-risk-of-diabetic-
ketoacidos is)

8' •• • Discuss (5) Improve


[ ,6 •• t9 Discuss (5) Improve

Next question >

SGLT-2 inhibitors

SGLT-2 inhibitors reversibly inhibit sodium-glucose co-transporter 2


(SGLT-2) in the renal proximal convoluted tubule to reduce glucose
reabsorption aind increase urinary glucose excretion.

Examples include canagliflozin, dapagliflozin and empagliflozin.

Important adverse effects include


• urinary and genital infection (secondary to glycosuria). Fournier's
gangrene has also been reported
• normoglycaemic ketoacidosis
• increased risk of lower-limb amputation: feet should be closely
monitored

Patients taking SGLT-2 drugs often lose weight, which can be beneficial
in type 2 diabetes mellitus.

Next question )

B I A .... c-:::>

Save my notes
Q16

Which one of the following statements regarding dipeptidyl peptidase-4


inhibitors in the management of type 2 diabetes mellitus is correct?

Metformin should always be co-prescribed

Do not cause weight gain

Is given via a subcutaneous injection

An example is exenatide

Patients should be warned that hypoglycaemia is the most


common side-effect

Submit answer

Reference ranges v
Do not cause weight gain

Is given via a subcutaneous injection

An example is exenatide

Patients should be warned that hypoglycaemia is the most common


side-effect

Hypoglycaemia is rare in patients taking dipeptidyl peptidase-4


inhibitors.

,t •• • Discuss (1) Improve

Next question )

Diabetes mellitus: GLP-1 and the new drugs

A number of new drugs to treat diabetes mellitus have become


available in recent years. Much research has focused around the role of
glucagon-like peptide-1 (GLP-1 ), a hormone released by the small
intestine in response to an oral glucose load

Whilst it is well known that insulin resistance and insufficient B-cell


compensation occur other effects are also seen in type 2 diabetes
mellitus (T2DM). In normal physiology an oral glucose load results in a
greater release of insulin than if the same load is given intravenously -
this known as the incretin effect. This effect is largely mediated by GLP-
1 and is known to be decreased in T2DM.

Increasing GLP-1 levels, either by the administration of an analogue


(glucagon-like peptide-1, GLP-1 mimetics, e.g. exenatide) or inhibiting
its breakdown (dipeptidyl peptidase-4 ,DPP-4 inhibitors - the gliptins), is
therefore the target of two recent classes of drug.
Glucagon-like peptide-1 (GLP-1) mimetics (e.g. exenatide)

Exenatide is an example of a glucagon-like peptide-1 (GLP-1) mimetic.


These drugs increase insulin secretion and inhibit glucagon secretion.
One of the major advances of GLP-1 mimetics is that they typically
result in weight loss, in contrast to many medications such as insulin,
sulfonylureas and thiazolidinediones. They are sometimes used in
combination with insulin in T2DM to minimise weight gain.

Exenatide must be given by subcutaneous injection within 60 minutes


before the morning and evening meals. It should not be given after a
meal.

Liraglutide is the other GLP-1 mimetic currently available. One the main
advantages of liraglutide over exenatide is that it only needs to be given
once a day.

Both exenatide and liraglutide may be combined with metformin and a


sulfonylurea. Standard release exenatide is also licensed to be used
with basal insulin alone or with metformin. Please see the BNF for a
more complete list of licensed indications.

NICE state the following :

Consider adding exenatide to metformin and a sulfonylurea if


• BMI >= 35 kg/ m 2 in people of European descent and there are
problems associated with high weight, or
• BMI < 35 kg/ m 2 and insulin is unacceptable because of
occupational implications or weight loss would benefit other
comorbidities.
NICE like patients to have achieved a 11 mmol/mol (1 %) reduction in
HbA1c and 3% weight loss after 6 months to justify the ongoing
prescription of GLP-1 mimetics.

The major adverse effect of GLP-1 mimetics is nausea and vomiting.


The Medicines and Healthcare products Regulatory Agency has issued
specific warnings on the use of exenatide, reporting that is has been
linked to severe pancreatitis in some patients.

Dipeptidyl peptidase-4 (DPP-4) inhibitors ( e.g. Vildagliptin, sitagliptin)

Key points
• oral preparation
• trials to date show that the drugs are relatively well tolerated with
no increased incidence of hypoglycaemia
• do not cause weight gain

NICE guidelines on DPP-4 inhibitors


• NICE suggest that a DPP-4 inhibitor might be preferable to a
thiazolidinedione if further weight gain would cause significant
problems, a thiazolidinedione is contraindicated or the person has
had a poor response to a thiazolidinedione

Next question >

BI l!J A ...

Save my notes

Search
Q17 [CJ

A 6-year-old South Sudanese boy is admitted progressive worsening of


his hearing loss. His mother is extremely concerned with his lack of
progress at school. Systems review reveals a 2-month history of
malaise, arthralgia and constipation. He has a past medical history of
deafness. On examination he has dry skin and thin hair; there were no
thyroid eye signs, no ophthalmoplegia and no myxoedema. He appears
to have a smooth symmetrically enlarged goitre, which is not painful.

Thyroid function tests:

Thyroid stimulating hormone 5.7 (mu/ I)

Free T4 9 pmol/1

Total T4 67 nmol/1

Which of the following causes of hypothyroidism is the patient suffering


from?

Hashimoto's thyroiditis

Iodine deficiency

Pendred syndrome

Thyroid agenesis

Atrophic hypothyroidism

Submit answer
Thyroid agenesis

Atrophic hypothyroidism

The patient is suffering from mild signs of hypothyroidism and


progressive bilateral deafness. Out of the following answers only
Pendred syndrome presents with signs of deafness and
hypothyroidism.

a6 •• tt Discuss (2) Improve

Next question )

Pendred's syndrome

Pendred is an autosomal recessive genetic disorder that is


characterised by bilateral sensorineural deafness, with mild
hypothyroidism and a goitre. The patients tend to present with
progressive hearing loss and delay in academic progression. Often
head trauma tends to make the sensorineural deafness worse, leading
to patients having to avoid contact sports.

In Pendred syndrome there is a defect in the organification of iodine,


leading to dyshormonogenesis. However thyroid symptoms in pendred
syndrome are often mild and patients are often clinically euthyroid,
presenting only with a goitre. Thyroid function tests are also often
normal, requiring the perchlorate discharge test to aid diagnosis.

The syndrome can be diagnosed via genetic testing (Pendred syndrome


(PDS) gene, chromosome 7), audiometry and MRI imaging to look for
characteristic one and a half turns in the cochlea, compared to the
normal two and a half turns.

Treatment is with thvroid hormone replacement and cochlear implants.


Q18

A General Practitioner refers a 45-year-old fema le patient to the


endocrinology department with hypercalcaemia and raised parathyroid
hormone levels. Her blood tests are highly suggestive of primary
hyperparathyroidism. Her past medical history includes type 2 diabetes,
which is well controlled on metformin alone.

Which feature would be the strongest indication for referral of the


patient for consideration of parathyroid surgery?

Co-existing type 2 diabetes

Post menopausal patient

Persistent hypercalcaemia over 4 years

Vitamin D deficiency

Age of 45

Submit answer

Reference ranges v
Persistent hypercalcaemia over 4 years

Vitamin D deficiency

l Age of 45

NICE guidelines clearly stipulate the circumstances under which


parathyroidectomy should be considered in primary
hyperparathyroidism. These are listed below:

• Age under 50 years.


• Adjusted serum calcium concentration that is 0.25 mmol/L or
more above the upper end of the reference range.
• Estimated glomerular filtration rate (eGFR) less than 60
ml/ min/ 1.73 m2 although this threshold depends on other
factors, such as age.
• Renal stones or presence of nephrocalcinosis on ultrasound or CT.
• Presence of osteoporosis or osteoporotic fracture.
• Symptomatic disease

From the potential answers offered, the patient's age under 50 is the
only answer that meets the NICE criteria.

(Source: NICE guidelines, hyperparathyroidism -


http://cks. nice. org. u k/hyperca lcaem ia)

[ ,6 •t tit Discuss (5) Improve

Next question )
Primary hyperparathyroidism

In exams, primary hyperparathyroidism is stereotypically seen in elderly


females with an unquenchable thirst and an inappropriately normal or
raised parathyroid hormone level. It is most commonly due to a solitary
adenoma

Causes of primary hyperparathyroidism


• 80%: solitary adenoma
• 1 5%: hyperplasia
• 4%: multiple adenoma
• 1%: carcinoma

Features - 'bones, stones, abdominal groans and psychic moans'


• polydipsia, polyuria
• peptic ulceration/constipation/ pancreatitis
• bone pain/fracture
• renal stones
• depression
• hypertension

Associations
• hypertension
• multiple endocrine neoplasia: MEN I and II

Investigations
• raised calcium, low phosphate
• PTH may be raised or (inappropriately, given the raised ca lcium)
normal
• technetium-MIBI subtraction scan
• pepperpot skull is a characteristic X-ray finding of
hyperparathyroidism
Treatment
• the definitive management is total parathyroidectomy
• conservative management may be offered if the calcium level is
less than 0.25 mmol/L above the upper limit of normal AND the
patient is > 50 years AND there is no evidence of end-organ
damage
• calcimimetic agents such as cinacalcet are sometimes used in
patients who are unsuitable for surgery
Q19

A 58-year-old man comes for review in the diabetes clinic. He was


diagnosed as having type 2 diabetes mellitus (T2DM) around 1 O years
ago and currently only takes gliclazide and atorvastatin. Three years
ago he was successfully treated for bladder cancer. A recent trial of
metformin was unsuccessful due to gastrointestinal side-effects. He
works as an accountant, is a non-smoker and his BMI is 31 kg/ m 2 • His
annual bloods show the following:

Na+ 138 mmol/I

K+ 4 .1 mmol/1

Urea 4.3 mmol/1

Creatinine 104 µmol/1

HbA1c 62 mmol/mol (7.8%)

What is the most appropriate next step in management?

Add pioglitazone

Add exenatide

Add acarbose

Add repaglinide

Add sitagliptin

Submit answer
Add sitagliptin

Pioglitazone is contraindicated by his history of bladder cancer and may


contribute to his obesity. A DPP-4 inhibitor such as sitagliptin is
therefore the best option.

Exenatide generally causes weight loss and is therefore useful in obese


diabetics but he does not meet the NICE body mass index criteria of 35
kg/m 2 •

16 •t tt Discuss (4) Improve

Next question >

Diabetes mellitus: management of type 2

NICE updated its guidance on the management of type 2 diabetes


mellitus (T2DM) in 2015. Key points are listed below:
• HbA 1c targets have changed. They are now dependent on what
antidiabetic drugs a patient is receiving and other factors such as
frailty
• there is more flexibility in the second stage of treating patients
(i.e. after metformin has been started) - you now have a choice of
4 oral antidiabetic agents

It's worthwhile thinking of the average patient who is taking


metformin for T2DM, you can titrate up metformin and encourage
lifestyle changes to aim for a HbA1c of 48 mmol/mol (6.5%), but
should only add a second drug if the HbA 1c rises to 58 mmol/mol
(7. 5%)
Dietary advice
• encourag1e high fibre, low glycaemic index sources of
carbohydrates
• include low-fat dairy products and oily fish
• control the intake of foods containing saturated fats and trans
fatty acids
• limited substitution of sucrose-containing foods for other
carbohydrates is allowable, but care should be taken to avoid
excess energy intake
• discourage use of foods marketed specifically at people with
diabetes
• initial target weight loss in an overweight person is 5-10%

HbA1c targets

This is area which has changed in 2015


• individual targets should be agreed with patients to encourage
motivation
• HbA 1 c should be checked every 3-6 months until stable, then 6
monthly
• NICE encourage us to consider relaxing targets on 'a case-by-case
basis, with particular consideration for people who are older or frail,
for adults with type 2 diabetes'
• in 2015 the guidelines changed so HbA 1 c targets are now
dependent on treatment:
Lifestyle or single drug treatment

HbA1c
Management of T2DM target

Lifestyle 48
mmol/mol
(6.5%)

Lifestyle+ metformin 48
mmol/mol
(6.5%)

Includes any drug which may cause hypoglycaemia 53


(e.g. lifestyle+ sulfonylurea) mmol/mol
(7.0%)

Practical examples
• a patient is newly diagnosed with HbA 1c and wants to try lifestyle
treatment first. You agree a target of 48 mmol/mol (6.5%)
• you review a patient 6 months after starting metformin. His
HbA 1c is 51 mmol/mol (6.8%). You increase his metformin from
500mg bd to 500mg tds and reinforce lifestyle factors

Patient already on treatment

Management of T2DM HbA1c target

Already on one drug, but HbA1c has risen to 58 53 mmol/mol


mmol/mol (7.5%) (7.0%)
Drug treatment

The 2015 NICE guidelines introduced some changes into the


management of type 2 diabetes. There are essentially two pathways,
one for patients who can tolerate metformin, and one for those who
can't

tMtrotmln not tolerate-a or Cl

Tolerates metformin:
• metformin is still first-line and should be offered if the HbA 1c
rises to 48 mmol/mol (6.5%)* on lifestyle interventions
• if the HbA 1c has risen to 58 mmol/mol (7.5%) then a second drug
should be added from the following list
• - sulfonylurea
• - gliptin
• - pioglitazone
• - SGLT-2 inhibitor
• if despite this the HbA1 c rises to, or remains above 58 mmol/mol
(7.5%) then triple therapy with one of the following combinations
should be offered:
• - metformin + gliptin + sulfonylurea
• - metformin + pioglitazone + sulfonylurea
• - metformin + sulfonylurea + SGLT-2 inhibitor
• if despite this the HbA1c rises to, or remains above 58 mmol/mol
(7.5%) then triple therapy with one of the following combinations
should be offered:
• - metformin + gliptin + sulfonylurea
• - metformin + pioglitazone + sulfonylurea
• - metformin + sulfonylurea + SGLT-2 inhibitor
• - metformin + pioglitazone + SGLT-2 inhibitor
• - OR insulin therapy should be considered

Criteria for glucagon-like peptide1 (GLP1) mimetic (e.g. exenatide)


• if triple therapy is not effective, not tolerated or contraindicated
then NICE advise that we consider combination therapy with
metformin, a sulfonylurea and a glucagonlike peptide1 (GLP1 )
mimetic if:
• - BMI >= 35 kg/ m 2 and specific psychological or other medical
problems associated with obesity or
• - BMI < 35 kg/ m 2 and for whom insulin therapy would have
significant occupational implications or

weight loss would benefit other significant obesityrelated comorbidities


• only continue if there is a reduction of at least 11 mmol/mol
[1.0%] in HbA1c and a weight loss of at least 3% of initial body
weight in 6 months

Practical examples
• you review an established type 2 diabetic on maximum dose
metformin. Her HbA1c is 55 mmol/ mol (7.2%). You do not add
another drug as she has not reached the threshold of 58
mmol/ mol (7.5%)
• a type 2 diabetic is found to have a HbA1c of 62 mmol/mol (7.8%)
at annual review. They are currently on maximum dose metformin.
You elect to add a sulfonylurea

CamScanne, -? Y.¥-> i.b . ~I


Cannot tolerate metformin or contraindicated
• if the HbA1 c rises to 48 mmol/ mol (6.5%)* on lifestyle
interventions, consider one of the following :
• - sulfonylurea
• _. gliptin
• -+ pioglitazone
• if the HbA 1c has risen to 58 mmol/mol (7.5%) then a one of the
following combinations should be used:
• -+ gliptin + pioglitazone
• -+ gliptin + sulfonylurea
• _. pioglitazone + sulfonylurea
• if despite this the HbA1 c rises to, or remains above 58 mmol/mol
(7.5%) then consider insulin therapy

Starting insulin
• metformin should be continued. In terms of other drugs NICE
advice: 'Review the continued need for other blood glucose lowering
therapies'
• NICE recommend starting with human NPH insulin (isophane,
intermediate acting) taken at bed-time or twice daily according to
need

Risk factor modification

Blood pressure
• target is < 140/ 80 mm Hg (or < 130/ 80 mm Hg if end-organ
damage is present)
• ACE inhibitors are first-line

Antiplatelets
• should not be offered unless a patient has existing cardiovascular
disease
Lipids
• following the 2014 NICE lipid modification guidelines only patients
with a 10-year cardiovascular risk > 10% (using QRISK2) should be
offered a statln. The first-line statin of choice is atorvastatin 20mg
on

Pnmary pnr,enuon &teond.lry ~ventton


10.yu. Ca'dio'laltlDf ~ n 10'!<. CA 1thH1rt0HN,tOR
moat type 1 clalletlcs OR IC!Mr . . . . OR
CKDIIGf'R 1011~ -..twall . . . . . . . . . .

- - - - -•- - - - - - - - -•- - - - -
Atorva5taun 20mg od
AtotVastat n IOmg oa
noM-iDL tl • noc fMn DY u .& 1tltn
CCl'l.lid9f lmlli!lg ~ to 80n,g

Graphic showing choice of statin.

*this is a bit confusing because isn't the diagnostic criteria for T2DM
HbA1c 48 mmol/ mol (6.5%)? So shouldn't all patients be offered
metformin at diagnosis? Our interpretation of this is that some patients
upon diagnosis will elect to try lifestyle measures, which may reduce
their HbA1c below this level. If it then rises to the diagnostic threshold
again metformin should be offered

Next question )

Save my notes
Q20

Each one of the following is associated with


pseudohypoparathyroidism, except:

Low calcium levels

Low PTH levels

Shortened 4th and 5th metacarpals

Low IQ

Short stature

Submit answer

Reference ranges v
I Low PTH levels

Shortened 4th and 5th metacarpals

Low IQ

Short stature

116 •t tt Discuss (2) Improve

Next question )

Hypoparathyroidism

Primary hypoparathyroidism
• decrease PTH secretion
• e.g. secondary to thyroid surgery*
• low calcium, high phosphate
• treated with alfacalcidol

The main symptoms of hypoparathyroidism are secondary to


hypocalcaemia:
• tetany: muscle twitching, cramping and spasm
• perioral paraesthesia
• Trousseau's sign: carpal spasm if the brachia I artery occluded by
inflating the blood pressure cuff and maintaining pressure above
systolic
• Chvostek's sign: tapping over parotid causes facial muscles to
twitch
• if chronic: depression, cataracts
• ECG: prolonged QT interval
Pseudohypoparathyroidism
• target cells being insensitive to PTH
• due to abnormality in a G protein
• associated with low IQ, short stature, shortened 4th and 5th
metacarpals
• low calcium, high phosphate, high PTH
• diagnosis is made by measuring urinary cAMP and phosphate
levels following an infusion of PTH. In hypoparathyroidism this
will cause an increase in both cAMP and phosphate levels. In
pseudohypoparathyroidism type I neither cAMP nor phosphate
levels are increased whilst in pseudohypoparathyroidism type II
only cAMP rises.

Pseudopseudohypoparathyroidism
• similar phenotype to pseudohypoparathyroidism but normal
biochemistry

*this may seem an oxymoron, but most medical textbooks classify


hypoparathyroidism which is secondary to surgery as being 'primary
hypoparathyroidism'

Next question )

Save my notes

Search

Search textbook ...


-
Q21

A 68-year-old woman is found to have the following blood tests:

TSH 0.05 mu/ I

Free T4 19 pmol/ 1(range 9-25 pmol/1)

Free T3 7 pmol/1 (range 3-9 pmol/1)

If left untreated, what are the most likely possible consequences?

Supraventricular arrhythmias and osteoporosis

Supraventricular arrhythmias and hyperlipidaemia

Hypothyroidism and impaired glucose tolerance

Myasthenia gravis and hypothyroidism

Impaired glucose tolerance and hyperlipidaemia

Submit answer

Reference ranges v
Q21

A 68-year-old woman is found to have the following blood tests:

TSH 0.05 mu/ I

Free T4 19 pmol/I (range 9-25 pmol/ 1)

Free T3 7 pmol/1 (range 3-9 pmol/ I)

If left untreated, what are the most likely possible consequences?

I Supraventricular arrhythmias and osteoporosis

Supraventricular arrhythmias and hyperlipidaemia

Hypothyroidism and impaired glucose tolerance

Myasthenia gravis and hypothyroidism

Impaired glucose tolerance and hyperlipidaemia

,. ,. • Discuss (2) Improve ]

Next question )
Next question )

Subclinical hyperthyroidism

Subclinical hyperthyroidism is an entity which is gaining increasing


recognition. It is defined as:
• normal serum free thyroxine and triiodothyronine levels
• with a thyroid stimulating hormone (TSH) below normal range
(usually< 0.1 mu/I)

Causes
• multinodular goitre, particularly in elderly females
• excessive thyroxine may give a similar biochemical picture

The importance in recognising subclinical hyperthyroidism lies in the


potential effect on the cardiovascular system (atrial fibrillation) and
bone metabolism (osteoporosis). It may also impact on quality of life
and increase the likelihood of dementia

Management
• TSH levels often revert to normal - therefore levels must be
persistently low to warrant intervention
• a reasonable treatment option is a therapeutic trial of low-dose
antithyroid agents for approximately 6 months in an effort to
induce a remission

Next question )

B I A ... ·-
·- 2,-: -- . . . T I""
Q22

A 45-year-old woman who has a history of Graves' disease presents


with visual problems. She is known to have Graves' ophthalmopathy
and does not currently smoke. Her most recent thyroid function tests
are shown below:

Free T4 15 pmol/1

TSH 1.6 mu/I

Which one of the following features is the strongest indicator of the


need for urgent ophthalmology review?

Sensitivity of eyes to light

Diplopia

Troublesome eyelid retraction

Awareness of change in intensity or quality of colour vision

Erythema of the conjunctiva

Submit answer
Awareness of change in intensity or quality of colour vision

Erythema of the conjunctiva

The other symptoms/signs indicate the need for non-urgent review by


an opthalmologist. Please see the EUGOGO guidelines for more details.

" 'f - Discuss (2) Improve ]

Next question )

Thyroid eye disease

Thyroid eye disease affects between 25-50% of patients with Graves'


disease.

Pathophysiology
• it is thought to be caused by an autoimmune response against an
autoantigen, possibly the TSH receptor - retro-orbital
inflammation
• the inflammation results in glycosaminoglycan and collagen
deposition in the muscles

Prevention
• smoking is the most important modifiable risk factor for the
development of thyroid eye disease
• radioiodine treatment may increase the inflammatory symptoms
seen in thyroid eye disease. In a recent study of patients with
Graves' disease around 15% developed, or had worsening of, eye
disease. Prednisolone may help reduce the risk

CamScanne, -? Y.¥-> i.b . ~I


Features
• the patient may be eu-, hypo- or hyperthyroid at the time of
presentation
• exophthalmos
• conjunctiva! oedema
• optic disc swelling
• ophthalmoplegia
• inability to close the eyelids may lead to sore, dry eyes. If severe
and untreated patients can be at risk of exposure keratopathy

Management
• topical lubricants may be needed to help prevent corneal
inflammation caused by exposure
• steroids
• radiotherapy
• surgery

Monitoring patients with established thyroid eye disease

For patients with established thyroid eye disease the following


symptoms/signs should indicate the need for urgent review by an
ophthalmologist (see EUGOGO guidelines):
• unexplained deterioration in vision
• awareness of change in intensity or quality of colour vision in one
or both eyes
• history of eye suddenly 'popping out' (globe subluxation)
• obvious corneal opacity
• cornea still visible when the eyelids are closed
• disc swelling

Next question )

BI !!f A~
Q23

A 53-year-old man with a history of type 2 diabetes mellitus is reviewed


in the diabetes clinic. Twelve months ago his HbA1 c was 9 .7% despite
maximal oral hypoglycaemic therapy. Insulin was started and his most
recent HbA1c is 8.2%. He is considering applying for a HGV licence and
asks for advice. What is the most appropriate advice?

He cannot drive a heavy goods vehicle if he is taking insulin

He may be able to apply for a HGV licence if he meets strict


criteria relating to hypoglycaemia

He should stop insulin and start meglitinide

As under 55 years of age there is no requirement to inform the


DVLA

He needs to have been stable on insulin for at least 5 years


before applying

Submit answer

Reference ranges v
He may be able to apply for a HGV licence if he meets strict criteria
relating to hypoglycaemia

He should stop insulin and start meglitinide

As under 55 years of age there is no requirement to inform the DVLA

He needs to have been stable on insulin for at least 5 years before


applying

Patients on insulin may now hold a HGV licence if they meet strict
DVLA criteria
Importance: 50

116 11
f • Discuss (2) Improve

Next question )

DVLA: diabetes mellitus

Until recently people with diabetes who used insulin could not hold a
HGV licence. The DVLA changed the rules in October 2011. The
following standards need to be met (and also apply to patients using
other hypoglycaemic inducing drugs such as sulfonylureas):
• there has not been any severe hypoglycaemic event in the
previous 12 months
• the driver has full hypoglycaemic awareness
• the driver must show adequate control of the condition by regular
blood glucose monitoring*, at least twice daily and at times
relevant to driving
• the driver must demonstrate an understanding of the risks of
hypoglycaemia
• here are no other debarring complications of diabetes

From a practical point of view patients on insulin who want to apply for
a Group 2 (HGV) licence need to complete a VDIAB1 I form.

Other specific points for group 1 drivers:


• if on insulin then patient can drive a car as long as they have
hypoglycaemic awareness, not more than one episode of
hypoglycaemia requiring the assistance of another person within
the preceding 12 months and no relevant visual impairment.
Drivers are normally contacted by DVLA
• if on tablets or exenatide no need to notify DVLA. If tablets may
induce hypoglycaemia (e.g. sulfonylureas) then there must not
have been more than one episode of hypoglycaemia requiring the
assistance of another person within the preceding 12 months
• if diet controlled alone then no requirement to inform DVLA

*to demonstrate adequate control, the Secretary of State's Honorary


Medical Advisory Panel on Diabetes Mellitus has recommended that
applicants will need to have used blood glucose meters with a memory
function to measure and record blood glucose levels for at least 3
months prior to submitting their application

Next question )

Save my not es

Search
Q24

Which one of the following types of oral steroid has the least amount of
mineralocorticoid activity?

Fludrocortisone

Hydrocortisone

Dexamethasone

Prednisolone

Cortisone

Submit answer

Reference ranges v
Dexamethasone

Prednisolone

Cortisone

This is clinically relevant as there are some situations where it is


important to combine high glucocorticoid (anti-inflammatory) activity
with minimal mineralocorticoid (fluid-retention) effects. A good
example is the use of dexamethsone for patients with raised
intracranial pressure secondary to brain tumours .

116 11
f • Discuss Improve

Next question )

Corticosteroids

Corticosteroids are amongst the most commonly prescribed therapies


in clinical practice. They are used both systemically (oral or
intravenous) or locally (skin creams, inhalers, eye drops, intra-articular).
They augment and in some cases replace the natural glucocorticoid
and mineralocorticoid activity of endogenous steroids.

The relative glucocorticoid and mineralocorticoid activity of commonly


used steroids is shown below:
The relative glucocorticoid and mineralocorticoid activity of commonly used steroids is shown
below:

Minimal Predominant Very high


glucocorticoid Glucocorticoid glucocorticoid glucocorticoid
activity, very high activity, high activity, low activity, minimal
mineralocorticoid mineralocorticoid mineralocorticoid mineralocorticoid
activity, activity, activity activity

Fludrocortisone Hydrocortisone Prednisolone Dexamethasone


Betmethasone

Side-effects

The side-effects of corticosteroids are numerous and represent the single greatest limitation on
their usage. Side-effects are more common with systemic and prolonged therapy.

Glucocorticoid side-effects
• endocrine: impaired glucose regulation, increased appetite/weight gain, hirsutism,
hyperlipidaemia

Cam Scanner -, I.&~ a.:..~ I


Glucocorticoid side-effects
• endocrine: impaired glucose regulation, increased appetite/weight
gain, hirsutism, hyperlipidaemia
• Cushing's syndrome: moon face, buffalo hump, striae
• musculoskeletal: osteoporosis, proximal myopathy, avascular
necrosis of the femoral head
• immunosuppression: increased susceptibility to severe infection,
reactivation of tuberculosis
• psychiatric: insomnia, mania, depression, psychosis
• gastrointestinal: peptic ulceration, acute pancreatitis
• ophthalmic: glaucoma, cataracts
• suppression of growth in children
• intracranial hypertension
• neutrophilia

Mineralocorticoid side-effects
• fluid retention
• hypertension

Selected points on the use of corticosteroids:


• patients on long-term steroids should have their doses doubled
during intercurrent illness
• the BNF suggests gradual withdrawal of systemic corticosteroids
if patients have: received more than 40mg prednisolone daily for
more than one week, received more than 3 weeks treatment or
recently received repeated courses

Next question )

B / ~ ~
.t. l ,..
== 1-
i:
=..,.

Save my notes
Q25

A 45-year-old man with a history of depression and gastro-oesophageal


reflux disease presents due to a milky discharge from his nipples. The
following blood results are obtained :

Prolactin 700 mu/ I

Which one of his medications is most likely to be responsible?

Omeprazole

Fluoxetine

Metoclopramide

Cimetidine

Amitriptyline

Submit answer

Reference ranges v
Fluoxetine

Metoclopramide

Cimetidine

Amitriptyline

Causes of raised prolactin - the p's


• pregnancy
• prolactinoma
• physiological
• polycystic ovarian syndrome
• primary hypothyroidism
• phenothiazines, metoclopramide, domperidone

Importance: 50

Selective serotonin reuptake inhibitors such as fluoxetine have rarely


been associated with hyperprolactinaemia but the most likely cause in
this patient is metoclopramide. Cimetidine is generally associated with
gynaecomastia, rather than galactorrhoea.

"' If • Discuss (3) Improve

Next question )
Next question )

Prolactin and galactorrhoea

Prolactin is secreted by the anterior pituitary gland w ith release being


controlled by a wide variety of physiological factors . Dopamine acts as
the primary prolactin releasing inhibitory factor and hence dopamine
agonists such as bromocriptine may be used to control galactorrhoea.
It is important to differentiate the causes of galactorrhoea (due to the
actions of prolactin on breast tissue) from those of gynaecomastia

Features of excess prolactin


• men: impotence, loss of libido, galactorrhoea
• women: amenorrhoea, galactorrhoea

Causes of raised prolactin


• prolactinoma
• pregnancy
• oestrogens
• physiological: stress, exercise, sleep
• acromegaly: 1/3 of patients
• polycystic ovarian syndrome
• primary hypothyroidism (due to thyrotrophin releasing hormone
(TRH) stimulating prolactin release)

Drug causes of raised prolactin


• metoclopramide, domperidone
• phenothiazines
• haloperidol
• very rare: SSRls, opioids

Next question )

B / ~ A "" := ~= ="" T I"" !ffl "" ~ c-:,


Q26 [Cl

A 54-year-old woman who has had a hysterectomy presents for advice


about hormone replacement therapy. Which one of the follow ing would
result from the use of a combined oestrogen-progest ogen preparat ion
compared to an oestrogen-only preparation?

Decreased risk of venous thromboembolism

Increased risk of a stroke

Increased risk of breast cancer

Increased risk of endometrial cancer

Better control of symptoms

Submit answer

Reference ranges v
Increased risk of a stroke

Increased risk of breast cancer

Increased risk of endometrial cancer

Better control of symptoms

HRT: adding a progestogen increases the risk of breast cancer


Importance: 50

This is the rationale behind giving women who've had a hysterectomy


oestrogen-only treatment. The BNF states that the stroke risk is the
same regardless of whether the HRT preparation contains
progesterone.

116 •t • Discuss (2) Improve

Next question )

Hormone replacement therapy: adverse effects

Hormone replacement therapy {HRT) involves the use of a small dose


of oestrogen (combined with a progestogen in women with a uterus) to
help alleviate menopausal symptoms.

Side-effects
• nausea
• breasttenderness
• fluid retention and weight gain
Potential complications
• increased risk of breast cancer: increased by the addition of a
progestogen
• increased risk of endometrial cancer: reduced by the addition of a
progestogen but not eliminated completely. The BNF states that
the additional risk is eliminated if a progestogen is given
continuously
• increased risk of venous thromboembolism: increased by the
addition of a progestogen
• increased risk of stroke
• increased risk of ischaemic heart disease if taken more than 1 O
years after menopause

Breast cancer
• In the Women's Health Initiative (WH I) study there was a relative
risk of 1.26 at 5 years of developing breast cancer
• the increased risk relates to duration of use
• breast cancer incidence is higher in women using combined
preparations compared to oestrogen-only preparations
• the risk of breast cancer begins to decline when HRT is stopped
and by 5 years it reaches the same level as in women who have
never taken H RT

Next question )

Save my notes

Search
Q27

A 52-year-old woman who was diagnosed as having primary atrophic


hypothyroidism 12 months ago is reviewed following recent thyroid
function tests (TFTs):

TSH 12.5 mU/ 1

Free T4 14 pmol/ 1

She is currently taking 75mcg of levothyroxine once a day. How should


these results be interpreted?

Poor compl iance with medication

Her thyroxine dose needs to be increased

Evidence of recent systemic steroid therapy

She is on the correct dose

T 4 to T3 conversion disorder

Submit answer

Reference ranges v

Ca mSc.an,ner ~ '....,.i;lo-" ~ ~ I
A 52-year-old woman who was diagnosed as having primary atrophic
hypothyroidism 12 months ago is reviewed following recent thyroid
function tests (TFTs):

TSH 1 2. 5 mU/ 1

Free T4 14 pmol/1

She is currently taking 75mcg of levothyroxine once a day. How should


these results be interpreted?

I Poor compliance with medication

Her thyroxine dose needs to be increased

Evidence of recent systemic steroid therapy

She is on the correct dose

T 4 to T3 conversion disorder

The TSH level is high. This implies that over recent days/ weeks her
body is thyroxine deficient. However, her free T 4 is within normal range.
The most likely explanation is that she started taking the thyroxine
properly just before the blood test. This would correct the thyroxine
level but the TSH takes longer to norma lise.

16 11, tl Discuss (5) Improve

Next question )
Thyroid function tests

The interpretation of thyroid function tests is usually straightforward:

Free
Diagnosis TSH T4 Notes

Thyrotoxicosis (e.g. Low High In T3


Graves' disease) thyrotoxicosis
the free T 4 will
be normal

Primary hypothyroidism High Low


(primary atrophic
hypothyroidism)

Secondary hypothyroidism Low Low Replacement


steroid therapy
is required prior
to thyroxine

Sick euthyroid syndrome* Low** Low Common in


hospital
inpatients
T3 is particularly
low in these
patients

Subclinical hypothyroidism High Normal

Poor compliance with High Normal


thyroxine

Steroid therapy Low Normal


Hypothyroid ism Hyperthyrodfsm

H s.h1moto's

Rt
--
th~ro;dr IS

ri lhyrotd1US
Subacule lh~ ro1d I.is
(de Ou rv n's)

Amx>darone
Graves' a,s as.e

Toxic mu I nodular gc)llre


Postpartum thyroktrtis
lodme defic1 nc ,.,...~tt.- -
Lllhlum

Venn diagram showing how different causes of thyroid


dysfunction may manifest. Note how many causes of
hypothyroidism may have an initial thyrotoxic phase.

*now referred to as non-thyroidal illness


**TSH may be normal in some cases

Next question )

B / l!J

Save my notes
Q28

A 20-year-old man presents with a nine month history of weight gain.


Prior to this he was of a normal weight and cannot identify any obvious
lifestyle changes that would account for his obesity. On examination he
is noted to have abdominal striae and a degree of proximal myopathy.
Blood pressure is 130/80 mmHg. Bloods show the following:

Na+ 141 mmol/1

K+ 3.3 mmol/1

Bicarbonate 26 mmol/1

Urea 3.3 mmol/1

Creatinine 72 µmol/ 1

What is the most appropriate next test?

High-dose dexamethasone suppression test

Plasma ACTH

Short ACTH test

24 hour urinary free cortisol

Renin:aldosterone ratio

Submit answer

Reference ranges v
Plasma ACTH

Short ACTH test

24 hour urinary free cortisol

Renin:aldosterone ratio

The overnight dexamethasone suppression test is the best test to


diagnosis Cushing's syndrome
Importance: 50

There is some debate as to whether a 24 hour urinary free cortisol or an


overnight dexamethasone suppression test should be used to screen
patients for Cushing's. The overnight (not high-dose) dexamethasone
suppression test has however been shown to be more sensitive and is
now much more commonly used in clinical practice. As this is not
offered then 24 hour urinary free cortisol is the next best answer

For a review comparing diagnostic methods see 'Specificity of first-line


tests for the diagnosis of Cushing's syndrome: assessment in a large
series, J Clin Endocrinol Metab. 2007 Nov;92(11 ):4123-9'

The high-dose dexamethasone suppression test is used to help


differentiate the cause of Cushing's syndrome

[ ,6 ,. - Discuss (9) Improve ]

Next question )

Ca mSc.an,ner ~ '....,.i;lo-" ~ ~ I
Q29

A 44-year-old woman presents to her GP as she is feeling 'hot an the


t ime' and is consequently worried she is going through an early
menopause. Her husband has also noticed a 'fullness' of her neck
which has become apparent over the past few weeks. On examination
her pulse is 90/minute and she has a small, non-tender goitre. Blood
tests are arranged:

TSH < 0.05 mu/I

Free T4 24 pmol/I

Anti-thyroid peroxidase antibodies 102 IU/ml (< 35 IU/ml)

ESR 23 mm/hr

What is the most likely diagnosis?

Hashimoto's thyroiditis

Toxic multinodular goitre

Thyroid cancer

De Quervain's thyroiditis

Graves' disease

I
Submit answer
Q29

A 44-year-old woman presents to her GP as she is feeling 'hot all the


time' and is consequently worried she is going through an early
menopause. Her husband has also noticed a 'fullness' of her neck
which has become apparent over the past few weeks. On examination
her pulse is 90/minute and she has a small, non-tender goitre. Blood
tests are arranged:

TSH < 0.05 mu/ I

Free T4 24 pmol/1

Anti-thyroid peroxidase antibodies 102 IU/ ml (< 35 IU/ml )

ESR 23 mm/ hr

What is the most likely diagnosis?

Hashimoto's thyroiditis

Toxic multinodular goitre

Thyroid cancer

De Quervain's thyroiditis

Graves' disease

Submit answer

Reference ranges v
Toxic multinodular goitre

Thyroid cancer

De Quervain's thyroiditis

Graves' disease

The thyrotoxic symptoms and blood tests, goitre and anti-thyroid


peroxidase antibodies suggest a diagnosis of Graves' disease.

Whilst anti-thyroid peroxidase antibodies are seen in 90% of


Hashimoto's disease they are also seen in 75% of patients with Graves'
disease. Hashimoto's thyroiditis is also generally associated with
hypothyroidism, which is not in keeping with this presentation.

The ESR result is within normal range.

11• at • Discuss (6) Improve

Next question )
Next question )

Graves' disease: features

Graves' disease is the most common cause of thyrotoxicosis. It is


typically seen in women aged 30-50 years.

Features
• typica l features of thyrotoxicosis
• specific signs limited to Grave's (see below)

Features seen in Graves' but not in other causes of thyrotoxicosis


• eye signs (30% of patients)
o exophthalmos
o ophthalmoplegia
• pretibial myxoedema
• thyroid acropachy, a triad of:
o digital clubbing
o soft tissue swelling of the hands and feet
o periosteal new bone formation

Autoantibodies
• TSH receptor stimulating antibodies (90%)
• anti-thyroid peroxidase antibodies (75%)

Next question )

B / f!!j
Q30 Jc:::1

Which one of the following is most likely to be found in a patient with


Hashimoto's thyroiditis?

Raised ESR

Anti-TSH receptor stimulating antibodies

Anti-thyroid peroxidase antibodies

Decreased TSH

Co-existing type 2 diabetes mellitus

Submit answer

Reference ranges v
Anti-TSH receptor stimulating antibodies

Anti-thyroid peroxidase antibodies

Decreased TSH

Co-existing type 2 diabetes mellitus

Hashimoto's thyroiditis =hypothyroidism+ goitre+ anti-TPO


Importance: 50

"' .. t9 Discuss (1) Improve ]

Next question )

CamScanne, -? Y.¥-> i.b . ~I


Next question )

Hashimoto's thyroiditis

Hashimoto's thyroiditis (chronic autoimmune thyroiditis) is an


autoimmune disorder of the thyroid gland. It is typically associated with
hypothyroidism although there may be a transient thyrotoxicosis in the
acute phase. It is 1 Otimes more common in women

Features
• features of hypothyroidism
• goitre: firm, non-tender
• anti-thyroid peroxidase and also anti-Tg antibodies

Hypothyroidism Hyperthyrodlsm

Hashrmoto's
thyroidius &lbacute thyroid tis
(da Querva:n's) Graves· disease
0.

Anuodarone lox1c mu t nodular goitre


Riedel's thyroid1 s
Postpartum thyrotd,tis

..

Venn diagram showing how different causes of thyroid


dysfunction may manifest. Note how many causes of
hypothyroidism may have an initial thyrotoxic phase.

Next question )
Q1

You review a 68-year-old man who has chron ic obst ructive pulmonary
disease (COPD). Each year he typically has around 7-8 courses of ora l
prednisolone to treat infective exacerbations of his COPD. Which one of
the following adverse effects is linked to long-term steroid use?

Osteomalacia

Enophthalmos

Leucopaenia

Avascular necrosis

Constipation

Submit answer

Reference ranges v
Enophthalmos

Leucopaenia

Avascular necrosis

Constipation

Long-term corticosteroid use is linked to osteopaenia and osteoporosis,


rather than osteomalacia.

[ "' If • Discuss Improve ]

Next question )

r,. - _ ... : - - - .a. - - - : J -

Ca mSc.an,ner ~ '....,.i;lo-" ~ ~ I
Q2

A 75-year-old woman with a history of hypothyoidism is admitted to the


Emergency Department following an episode of chest pain. She is
diagnosed as having an acute coronary syndrome and iron-deficiency
anaemia. A percutaneous coronary intervention is performed and a
coronary artery stent is inserted. Endoscopies of the upper and lower
gastrointestinal tract are performed and reported as normal. She is
discharged on the following drugs in addition to her regular
levothyroxine: aspirin, clopidogrel, ramipril, lansoprazole, simvastatin
and ferrous sulphate. Six weeks later she complains of feeling tired all
the time. Her GP arranges some routine blood tests:

Hb 11 .9 g/ dl

Platelets 1 55 * 1o9/ I

WBC 5.2 * 109 / 1

Free T 4 8.1 pmol/ 1

TSH 8.2 mu/ I

Prior to her recent admission the TSH has been within range for the
past two years. Which one of the following new drugs most likely
explains the raised TSH?

Simvastatin

Clopidogrel

Ferrous sulphate

Ramipril

Lansoprazole
Clopidogrel

Ferrous sulphate

Ramipril

Lansoprazole

Iron reduces the absorption of thyroxine


Importance: 50

[ ,t •t • Discuss (3) Improve ]

Next question )

I I _ __ _ _ _._ 1 _ _ • I•
Hypothyroidism: management

Key points
• initial starting dose of levothyroxine should be lower in elderly
patients and those with ischaemic heart disease. The BNF
recommends that for patients with cardiac disease, severe
hypothyroidism or patients over 50 years the initial starting dose
should be 25mcg od with dose slowly titrated. Other patients
should be started on a dose of 50-1 00mcg od
• following a change in thyroxine dose thyroid function tests should
be checked after 8-1 2 weeks
• the therapeutic goal is 'normalisation' of the thyroid stimulating
hormone (TSH) level. As the majority of unaffected people have a
TSH value 0.5-2.5 mU/I it is now thought preferable to aim for a
TSH in this range
• women with established hypothyroidism who become pregnant
should have their dose increased 'by at least 25-50 micrograms
levothyroxine'* due to the increased demands of pregnancy. The
TSH should be monitored carefully, aiming for a low-normal value
• there is no evidence to support combination therapy with
levothyroxine and liothyronine

Side-effects of thyroxine therapy


• hyperthyroidism: due to over treatment
• reduced bone mineral density
• worsening of angina
• atrial fibrillation

Interactions
• iron, calcium carbonate
o absorption of levothyroxine reduced, give at least 4 hours
apart
Q3 fc::J

A 42-year-old man presents to the emergency department complaining


of long-standing muscle weakness and headaches. His blood pressure
is 176/78 mmHg, but all other observations are within normal limits.
Examination is unremarkable.

Blood results are as follows:

Na+ 145 mmol/L (135 - 145)

K+ 2.8 mmol/L (3.5- 5.0)

Bicarbonate 34 mmol/L (22 - 29)

Urea 4 mmol/L (2.0 - 7.0)

Creatinine 72 µmol/L (55 - 120)

What is the most appropriate first-li ne investigation?

Adrenal vein sampling

CTabdomen

Dexamethasone suppression test

Plasma aldosterone/ renin ratio

Short synacthen test

Submit answer

Reference ranges v
CTabdomen

Dexamethasone suppression test

Plasma aldosterone/ renin ratio

Short synacthen test

A plasma aldosterone/renin ratio is the first-line investigation in


suspected primary hyperaldosteronism
Importance: 50

Features of hypokalaemia coupled with hypertension are suggestive of


primary hyperaldosteronism. This can be caused by an adrenal
adenoma (Conn's syndrome) or bilateral idiopathic adrenal hyperplasia.
The first line investigation for this is a plasma aldosterone/renin ratio,
which should show high aldosterone levels alongside low renin levels.

Adrenal vein sampling and CT abdomen can be useful in differentiating


between the different causes of primary hyperaldosteronism, but would
not be appropriate first-line investigations.

The short synacthen test is the first-line investigation for adrenal


insufficiency. This would typically present with hyponatraemia and
hypotension.

Dexamethasone suppression tests are useful in the investigation


adrenal gland function and is typically used to investigate for Cushing's
disease.

a6 f
11 tt Discuss (3) Improve

Next question )

CamScanner -! W~ :.i.;,-~\
Primary hyperaldosteronism

Primary hyperaldosteronism was previously thought to be most


commonly caused by an adrenal adenoma, termed Conn's syndrome.
However, recent studies have shown that bilateral idiopathic adrenal
hyperplasia is the cause in up to 70% of cases. Differentiating between
the two is important as this determines treatment. Adrenal carcinoma is
an extremely rare cause of primary hyperaldosteronism.

Features
• hypertension
• hypokalaemia
o e.g. muscle weakness
o this is a classical feature in exams but studies suggest this
is seen in only 10-40% of patients
• alkalosis

Investigations
• the 2016 Endocrine Society recommend that a plasma
aldosterone/renin ratio is the first-line investigation in suspected
primary hyperaldosteronism
o should show high aldosterone levels alongside low renin
levels (negative feedback due to sodium retention from
aldosterone)
• following this a high-resolution CT abdomen and adrenal vein
sampling is used to differentiate between unilateral and bilateral
sources of aldosterone excess
• Adrenal Venous Sampling (AVS) can be done to identify the gland
secreting excess hormone in primary hyperaldosteronism

Management
• adrenal adenoma: surgery
• bilateral adrenocortical hyperplasia: aldosterone antagonist e.g.
spironolactone
Q4

Which one of the following increases the risk of developing peripheral


oedema in a patient taking pioglitazone?

Concomitant use with gliclazide

Serum sodium < 140 mmol/1

Concomitant use with insulin

Concomi1ant use with metformin

Serum potassium< 4.0 mmol/1

Submit answer

Reference ranges v
I Concomitant use with insulin

Concomitant use with metformin

Serum potassium < 4.0 mmol/1

•• •• • Discuss (5) Improve

Next question )

Thiazolidinediones

Thiazol idinediones are a class of agents used in the treatment of type 2


diabetes mellitus. They are agonists to the PPAR-gamma receptor and
reduce peripheral insulin resistance. Rosiglitazone was withdrawn in
201 0 following concerns about the cardiovascular side-effect profile.

The PPAR-gamma receptor is an intracellular nuclear receptor. It's


natural ligands are free fatty acids and it is t hought to control adipocyte
differentiation and function.

Adverse effects
• weight gain
• liver impairment: monitor LFTs
• fluid retention - therefore contraindicated in heart failu re. The risk
of fluid retention is increased if the patient also takes insulin
• recent studies have indicated an increased risk of fractures
• bladder cancer: recent studies have shown an increased risk of
bladder cancer in patients taking pioglitazone (hazard ratio 2.64)

Next question )
QS

A 67-year-old man who has a history of type 2 diabetes mellitus and


benign prostatic hypertrophy presents with burning pain in his feet. This
has been present for the past few months and is getting gradually
worse. He has tried taking duloxetine but unfortunately has received no
benefit. Clinical examination is unremarkable other than diminished
sensation to fine touch on both soles. What is the most suitable initial
management?

Carbamazepine

Amitriptyline

Pregabalin

Fluoxetine

Sodium valproate

Submit answer

Reference ranges v
Pregabalin

Fluoxetine

Sodium valproate

Amitriptyllne would normally be first choice but given his history of


benign prostatic hyperplasia it is better to avoid amitriptyline due to the
risk of urinary retention.

•• •• • Discuss (1) Improve

Next question )

Diabetic neuropathy

Diabetes typically leads to sensory loss and not motor loss in peripheral
neuropathy. Painful diabetic neuropathy is a common problem in
cliniical practice.

NICE updated it's guidance on the management of neuropathic pain in


2013. Diabetic neuropathy is now managed in the same way as other
forms of neuropathic pain:
• first-line treatment: amitriptyline, duloxetine, gabapentin or
pregabalin
• if the first-line drug treatment does not work try one of the other 3
drugs
• tramadol may be used as 'rescue therapy' for exacerbations of
neuropathic pain
• topical capsaicin may be used for localised neuropathic pain (e.g.
post-herpetic neuralgia)
• pain management clinics may be useful in patients with resistant
problems
Gastrointestinal autonomic neuropathy

Gastroparesis
• symptoms include erratic blood glucose control, bloating and
vomiting
• management options include metoclopramide, domperidone or
erythromycin (prokinetic agents)

Chronic diarrhoea
• often occurs at night

Gastro-oesophageal reflux disease


• caused by decreased lower esophageal sphincter (LES) pressure

Next question )

B I l!J A y
·- 1-
~
-- y
T IY lffl y

Save my notes

Search

Search textbook ...



Q6

A 45-year-old man is investigated following referra l to the endocrinology


clinic with polydipsia. Plasma glucose and calcium are normal. A water
deprivation test is performed with the following results:

Starting plasma osm. 319 mOsmol/ 1(275-295 mOsmol/ 1)

Final urine osm. 142 mOsmol/1

Urine osm. post-DOAVP 885 mOsmol/ 1

What is the most likely diagnosis?

Psychogenic polydipsia

Nephrogenic diabetes insipidus

Primary hyperparathyroidism

Pseudohypoparathyroidism

Cranial diabetes insipidus

Submit answer

Reference ranges v
I Cranial diabetes insipid_u_s _ _ _ _ ~
A dramatic improvement is seen in the ability of the kidneys to
concentrate urine following the administration of DDAVP. This points
towards a diagnosis of cranial diabetes insipidus

"' •• tt Discuss (3) Improve

Next question )

Water deprivation test

Method
• prevent patient drinking water
• ask patient to empty bladder
• hourly urine and plasma osmolalities

Final
Starting urme Urine osm.
plasma osm. osm. post-DDAVP

Normal Normal > 600 > 600

Psychogenic Low > 400 > 400


polydipsia

Cranial DI High < 300 > 600

Nephrogenic DI High < 300 < 300

Next question )
Q7

An 24-year-old woman is reviewed due to facial hirsutism. You suspect


a diagnosis of polycystic ovarian syndrome (PCOS). Which one of the
following features would suggest the need for further investigations
before confidently making a diagnosis of PCOS?

Clitoromegaly

Acanthosis nigricans

Obesity

Amenorrhoea

Acne

Submit answer

Reference ranges v
Q7

An 24-year-old woman is reviewed due to facial hirsutism. You suspect


a diagnosis of polycystic ovarian syndrome (PCOS). Which one of the
following features would suggest the need for further investigations
before confidently making a diagnosis of PCOS?

I Clitoromegaly
---------

Acanthosis nigricans

Obesity

Amenorrhoea

Acne

Clitoromegaly is seen occasionally in PCOS but is normally associated


with very high androgen levels. If clitoromegaly is found then further
investigations to exclude an ovarian or adrenal androgen secreting
tumour are required.

[ ,, .. t9 Discuss (2) Improve ]


• f
1 • Discuss (2) Improve

Next question )

Polycystic ovarian syndrome: features and


investigation

Polycystic ovary syndrome (PCOS) is a complex condition of ovarian


dysfunction thought to affect between 5-20% of women of reproductive
age. The aetiology of PCOS is not fully understood. Both
hyperinsulinaemia and high levels of luteinizing hormone are seen in
PCOS and there appears to be some overlap with the metabolic
syndrome.

Features
• subfertility and infertility
• menstrual disturbances: oligomenorrhea and amenorrhoea
• hirsutism, acne (due to hyperandrogenism)
• obesity
• acanthosis nigricans (due to insulin resistance)

Investigations
• pelvic ultrasound: multiple cysts on the ovaries
• FSH, LH, prolactin, TSH, and testosterone are useful
investigations: raised LH:FSH ratio is a 'classical' feature but is no
longer thought to be useful in diagnosis. Prolactin may be normal
or mildly elevated. Testosterone may be normal or mildly elevated
- however, if markedly raised consider other causes
• check for impaired glucose tolerance

Next question )
'
QB

A 66-year-old man attends clinic for review of his diabetic control. He


has a history of type 2 diabetes mellitus, and was commenced on
metformin six months' ago. The patient's HbA1c today is 60mmol/ mol.
You discuss the patient with your consultant, who recommends
addition of empagliflozin for glycaemic control.

What is the mechanism of action of this medication?

Biguanide

Sulfonylurea

GLP-1 mimetic

SGLT-2 inhibitor

DPP-4 inhibitor

Submit answer

Reference ranges v
SGLT-2 inhibitor

DPP-4 inhibitor

SGLT-2 inhibitors reversibly inhibit sodium-glucose co-transporter


2 (SGLT-2) in the renal proximal convoluted tubule
Importance: 50

SGLT-2 inhibitors reversibly inhibit sodium-glucose co-transporter 2


(SGLT-2) in the renal proximal convoluted tubule. In doing so, they
reduce glucose reabsorption and increase urinary glucose excretion.
This mechanism explains their main side effects - increased urine
output, weight loss and urinary infections. Examples include
canagliflozin, dapagliflozin and empagliflozin.

SGLT-2 inhibitors are often the best option in patients with ischaemic
heart disease and heart failure who have not achieved adequate
glycaemic control on metformin alone.

Important adverse effects include:


• Urinary and genital infection (secondary to glycosuria).
• Normoglycaemic ketoacidosis
• Increased risk of lower-limb amputation: feet should be closely
monitored

Biguanides (such as metformin) act to decrease gluconeogenesis in the


liver and increase insulin sensitivity.

Sulfonylureas (such as gliclazide) act to increase insulin release from


beta-cells in the pancreas.

GLP-1 mimetics (such as exenatide) mimic incretin which is usually


released in the gastrointestinal tract, and has the effect of increasing
insulin production.
DPP-4 inhibitors (such as sitagliptin) act to block the action of DPP-4,
an enzyme which breaks down incretin.

11• If tt Discuss Improve

Next question )

SGLT-2 inhibitors

SGLT-2 inhibitors reversibly inhibit sodium-glucose co-transporter 2


(SGLT-2) in the renal proximal convoluted tubule to reduce glucose
reabsorption and increase urinary glucose excretion.

Examples include canagliflozin, dapagliflozin and empagliflozin.

Important adverse effects include


• urinary and genital infection (secondary to glycosuria). Fournier's
gangrene has also been reported
• normoglycaemic ketoacidosis
• increased risk of lower-limb amputation: feet should be closely
monitored

Patients taking SGLT-2 drugs often lose weight, which can be beneficial
in type 2 diabetes mellitus.

Next question )

B I tj A • ·- J= =• T I• ~• ~ c-::>

Save my notes
Q9

A 29-year-female, who is 4 months post-partum, presents with a 3-week


history of weight loss, heat intolerance, tremor, palpitation and
diarrhoea. Pregnancy and birth were uncomplicated. On further
questioning, she admits having taken off-license weight loss
medication bought from the internet 1 month ago. Past medical history
and family history are insignificant. She does not smoke or drink
alcohol.

On physical examination, she has exophthalmos, brisk reflexes and fine


tremor. Her vital signs were heart rate 98/minute, blood pressure
136/76 mmHg, temperature 36.5°C. The thyroid gland was diffusely
enlarged.

Thyroid Stimulating Hormone (TSH) 0.02 mU/1

Free thyroxine (T4) 24 pmol/1

Total thyroxine (T4) 1 SO nmol/1

What is the most likely diagnosis?

Exogenous thyroxine

Graves' Disease

Hashimoto's thyroiditis

Post-partum thyroiditis

De Quervain's thyroiditis

Submit answer
I Graves' Disease

Hashimoto's thyroiditis

Post-partum thyroiditis

De Quervain's thyroiditis

Graves' disease may present first or become worse during the


post-natal period
Importance: 50

This is a case of hyperthyroidism due to Graves' disease. Graves'


disease may manifest itself or worsen during pregnancy and the post-
natal period.

Exophthalmos is a specific sign seen in Graves' disease and not in other


hyperthyroid conditions.

Hashimoto's thyroiditis leads to hypothyroidism.

In post-partum thyroiditis, the woman initially develops hyperthyroidism


immediately after birth followed by normal or sometimes decreased
thyroid levels.

De Quervain's thyroiditis can present with pain and dysphagia and may
lead to high, normal or low thyroid levels.

[ ,. •• • Discuss (4) Improve ]

Next question )
l 'f CJ\l \.{ UC.:>l l UI I r

Pregnancy: thyroid problems

In pregnancy, there is an increase in the levels of thyroxine-binding


globulin (TBG). This causes an increase in the levels of total thyroxine
but does not affect the free thyroxine level.

Thyrotoxicosis

Untreated thyrotoxicosis increases the risk of feta! loss, maternal heart


failure and premature labour

Graves' disease is the most common cause of thyrotoxicosis in


pregnancy. It is also recognised that activation of the TSH receptor by
HCG may also occur - often termed transient gestational
hyperthyroidism. HCG levels will fall in the second and third trimester

Management
• propylthiouracil has traditionally been the antithyroid drug of
choice
• however, propylthiouracil is associated with an increased risk of
severe hepatic injury
• therefore NICE Clinical Knowledge Summaries advocate the
following: 'Propylthiouracil is used in the first trimester of pregnancy
in place of carbimazo/e, as the latter drug may be associated with
an increased risk of congenital abnormalities. At the beginning of
the second trimester, the woman should be switched back to
carbimazole'
• maternal free thyroxine level s should be kept in the upper third of
the normal reference range to avoid fetal hypothyroidism
• thyrotrophin receptor stimulating antibodies should be checked at
30-36 weeks gestation - helps to determine the risk of neonatal
thyroid problems
• block-and-replace regimes should not be used in pregnancy
• radioiodine therapy is contraindicated
Hypothyroid ism

Key points
• thyroxine is safe during pregnancy
• serum thyroid stimulating hormone measured in each trimester
and 6-8 weeks post-partum
• some women require an increased dose of thyroxine during
pregnancy
• breast feeding is safe whilst on thyroxine

Next question )

B / !!! T 1-
~ -
-T T IT

Save my notes

Search


Search textbook...
---- ·
0. Google search on "Pregnancy: thyroid problems"
Q10

A 72-year-old male is seen in clinic complaining of pain in both feet. He


reports the pain is burning in nature and worse at night when sitting
watching TV. It can often be severe and he is struggling to manage this
with paracetamol alone. He has a past medical history of poorly
controlled diabetes, hypertension and high cholesterol. On examination,
he has normal power bilaterally but reduced sensation. Pedal pulses are
present and there are no vascular related skin changes.

What is the most appropriate first line treatment for his pain?

Tramadol

Topical capsaicin

Codeine

Duloxetine

Naproxen

Submit answer

Reference ranges v
Topical capsaicin

Codeine

Duloxetine

Naproxen

First line treatment in diabetic neuropathy is with amitriptyline,


duloxetine, gabapentin or pregabalin
Importance: 50

The correct answer is duloxetine. This patient has a history of poorly


controlled diabetes and combined with the examination findings of
reduced sensation and normal pulses a diagnosis of diabetic
neuropathy is most likely. NICE guidelines suggest first-line treatment
for diabetic neuropathy is the same as for other causes of neuropathic
pain with amitriptyline, duloxetine, gabapentin or pregabalin.

Topical capsaicin can be used for localised neuropathic pain such as


postherpetic neuralgia but would not be first-line for this patient.

Tramadol is also not the first line treatment but can be used as rescue
therapy for exacerbations.

Neither codeine or naproxen are recommended in neuropathic pain.

a6 Rf t9 Discuss Improve

Next question >


Diabetic neuropathy

Diabetes typically leads to sensory loss and not motor loss in peripheral
neuropathy. Painful diabetic neuropathy is a common problem in
clinical practice.

NICE updated it's guidance on the management of neuropathic pain in


2013. Diabetic neuropathy is now managed in the same way as other
forms of neuropathic pain:
• first-line treatment: amitriptyline, duloxetine, gabapentin or
pregabalin
• if the first-line drug treatment does not work try one of the other 3
drugs
• tramadol may be used as 'rescue therapy' for exacerbations of
neuropathic pain
• topical capsaicin may be used for localised neuropathic pain (e.g.
post-herpetic neuralgia)
• pain management clinics may be useful in patients with resistant
problems

Gastrointestinal autonomic neuropathy

Gastroparesis
• symptoms include erratic blood glucose control, bloating and
vomiting
• management options include metoclopramide, domperidone or
erythromycin (prokinetic agents)

Chronic diarrhoea
• often occurs at night

Gastro-oesophageal reflux disease


• caused by decreased lower esophageal sphincter (LES) pressure
Q11

A 41-year-old woman presents with palpitations and heat intolerance.


On examination her pulse is 90/min and a small, diffuse goitre is noted
which is tender to touch. Thyroid function tests show the following :

Free T 4 24 pmoVI

TSH < 0.05 mu/ I

What is the most likely diagnosis?

Grave's disease

Sick thyroid syndrome

De Quervain's thyroiditis

Hashimoto's thyroiditis

Toxic multinodular goitre

Submit answer

Reference ranges v
Sick thyroid syndrome

De Quervain's thyroiditis

Hashimoto's thyroiditis

Toxic multinodular goitre

Thyrotoxicosis with tender goitre= subacute (De Quervain's)


thyroiditis
Importance: 50

Whilst Grave's disease is the most common cause of thyrotoxicosis it


would not cause a tender goitre. In the context of thyrotoxicosis this
finding is only really seen in De Quervain's thyroiditis.

Hashimoto's thyroiditis is an autoimmune disorder of the thyroid gland.


It is typically associated with hypothyroidism although there may be a
transient thyrotoxicosis in the acute phase. The goitre is non-tender in
Hashimoto's.

116 •t - Discuss Improve

Next question >

Subacute (De Quervain's) thyroiditis


Subacute thyroiditis (also known as De Quervain's thyroiditis and
subacute granulomatous thyroiditis) is thought to occur following viral
infection and typically presents with hyperthyroidism.

There are typically 4 phases;


• phase 1 (lasts 3-6 weeks): hyperthyroidism, painful goitre, raised
ESR
• phase 2 (1-3 weeks) : euthyroid
• phase 3 (weeks - months): hypothyroidism
• phase 4: thyroid structure and function goes back to normal

Investigations
• thyroid scintigraphy: globally reduced uptake of iodine-131

Management
• usually self-limiting - most patients do not require treatment
• thyroid pain may respond to aspirin or other NSAIDs
• in more severe cases steroids are used, particularly if
hypothyroidism develops

Hypothyroidism .«yperthyrodlsm

H hlfflOto's
thyroid us
__
Subacute thyroid I.ls
(de Ouervam's) ..,

Anuodarone Tox c mull nodular 110«lre


R s 1hyroid1 tS
Postpartum lhynMditrs
lod~ de tciency ~ ··~·- · ..

L1th1Um

Venn diagram showing how different causes of thyroid


dysfunction may manifest. Note how many causes of
hypothyroidism may have an initial thyrotoxic phase.
Q12

A 31-year-old female is under investigation for Addison's disease after


presenting with feeling tired all the time, weight loss and postural
hypotension. A 9 am cortisol result has returned at 150 nmol/1.

What is the most appropriate interpretation of this result?

Inconclusive result - should be repeated

Confirms a diagnosis of Addison's disease

Confirms a diagnosis of Conn's disease

Inconclusive result - should be referred for short Synacthen test

Excludes a diagnosis of Addison's disease

Submit answer

Reference ranges v
Confirms a diagnosis of Conn's disease

Inconclusive result - should be referred for short Synacthen test

Excludes a diagnosis of Addison's disease

9 am cortisol between 100-SOOnmol/l is inconclusive and requires


further investigation with a short synacthen test
Importance: 50

The correct answer is that this is an inconclusive result and the patient
should be further investigated with a short Synacthen test. 9 am
cortisol results between 100-500nmol/l are inconclusive and patients
should be referred to secondary care for further investigation. A result
less than 1 oonmol/I makes Addison's disease or hypoadrenalism likely,
whilst a result >S00nmol/1 makes a diagnosis of Addison's disease very
unlikely.

Simply repeating the test would not be the correct answer unless there
was concern that the test was not taken at the correct time.

A result of 150nmol/l would not confirm a diagnosis of Addison's


disease. A result <1 00nmol/I would suggest a definite abnormality but
the definitive investigation is a short Synacthen test.

A 9 am cortisol of 1 S0nmol/I would not confirm a diagnosis of Conn's


syndrome. Whilst 9 am cortisol can be useful in the diagnosis of Conn's,
patients typically require plasma renin:aldosterone level.

A 9 am cortisol above S00nmol/ I not below would make a diagnosis of


Addison's very unlikely.

16 .. • Discuss (2) Improve

Next question )
Next question )

Addison's disease: investigations

In a patient with suspected Addison's disease the definite investigation


is an ACTH stimulation test (short Synacthen test) . Plasma cortisol is
measured before and 30 minutes after giving Synacthen 250ug IM .
Adrenal autoantibodies such as anti-21-hydroxylase may also be
demonstrated.

If an ACTH stimulation test is not readily available (e.g. in primary care)


then sending a 9 am serum cortisol can be useful:
• > 500 nmol/ I makes Addison's very unlikely
• < 100 nmol/ I is definitely abnormal
• 100-500 nmol/ I should prompt a ACTH stimu lation test to be
performed

Associated electrolyte abnormalities are seen in around one-third of


undiagnosed patients:
• hyperkalaemia
• hyponatraemia
• hypoglycaemia
• metabolic acidosis

Next question )

B / ~ T I•
Q13

A 43-year-old man is admitted to hospital with pneumonia. His past


medical history includes Addison's disease for which he takes
hydrocortisone (20mg in the mornings and 10mg in the afternoon).
What is the most appropriate action with respect to his steroid dose?

Continue to take the same dose

Double hydrocortisone to 40mg mornings and 20mg afternoon

Halve hydrocortisone to 10mg mornings and 5mg afternoon

Continue to take the same dose + prescribe a proton pump


inhibitor

Continue the same morning dose + stop the afternoon dose

Submit answer

Reference ranges v
I Double hydrocortisone to 40mg mornings and 20mg afternoon

Halve hydrocortisone to 10mg mornings and 5mg afternoon

Continue to take the same dose + prescribe a proton pump inhibitor

Continue the same morning dose + stop the afternoon dose

Patients on long-term steroids should have their doses doubled


during intercurrent illness
Importance: 50

" •t • Discuss (4) Improve ]

Next question )
Q14

A 34-year-old female with a history of Addison's disease presents for


review in endocrinology clinic. She is generally well but complains of a
decrease in her libido. On examination there is a slight loss of pubic
hair. What is the most likely cause?

Adverse effect of hydrocortisone therapy

11-hydroxylase deficiency

Diethylstilbestrol deficiency

Oestrogen deficiency

Dehydroepiandrosterone (DHEA) deficiency

Submit answer

Reference ranges v
11-hydroxylase deficiency

Diethylstilbestrol deficiency

Oestrogen deficiency

Dehydroepiandrosterone (DHEA) deficiency

Dehydroepiandrosterone is the most abundant circulating adrenal


steroid. Adrenal glands are the main source of dehydroepiandrosterone
in females - loss of functioning adrenal tissue as in Addison's disease
may result in symptoms secondary to androgen deficiency, such as loss
of libido. Research is ongoing as to whether routine replacement of
DHEA is beneficial

[ ,6 "f • Discuss (1) Improve ]

'

Next question )
1'4CJU ~UC~l l UI I r

Addison's disease

Autoimmune destruction of the adrenal glands is the commonest cause


of primary hypoadrenalism in the UK, accounting for 80% of cases. This
is termed Addison's disease and results in reduced cortisol and
aldosterone being produced.

Features
• lethargy, weakness, anorexia, nausea & vomiting, weight loss, 'salt-
craving'
• hyperpigmentation (especially palmar creases)*, vitiligo, loss of
pubic hair in women, hypotension, hypoglycaemia
• hyponatraemia and hyperkalaemia may be seen
• crisis: collapse, shock, pyrexia

Other causes of hypoadrenalism

Primary causes
• tuberculosis
• metastases (e.g. bronchial carcinoma)
• meningococcal septicaemia (Waterhouse-Friderichsen syndrome)
• HIV
• antiphospholipid syndrome

Secondary causes
• pituitary disorders (e.g. tumours, irradiation, infiltration)

Exogenous glucocorticoid therapy

*Primary Addison's is associated with hyperpigmentation whereas


secondary adrenal insufficiency is not

Next question )
0 Q15

A 51-year-old woman is reviewed in the diabetes clinic. She was


diagnosed with type 2 diabetes mellitus 12 months ago and still has
poor glycaemic control (63 mmol/mol). She has recently had to stop
taking gliclazide due to repeated episodes of hypoglycaemia and is only
taking maximum dose metformin. Her BMI is 26 kg/mA2. What is the
most appropriate next step in management?

Add either pioglitazone, a DPP-4 inhibitor or a SGLT-2 inhibitor

Refer her for a laparoscopic gastric band

Refer her for insulin therapy

Add either a thiazolidinedione or exenatide

Add either a DPP-4 inhibitor or exenatide

Submit answer

Reference ranges v
Add either pioglitazone, a DPP-4 inhibitor or a SGLT-2 inhibitor

Refer her for a laparoscopic gastric band

Refer her for insulin therapy

Add either a thiazolldinedione or exenatide

Add either a DPP-4 inhibitor or exenatide

a6 •t tt Discuss (5) Improve

Next question )

Diabetes mellitus: management of type 2

NICE updated its guidance on the management of type 2 diabetes


mellitus (T2DM) in 2015. Key points are listed below:
• HbA1c targets have changed. They are now dependent on what
antidiabetic drugs a patient is receiving and other factors such as
frailty
• there is more flexibility in the second stage of treating patients
(i.e. after metformin has been started) - you now have a choice of
4 oral antidiabetic agents

It's worthwhile thinking of the average patient who is taking


metformin for T2DM, you can titrate up metformin and encourage
lifestyle changes to aim for a HbA1c of 48 mmol/mol (6.5%), but
should only add a second drug if the HbA 1c rises to 58 mmol/mol
(7.5%)
Dietary advice
• encourage high fibre, low glycaemic index sources of
carbohydrates
• include low-fat dairy products and oily fish
• control the intake of foods containing saturated fats and trans
fatty acids
• limited substitution of sucrose-containing foods for other
carbohydrates is allowable, but care should be taken to avoid
excess energy intake
• discourage use of foods marketed specifically at people with
diabetes
• initial target weight loss in an overweight person is 5-10%

HbA1c targets

This is area which has changed in 201 5


• individual targets should be agreed with patients to encourage
motivation
• HbA 1c should be checked every 3-6 months until stable, then 6
monthly
• NICE encourage us to consider relaxing targets on 'a case-by-case
basis, with particular consideration for people who are older or frail,
for adults with type 2 diabetes'
• in 2015 the guidelines changed so HbA1 c targets are now
dependent on treatment:
Lifestyle or single drug treatment

HbA1c
Management of T2DM target

Lifestyle 48
mmol/mol
(6.5%)

Lifestyle+ metformin 48
mmol/mol
(6.5%)

Includes any drug which may cause hypoglycaemia 53


(e.g. lifestyle+ sulfonylurea) mmol/mol
(7.0%)

Practical examples
• a patient is newly diagnosed with HbA1 c and wants to try lifestyle
treatment first. You agree a target of 48 mmol/mol (6.5%)
• you review a patient 6 months after starting metformin. His
HbA 1c is 51 mmol/ mol (6.8%). You increase his metformin from
500mg bd to 500mg tds and reinforce lifestyle factors

Patient already on treatment

Management of T2DM HbA1c target

Already on one drug, but HbA1c has risen to 58 53 mmol/mol


mmol/ mol (7. 5%) (7.0%)
Drug treatment

The 2015 NICE guidelines introduced some changes into the


management of type 2 diabetes. There are essentially two pathways,
one for patients who can tolerate metformin, and one for those who
can't:

,..t1orm1n not tolerated or Cl

Tolerates metformin:
• metformin is still first-line and should be offered if the HbA 1c
rises to 48 mmol/mol (6.5%)* on lifestyle interventions
• if the HbA1c has risen to 58 mmol/mol (7.5%) then a second drug
should be added from the following list:
• - sulfonylurea
• - gliptin
• - pioglitazone
• - SGLT-2 inhibitor
• if despite this the HbA1c rises to, or remains above 58 mmol/ mol
(7.5%) then triple therapy with one of the following combinations
should be offered:
• - metformin + gliptin + sulfonylurea
• - metformin + pioglitazone + sulfonylurea
• - metformin + sulfonylurea + SGLT-2 inhibitor
• - metformin + pioglitazone + SGLT-2 inhibitor
• - OR insulin therapy should be considered

Criteria for glucagon-like peptide1 (GLP1) mimetic (e.g. exenatide)


• if triple therapy is not effective, not tolerated or contraindicated
then NICE advise that we consider combination therapy with
metformin, a sulfonylurea and a glucagonlike peptide1 (GLP1)
mimetic if:
• - BMI >= 35 kg/ m 2 and specific psychological or other medical
problems associated with obesity or
• - BMI < 35 kg/ m 2 and for whom insulin therapy wou ld have
significant occupational implications or

weight loss would benefit other significant obesityrelated comorbidities


• only continue if there is a reduction of at least 11 mmol/mol
[1 .0%] in HbA1c and a weight loss of at least 3% of initial body
weight in 6 months

Practical examples
• you review an established type 2 diabetic on maximum dose
metformin. Her HbA1 c Is 55 mmol/ mol (7.2%). You do not add
another drug as she has not reached the threshold of 58
mmol/mol (7.5%)
• a type 2 diabetic is found to have a HbA1c of 62 mmol/mol (7.8%)
at annual review. They are currently on maximum dose metformin.
You elect to add a sulfonylurea

CamScanne, -? Y.¥-> i.b.~I


Cannot tolerate metformin or contraindicated
• if the HbA1 c rises to 48 mmol/ mol (6.5%)* on lifestyle
interventions, consider one of the following:
• .... sulfonylurea
• .... gliptin
• .... pioglitazone
• if the HbA1 c has risen to 58 mmol/mol (7.5%) then a one of the
following combinations should be used:
• .... gliptin + pioglitazone
• .... gliptin + sulfonylurea
• -- pioglitazone + sulfonylurea
• if despite this the HbA1 c rises to, or remains above 58 mmol/mol
(7.5%) then consider insulin therapy

Starting insulin
• metformin should be continued. In terms of other drugs NICE
advice: 'Review the continued need for other blood glucose lowering
therapies'
• NICE recommend starting with human NPH insulin (isophane,
intermediate acting) taken at bed-time or twice daily according to
need

Risk factor modification

Blood pressure
• target is< 140/ 80 mmHg (or< 130/ 80 mmHg if end-organ
damage is present)
• ACE inhibitors are first-line

Antiplatelets
• should not be offered unless a patient has existing cardiovascular
disease
Lipids
• following the 2014 NICE lipid modification guidelines on ly patients
with a 10-year cardiovascular risk> 10% (using QRISK2) shou ld be
offered a statin. The first-line statin of choice is atorvastatin 20mg
on

Primary prevention Secondary prevention


10 yur CM:iiovntiar risk•• 10% OR nown IICMlffllC heawt OM-. OR
mott rp i clalletit1 OR ~ovnc-.._.OR
PII......,.,__,..._
CICDI~ ~

Atorva,tat111 20mg od
! non-HOL t1a1 not ~ by ,.,. , thin AtOfVHtat " IOmg od
c0f!Sidsr llhllllg up to 81lm;

Graphic showing choice of statin.

"1his is a bit confusing because isn't the diagnostic criteria for T2DM
HbA 1c 48 mmol/ mol (6.5%)? So shouldn't all patients be offered
metformin at diagnosis? Our interpretation of this is that some patients
upon diagnosis will elect to try lifestyle measures, which may reduce
their HbA1c below this level. If it then rises to the diagnostic threshold
again metformin shou ld be offered

Next question )

B I !!J A .... ·-
·- 2=
1- -- 'IP'
01

Which one of the following features is least common ly seen in


Gitelman's syndrome?

Hypokalaemia

Hypertension

Metabolic alkalosis

Hypocalciuria

Hypomagnesaemia

Submit answer

Reference ranges v
Hypertension

Metabolic alkalosis

Hypocalciuria

Hypomagnesaemia

Gitelman's syndrome: normotension, hypokalaemia +


hypocalciuria
Importance: 50

9' f
11
• Discuss (3) Improve

Next question >

Gitelman's syndrome

Gitelman's syndrome is due to a defect in the thiazide-sensitive Na+ Cl-


transporter in the distal convoluted tubule.

Features
• normotension
• hypokalaemia
• hypocalciuria
• hypomagnesaemia
• metabolic alkalosis
Q2

Which one of the following is not part of the diagnostic criteria for the
metabolic syndrome?

High triglycerides

Low HDL

High LDL

Central obesity

Hypertension

Submit answer

Reference ranges v
LUVV nuL

High LDL

Central obesity

Hypertension

High LDL levels are not part of the World Health Organization or
International Diabetes Federation diagnostic criteria

[ ,6 •• tt Discuss Improve

Next question )

Metabolic syndrome

Unfortunately there are a number of competing definitions of the


metabolic syndrome around at the present time. It is thought that the
key pathophysiological factor is insulin resistance.

SIGN recommend using criteria similar to those from the American


Heart Association . The similarity of the International Diabetes
Federation criteria should be noted. For a diagnosis of metabolic
syndrome at least 3 of the following should be identified:
• elevated waist circumference: men > 102 cm, women > 88 cm
• elevated triglycerides: > 1.7 mmol/L
• reduced HDL: < 1.03 mmol/L in males and < 1.29 mmol/L in
females
• raised blood pressure: > 130/85 mmHg, or active treatment of
hypertension
• raised fasting plasma glucose > 5.6 mmol/L, or previously
diagnosed type 2 diabetes
s set of
The International Diabetes Federation produced a cons ensu
These require
diag nost ic criteria in 2005, whic h are now widely in use.
ence > 94cm
the presence of central obes ity (defined as wais t circu mfer
icity spec ific
for Europid men and > 80cm for Europid women, with ethn
facto rs:
values for othe r groups) plus any two of the follo wing four
tmen t for
• raised triglycerides level : > 1.7 mmo l/L, or spec ific trea
this lipid abnormality
< 1.29
• reduced HDL cholesterol: < 1.03 mm ol/L in male s and
rma lity
mmo l/L in females, or spec ific trea tmen t for this lipid abno
tmen t of
• raised blood pressure : > 130/ 85 mm Hg, or active trea
hypertension
ly
• raised fasti ng plas ma gluc ose> 5.6 mmo l/L, or previous
diagnosed type 2 diab etes

ic crite ria
In 1999 the World Health Organization produced diag nost
gluc ose
which required the presence of diabetes mellitus, impaired
AND two of
tolerance, impaired fasti ng gluc ose or insulin resistance,
the follo wing :
• blood pressure: > 140/ 90 mmH g
-den sity
• dyslipidaemia : trigly cerid es: > 1.695 mm ol/L and/ or high
mm ol/L
lipoprotein cholesterol (HDL-C) < 0.9 mm ol/L (male),< 1.0
(female)
ale),
• central obes ity: wais t:hip ratio > 0.90 (male), > 0.85 (fem
and/ or body mas s index > 30 kg/ m2
mg/ min or
• microalbuminuria: urinary albumin excretion ratio > 20
albumin:creatinine ratio > 30 mg/ g

Other associated features include:


• raised uric acid levels
• non-alco holic fatty liver disease
• polycystic ovarian synd rome

Nex t ques tion )

CamSca nne r ➔ •..;~ L - ~1


0 Q3

A 45-year-old woman presents with weight gain and recurrent 'dizzy'


episodes. Over the past four months she has gained 20 kg. The
episodes occur on an almost daily basis and are characterised by
blurred vision, sweating, headaches and palpitations. Her GP checked a
blood sugar during one of these episodes which was record as being
1.4 mmol/1. What is the single most useful test?

Glucagon stimulation test

Oral glucose tolerance test with growth hormone measurements

Insulin + C-peptide levels during a hypoglycaemic episode

Sh art ACTH test

Insulin tolerance test

Submit answer

Reference ranges v
Oral glucose tolerance test with growth hormone measurements

Insulin + C-peptide levels during a hypoglycaemic episode

Short ACTH test

Insulin tolerance test

This patient has symptoms typical of an insulinoma. Whilst supervised


fasting is normally the investigation of choice if this option is not given
then insulin + C-peptide levels during an acute hypoglycaemic episode
are useful.

,t If t9 Discuss (1) Improve ]

Next question )
Next question >

lnsulinoma

An insulinoma is a neuroendocrine tumour deriving mainly from


pancreatic Islets of Langerhans cells

Basics
• most common pancreatic endocrine tumour
• 10% malignant, 10% multiple
• of patients with multiple tumours, 50% have MEN-1

Features
• of hypoglycaemia: typically early in morning or just before meal,
e.g. diplopia, weakness etc
• rapid weight gain may be seen
• high insulin, raised proinsulin:insulin ratio
• high C-peptide

Diagnosis
• supervised, prolonged fasting (up to 72 hours)
• CT pancreas

Management
• surgery
• diazoxide and somatostatin if patients are not candidates for
surgery

Next question )
Q4

A 60-year-old man who is known to have lung cancer comes for review.
For the past three weeks he has lost his appetite, has been feeling sick
and generally feels tired. On examination he appears to be mildly
dehydrated. You order some blood tests:

Calcium 3.12 mmol/I

Albumin 40 g/ I

Glucose (random) 6.7 mmol/I

Urea 10.2 mmol/1

Creatinine 115 µmol/ 1

Which one of his existing medications is most likely to be contributing


to his presentation?

Amlodipine

Simvastatin

Bendroflu methiazide

Aspirin

Lisinopril

Submit answer
Simvastatin

Bendroflumethiazide

Aspirin

Lisinopril

Thiazides cause hypercalcaemia


Importance: 50

16 .. • Discuss Improve

Next question )
- -.... ......., .....""!II,.....- - --,-- . .,

Next quest ion )

Hypercalcaemia: causes

Two condit ions accou nt for 90% of cases of hypercalcaemia:


• 1. Primary hyperparathyroidism: comm onest cause in non-
hospitalised patien ts
• 2. Malignancy: the comm onest ca use in hospit alised patien ts.
This may be due to numb er of processes, including; bone
metastases, myeloma, PTHrP from squam ous cell lung cance r

Other causes include


• sarco idosis *
• vitamin D intoxic ation
• acromegaly
• thyrotoxicosis
• Milk-alkali syndrome
• drugs: thiazides, calciu m contai ning antac ids
• dehydration
• Addison's disease
• Paget's disease of the bone* *

*other causes of granulomas may lead to hypercalcaemia e.g.


Tuberculosis and histop lasmo sis

**usually normal in this condi tion but hypercalcaemia may occur with
prolonged immob ilisatio n

Next quest ion )


QS

A 40-year-old man presents to the GP concerned about his risk of


developing cancer. You notice that he has large spade-like hands, a
prominent forehead and nose, and thick skin. Which of the following
cancers is he at increased risk of developing?

Pancreatic carcinoma

Colorectal carcinoma

Adrenal carcinoma

Malignant melanoma

Lung cancer

Submit answer

Reference ranges v
Colorectal carcinoma ]
Adrenal carcinoma

Malignant melanoma

Lung cancer

Patients with acromegaly have an increased risk of colorectal


carcinoma
Importance: 50

Patients with acromegaly have an increased incidence of colorectal


polyps and carcinoma. This is a fairly recent finding, now that patients
with acromegaly are surviving longer due to better management of their
other complications, particularly diabetes and cardiovascular disease. It
is recommended that patients with acromegaly have an initial
colonoscopy at age 40, and enter a surveillance program based on the
resu lts of the colonoscopy.

Guidelines for colorectal cancer screening and surveillance in moderate


and high risks. Update
https:// www.ncbi.nlm.nih.gov/pubmed/20427401

[ ,. ,. - Discuss Improve ]

Next question )

CamSc.an,ner ~ '....,.i;lo-" ~ ~ I
Acromegaly: features

In acromegaly 1there is excess growth hormone secondary to a pituitary


adenoma in over 95% of cases. A minority of cases are caused by
ectopic GHRH or GH production by tumours e.g. pancreatic.

Features
• coarse facial appearance, spade-like hands, increase in shoe size
• large tongue, prognathism, interdental spaces
• excessive sweating and oily skin: caused by sweat gland
hypertrophy
• features of pituitary tumour: hypopituitarism, headaches,
bitemporal hemianopia
• raised prolactin in 1/ 3 of cases - galactorrhoea
• 6% of patients have MEN-1

Complications
• hypertension
• diabetes (>10%)
• cardiomyopathy
• colorectal cancer

Next question )

B I l!J A ... ·- 1-
~
- - ...

Save my notes
Q6

A 23-year-old woman is diagnosed with Graves' disease. Which one of


the following statements regarding treatment is correct?

Block-and-replace regimes are usually of a shorter duration than


carbimazole titration therapy

Concurrent administration of propranolol and carbimazole should


be avoided

Patients on block-and-replace regimes have fewer side-effects


than those using titration therapy

Carbimazole should be started at no higher than 1Omg/ day for


patients commencing a titration regime

In the block-and-replace regime levothyroxine should be started


at the same time as carbimazole

'

Submit answer

Reference ranges v
A 23-year-old woman is diagnosed with Graves' disease. Which one of
the following statements regarding treatment is correct?

Block-and-replace reg imes are usually of a shorter duration than


carbimazole titration therapy

Concurrent administration of propranolol and carbimazole should be


avoided

Patients on block-and-replace regimes have fewer side-effects than


those using t itration therapy

Carbimazole should be started at no higher than 1Omg/ day for


patients commencing a titration regime

In the block-and-replace regime levothyroxine should be started at


the same time as carbimazole

a6 •• - Discuss (6) Improve

Next question )
Graves' disease: ma nag em ent

al
Despite man y trials there is no clea r guid ance on the optim
de titra tion of
man agem ent of Graves' dise ase. Trea tmen t optio ns inclu
k-and-replace
anti-thyroid drugs (ATDs, for exam ple carbimazole), bloc
is ofte n given
regimes, radioiodine trea tmen t and surgery. Propranolol
initially to bloc k adrenergic effe cts

ATD titra tion


to main tain
• carbimazole is start ed at 40m g and reduced grad ually
euthyroidism
• typic ally continued for 12-18 mon ths
show n to
• patie nts follo wing an ATD titra tion regim e have been
e
suffe r fewe r side -effe cts than thos e on a bloc k-and-replac
regime

Block-and-replace
• carbimazole is start ed at 40m g
• thyroxine is added when the patie nt is euthyroid
• trea tmen t typic ally lasts for 6-9 mon ths

nulo cyto sis


The majo r com plica tion of carb imaz ole ther apy is agra

Radioiodine trea tmen t


for 4-6
• cont raind icati ons include pregnancy (should be avoided
eye
mon ths follo wing trea tmen t) and age < 16 years. Thyr oid
the
disease is a relative cont raind icati on, as it may wors en
cond ition
nds on
• the prop ortio n of patie nts who beco me hypothyroid depe
require
the dose given, but as a rule the majo rity of patie nt will
thyroxine supp leme ntati on afte r 5 years

Nex t ques tion )

CamScanne r ➔ •..;~ L - ~1
Q7

Which one of the following is least associat ed with gynaecomastia?

Klinefelter's syndrome

Seminoma

Liver disease

Puberty

Hypothyroidism

Submit answer

Reference ranges v
Seminoma

Liver disease

Puberty

Hypothyroidism

Gynaecomastia is seen in up to a third of men with thyrotoxicosis, but is


not a feature of hypothyroidism

[ "' .. • Discuss (1) Improve ]

Next question )

Gynaecomastia

Gynaecomastia describes an abnormal amount of breast tissue in


males and is usually caused by an increased oestrogen:androgen ratio.
It is important to differentiate the causes of galactorrhoea (due to the
actions of prolactin on breast tissue) from those of gynaecomastia
causes of gynaecomastia
• physiological: normal in puberty
• syndromes with androgen deficiency: Ka llman's, Klinefelter's
• testicular failure: e.g. mumps
• liver disease
• testicular cancer e.g. seminoma secreting hCG
• ectopic tumour secretion
• hyperthyroidism
• haemodialysis
• drugs: see below

Drug causes of gynaecomastia


• spironolactone (most common drug cause)
• cimetidine
• digoxin
• cannabis
• finasteride
• gonadorelin analogues e.g. Goserelin, buserelin
• oestrogens, anabolic steroids

Very rare drug causes of gynaecomastia


• tricyclics
• isoniazid
• calcium channel blockers
• heroin
• busulfan
• methyldopa

Next question )

B I ~ A ,..

Save my notes
Q8

A 25-year-old woman presents for her first cervical smear. What is the
most important aetiological factor causing cervical cancer?

Human papilloma virus 6 & 11

Herpes simplex virus 2

Smoking

Combined oral contraceptive pill use

Human papilloma virus 16 & 18

Submit answer

Reference ranges v
Herpes simplex virus 2

Smoking

Combined oral contraceptive pill use

Human papilloma virus 16 & 18

Cervical cancer: Human papillomavirus infection (particularly


16,18 & 33) is by far the most important risk factor
Importance: 50

Whilst a number of the above are known to contribute to the


development of cervica l cancer infection with human papilloma virus 16
& 18 is by far the most important factor.

116 .. • Discuss (1) Improve

Next question )
Cervical cancer

Around 50% of cases of cervical cance r occur in wome n under the age
of 45 years, with incidence rates for cervical cance r in the UK are
highes t in people aged 25-29 years, accord ing to Cancer Research UK. It
may be divided into:
• squam ous cell cancer (80%)
• adenocarcinoma (20%)

Features
• may be detected during routine cervical cance r screen ing
• abnormal vaginal bleeding: postco ital, interm enstru al or
postm enopa usal bleeding
• vaginal discharge

Human papillomavirus (HPV), partic ularly serotypes 16,18 & 33 is by far


the most impor tant factor in the develo pmen t of cervical cancer. Other
risk factor s include:
• smoki ng
• human immun odefic iency virus
• early first intercourse, many sexual partne rs
• high parity
• lower socioe conom ic status
• combi ned oral contra ceptiv e pill*

Mecha nism of HPV causin g cervical cance r


• HPV 16 & 18 produces the oncog enes E6 and E7 genes
respectively
• E6 inhibit s the p53 tumou r suppr essor gene
• E7 inhibit s RB suppr essor gene

*the strength of this assoc iation is some times debated but a large
study published in the Lancet (2007 Nov 10;370 (9599 ):1609-21)
confir med the link
0 Q9

A 45-year-old man is reviewed in the diabetes clinic. The following


results are obtained:

Urinalysis NAO

HbA1 c 69 mmol/ mol

Gliclazide is added to the metformin he already takes. What is the


minimum time period after which the HbA 1c should be repeated?

6 months

1 month

2 weeks

3 months

4 months

Submit answer

Reference ranges v
13months
4 month s

A more accurate answe r would proba bly be 2 month s but this is not
given as an option . See the explan ation below

[ ,t 'f • Discuss (4) Impro ve ]

Next quest ion )

Glycosylated haemoglobin

Glycosylated haemo globin (HbA1c) is the most widely used measu re of


long-t erm glycae mic contro l in diabet es mellitu s. HbA1 c is produ ced by
the glycos ylation of haem oglobi n at a rate propo rtiona l to the gluco se
conce ntratio n. The level of HbA 1c theref ore is depen dant on
• red blood cell lifespan
• average blood glucos e conce ntratio n

A numb er of condit ions can interfe re with accur ate HbA 1c


interp retatio n:

Lower-than-expected levels of Highe r-than -expe cted levels of


HbA1 c ( due to reduced red HbA1 c (due to increased red
blood cell lifespan) blood cell lifespan)

Sickle-cell anaemia Vitam in 812/ folic acid


GP6D deficie ncy deficie ncy
Hereditary spher ocytos is Iron-d eficien cy anaemia
Splen ectom y
over the
HbA1c is generally thou ght to refle ct the bloo d gluc ose
is weig hed mor e
previous '3 mon ths' altho ugh ther e is som e evidence it
mme nd
stron gly to gluc ose levels of the past 2-4 weeks. NICE reco
then 6 mon thly' .
'HbA 1c shou ld be chec ked every 3-6 mon ths until stable,

ose is com plex


The relationship between HbA 1c and average bloo d gluc
tions Trial
but has been studied by the Diabetes Control and Com plica
repo rting HbA1c
(DCCT). A new inter natio nally stan dard ised meth od for
cal
has been developed by the International Federation of Clini
of
Chemistry (IFCC). This will repo rt HbA 1c in mm ol per mol
haem oglo bin with out gluc ose attac hed.

HBA 1c Average plas ma gluc ose


(%) (mm ol/I) IFCC -HbA1 c (mm ol/m ol)

5 5.5

6 7.5 42

7 9.5 53

8 11 .5 64

9 13.5 75

10 15.5

11 17.5

12 19.5

= (2 * HbA 1c)
From the above we can see that average plas ma gluc ose
-4.5

CamSca nne r ➔ •..;~ L - ~1


Q10

Dynamic pituitary function tests may be used to assess each one of the
following, except:

Cortisol

Prolactin

Growth hormone

Follicular stimulating hormone

Antidiuretic hormone

Submit answer

Reference ranges v
I Antldi uretic hormo ne

[ ,6 ,. tit Discuss (4) Impro ve ]

Next quest ion )

Dynamic pituitary function tests

A dynam ic pituita ry functi on test is used to asses s patien ts with


suspe cted prima ry pituita ry dysfun ction

Insulin, TRH and LHRH are given to the patien t follow ing which the
serum glucose, cortiso l, growt h hormo ne, TSH, LH and FSH levels are
recorded at regular interva ls. Prolactin levels are also some times
measu red*

A norma l dynam ic pituita ry functi on test has the follow ing


chara cterist ics:
• GH level rises > 20mu / l
• cortis ol level rises > 550 mmol/1
• TSH level rises by> 2 mu/ I from baseli ne level
• LH and FSH should doubl e

*dopa mine antag onist tests using metoc lopram ide may also be used in
the invest igation of hyper prolac tinaem ia. A norma l respo nse is at least
a twofo ld rise in prolactin. A blunte d prolac tin respo nse sugge sts a
prolac tinoma

Next question )
Q11

A 49-year-old man with type 2 diabetes mellitus is reviewed. Despite


weight loss and therapy with metformin and gliclazide his last HbA 1c is
7.2%. Which one of t he following factors would suggest that the patient
may benefit from a meglitinide?

Obesity

Not adhering to diabetic diet

Problems with hypoglycaemia from gliclazide

Erratic lifestyle

Elderly and frail patients

Submit answer

Reference ranges v
Erratic lifestyle

Elderly and frail patients

Meglitinides - stimulate insulin release - good for erratic lifestyle


Importance: 50

Meglitinides stimulate insulin release and are particularly useful for


post-prandial hyperglycaemia or an erratic eating schedule, as patients
take them shortly before meals

•• .. tt Discuss (7) Improve

Next question )

Diabetes mellitus: management of type 2

NICE updated its guidance on the management of type 2 diabetes


mellitus (T2DM) in 2015. Key points are listed below:
• HbA 1 c targets have changed. They are now dependent on what
antidiabetic drugs a patient is receiving and other factors such as
frailty
• there is more flexibility in the second stage of treating patients
(i.e. after metformin has been started) - you now have a choice of
4 oral antidiabetic agents

It's worthwhile thinking of the average patient who is taking


metformin for T2DM, you can titrate up metformin and encourage
lifestyle changes to aim for a HbA1c of 48 mmol/mol (6. 5%), but
should only add a second drug if the HbA 1 c rises to 58 mmol/mol
(7.5%)
Dietary advice
• encourage high fibre, low glycaemic index sources of
carbohydrates
• include low-fat dairy products and oily fish
• control the intake of foods containing saturated fats and t rans
fatty acids
• limited substitution of sucrose-containing foods for other
carbohydrates is allowable, but care should be taken to avoid
excess energy intake
• discourage use of foods marketed specifically at people with
diabetes
• initial target weight loss in an overweight person is 5-1 0%

HbA1c targets

This is area which has changed in 2015


• individual targets should be agreed with patients to encourage
motivation
• HbA1c should be checked every 3-6 months until stable, then 6
monthly
• NICE encourage us to consider relaxing targets on 'a case-by-case
basis, with particular consideration for people who are older or frail,
for adults with type 2 diabetes'
• in 207 5 the guidelines changed so HbA1 c targets are now
dependent on treatment:

CamSc.an,ner ~ '....,.i;lo-" ~ ~ I
Lifestyle or single drug treatme nt

HbA1c
target
Manage ment of T2DM

48
Lifestyle
mmol/ mol
(6.5%)

Lifestyle + metform in 48
mmol/m ol
(6.5%)

Includes any drug which may cause hypoglycaemia 53


(e.g. lifestyle + sulfonylurea) mmol/m ol
(7.0%)

Practical examples
• a patient is newly diagnosed with HbA 1 c and wants to try lifestyle
treatme nt first. You agree a target of 48 mmol/m ol (6.5%)
• you review a patient 6 months after starting metform in. His
HbA 1 c is 51 mmol/m ol (6.8%). You increase his metform in from
500mg bd to 500mg tds and reinforce lifestyle factors

Patient already on treatme nt

Manage ment of T2DM HbA1c target

Already on one drug, but HbA1 c has risen to 58 53 mmol/ mol


mmol/m ol (7.5%) (7.0%)
Drug treatment

The 2015 NICE guidelines introduced some changes into the


management of type 2 diabetes. There are essentially two pathways,
one for patients who can tolerate metformin, and one for those who
can't:

Mettormtn not tolerated or e1


M4tform n

Tolerates metformin:
• metformin is still first-line and should be offered if the HbA 1c
rises to 48 mmol/mol (6.5%)* on lifestyle interventions
• if the HbA1c has risen to 58 mmol/ mol (7.5%) then a second drug
should be added from the following list:
• - sulfonylurea
• - gliptin
• - pioglitazone
• - SGLT-2 inhibitor
• if despite this the HbA1 c rises to, or remains above 58 mmo l/mo l
(7.5%) then triple therapy with one of the follow ing comb inatio ns
should be offered:
• -+ metfo rmin + glipti n + sulfo nylur
ea
• -+ metfo rmin + piogl itazo ne + sulfo
nylurea
• -+ metfo rmin + sulfo nylur ea + SGLT
-2 inhib itor
• - metfo rmin + pioglitazone + SGLT-2 inhib itor
• - OR insulin therapy should be considered

Criteria for glucagon-like peptide1 (GLP1) mime tic (e.g. exenatide)


• if triple therapy is not effective, not tolera ted or contr aindi cated
then NICE advise that we cons ider comb inatio n thera py with
metfo rmin, a sulfonylurea and a gluca gonli ke peptide1 (GLP1)
mime tic if:
• BMI >= 35 kg/ m 2 and spec ific psyc holog ical or othe r medi cal
-+

problems associated with obes ity or


• - BMI < 35 kg/ m and for whom insulin therapy woul d have
2

signi fican t occu patio nal impli catio ns or

weight loss would benefit other signi fican t obesityrelated como rbidit ies
• only continue if there is a reduction of at least 11 mmo l/mo l
[1 .0%] in HbA1c and a weig ht loss of at least 3% of initial body
weig ht in 6 mont hs

Practical examples
• you review an established type 2 diabe tic on maxi mum dose
metfo rmin . Her HbA 1c is 55 mmo l/ mol (7.2%). You do not add
another drug as she has not reached the thres hold of 58
mmo l/ mol (7.5%)
• a type 2 diabetic is found to have a HbA 1c of 62 mmo l/mo l (7.8%)
at annual review. They are currently on maxi mum dose metfo rmin.
You elect to add a sulfonylurea
Cannot tolerate metformin or contraindicated
• if the HbA1crises to 48 mmol /mol (6.5%) * on lifesty le
interventions, consid er one of the follow ing:
• _. sulfonylurea
• _. gliptin
• _. pioglitazone
• if the HbA 1c has risen to 58 mmol / mol (7.5%) then a one of the
follow ing combi nation s should be used:
• _. gliptin + pioglitazone
• -+ gliptin + sulfon ylurea

• _. pioglitazone + sulfonylurea
• if despite this the HbA 1c rises to, or remains above 58 mmol /mol
(7.5%) then consid er insulin therapy

Starting insulin
• metfo rmin should be contin ued. In terms of other drugs NICE
advice : 'Review the contin ued need for other blood glucos e loweri ng
therapies '
• NICE recommend startin g with huma n NPH insulin (isophane,
intermediate acting) taken at bed-tim e or twice daily accor ding to
need

Risk factor modification

Blood pressure
• target is< 140/8 0 mmHg (or< 130/ 80 mmHg if end-organ
damage is present)
• ACE inhibitors are first-line

Antiplatelet s
• should not be offered unless a patien t has existin g cardio vascu lar
disease
Lipids
• following the 2014 NICE lipid modifica tion guidelines only patients
with a 10-year cardiova scular risk > 10% (using QRISK2) should be
offered a statin. The first-line statin of choice is atorvast atin 20mg
on

Prtmary p,wventton Sec~ry prevention

10.,..,~1A afrisi••1 OR t.nown CNwtOIMeM


masl~1 ~0ft ~iat.._OR
CKD I eQfR c ~ papal ............

Atorvastatln 20mg od
Atorvastatin IOmg od
non-HOL nn noc t by .,. ,o,. tnen
cCtlllder wnng I.IP IO 81),,g

Graphic showing choice of statin.

-kthis is a bit confusin g because isn't the diagnos tic criteria for T2DM
HbA 1c 48 mmol/m ol (6.5%)? So shouldn 't all patients be offered
metform in at diagnosis? Our interpret ation of this is that some patients
upon diagnos is will elect to try lifestyle measures, which may reduce
their HbA 1c below this level. If it then rises to the diagnos tic threshol d
again metform ln should be offered

Next question )

B / tj A,..

Save my notes
0 Q12

Which of the fo llowing statements is true regarding the


pathophysiology of diabetes mellitus?

Concordance between identical twins is higher in type 2 diabetes


mellitus than type 1

Patients with type 1 diabetes mellitus are rarely HLA-DR4 positive

Type 2 diabetes mellitus is associated with HLA-DR3

Haemochromatosis is an example of primary diabetes

Type 1 diabetes mellitus is thought to be inherited in an


autosomal dominant fashion

Submit answer

Reference ranges v
Concordance between identical twins is higher in type 2 diabetes
mellitus than type 1

Patients with type 1 diabetes mellitus are rarely HLA-DR4 posiitive

Type 2 diabetes mellitus is associated with HLA-DR3

Haemochromatosis is an example of primary diabetes

Type 1 diabetes mellitus is thought to be inherited in an autosomal


dominant fashion

Type 1 diabetes mellitus is caused by autoimmune destruction of the


Beta-cells of the pancreas. Identical twins show a genetic concordance
of 40%. It is associated with HLA-DR3 and DR4. It is inherited in a
polygenic fashion

Type 2 diabetes mellitus is thought to be caused by a relative deficiency


of insulin and the phenomenon of insulin resistance. Age, obesity and
ethnicity are important aetiological factors. There is almost 100%
concordance in identical twins and no HLA associations.

Haemochromatosis is an example of secondary diabetes

[ ,. ,. - Discuss (3) Improve ]

Next question )

CamSc.an,ner ~ '....,.i;lo-" ~ ~ I
Next question )

Diabetes: pathophysiology

Type 1 OM
• autoimmune disease
• antibodies against beta cells of pancreas
• HLA OR4 > HLA DR3
• various antibodies such as islet-associated antigen (IAA) antibody
and glutamic acid decarboxylase (GAD) antibody are detected in
patients who later go on to develop type 1 OM - their prognostic
significance is not yet clear

Next question )

B I l!!J

Save my notes

Search


Search textbook...
---- ·
Q Google search on "Diabetes: pathophysiology"
Q13

Liddle's syndrome is associated with each one of the following, except

Alkalosis

Response to treatment with amiloride

Hypertension

Autosomal recessive inheritance

Hypokalaemia

Submit answer

Reference ranges v
Response to treatment with amiloride

Hypertension

Autosomal recessive inheritance

Hypokalaemia

•• f
11
• Discuss (1) Improve

Next question )

Liddle's syndrome

Liddle's syndrome is a rare autosomal dominant condition that causes


hypertension and hypokalaemic alkalosis. It is thought to be caused by
disordered sodium channels in the distal tubules leading to increased
reabsorption of sodium.

Treatment is with either amiloride or triamterene

Next question >

Save my notes

Search
Q14

Which one of the following types of thyroid cancer is associated with


the RET oncogene?

Anaplastic

Lymphoma

Follicular

Medullary

All types of thyroid cancer

Submit answer

Reference ranges v
Medullary

All types of thyroid cancer

The RET oncogene encodes a receptor tyrosine kinase and is


associated with MEN type 2.

Papillary thyroid cancer also appears to be associated with the RET


oncogene

116 f
11 tt Discuss Improve

Next question >

Multiple endocrine neoplasia

The table below summarises the three main types of multiple endocrine
neoplasia (MEN). MEN is inherited as an autosomal dominant disorder.

MEN type I MEN type Ila MEN type llb

Medullary thyroid Medullary thyroid


3 P's
cancer (70%) cancer

Parathyroid (95%):
hyperparathyroidism 2 P's 1P
due to parathyroid
hyperplasia Parathyroid (60%) Phaeochromocytomc
Pituitary (70%) Phaeochromocytoma
Pancreas (50%): e.g. Marfanoid body
insulinoma, habitus
gastrinoma (leading Neuromas
to recurrent peptic
ulceration)
Also: adrenal and
thyroid

MENl gene RET oncogene RET oncogene

Most common
presentation =
hypercalcaemia

MEN type 1

P oc, he tumours
(e g gastnnoma
su!inoma)

Pitultary tumours
<• g PfO cunornaJ

Primary
hyperparathyroidrsm

MEN type 2a MEN type 2b

PhaeocPuomocytoma
Marfanoid body habitus
Medullaiy thyroid
ca!lCer
Neuromas

RE T oncogene

Venn diagram showing the different types of MEN and their


associated features

Next question )

B J ~ A ...
Q15

A 30-year-old female is started on carbimazole 20mg bd following a


diagnosis of Grave's disease. What is the best biochemical marker to
assess her response to treatment?

Total T4

TSH

Free T4

ESR

Free T3

Submit answer

Reference ranges v
Free T4

ESR

Free T3

The answer the College are loo kin g for is TSH


. The re is how eve r a
sig nifi can t proportion of pat ien ts for wh om
TSH mo nito ring alo ne is
insufficient. TSH ma y remain suppressed for
several we eks as
continued production of thy roid stim ula ting
imm uno glo bul ins seen in
Grave's disease reduces the need for the pitu
itar y to sec rete TSH

[ ,. •• tt Discuss (7) Imp rov e ]

Ne xt que stio n )
Thyrotoxicosis: causes and investigation

Graves' disease accounts for around 50-60%of cases of thyrotoxicosis.

Causes
• Graves' disease
• toxic nodular goitre
• acute phase of subacute (de Quervain's) thyroiditis
• acute phase of post-partum thyroiditis
• acute phase of Hashimoto's thyroiditis (later results in
hypothyroidism)
• amiodarone therapy

Investigation
• TSH down, T4 and T3 up
• thyroid autoantibodies
• other investigations are not routinely done but includes isotope
scanning

Hypothyroidism Hyperthyrodism

H st11moo's
Chyroidi,:,s &lblacute th~roicl t.s
(de Ouervarn's) Graves' disease
Ill

Amiodarone Toxic muttinodular goitre


RI S lhyroid1lJS
Postpartum thYf01di1L~

Venn diagram showing how different causes of thyroid


dysfunction may manifest. Note how many causes of
hypothyroidism may have an initial thyrotoxic phase.
Q16

A 24-year-old female with a history of type 1 diabetes mellitus presents


to the Emergency Department with vom iting and abdominal pain.
Finger-prick testing estimates the blood sugar to be 25 mmol/1. Arterial
blood gases record a pH of 7.22. On examination the patient is
dehydrated and weighs 80 kg . An intravenous line is sited and bloods
are sent. One litre of 0.9% saline is infused and an intravenous insulin
pump is set-up . What rate should insulin be initially given?

1O unit / hour

1 unit / hour

2 unit / hour

6 unit / hour

8 unit / hour

Submit answer

Reference ranges v
I 8 unit/ hour

The Joint British Diabetes Societies produced guidelines in 201 0


recommending starting the insulin infusion at a rate of 0.1 unit/kg/hour.

116 •t tt Discuss (4) Improve

Next question )

Diabetic ketoacidosis

Diabetic ketoacidosis (DKA) may be a complication existing type 1


diabetes mellitus or be the first presentation, accounting for around 6%
of cases. Rarely, under conditions of extreme stress, patients with type
2 diabetes mellitus may also develop DKA.

Whilst DKA remains a serious condition mortality rates have decreased


from 8% to under 1% in the past 20 years.

Pathophysiology
• DKA is caused by uncontrolled lipolysis (not proteolysis) which
results in an excess of free fatty acids that are ultimately
converted to ketone bodies

The most common precipitating factors of DKA are infection, missed


insulin doses and myocardial infarction.

Features
• abdominal pain
• polyuria, polydipsia, dehydration
• Kussmaul respiration (deep hyperventilation)
• Acetone-smelling breath ('pear drops' smell)
Diagnostic criteria

American Diabetes Joint British Diabetes Societies


Association (2009) (2013)

Key points Key points


• glucose > 13.8 • glucose > 11 mmol/ 1or known
mmol/ 1 diabetes mellitus
• pH< 7.30 • pH< 7.3
• serum bicarbonate • bicarbonate < 1 5 mmol/ 1
<18 mmol/ 1 • ketones > 3 mmol/ 1or urine
• anion gap > 1 O ketones ++ on dipstick
• ketonaemia

Management
• fluid replacement: most patients with OKA are deplete around 5-8
litres. Isotonic saline is used initially. Please see an example fluid
regime below.
• insulin: an intravenous infusion should be started at 0.1
unit/ kg/ hour. Once blood glucose is < 15 mmol/ 1an infusion of 5%
dextrose should be started
• correction of hypokalaemia
• long-acting insulin should be continued, short-acting insulin
should be stopped

JBDS example of fluid replacement regime for patient with a systolic


BP on admission 90mmHg and over
BP on admission 9UmmHg and over

Fluid Volume

0.9% sodium chloride 1 L 1000ml over 1 st hour

0.9% sodium chloride 1 L with potassium 1000ml over next 2


chloride hours

0.9% sodium chloride 1 L with potassium 1000ml over next 2


chloride hours

0. 9% sodium chloride 1 L with potassium 1000ml over next 4


ch loride hours

0. 9% sodium chloride 1 L with potassium 1000ml over next 4


chloride hours

0.9% sodium chloride 1 L with potassium 1 000ml over next 6


ch loride hours

Please note that slower infusion may be indicated in young adults (aged
18-25 years) as they are at greater risk of cerebral oedema .

JBDS potassium guidelines

Potassium level in first 24 Potassium replacement in mmol/L


hours (mmol/L) of infusion solution

Over 5.5 Nil

3.5-5.5 40

Below 3.5 Senior review as additional


potassium needs to be given
Complications of OKA and its treatment
• gastric stasis
• thromboembolism
• arrhythmias secondary to hyperkalaemia/ iatrogenic hypokalaemia
• iatrogenic due to incorrect fluid therapy: cerebral oedema*,
hypokalaemia, hypoglycaemia
• acute respiratory distress syndrome
• acute kidney injury

* children/ young adults are particularly vulnerable to cerebral oedema


following fluid resuscitation in OKA and often need 1 :1 nursing to
monitor neuro-observations, headache, irritability, visual disturbance,
focal neurology etc. It usually occurs 4-12 hours following
commencement of treatment but can present at any time. If there is any
suspicion a CT head and senior review should be sought

Next question )

B I = ...
- T I ...

Save my notes

Search
Q17

A 54-year-old female presents to the Emergency Department concerned


about double vision . She is noted to have exophthalmos and
conjunctiva! oedema on examination and a diagnosis of thyroid eye
disease is suspected. What can be said regarding her thyroid status?

Hyper- or euthyroid

Hypothyroid

0 Hyperthyroid

Hypo- or euthyroid

Eu-, hypo- or hyperthyroid

Submit answer

Reference ranges v
I Eu-, hypo- or hyperthyro-id~

Whilst thyroid eye disease is mostly associated with hyperthyroidism


secondary to Graves' disease there is not always an association. A
minority of patients will either be euthyroid or hypothyroid.

It is also known that the severity of thyroid eye disease is not related to
the degree of thyrotoxicosis in Graves' disease.

16 if • Discuss (4) Improve

Next question )

Thyroid eye disease

Thyroid eye disease affects between 25-50% of patients with Graves'


disease.

Pathophysiology
• it is thought to be caused by an autoimmune response against an
autoantigen, possibly the TSH receptor -- retro-orbital
inflammation
• the inflammation results in glycosaminoglycan and collagen
deposition in the muscles

Prevention
• smoking is the most important modifiable risk factor for the
development of thyroid eye disease
• radioiodine treatment may increase the inflammatory symptoms
seen in thyroid eye disease. In a recent study of patients with
Graves' disease around 15% developed, or had worsening of, eye
disease. Prednisolone may help reduce the risk
Features
• the patient may be eu-, hypo- or hyperthyroid at the time of
presentation
• exophthalmos
• conjunctiva! oedema
• optic disc swelling
• ophthalmoplegia
• inability to close the eyelids may lead to sore, dry eyes. If severe
and untreated patients can be at risk of exposure keratopathy

Management
• topical lubricants may be needed to help prevent corneal
inflammation caused by exposure
• steroids
• radiotherapy
• surgery

Monitoring patients with established thyroid eye disease

For patients with established thyroid eye disease the following


symptoms/signs should indicate the need for urgent review by an
ophthalmologist (see EUGOGO guidelines):
• unexplained deterioration in vision
• awareness of change in intensity or quality of colour vision in one
or both eyes
• history of eye suddenly 'popping out' (globe subluxation)
• obvious corneal opacity
• cornea still visible when the eyelids are closed
• disc swelling

Next question )
Q18

A 16-year-old male is reviewed in the endocrinology clinic due to lack of


pubertal development. On examination his testes are undescended and
there is only scanty pubic hair. What is the most likely diagnosis?

Down's syndrome

Kallman's syndrome

Dubin-Johnson syndrome

Turner's syndrome

Klinefelter's syndrome

Submit answer

Reference ranges v
Dubin-Johnson syndrome

Turner's syndrome

Klinefelter's syndrome

Cryptorchidism is more suggestive of Kallman's than Klinefelter's


syndrome

116 •• • Discuss (3) Improve

Next question )

Kallmann's syndrome

Kallmann's syndrome is a recognised cause of delayed puberty


secondary to hypogonadotrophic hypogonadism. It is usually inherited
as an X-linked recessive trait. Kallmann's syndrome is thought to be
caused by failure of GnRH-secreting neurons to migrate to the
hypothalamus.

The clue given in many questions is lack of smell (anosmia) in a boy


with delayed puberty

Features
• 'delayed puberty'
• hypogonadism, cryptorchidism
• anosmia
• sex hormone levels are low
• LH, FSH levels are inappropriately low/ normal
• patients are typically of normal or above average height

Cleft lip/ palate and visual/hearing defects are also seen in some
Q19

Which one of the following statements regarding impaired glucose


regulation is correct?

All patient should have a repeat oral glucose tolerance test every
2 years

Patients with impaired glucose tolerance are more likely to


develop diabetes than patients with impaired fasting glycaemia

Impaired glucose tolerance (IGT) is defined as a fasting glucose


greater than or equal to 6.1 but less than 7.0 mmol/1

Around 1 in 20 adults in the UK have impaired glucose regulation

Patients should be offered pioglitazone if lifestyle changes fail to


improve their glucose profile

Submit answer

Reference ranges v
Patients with impaired glucose tolerance are more likely to develop
diabetes than patients with impaired fasting glycaemia

Impaired glucose tolerance (IGT) is defined as a fasting glucose


greater than or equal to 6.1 but less than 7.0 mmol/1

Around 1 in 20 adults in the UK have impaired glucose regulation

Patients should be offered pioglitazone if lifestyle changes fail to


improve their glucose profile

8' .. • Discuss (3) Improve

Next question >

Prediabetes and impaired glucose regulation

Prediabetes is a term which is increasingly used where there is


impaired glucose levels which are above the normal range but not high
enough for a diagnosis of diabetes mellitus. The term includes patients
who have been labelled as having either impaired fasting glucose (IFG)
or impaired glucose tolerance (IGT). Diabetes UK estimate that around
1 in 7 adults in the UK have prediabetes. Many individuals with
prediabetes will progress on to developing type 2 diabetes mellitus
(T2DM) and they are therefore at greater risk of microvascular and
macrovascular complications.

Terminology
• Diabetes UK currently recommend using the term prediabetes
when talking to patients and impaired glucose regulation when
talking to other healthcare professionals
• research has shown that the term 'prediabetes' has the most
impact and is most easily understood
Identification of patients with prediabetes
• NICE recommend using a validated computer based risk
assessment tool for all adults aged 40 and over, people of South
Asian and Chinese descent aged 25-39, and adults with
conditions that increase the risk of type 2 diabetes
• patients identified at high risk should have a blood sample taken
• a fasting plasma glucose of 6.1-6. 9 mmol/ 1or an HbA1c level of
42-47 mmol/ mol (6 .0-6.4%) indicates high risk

Fasting glucose: <= o mmol+1 >= 1.0 nnnol.11

,rediabetH
Normal g~m•c control Diabnn meliilus
.,.'bi\ le ~2....&7 rrm:,lmci 6 ~ -I
1$

or '
C: n~ g\leow 6 1-6 Pm~,

HbA1c· ,,=•s rt'mot•mot


<a •1
..mmol!mot
) I 5

I e ....,, $ eq 10 lhlS or i ~ C d gno$lC


tclo'N re com;idcred n I t,reS.~~ for o bees

Diagram showing the spectrum of diabetes diagnosis

Management
• lifestyle modification: weight loss, increased exercise, change in
diet
• at least yearly follow-up with blood tests is recommended
• NICE recommend metformin for adults at high risk 'whose blood
glucose measure (fasting plasma glucose or HbA 1c) shows they
are still progressing towards type 2 diabetes, despite their
participation in an intensive lifestyle-change programme'
Impaired fasting glucose and impaired glucose tolerance

There are two main types of IGR:


• impaired fasting glucose (IFG) - due to hepatic insulin resistance
• impaired glucose tolerance (IGT) - due to muscle insulin
resistance
• patients with IGT are more likely to develop T2DM and
cardiovascular disease than patients with IFG

Definitions
• a fasting glucose greater than or equal to 6.1 but less than 7.0
mmol/1 implies impaired fasting glucose (IFG)
• impaired glucose tolerance (IGT) is defined as fasting plasma
glucose less than 7.0 mmol/1 and OGTT 2-hour value greater than
or equal to 7.8 mmol/1 but less than 11.1 mmol/1
• people with IFG should then be offered an oral glucose tolerance
test to rule out a diagnosis of diabetes. A result below 11 .1
mmol/1 but above 7.8 mmol/1 indicates that the person doesn't
have diabetes but does have IGT

Next question )

B I A ... ·-
·- 21-: - - ...
0 Q20

A 37-year-old lady presents with weight gain and bruising. On


examination, she is hypertensive and has proximal myopathy. A low
dose dexamethasone suppression test showed a lack of appropriate
suppression of plasma cortisol. However, cortisol was suppressed
during a high dose dexamethasone suppression test. Plasma ACTH
was elevated. A pituitary MRI was normal.

What is the most likely diagnosis?

Cushing's disease

Exogenous steroid use

Small cell lung can cer

Conn's syndrome

Adrenal hyperplasia

Submit answer

Reference ranges v
Cushing's disease

Exogenous steroid use

Small cell lung cancer

Conn's syndrome

Adrena I hyperplasia

In Cushing's disease, cortisol is not suppressed by low-dose


dexamethasone but is suppressed by high-dose dexamethasone
Importance: 50

Cushing's disease is when there is an ACTH secreting pituitary


adenoma. In Cushing's disease, cortisol is not suppressed by low-dose
dexamethasone but is suppressed by high-dose dexamethasone. The
adenoma in Cushing's disease is often too small to be picked up on CT
or MRI imaging. A negative scan does not rule out this diagnosis.

There is no evidence from the history of exogenous steroid use. With


exogenous steroid use, the plasma ACTH would be decreased.

With ectopic ACTH from small cell lung cancer, cortisol would not be
suppressed by low dose or high-dose dexamethasone.

Conn's syndrome is an aldosterone secreting adrenal adenoma.

With cortisol secretion from an adrenal adenoma, cortisol would not be


suppressed by low-dose or high-dose dexamethasone.

r "' I 11
, I tt Discuss (2) I Improve l
Cushing's syndrome: investigations

Investigations are divided into confirming Cushing's synd rome and then
localising the lesion. A hypokalaemic metabolic alkalosis may be seen,
along with impaired glucose tolerance. Ectopic ACTH secretion (e.g.
secondary to small cell lung cancer) is characteristically associated
with very low potassium levels. An insulin stress test is used to
differentrate between true Cushing's and pseudo-Cushing's

Tests to confirm Cushing's syndrome

The two most commonly used tests are:


• overnight dexamethasone suppression test (most sensitive)
• 24 hr urinary free cortisol

Localisation tests

The first-line localisation is 9am and midnight plasma ACTH (and


cortisol) levels. If ACTH is suppressed then a non-ACTH dependent
cause is likely such as an adrenal adenoma

Both low- and high-dose dexamethasone suppression tests may be


used to localise the pathology resulting in Cushing's syndrome. These
tests may be interpreted as follows:
Both low- and high-dose dexamethasone suppression tests may be
used to localise the pathology resulting in Cushing's syndrome. These
tests may be interpreted as follows:

Cortisol result Interpretation

Not suppressed by low-dose Cushing's syndrome not due to


dexamethasone primary causes, i.e. likely secondary
to corticosteroid therapy

Not suppressed by low- Cushing's disease


dose, but suppressed by
high-dose dexamethasone

Not suppressed by low- or Ectopic ACTH syndrome likely


high-dose dexamethasone

CRH stimulation
• if pituitary source then cortisol rises
• if ectopic/adrenal then no change in cortisol

Petrosal sinus sampling of ACTH may be needed to differentiate


between pituitary and ectopic ACTH secretion

Next question )

B / A ,.. 1-
~
- - ,..
1..:1 c-:::,
01

In patients with suspected insulinoma, which one of the following is


considered the best investigation?

Oral glucose tolerance test

Insulin tolerance test

Early morning C-peptide levels

Glucagon stimulation test

Supervised fasting

Submit answer

Reference ranges v
Insulin tolerance test

Early morning C-peptide levels

Glucagon stimulation test

Supervised fasting

lnsulinoma is diagnosed with supervised prolonged fasting


Importance: 50

CT of the pancreas is also useful in demonstrating a lesion

[ ,6 ,. t9 Discuss (2) Improve ]

Next question )

CamScanner -! W~ :.i.;,-~ \
lnsulinoma

An insulinoma is a neuroendocrine tumour deriving mainly from


pancreatic Islets of Langerhans cells

Basics
• most common pancreatic endocrine tumour
• 10% malignant, 10% multiple
• of patients with multiple tumours, 50% have MEN-1

Features
• of hypoglycaemia: typically early in morning or just before meal,
e.g. diplopia, weakness etc
• rapid weight gain may be seen
• high insulin, raised proinsulin:insulin ratio
• high C-peptide

Diagnosis
• supervised, prolonged fasting (up to 72 hours)
• CT pancreas

Management
• surgery
• diazoxide and somatostatin if patients are not candidates for
surgery

Next question )

I T l ....
0 Q2

A 32-year-old woman presents to the emergency department drowsy.


On examination, she scores 12/ 1 5 on the Glasgow coma scale.

Capillary blood glucose is 1.9 mmol/L.

Which is the first hormone secreted physiologically in this scenario?

Cortisol

Glucagon

Growth hormone

lncretin

Insulin

Submit answer

Reference ranges v
Glucagon

Growth hormone

lncretin

Insulin

Glucagon is the first hormone secreted in response to


hypoglycaemia
Importance: 50

Low blood glucose can be caused by a number of factors, including


self-administration of insulin or alcohol. The first response to
hypoglycaemia physiologically is decreased insulin secretion, followed
by glucagon secretion from alpha cells of the pancreas. This causes the
liver to convert stored glycogen into glucose, thus raising circulating
blood glucose levels.

Growth hormone and cortisol are released later in response. Addison's


disease, where cortisol production is reduced, can therefore cause
hypoglycaemia. This understanding forms the basis of the insulin
tolerance test. Here, insulin is administered to induce hypoglycaemia
and an inadequate rise in serum cortisol suggests adrenal insufficiency.

lncretins, in contrast, are hormones which stimulate a decrease in blood


glucose levels, especially after eating. One particular incretin, glucagon-
like peptide 1 (GLP-1) is used in the treatment of type 2 diabetes;
exenatide is an example of an injectable GLP-1 analogue medication.

8' 1f 49 Discuss (1) Improve

Next question )
9' 11
f • Discuss (1) Improve

Next question )

Hypoglycaemia

Causes
• insulinoma - increased ratio of proinsulin to insulin
• self-administration of insulin/sulphonylureas
• liver failure
• Addison's disease
• alcohol

Other possible causes in children


• nesidioblastosis - beta cell hyperplasia

Physiological response to hypoglycaemia


• hormonal response: the first response of the body is decreased
insulin secretion. This is followed by increased glucagon
secretion . Growth hormone and cortisol are also released but later
• sympathoadrenal response: increased catecholam ine-mediated
(adrenergic) and acetylcholine-mediated (cholinergic)
neurotransmission in the peripheral autonomic nervous system
and in the central nervous system

Next question )

B /
Q3

A 55-year-old female is reviewed in the diabetes clinic. The following


results are obtained:

Urinalysis protein+

HbA 1c 86 mmol/mol (10.0%)

What average blood glucose level for the past 2 months is this most
likely to represent?

10

11

15

There is no relation between HbA 1c and average blood glucose

Submit answer

Reference ranges v
A 55-year-old female is reviewed in the diabetes clinic. The following
results are obtained:

Urinalysis protein +

HbA1c 86 mmol/ mol (10.0%)

What average blood glucose level for the past 2 months is this most
likely to represent?

10

11

15

There is no relation between HbA 1c and average blood glucose

[ "' •• tll Discuss (4) Improve

Next question )
Q4

A diabetic man is diagnosed as having painful diabetic neuropathy in


his feet. He has no other medical history of note. What is the most
suitable first-line treatment to relieve his pain?

Duloxetine

Sodium valproate

Carbamazepine

Referral to pain management clinic

Tramadol

Submit answer

Reference ranges v
Q4

A diabetic man is diagnosed as having painful diabetic neuropathy in


his feet. He has no other medical history of note. What is the most
suitable first-line treatment to relieve his pain?

Duloxetine

Sodium valproate

Carbamazepine

Referral to pain management clinic

Tramadol

16 1f • Discuss (1) Improve

Next question )
Diabetic neuropathy

Diabetes typically leads to sensory loss and not motor loss in peripheral
neuropathy. Painful diabetic neuropathy is a common problem in
clinical practice.

NICE updated it's guidance on the management of neuropathic pain in


2013. Diabetic neuropathy is now managed in the same way as other
forms of neuropathic pain:
• first-line treatment: amitriptyline, duloxetine, gabapentin or
pregabalin
• if the first-line drug treatment does not work try one of the other 3
drugs
• tramadol may be used as 'rescue therapy' for exacerbations of
neuropathic pain
• topical capsaicin may be used for localised neuropathic pain (e.g.
post-herpetic neuralgia)
• pain management clinics may be useful in patients with resistant
problems

Gastrointestinal autonomic neuropathy

Gastroparesis
• symptoms include erratic blood glucose control, bloating and
vomiting
• management options include metoclopramide, domperidone or
erythromycin (prokinetic agents)

Chronic diarrhoea
• often occurs at night

Gastro-oesophageal reflux disease


• caused by decreased lower esophageal sphincter (LES) pressure
QS

A 35-year-old female is referred to the endocrine clinic due to weight


loss and palpitations. The following results are obtained:

TSH < 0.05 mu/ I

T4 178 mmol/1

Which one of the following features would most suggest a diagnosis of


Grave's disease?

Atrial fibrillation

Lid lag

Family history of radioiodine treatment

Pretibial myxoedema

Multinodular goitre

Submit answer

Reference ranges v
Pret161al myxoedema

Multinodular goitre

Pretibial myxoedema is not seen in other causes of thyrotoxicosis and


points towards a diagnosis of Graves'

•• If tt Discuss (8) Improve

Next question )

Graves' disease: features

Graves' disease is the most common cause of thyrotoxicosis. It is


typically seen in women aged 30-50 years.

Features
• typical features of thyrotoxicosis
• specific signs limited to Grave's (see below)

Features seen in Graves' but not in other causes of thyrotoxicosis


• eye signs (30% of patients)
o exophthalmos
o ophthalmoplegia
• pretibial myxoedema
• thyroid acropachy, a triad of:
o digital clubbing
o soft tissue swelling of the hands and feet
o periosteal new bone formation

Autoantibodies
• TSH receptor stimulating antibodies (90%)
• anti-thyroid peroxidase antibodies (75%)
Q6

A 41-year-old woman is investigated for hot flushes and night sweats.


Bloods show a significantly raised FSH level and her symptoms are
attributed to the menopause. Following discussions with the patient
she elects to have hormone replacement treatment. What is the most
significant risk of prescribing an oestrogen-only preparation rather than
a combined oestrogen-progestogen preparation?

Increased risk of venous thromboembolism

Increased risk of ovarian cancer

Increased risk of endometrial cancer

Increased risk of breast cancer

Increased risk of colorectal cancer

Submit answer

Reference ranges v
A 41-year-old woman is investigated for hot flushes and night sweats.
Bloods show a significantly raised FSH level and her symptoms are
attributed to the menopause. Following discussions with the patient
she elects to have hormone replacement treatment. What is the most
significant risk of prescribing an oestrogen-only preparation rather than
a combined oestrogen-progestogen preparation?

Increased risk of venous thromboembolism

Increased risk of ovarian cancer

Increased risk of endometrial cancer

Increased risk of breast cancer

Increased risk of colorectal cancer

HRT: unopposed oestrogen increases risk of endometrial cancer


Importance: 50

,6 11, tt Discuss (2) Improve

Next question )
Hormone replacement therapy: adverse effects

Hormone replacement therapy (HRT) involves the use of a small dose


of oestrogen (combined with a progestogen in women with a uterus) to
help alleviate menopausal symptoms.

Side-effects
• nausea
• breast tenderness
• fluid retention and weight gain

Potential complications
• increased risk of breast cancer: increased by the addition of a
progestogen
• increased risk of endometrial cancer: reduced by the addition of a
progestogen but not eliminated completely. The BNF states that
the additional risk is eliminated if a progestogen is given
continuously
• increased risk of venous thromboembol ism: increased by the
addition of a progestogen
• increased risk of stroke
• increased risk of ischaemic heart disease if taken more than 1O
years after menopause

Breast cancer
• in the Women's Health Initiative (WHI) study there was a relative
risk of 1.26 at 5 years of developing breast cancer
• the increased risk relates to duration of use
• breast cancer incidence is higher in women using combined
preparations compared to oestrogen-only preparations
• the risk of breast cancer begins to decline when HRT is stopped
and by 5 years it reaches the same level as in women who have
never taken HRT

Next question )
0 Q7

A 30-year-old woman who is investigated for obesity, hirsutism and


oligomenorrhoea is diagnosed as having polycystic ovarian syndrome
(PCOS) following an ultrasound scan. She is hoping to start a fam ily
and her doctor starts metformin to try and improve her fertility. What is
the mechanism of action of metformin in PCOS?

Stimulates the release of insulin from the pancreas

Blocks the insulin mediated development of multiple immature


follicles in the ovaries

Increases peripheral insulin sensitivity

Blocks the conversion of oestradiol to testosterone

Increases hepatic gluconeogenesis

Submit answer

Reference ranges v
Increases peripheral insulin sensitivity

Blocks the conversion of oestradiol to testosterone

Increases hepatic gluconeogenesis

The majority of patients with polycystic ovarian syndrome have a


degree of insulin resistance which in turn can lead to complicated
changes in the hypothalamic-pituitary-ovarian axis.

,t •t tt Discuss Improve ]

Next question )

Polycystic ovarian syndrome: management

Polycystic ovarian syndrome (PCOS) is a complex condition of ovarian


dysfunction thought to affect between 5-20% of women of reproductive
age. Management is complicated and problem based partly because
the aetiology of PCOS is not fully understood. Both hyperinsulinaemia
and high levels of luteinizing hormone are seen in PCOS and there
appears to be some overlap with the metabolic syndrome.
General
• weight reduction if appropriate
• if a women requires contraception then a combined oral
contraceptive (COC) pill may help regulate her cycle and induce a
monthly bleed (see below)

Hirsutism and acne


• a COC pill may be used help manage hirsutism. Possible options
include a third generation COC which has fewer androgenic
effects or co-cyprindiol which has an anti-androgen action. Both
of these types of COC may carry an increased risk of venous
thromboembolism
• if doesn't respond to COC then topical eflornithine may be tried
• spironolactone, flutamide and flnasteride may be used under
specialist supervision

Infertility
• weight reduction if appropriate
• the management of infertility in patients with PCOS should be
supervised by a specialist. There is an ongoing debate as to
whether metformin, clomifene or a combination should be used to
stimulate ovulation
• a 2007 trial published in the New England Journal of Medicine
suggested clomifene was the most effective treatment. There is a
potential risk of multiple pregnancies with anti-oestrogen*
therapies such as clomifene. The RCOG published an opinion
paper in 2008 and concluded that on current evidence metformin
is not a first line treatment of choice in the management of PCOS
• metformin is also used, either combined with clomifene or alone,
particularly in patients who are obese
• gonadotrophins

*work by occupying hypothalamic oestrogen receptors without


activating them. This interferes with the binding of oestradiol and thus
prevents negative feedback inhibition of FSH secretion
Q8

A 68-year-old female with a background of stage four chronic kidney


disease (CKD) presents with the following blood results to the
nephrology clinic. She has had progressive CKD over many years
following a diagnosis of lgA nephropathy in her third decade.

PTH 19.1 pmol/1{1.05 - 6.83)

Adjusted ca lciu m 2.84 mmol/ 1{2.1-2.6)

What endocrine abnormality is she most likely to have given her


background?

Tertiary hyperparathyroidism

Secondary hyperparathyroidism

Multiple endocrine neoplasia 1 (MEN1)

Pseudohypoparathyroidism

Primary hyperparathyroidism

Submit answer

Reference ranges v
Q8

A 68-year-old female with a background of stage four chronic kidney


disease (CKD) presents with the following blood results to the
nephrology cl ini c. She has had progressive CKD over many years
following a diagnosis of lgA nephropathy in her third decade.

PTH 19.1 pmol/1(1.05 - 6.83)

Adjusted calcium 2.84 mmol/1 (2. 1-2.6)

What endocrine abnormality is she most likely to have given her


background?

I Tertiary hyperparathyroidism

Secondary hyperparathyroidism

Multiple endocrine neoplasia 1 (MEN1)

Pseudohypoparathyroidism

Primary hyperparathyroidism

The correct answer is tertiary hyperparathyroidism. The PTH level is


inappropriate given the high ca lcium level indicating autonomous
hypersecretion of PTH. Although such results could be possible with
primary hyperparathyroidism, the long history of renal impairment
points towards tertiary hyperparathyroidism as the hypertrophied
parathyroid glands no longer respond to serum calcium levels. Tertiary
hyperparathyroidism occurs after long t erm secondary
hyperparathyroidism.
Primary hyperparathyroidism
• PTH over-secretion usually from a parathyroid adenoma
• both PTH and calcium are elevated
• surgery to remove the adenoma is the most effective treatment
• conservative measures such as bisphosphonates can be used

Secondary hyperparathyroidism
• occurs in chronic kidney disease typically
• can be secondary to vitamin D deficiency
• PTH released due to low calcium, high phosphate and lack of
vitamin D activation by diseased kidneys
• PTH level high with calcium levels being low or normal
• medical management primarily: phosphate binders, calcium and
vitamin D supplementation

Tertiary hyperparathyroidism
• autonomous hypersecretion of PTH due to hypertrophied
parathyroid glands
• occurs after a period of long standing secondary
hyperparathyroidism
• treatment involves parathyroidectomy

Pseudohypoparathyroidism is caused by PTH resistance and is


associated with low calcium and high PTH levels. MEN1 is a rare
hereditary disorder involving multiple types of endocrine tissue
neoplasia.

16 •• • Discuss (3) Improve

Next question )
8' f
11
- Discuss (3) Improve

Next question )

Hypercalcaemia: causes

Two conditions account for 90% of cases of hypercalcaemia:


• 1. Primary hyperparathyroidism: commonest cause in non-
hospitalised patients
• 2. Malignancy: the commonest cause in hospitalised patients.
This may be due to number of processes, including; bone
metastases, myeloma, PTHrP from squamous cell lung cancer

Other causes include


• sarcoidosis*
• vitamin D intoxication
• acromegaly
• thyrotoxicosis
• Milk-alkali syndrome
• drugs: thiazides, calcium containing antacids
• dehydration
• Addison's disease
• Paget's disease of the bonett

*other causes of granulomas may lead to hypercalcaemia e.g.


Tuberculosis and histoplasmosis

**usually normal in this condition but hypercalcaemia may occur with


prolonged immobilisation

Next question )
Q9

A 52-year-old woman with suspected diabetes mellitus has an oral


glucose tolerance test, following the standard WHO protocol. The
following results are obtained:

Time (hours) Blood glucose (mmol/1)

0 5.9

2 8.4

How should these results be interpreted?

Impaired fasting glucose and impaired glucose to lerance

Normal

Diabetes mellitus

Impaired glucose tolerance

Impaired fasting glucose

Submit answer

Reference ranges v
Impaired glucose tolerance

Impaired fasting glucose

a6 •t • Discuss (3) Improve

Next question )

Diabetes mellitus (type 2): diagnosis

The diagnosis of type 2 diabetes mellitus can be made by either a


plasma glucose or a HbA1 c sample. Diagnostic criteria vary according
to whether the patient is symptomatic (polyuria, polydipsia etc) or not.

If the patient is symptomatic:


• fasting glucose greater than or equal to 7.0 mmol/ 1
• random glucose greater than or equal to 11.1 mmol/1(or after 75g
oral glucose tolerance test)

If the patient is asymptomatic the above criteria apply but must be


demonstrated on two separate occasions.

Fasting glucose: <=• '-0 mmon >• 7.0 mmol·I

l"n-mabt-1"
Normal 9'Y'CHffllC Control Duioelts--.
H:!Alt Ill ~Z-H l!'mQh nel e ~,I I
0f
c., r.g g\l:oM fl I~ g mmci l

HbA1c· o .c1 rnmOllinOI >-a 41 mmollmol


(5ft) {E ,51M,)

1 o ,aluu equel lo lhG or I e cne OIIIQl"IO$- c


be low.,. COMldered norrNII thu:s."!Ob for Cl8beles •n,_..,.,...,_,
1n 20 1 1 vvAo released s□ pplementary g□ laance on tne use ot AoA Icon
the diagnosis of diabetes:
• a HbA 1 c of greater than or equal to 48 mmol/mol (6.5%) is
diagnostic of diabetes mellitus
• a HbAlc value of less than 48 mmol/mol (6.5%) does not exclude
diabetes (i.e. it is not as sensitive as fasting samples for detecting
diabetes)
• in patients without symptoms, the test must be repeated to
confirm the diagnosis
• it should be remembered that misleading HbA1 c results can be
caused by increased red cell turnover (see below)

Conditions where HbA1 c may not be used for diagnosis:


• haemoglobinopathies
• haemolytic anaemia
• untreated iron deficiency anaemia
• suspected gestational diabetes
• children
• HIV
• chronic kidney disease
• people taking medication that may cause hyperglycaemia (for
example corticosteroids)

Impaired fasting glucose and impaired glucose tolerance

A fasting glucose greater than or equal to 6.1 but less tha n 7.0 mmol/1
implies impaired fasting glucose (IFG)

Impaired glucose tolerance (IGT) is defined as fasting plasma glucose


less than 7.0 mmol/1and OGTT 2-hour value greater than or equal to 7.8
mmol/1 but less than 11 .1 mmol/1

Diabetes UK suggests:
• 'People with IFG should then be offered an ora l glucose tolerance
test to rule out a diagnosis of diabetes. A result below 11.1
mmol/1 but above 7.8 mmol/1indicates that the person doesn't
Q1

What is the most common cause of primary hyperaldosteronism?

Pituitary tumour

Adrenocortical adenoma

Adrenal carcinoma

Ectopic secretion

Bilateral idiopathic adrenal hyperplasia

Submit answer

Reference ranges v
l Bilateral idiopathic adrenal hyperplasia

Bilateral idiopathic adrenal hyperplasia is the most common


cause of primary hyperaldosteronism
Importance: so

[ 8' •• t9 Discuss (1) Improve

Next question )

Primary hyperaldosteronism

Primary hyperaldosteronism was previously thought to be most


1
commonly caused by an adrenal adenoma, termed Conn s syndrome.
However, recent studies have shown that bilateral idiopathic adrenal
hyperplasia is the cause in up to 70% of cases. Differentiating between
the two is important as this determines treatment. Adrenal carcinoma is
an extremely rare cause of primary hyperaldosteronism.

Features
• hypertension
• hypokalaemia
o e.g. muscle weakness
o this is a classical feature in exams but studies suggest this
is seen in only 10-40% of patients
• alkalosis
Investigations
• the 2016 Endocrine Society recommend that a plasma
aldosterone/renin ratio is the first-line investigation in suspected
primary hyperaldosteronism
o should show high aldosterone levels alongside low renin
levels (negative feedback due to sodium retention from
aldosterone)
• following this a high-resolution CT abdomen and adrenal vein
sampling is used to differentiate between unilateral and bilateral
sources of aldosterone excess
• Adrenal Venous Sampling (AVS) can be done to identify the gland
secreting excess hormone in primary hyperaldosteronism

Management
• adrenal adenoma: surgery
• bilateral adrenocortical hyperplasia: aldosterone antagonist e.g.
spironolactone
Q2

A 22-year-old female presents with recurrent painful oral ulceration.


Examination reveals signs of oral Candida I infection. Which one of the
following would most suggest type 1 polyglandular syndrome?

Hypocal caemia

Rheumatoid arthritis

Type II diabetes mellitus

Coeliac disease

Hypercalcaemia

Submit answer

Reference ranges v
Q2

A 22-year-old female presents with recurrent painful oral ulceration.


Examination reveals signs of oral Candida! infection . Which one of the
following would most suggest type 1 polyglandular syndrome?

I Hypocalcaemia

Rheumatoid arthritis

Type II diabetes mellitus

Coeliac disease

Hypercalcaemia

Primary hypoparathyroidism is usually the first endocrine manifestation


of type 1 autoimmune polyendocrinopathy syndrome. The contrast to
multiple endocrine neoplasia (MEN), where hyperparathyroidism is a
common finding, should be noted

The question gives a slightly atypical history as this is the upper end of
the age range in which patients would be expected to present

1
"' f • Discuss (2) Improve

Next question )
Autoimmune polyendocrinopathy syndrome

Addison's disease (autoimmune hypoadrenalism) is associated with


other endocrine deficiencies in approximately 10% of patients. There
are two distinct types of autoimmune polyendocrinopathy syndrome
(APS), with type 2 (sometimes referred to as Schmidt's syndrome) being
much more common .

APS type 2 has a polygenic inheritance and is linked to H LA DR3/ DR4.


Patients have Addison's disease plus either:
• type 1 diabetes mellitus
• autoimmune thyroid disease

APS type 1 is occasionally referred to as Multiple Endocrine Deficiency


Autoimmune Candidiasis (MEDAC). It is a very rare autosomal recessive
disorder caused by mutation of AIRE1 gene on chromosome 21

Feat ures of APS type 1 (2 out of 3 needed)


• chronic mucocutaneous candldlasis (typically first feature as
young chi ld)
• Addison's disease
• primary hypoparathyroidism

Vitiligo can occur in both types

Next question )

B / !! A ...
0 Q3

An 80-year-old man is admitted with a 3 month history of gradual


decline and dizziness on standing, followed by a 3 day history of
inability to mobilise, general weakness and nausea. The medical
consultant asks you to perform a short synact hen test which returns as
normal. Which cause of adrenocortical insufficiency has not been
excluded?

Infiltration of the adrenal gland by amyloidosis

Enlarging pituitary malignancy

Haemorrhage into the adrenal gland

Autoimmune adrenal failure

Infiltration of the adrenal gland by tuberculosis

Submit answer

Reference ranges v
I Enlarging pituita ry malign ancy

Haemo rrhage into the adrenal gland

Autoim mune adrenal failure

Infiltra tion of the adrenal gland by tuberc ulosis

A norma l short synacthen test does not exclude adrenocortical


insuffi ciency due to pituita ry failure
Importance: 50

The short Synacthen test is a metho d of excluding adrenal insufficiency.


A baseline cortiso l level is taken, IV synthe tic ACTH is then
admin istered and a second cortiso l level is taken 30 minutes later. If the
cortiso l post ACTH rises to > 420 nmol/L at 30 minutes, the adrenal
response to ACTH is adequate and Addison's disease (adrenal failure)
can be excluded.

However, this excludes only primar y adrenal failure and does not
exclude cortiso l deficie ncy secondary to failure of the pituita ry to
produc e ACTH . The correc t answer is therefore pituitary failure due to
damag e by an enlarging malignancy. The other answers all cause
damag e to the adrenal gland.

NICE CKS: Addison's disease


https:/ /cks.nice.org.uk/ addisons-disease

[ " •• - Discuss (2) Improve ]

Next questi on )
Next question )

Addison's disease: investigations

In a patient with suspected Addison's disease the definite investigation


is an ACTH stimulation test (short Synacthen test) . Plasma cortisol is
measured before and 30 minutes after giving Synacthen 250ug IM.
Adrenal autoantibodies such as anti-21-hydroxylase may also be
demonstrated.

If an ACTH stimulation test is not readily available (e.g. in primary care)


then sending a 9 am serum cortisol can be useful:
• > 500 nmol/ I makes Addison's very unlikely
• < 100 nmol/I is definitely abnormal
• 100-500 nmol/1 should prompt a ACTH stimulation test to be
performed

Associated electrolyte abnormalities are seen in around one-third of


undiagnosed patients:
• hyperkalaemia
• hyponatraemia
• hypoglycaemia
• metabolic acidosis

Next question )

Save my notes
Q4

A woman presents to the emergency department with confusion. She is


found to be hypothermic with a low blood pressure and bradycardia.
After further examination and investigation, she is found to be in a
myxoedemic coma.

What is the most appropriate first-line treatment for this lady's


presentation?

Adrenaline and levothyroxine

Prednisolone and levothyroxine

Hydrocortisone and levothyroxine

Hydrocortisone and fludrocortisone

Adrenaline and hydrocortisone

Submit answer

Reference ranges v
Prednisolone and levothyroxine

Hyd rocort isone and levothyroxine

Hydrocortisone and fludrocortisone

Adrenaline and hydrocortisone

Myxoedemic coma is treated w ith thyroxine and hydrocortisone


Importance: 50

This question is asking about a woman presenting with confusion,


bradycardia, hypotension who has been diagnosed with a myxoedemic
coma . It is asking for the first initial treatment, thus the correct answer
is hydrocortisone and levothyroxine.

Levothyroxine is used to replace the low levels of thyroid hormone


causing the patient's symptoms.

Hydrocortisone is given to treat adrenal insufficiency. Patients suffering


from a myxoedemic coma due to secondary hypothyroidism are at risk
of hypopituitarism due to the location of the lesion. Thus patients are
treated as presumed adrenal insufficiency until it has been ruled out.

8' •t tit Discuss (2) Improve ]

Next question )
Next question )

Hypothyroidism: features

General
• Weight gain
• Lethargy
• Cold intolerance

Skin
• Dry (anhydrosis), cold, yellowish skin
• Non-pitting oedema (e.g. hands, face)
• Dry, coarse scalp hair, loss of lateral aspect of eyebrows

Gastrointestinal
• Constipation

Gynaecological
• Menorrhagia

Neurological
• Decreased deep tendon reflexes
• Carpal tunnel syndrome

A hoarse voice is also occasionally noted.

Next question )

..,,. T - - •- 1- - - m •- rrT1 - r--:i


QS

A 52-year-old man has a set of fasting bloods as part of a work-up for


hypertension. The fasting glucose comes back as 6.5 mmol/1. The test
is repeated and reported as 6.7 mmol/1. He says he feels constantly
tired but denies any polyuria or polydipsia. How should these results be
interpreted?

Impaired fasting glycaemia

Suggestive of diabetes mellitus but not diagnostic

Diabetes mellitus

Normal

Impaired glucose tolerance

Submit answer

Reference ranges v
A 52-year-old man has a set of fasting bloods as part of a work-up for
hypertension. The fasting glucose comes back as 6.5 mmol/1. The test
is repeated and reported as 6. 7 mmol/1. He says he feels constantly
tired but denies any polyuria or polydipsia. How should these results be
interpreted?

Impaired fasting glycaemia

Suggestive of diabetes mellitus but not diagnostic

Diabetes mellitus

Normal

Impaired glucose tolerance

" •• tit Discuss (3) Improve ]

Next uestion)
Q6

A SS-year-old woman is investigated following an osteoporotic hip


fracture. The following results are obtained:

TSH < 0.05 mu/ I

Free T4 29 pmol/1

Which one of the following autoantibodies is most likely to be present?

TSH receptor stimulating autoantibodies

Anti-nuclear antibodies

Anti-thyroglobulin autoantibodies

Anti-microsomal antibodies

Anti-thyroid peroxidase autoantibodies

Submit answer

Reference ranges v
TSH < 0.05 mu/ I

Free T 4 29 pmol/ 1

Which one of the following autoantibodies is most likely to be present?

TSH receptor stimulating autoantibodies

Anti-nuclear antibodies

Anti-thyroglobulin autoantibodies

Anti-microsomal antibodies

Anti-thyroid peroxidase autoantibodies

TSH receptor stimulating autoantibodies (often referred to as Thyroid


Stimulating lmmunoglobulins) are almost diagnostic of Graves' disease,
the most common cause of thyrotoxicosis in the UK

[ ,6 •• • Discuss {6) Improve ]

Next question )
Next question )

Graves' disease: features

Graves' disease is the most common ca use of thyrotoxicosis. It is


typically seen in women aged 30-50 years.

Features
• typical features of thyrotoxicosis
• specific signs limited to Grave's (see below)

Features seen in Graves' but not in other causes of thyrotoxicosis


• eye signs (30% of patients)
o exophthalmos
o ophthalmoplegia
• pretibial myxoedema
• thyroid acropachy, a triad of:
o digital clubbing
o soft tissue swelling of the hands and feet
o periosteal new bone formation

Autoantibodies
• TSH receptor stimulating antibodies (90%)
• anti-thyroid peroxidase antibodies (75%)

Next question )
Q7 jc::J

A 54-year-old woman presents to the Emergency Depart ment with


confusion and fever. She has a past history of thyrotoxicosis previously
treated with radioiodine therapy. On examination she has a pulse of
120/ min regular, blood pressure 150/ 90 mmHg, temperature of 39.1°c
and a respiratory rate of 18/ min. Examination of the cardiorespiratory
system is unremarkable and urine dipstick is clear. Blood results
showed the following:

Free T4 84 pmol/ 1(normal range 10-22 pmol/1)

Free T3 29 pmol/ 1(2.5-5.5 pmol/1)

TSH < 0.01 mU/1 {0.5-4.0 mU/ 1)

Which one of the following does not have a role in the subsequent
management?

Lugol's iodine

Propranolol

Propylthiouracil

Bicarbonate

Dexamethason e

Submit answer
Free T4 84 pmol/1 (normal range 10-22 pmol/ 1)

Free T3 29 pmol/1 (2.5-5.5 pmol/ 1)

TSH < 0.01 mU/1 (0.5-4.0 mU/ 1)

Which one of the following does not have a role in the subsequent
management?

Lugol 's iodine

Propranolol

Propylthiouracil

Bicarbonate

Dexamethasone

There is no indication for giving bicarbonate in this scenario.

11t If - Discuss (4) Improve ]

Next question )
Thyroid storm

Thyroid storm is a rare but life-threatening complication of


thyrotoxicosis. It is typically seen in patients with established
thyrotoxicosis and is rarely seen as the presenting feature. Iatrogenic
thyroxine excess does not usually result in thyroid storm.

Precipitating events:
• thyroid or non-thyroidal surgery
• trauma
• infection
• acute iodine load e.g. CT contrast media

Clinical features include:


• fever > 38. 5°C
• tachycardia
• confusion and agitation
• nausea and vomiting
• hypertension
• heart failure
• abnormal liver function test - jaundice may be seen clinica lly

Management:
• symptomatic treatment e.g. paracetamol
• treatment of underlying precipitating event
• beta-blockers: typically IV propranolol
• anti-thyroid drugs: e.g. methimazole or propylthiouracil
• Lugol's iodine
• dexamethasone - e.g. 4mg IV qds - blocks the conversion of T4 to
T3

Next question )

'Y
R 1 •
Q8

A 33-year-old woman is referred to the endocrinology clinic with


thyrotoxicosis. Recent blood tests show the following:

TSH < 0.05 mu/I

Free T 4 25 pmol/1

Anti-thyroid peroxidase antibodies 115 IU/ml (< 35 IU/ ml)

A smooth, non-tender goitre is noted on examination the neck. The


patient also has exophthalmos although there is no ophthalmoplegia,
no reduction in visual acuity and no eye symptoms present.

What is the most appropriate management?

Radioiodine treatment

Carbimazole

Propranolol

Fine need le aspiration biopsy of the thyroid gland

Intravenous corticosteroids

Submit answer
Carbimazole

Propranolol

Fine needle aspiration biopsy of the thyroid gland

Intravenous corticosteroids

This patient has Graves' disease as evidenced by the thyrotoxicosis,


goitre, thyroid eye disease and anti-thyroid peroxidase antibodies.

Radioiodine treatment should be avoided given the presence of thyroid


eye disease so carbimazole is a better treatment option.

If her eye disease was severe then an ophthalmologist should be


consulted. Options for severe thyroid eye disease include systemic
steroids and radiotherapy.

9' f
11 _. Discuss (3) Improve

Next question )
Graves' disease: management

Despite many trials there is no clear guidance on the optimal


management of Graves' disease. Treatment options include titration of
anti-thyroid drugs (ATDs, for example carbimazole), block-and-replace
regimes, radioiodine treatment and surgery. Propranolol is often given
initially to block adrenergic effects

ATD titration
• carbimazole is started at 40mg and reduced gradually to maintain
euthyroidism
• typically continued for 12-18 months
• patients following an ATD titration regime have been shown to
suffer fewer side-effects than those on a block-and-replace
regime

Block-and-replace
• carbimazole is started at 40mg
• thyroxine is added when the patient is euthyroid
• treatment typically lasts for 6-9 months

The major complication of carbimazole therapy is agranulocytosls

Radioiodine treatment
• contraindications include pregnancy (should be avoided for 4-6
months following treatment) and age < 16 years. Thyroid eye
disease is a relative contraindication, as it may worsen the
condition
• the proportion of patients who become hypothyroid depends on
the dose given, but as a rule the majority of patient will require
thyroxine supplementation after 5 years

Next question )
Q9

You are conducting the annual review of a 44-year-old woman who has
type 1 diabetes mellitus. You want to assess for diabetic neuropathy
affecting the feet.

What is the most appropriate screening test to use?

A standardi sed questionnaire

Doppler flow studies of the dorsalis pedis pulse

Nerve conduction studies

Test sensation using cotton wool

Test sensation using a 1O g monofilament

Submit answer

Reference ranges v
Test sensation using cotton wool

Test sensation using a 1o g monofllament

A 10 g monofilament should be used to assess for diabetic


neuropathy in the feet
Importance: 50

16 •• tt Discuss (1) Improve

Next question )

Diabetic foot disease

Diabetic foot disease is an important complication of diabetes mellitus


which should be screen for on a regular basis. NICE produced
guidelines relating to diabetic foot disease in 2015.

It occurs secondary to two main factors:


• neuropathy: resulting in loss of protective sensation (e.g. not
noticing a stone in the shoe), Charcot's arthropathy, dry skin
• peripheral arterial disease: diabetes is a risk factor for both macro
and microvascular ischaemia

Presentations
• neuropathy: loss of sensation
• ischaemia: absent foot pulses, reduced ankle-brachia! pressure
index (ABPI), intermittent claudication
• complications: calluses, ulceration, Charcot's arthropathy,
cellulitis, osteomyelitis, gangrene
All patients with diabetes should be screened for diabetic foot disease
on at least an annual basis
• screening for ischaemia: done by palpating for both the dorsalis
pedis pulse and posterial tibial artery pulse
• screening for neuropathy: a 1o g monofllament is used on various
parts of the sole of the foot

NICE recommend that we risk stratify patients:

Moderate
Low risk risk High risk

• no risk • deformity • previous ulceration or


factors except or • previous amputation or
callus alone • • on renal replacement therapy or
neuropathy • neuropathy and non-critical limb
or ischaemia together or
• non- • neuropathy in combination with
critical limb callus and/ or deformity or
ischaemia. • non-critical limb ischaemia in
combination with callus and/or
deformity.

All patients who are moderate or high risk (I.e. any problems other than
simple calluses) should be followed up regularly by the local diabetic
foot centre.

Next question )

B / ~ A .... :: E = .... T I.... fill .... ~ e:-:>

Save my notes J
A 39-year-old woman is reviewed in the clinical pharmacology clinic
following referral by her GP for management of her hypertension. She
has a blood pressure of 159/ 90 mmHg despite 3 oral anti-hypertensive
medications including full dose ramipril. Examination in t he clinic
confirms the elevated blood pressure. Her pulse is 72 and regu lar. Her
chest is clear and her abdomen is soft and non-tender with no palpable
masses. Her body mass index is 28 kg/ m 2 •

Hb 130 g/ 1

Platelets 221 * 109/I

WBC 6.0 * 109 /1

Na+ 141 mmol/I

K+ 3.1 m mol/I

Bicarbonate 31 mmol/I

Urea 4.1 m mol/I

Creat inine 102 µmol/I

Calcium 2.45 mmol/ I

CT abdomen: Right adrenal adenoma, thick walled gallbladder with a


solitary stone

Which of the following is the most likely diagnosis?

Conn's syndrome

Cushing's syndrome

Essential hypertension

Phaeochromocytoma

Renal artery stenosis


Conn's syndrome

Cushing's syndrome

Essential hypertension

Phaeochromocytoma

Renal artery stenosis

Primary hyperaldosteronism can present with hypertension,


hypernatraemia, and hypokalemia
Importance: 50

The relatively normal weight, coupled with hypertension and


hypokalaemic metabolic alkalosis fits well with a diagnosis of Conn's
syndrome, (primary hyperaldosteronism). The right adrenal adenoma is
the likely source of excess aldosterone production. In the presence of
an ACE inhibitor prescribed for hypertension, significant hypokalaemia
is very likely to be related to Conn's.

Cushing's syndrome is unlikely given that the body mass index is only
slightly elevated, and the presence of an adrenal adenoma and
biochemical abnormalities effectively rules out essential hypertension.
Phaeochromocytoma may be associated with hypokalaemia, but is
more likely to be associated with episodic hypertension associated with
bursts of catecholamine release. In renal artery stenosis, a significant
rise in creatinine would be expected in association with the introduction
of the ramipril.

116 If • Discuss (4) Improve

Next question )
Primary hyperaldosteronism

Primary hyperaldosteronism was previously thought to be most


commonly caused by an adrenal adenoma, termed Conn's syndrome.
However, recent studies have shown that bilateral idiopathic adrenal
hyperplasia is the cause in up to 70% of cases. Differentiating between
the two is important as this determines treatment. Adrenal carcinoma is
an extremely rare cause of primary hyperaldosteronism.

Features
• hypertension
• hypokalaemia
o e.g. muscle weakness
o this is a classical feature in exams but studies suggest this
is seen in only 10-40% of patients
• alkalosis

Investigations
• the 2016 Endocrine Society recommend that a plasma
aldosterone/renin ratio is the first-line investigation in suspected
primary hyperaldosteronism
o should show high aldosterone levels alongside low renin
levels (negative feedback due to sodium retention from
aldosterone)
• following this a high-resolution CT abdomen and adrenal vein
sampling is used to differentiate between unilateral and bilateral
sources of aldosterone excess
• Adrenal Venous Sampling (AVS) can be done to identify the gland
secreting excess hormone in primary hyperaldosteronism

Management
• adrenal adenoma: surgery
• bilateral adrenocortical hyperplasia: aldosterone antagonist e.g.
spironolactone
Q11

Mr Bevan is a 52-year-old patient with type 2 diabetes. He was unable to


tolerate metformin due to nausea. He has been doing some of his own
research into other options and suggests an SGLT-2 inhibitor,
empagliflozin, because he has read it might help him lose weight and
improve his blood pressure, as well as improve his blood sugar.

What is the mechanism of action of this drug?

Increase insulin release from pancreas

Decrease glucose absorption in the gut

Decrease glucagon release from pancreas

Increase urinary glucose excretion

Slows gastric emptying

Submit answer

Reference ranges v
Decrease glucose absorption in the gut

Decrease glucagon release from pancreas

Increase urinary glucose excretion

Slows gastric emptying

SGLT-2 inhibitors work by increasing urinary excretion of glucose


(Important as it is the cause of main side effects - increased urine
output, weight loss, UTI)
Importance: 50

Increase insulin release from pancreas - this is the mechanism of action


of sulphonylureas e.g. gliclazide.

Decrease glucose absorption in the gut - this is the mechanism of


action of acarbose, which is not routinely prescribed in the UK.

Decrease glucagon release from pancreas - DPP4-inhibitors reduce the


breakdown of incretins, which thereby decreases glucagon secretion.

SGLT-2 inhibitors such as empagliflozin work by reducing glucose


reabsorption in the proximal convoluted tubule, therefore increasing the
amount of glucose excreted in the urine. An additional 70g of glucose a
day (approximately) is excreted; as well as improving blood sugar this is
also likely to lead to weight loss, in contrast to some other diabetic
medications such as sulphonylureas and insulin which cause weight
gain. The slight diuresis caused by increased glucose excretion may
also improve blood pressure. Unfortunately, increased glucose in the
urine can also cause adverse events such as urinary tract or genital
infections.

SGLT-2 inhibitors do not slow gastric emptying.

r • I •• I • Discuss I Improve l
[ ,6 •t • Discuss Improve ]

Next question )

SGLT-2 inhibitors

SGLT-2 inhibitors reversibly inhibit sodium-glucose co-transporter 2


(SGLT-2) in the renal proximal convoluted tubule to reduce glucose
reabsorption and increase urinary glucose excretion.

Examples include canagliflozin, dapagliflozin and empagliflozin.

Important adverse effects include


• urinary and genital infection (secondary to glycosuria). Fournier's
gangrene has also been reported
• normoglycaemic ketoacidosis
• increased risk of lower-limb amputation: feet should be closely
monitored

Patients taking SGLT-2 drugs often lose weight, which can be beneficial
in type 2 diabetes mellitus.

Next question )

B I A ... ., _
·- 1-
~
- - ....

Save my notes
Q12

A 33-year-old white male attends his GP with a two month history of


weight loss, lethargy and polydipsia. He has a past medical history of a
thyroidectomy for Grave's disease, no significant family history and
currently takes levothyroxine. The GP does a capillary glucose
measurement, which is 18.1 mmol/ L and does a urinalysis revealing 2+
glucose and 2+ ketones. His blood pressure is 134/86mmHg. What is
the most likely diagnosis?

Type 2 diabetes mellitus

Addison's disease

Latent Autoimmune Diabetes of Adulthood

Maturity Onset Diabetes of the Young

Levothyroxine-induced diabetes mellitus

Submit answer

Reference ranges v
I Latent Autoimmune Diabetes of Adulthood

Maturity Onset Diabetes of the Young

Levothyroxine-induced diabetes mellitus

Latent autoimmune diabetes of adulthood (LADA) is a disorder in


which, despite the presence of islet antibodies at diagnosis of diabetes,
the progression of autoimmune -cell failure is slow.

In contrast to type 2 diabetes, patients are typically younger and without


an increased body habitus. In contrast to type 1 diabetes, insulin is not
usually required in the early stages of the disease.

Diagnosis may be aided through a Glutamic Acid Decarboxylase (GAD)


Autoantibodies test and evidence of other autoimmune diseases.

Levothyroxine is not associated with inducing diabetes. In patients with


diabetes starting thyroxine, doses of antidiabetic drugs including insulin
may need to be increased.

1
Addison s disease is associated with hypoglycaemia.

8' f
1 - Discuss (5) Improve

Next question )

Diabetes mellitus: a very basic introduction

Diabetes mellitus is one of the most common conditions encountered


in clinical practice and represents a significant burden on the health
systems of the developed world. It is now estimated that 8% of the total
NHS budget is now spent on managing patients with diabetes mellitus.
What is diabetes mellitus?

rised
Diabetes melli tus may be defined as a chronic cond ition char acte
by abnormally raised levels of blood glucose.

?
Why is the man agem ent of diabetes mellitus so important

wou ld
Before the advent of insulin therapy untreated type 1 diab etes
can still
usually result in death. Poorly treated type 1 diab etes mell itus
etic
result in signi fican t morb idity and mort ality (as a resu lt of diab
ent now
ketoacidosis). However, the main focu s of diabetes man agem
se,
is reducing the incidence of mac rova scula r (isch aem ic hear t disea
stroke) and micr ovas cular (eye, nerve and kidney damage)
com plica tions .

Type Notes

Type 1 Auto imm une disor der where the insul in-pr oduc ing
diabetes beta cells of the islets of Langerhans in the
mellitus pancreas are destroyed by the imm une syste m
(T1D M) This results in an abso lute defic iency of insulin
resulting in raised gluco se levels
Patients tend to develop T1 OM in child hood/ early
adult life and typic ally pres ent unwell, poss ibly in
diab etic ketoacidosis

Type 2 This is the mos t com mon cause of diab etes in the
diabetes developed world . It is caused by a relative
mellitus deficiency of insulin due to an excess of adip ose
(T2D M) tissu e. In simp le term s there isn't enough insulin to
'go around' all the excess fatty tissue, leading to
blood gluco se creeping up.
Prediabetes This term is used for patients who don't yet meet
the criteria for a formal diagnosis of T2DM to be
made but are likely to develop the condition over
the next few years . They, therefore, require closer
monitoring and lifestyle interventions such as
weight loss

Gestational Some pregnant develop raised glucose levels


diabetes during pregnancy. This is important to detect as
untreated it may lead to adverse outcomes for the
mother and baby

Maturity A group of inherited genetic disorders affecting the


onset production of insulin. Results in younger patients
diabetes of developing symptoms similar to those with T2DM,
the young i.e. asymptomatic hyperglycaemia with progression
{MODY) to more severe complications such as diabetic
ketoacidosis

Latent The majority of patients with autoimmune-related


autoimmune diabetes present younger in life. There are however
diabetes of a small group of patients who develop such
adults problems later in life. These patients are often
(LADA) misdiagnosed as having T2DM

Other types Any pathological process which damages the


insulin-producing cells of the pancreas may cause
diabetes to develop. Examples include chronic
pancreatitis and haemochromatosis.

Drugs may also cause raised glucose levels. A


common example is glucocorticoids which
commonly result in raised blood glucose levels
Symptoms and signs

The presentation of diabetes mellitus depends very much on the type:

Type 1 DM Type 2 DM

Weight loss Often picked up incidentally on


Polydipsia routine blood tests
Polyuria Polydipsia
Polyuria
May present with diabetic
ketoacidosis
• abdominal pain
• vomiting
• reduced
consciousness level

Remember that the polyuria and polydipsia are due to water being
'dragged' out of the body due to the osmotic effects of excess blood
glucose being excreted in the urine (glycosuria).

Investigations

There are 4 main ways to check blood glucose:


• a finger-prick bedside glucose monitor
• a one-off blood glucose. Th is may either be fasting or non-fasting
• a HbA1c. This measures the amount of glycosylated haemoglobin
and represents the average blood glucose over the past 2-3
months
• a glucose tolerance test. In this test, a fasting blood glucose is
taken after which a 75g glucose load is taken. After 2 hours a
second blood glucose reading is then taken
The diagnostic criteria are determined by WHO.

If the patient is symptomatic:


• fasting glucose greater than or equal to 7.0 mmol/I
• random glucose greater than or equal to 11.1 mmol/I (or after 75g
oral glucose tolerance test)

If the patient is asymptomatic the above criteria apply but must be


demonstrated on two separate occasions.

In 2011 WHO released supplementary guidance on the use of HbA 1c


for the diagnosis of diabetes:
• a HbA1c of greater than or equal to 6.5% (48 mmol/mol) is
diagnostic of diabetes mellitus
• a HbAlc value of less than 6.5% does not exclude diabetes (i.e. it
is not as sensitive as fasting samples for detecting diabetes)
• in patients without symptoms, the test must be repeated to
confirm the diagnosis
• it should be remembered that misleading HbA 1c results can be
caused by increased red cell turnover

f asting glucose: <a: so mmol,1 >• 7.0 mmol l

,,edabetn
Honn.al flVcNmlC control r o-...--.
HoA1c a .&;'-'T nTn:l fflCI ,e c..e ~ l
or
ea1~~ g\lCOM I! 1~ p Ml!'ICl(.I

HbA1c; <a.,r~W)
mmOlmol

I L' ~luu ~ :0 thit Of I e lne OJ,li!i)n0$'C


low are com10ered nom,ml t n r ~ for ~les

Diagram showing the spectrum of diabetes diagnosis


Management

The principle of managing diabetes mellitus are as follows:


• drug therapy to normalise blood glucose levels
• monitoring for and treating any complications related to diabetes
• modifying any other risk factors for other conditions such as
cardiovascular disease

Type 1 diabetes
• patients always require insulin to control the blood sugar levels.
This is because there is an absolute deficiency of insulin with no
pancreatic tissue left to stimulate with drugs
• different types of insulin are available according to their duration
of action

Type 2 diabetes
• the majority of patients with type 2 diabetes are controlled using
oral medication
• the first-line drug for the vast majority of patients is metformin
• second-line drugs include sulfonylureas, gliptins and pioglitazone.
Please see the table below for further information
• if oral medication is not controlling the blood glucose to a
sufficient degree then insulin is used
Mechanism of Main side-
Drug class action Route effects Notes

Insulin Direct Subcutaneous Hypoglycaemia Used in all


replacement for Weight gain patients with
endogenous Lipodystrophy T1 OM and some
insulin patients with
poorly controlled
T2DM

Can be
classified
according to
source
(analogue,
human
sequence and
porcine) and
duration of
action (short,
immediate, long-
acting)

CamScanner ~ Y~ 4!>~1
Metformin Increases insulin Oral Gastrointestinal First-line
sensitivity upset medication in
Decreases Lactic the managemen1
hepatic acidosis* ofT2DM
gluconeogenesis
Cannot be used
in patients with
an eGFR of< 30
ml/min

Sulfonylureas stimulate Oral Hypoglycaemia Examples


pancreatic beta Weight gain include
cells to secrete Hyponatraemia gliclazide and
insulin glimepiride

Thiazolidinediones Activate PPAR- Oral Weight gain Only currently


gamma receptor Fluid retention available
in adipocytes to thiazolidinedione
promote is pioglitazone
adipogenesis
and fatty acid
uptake

CamScanner -; Y~ b - ~ I
DPP-4 inhibitors (- Increases Oral Generally well
gliptins) incretin levels tolerated but
which inhibit increased risk
glucagon of pancreatitis
secretion

SGLT-2 inhibitors Inhibits Oral Urinary tract Typically result


(-gliflozins) reabsorption of infection in weight loss
glucose in the
kidney

GLP-1 agonists (- lncretin mimetic Subcutaneous Nausea and Typically result


tides) which inhibits vomiting in weight loss
glucagon Pancreatitis
secretion

NICE provide guidelines on how drug therapy should be used in T2DM:

Metfonnln Melformln not toteroi.ci « Cl

CamSca nner -; Y~ b - ~ I
Metformin Metformin not tolerated or Cl

Metformin Glitpln
OR
Sulfonylurea
OR
HbA1c > 58 mmovmol (7 5%) Pk>glltazone

!
t.tetformln + gllpt1n HbA1c > 58 mmovmo1 (7.5%}
OR
Metformm + sulfonyrurea
OR
Metform,n + p109htazone Ghplln • p1oghtazone
OR OR
Metformln + SGLT-2 Inhibitor Gllptm + sulfonyturea
OR
PloglttaZone + sulfonylUrea
HbA 1C > 58 mmoVmol (7 5°0)

t.tetrormtn + gllptln + su1rony1urea


OR
Metform,n • ptoglltazone + sulfonylurea HbA1c > 58 mmolmol (7 .5°0)
OR
Metformln + sulfonylurea + SGLT -2 lnh1bnor
OR
Metform,n + ploglitazone + SGLT-2 Inhibitor
Insulin
Triple therapy *nor effec(/1/e not to erated or contra nd,ca1ed·
AND
BMI >35

i
Metformln + sulfonylurea + GLP-1 mimetic C Passmed,cme corn
0 Q13

An 18-year-old girl is admitted to the Emergency Department with an


episode of sweating and dizziness. She is brought in by her f ather who
has type 2 diabetes mellitus as he is worried she may be diabetic. He
describes a number of similar episodes for the past two weeks. Her BM
on admission is 1.9 mmol/ 1so the following bloods are taken:

Plasma glucose 1.8 mmol/1

Insulin 15 mg/ ml {6-10 mg/ ml)

Prolnsulln 22% (22-24%)

C-peptide 0.15 nmol/ 1(0.2-0.4 nmol/1)

What is the most likely diagnosis?

Diabetes mellitus

lnsulinoma

Nesidioblastosis

Insulin abuse

Sulfonylurea abuse

Submit answer

Reference ranges v
Insulin abuse

Sulfonylurea abuse

The raised insulin with low c-peptide level points to a diagnosis of


insulin abuse. C-peptide levels would be raised in a patient following
sulfonylurea abuse

at •• tt Discuss (5) Improve ]

Next question )

Hypoglycaemia

Causes
• insulinoma - increased ratio of proinsulln to insulin
• self-administration of insulin/sulphonylureas
• liver failure
• Addison's disease
• alcohol

Other possible causes in children


• nesidioblastosis - beta cell hyperplasia

Physiological response to hypoglycaemia


• hormonal response: the first response of the body is decreased
insulin secretion. This is followed by increased glucagon
secretion. Growth hormone and cortisol are also released but later
• sympathoadrenal response: increased catecholamine-mediated
(adrenergic) and acetylcholine-mediated (cholinergic)
neurotransmission in the peripheral autonomic nervous system
and in the central nervous system

Next uestion )
Q14

Each one of the following is associated with Pendred's syndrome,


except

Goitre

Short 4th and 5th metacarpals

Autosomal recessive inheritance

Sensorineural deafness

Euthyroid status

Submit answer

Reference ranges v
Short 4th and 5th metacarpals

Autosomal recessive inheritance

Sensorineural deafness

Euthyroid status

9' 'f • Discuss (2) Improve

Next question )

Pendred's syndrome

Pendred is an autosomal recessive genetic disorder that is


characterised by bilateral sensorineural deafness, with mild
hypothyroidism and a goitre. The patients tend to present with
progressive hearing loss and delay in academic progression. Often
head trauma tends to make the sensorineural deafness worse, leading
to patients having to avoid contact sports.

In Pendred syndrome there is a defect in the organification of iodine,


leading to dyshormonogenesis. However thyroid symptoms in pendred
syndrome are often mild and patients are often clinical ly euthyroid,
presenting only with a goitre. Thyroid function tests are also often
normal, requiring the perchlorate discharge test to aid diagnosis.

The syndrome can be diagnosed via genetic testing (Pendred syndrome


(PDS) gene, chromosome 7), audiometry and MRI imaging to look for
characteristic one and a half turns in the cochlea, compared to the
normal two and a half turns.

Treatment is with thyroid hormone replacement and cochlear implants.


Q15

The first-line treatment in remnant hyperlipidaemia


(dysbetalipoproteinaemia) is:

Ursodeoxycholic acid

Vitamin A

Statins

Fish oil

Fibrates

Submit answer

Reference ranges v
IFibrates

9' .. • Discuss Improve

Next question )

Remnant hyperlipidaemia

Overview
• rare cause of mixed hyperlipidaemia (raised cholesterol and
triglyceride levels)
• also known as Fredrickson type Ill hyperlipidaemia, broad-beta
disease and dysbetalipoproteinaemia
• associated with apo-e2 homozygosity
• high incidence of ischaemic heart disease and peripheral vascular
disease
• thought to be caused by impaired removal of intermediate density
lipoprotein from the circulation by the liver

Features
• yellow palmar creases
• palmer xanthomas
• tuberous xanthomas

Management
• flbrates are first line treatment

Next question )
Q16 [Cl

You are on the post-take ward round reviewing a 55-year-old lady who
has been admitted with her third urinary tract infection in as many
months. Of note she is a type II diabetic patient and was commenced
on empagliflozin by her GP four months ago. You suspect recurrent
urinary tract infections secondary to her empagliflozin, a sodium
glucose co-transporter 2 inhibitor used in the treatment of type 2
diabetes mellitus. Where is its main site of action?

Early distal convoluted tubule

Collecting duct

Early prox imal convoluted tubule

Late proximal convoluted tubule

Late distal convoluted tubule

Submit answer

Reference ranges v
Early proximal convoluted tubule

Late proximal convoluted tubule

Late distal convoluted tubule

SGLT-2 inhibitors reversibly inhibit sodium-glucose co-transporter


2 (SGLT-2) in the renal proximal convoluted tubule
Importance: 50

In the normal kidney up to 180 grams/ day of glucose is filtered by the


renal glomerulus and virtually all of it is reabsorbed in the proximal
convoluted tubule. This reabsorption is carried out by two sodium-
dependent glucose co-transporter (SGLT) proteins, SGLT1, which
reabsorbs 10%, and SGLT2, which reabsorbs the remaining 90%. While
SGLT1 is expressed elsewhere in the body, SGLT2 is expressed solely in
the kidney, making it an attractive target for novel diabetic treatments.

To date a number of SGLT2 inhibitors have been developed. These


agents have been shown to enhance renal glucose excretion by
inhibiting renal glucose reabsorption with consequent improvements in
HbA1C and insulin resistance. Furthermore, they have been shown to
have protective effects in the progression of chronic kidney disease,
blood pressure lowering effects and reduce cardiovascular events in
high risk type II diabetic patients. To date they are they only oral
hypoglycaemic agent to demonstrate any benefit on macrovascular
outcomes in type II diabetic patients.

Important side effects to be aware of with this class of drug are genital
tract infections and euglycaemic diabetic ketoacidosis .

,, •• • Discuss (3) Improve

Next question )
••
• i passmedicine.com/ quest ion/ qu, * ♦ ➔ 0

SGLT-2 inhibitors

SGLT-2 inhibitors reversibly inhibit sodium-glucose co-transporter 2


(SGLT-2) in the renal proximal convoluted tubule to reduce glucose
reabsorption and increase urinary glucose excretion.

Examples include canagliflozin, dapagliflozin and empagliflozin.

Important adverse effects include


• urinary and genital infection (secondary to glycosuria). Fournier's
gangrene has also been reported
• normoglycaemic ketoacidosis
• increased risk of lower-limb amputation: feet should be closely
monitored

Patients taking SGLT-2 drugs often lose weight, which can be beneficial
in type 2 diabetes mellitus.

Next question )

B / ~
1
2-
= _
-
T
TI T

Save my notes

Search
Q17

A 71-year-old woman with a history of type 2 diabetes mellitus presents


with lethargy and polyuria. A diagnosis of hyperosmolar hyperglycaemic
state is considered. Which one of the following findings would be least
consistent with this diagnosis?

pH of 7.38

Ketones 1 + in urine

Serum osmolality of 31 O mosmol/ kg

Serum bicarbonate of 19 mmol/ I

Glucose of 45 mmol/ I

Submit answer

Reference ranges v
OCI 01 I I OJI I IOIUiilj 01 JI O I I 1031 I 101/ l(Q

Serum bicarbonate of 19 mmol/ 1

Glucose of 45 mmol/1

A trace of ketones may be found in hyperosmolar hyperglycaemic state.


Serum osmolality is typically > 320 mosmol/kg

"' If t9 Discuss (7) Improve

Next question >

Hyperosmolar hyperglycaemic state

Hyperosmolar hyperglycaemic state (HHS) is a medical emergency


which is extremely difficult to manage and has a significant associated
mortality. Hyperglycaemia results in osmotic diuresis, severe
dehydration, and electrolyte deficiencies. HHS typically presents in the
elderly with type 2 diabetes mellitus (T2DM), however the incidence in
younger adults is increasing. It can be the initial presentation of T2OM.

It is extremely important to differentiate HHS from diabetic


ketoacidosis (OKA) as the management is different, and treatment of
HHS with insulin (e.g. as part of a OKA protocol) can result in adverse
outcomes. The first 24 hours of treatment is very labour intensive so
these patients are best managed in either a medical high dependency
unit.

HHS has a higher mortality than OKA and may be complicated by


vascular complications such as myocardial infarction, stroke or
peripheral arterial thrombosis. Seizures, cerebral oedema and central
pontine myelinolysis (CPM) are uncommon but documented
complications of HHS. Whilst OKA presents within hours of onset, HHS
comes on over many days, and consequently the dehydration and
metabolic disturbances are more extreme.
Pathophys1oiogy
• Hyperglycaemia results in osmotic diuresis with associated loss
of sodium and potassium
• Severe volume depletion results in a significant raised serum
osmolarity (typically> than 320 mosmol/kg), resulting in
hyperviscosity of blood.
• Despite these severe electrolyte losses and total body volume
depletion, the typical patient with HHS, may not look as
dehydrated as they are, because hypertonicity leads to
preservation of intravascular volume.

Clinical features
• General: fatigue, lethargy, nausea and vomiting
• Neurological: altered level of consciousness, headaches,
papilloedema, weakness
• Haematological: hyperviscosity (may result in myocardial
infarctions, stroke and peripheral arterial thrombosis)
• Cardiovascular: dehydration, hypotension, tachycardia

Diagnosis
• 1. Hypovolaemia
• 2. Marked Hyperglycaemia (>30 mmol/L) without significant
ketonaemia or acidosis
• 3. Significantly raised serum osmolarity (> 320 mosmol/kg)
• Note: A precise definition of HHS does not exist, however the
above 3 criteria are helpful in distinguishing between HHS and
OKA. It is also important to remember that a mixed HHS / OKA
picture can occur.

Management

The goals of management of HHS can be summarised as follows:

• 1 . Normalise the osmolality (gradually)


• 2. Replace fluid and electrolyte losses
• 3. Normalise blood olucose (qraduallv)
Fluid replacement
• Fluid losses in HHS are estimated to be between 100 - 220 ml/kg
(e.g. 10-22 litres in an individual weighing 100 kg) .
• The rate of rehydration will be determined by assessing the
combination of initial severity and any pre-existing co-morbidities
(e.g. heart failure and chronic kidney disease). Caution is needed,
particularly in the elderly, where too rapid rehydration may
precipitate heart failure but insufficient may fail to reverse an
acute kidney injury.
• Intravenous (IV) 0.9% sodium chloride solution is the first line fluid
for restoring total body fluid .
• It is important to remember that isotonic 0.9% sodium chloride
solution is already relatively hypotonic compared to the serum in
someone with HHS. Therefore in most cases it is very effective at
restoring normal serum osmolarity.
• If the serum osmolarity is not declining despite positive balance
with O. 9% sodium chloride, then the fluid should be switched to
0.45% sodium chloride solution which is more hypotonic relative
to the HHS patients serum osmolarity
• IV fluid replacement should aim to achieve a positive balance of 3-
6 litres by 12 hours and the remaining replacement of estimated
fluid losses within the next 12 hours.
• Existing guidelines encourage vigorous initial fluid replacement
and this alone (without insulin) will result in a gradual decline in
plasma glucose and serum osmolarity. A rapid decline is
potentially harmful (see below) therefore insulin should NOT be
used in the first instance unless there is significant ketonaemia or
acidosis
• The aim of treatment should be to replace approximately 50% of
estimated fluid loss within the first 12 hours and the remainder in
the following 12 hours. However this is just a guide, and clinical
judgement should be applied, particularly in patient with co-
morbidities such as heart failure and chronic kidney disease
(which may limit the speed of correction).
Monitoring response to treatment
• The key parameter in managing HHS is the osmolality to which
glucose and sodium are the main contributors. Rapid changes of
serum osmolarity are dangerous and can result in cardiovascular
collapse and central pontine myelinolysis (CPM).
• Guidelines suggest that serum osmolarity, sodium and glucose
levels should be plotted on a graph to permit appreciation of the
rate of change. They should be plotted hourly initially.
• Not all laboratories have readily available access to serum
osmolarity measurements. If not available then a calculated
osmolarity can be estimated with 2Na + glucose + urea
• Fluid replacement alone (without insulin) will gradually lower
blood glucose which will reduce osmolality
• A reduction of serum osmolarity will cause a shift of water into
the intracellular space. This inevitably results in a rise in serum
sodium (a fall in blood glucose of 5.5 mmol/L will result in a 2.4
mmol/ L rise in sodium). This is not necessarily an indication to
give hypotonic solutions. If the inevitable rise in serum Na+ is
much greater than 2.4 mmol/ L for each 5.5 mmol/L fall in blood
glucose this would suggest insufficient fluid replacement. Rising
sodium is only a concern if the osmolality is NOT declining
concurrently.
• Rapid changes must be avoided. A safe rate of fall of plasma
glucose of between 4 and 6 mmol/hr is recommended. The rate of
fall of plasma sodium should not exceed 1 O mmol/L in 24 hours.
• A target blood glucose of between 1 O and 1 5 mmol/L is a
reasonable goal.
• Complete normalisation of electrolytes and osmolality may take
up to 72 hours.
Insulin
• Fluid replacement alone with 0.9% sodium chloride solution will
result in a gradual decline of blood glucose and osmolarity
• Because most patients with HHS are insulin sensitive (e.g. it
usually occurs in T2DM), administration of insulin can result in a
rapid decline of serum glucose and thus osmolarity.
• Insulin treatment prior to adequate fluid replacement may result in
cardiovascular collapse as the water moves out of the
intravascular space, with a resulting decline in intravascular
volume.
• A steep decline in serum osmolarity may also precipitate CPM.
• Measurement of ketones is essential for determining if insulin is
required.
• If significant ketonaemia is present (3~-hydroxy butyrate is more
than 1 mmol/L) this indicates relative hypoinsulinaemia and
insulin should be started at time zero (e.g. mixed DKA / HHS
picture). The recommended insulin dose is a fixed rate
intravenous insulin infusion given at 0.05 units per kg per hour.
• If significant ketonaemia is not present (3~-hydroxy butyrate is
less than 1 mmol/L) then do NOT start insulin.

Potassium
• Patients with HHS are potassium deplete but less acidotic than
those with DKA so potassium shifts are less pronounced
• Hyperkalaemia can be present with acute kidney injury
• Patients on diuretics may be profoundly hypokalaemic
• Potassium should be replaced or omitted as required

Next question )

B / /!!j A,..

Save my notes
Q18

A 43-year-old man is found to have a phaeochromocytoma. Which anti-


hypertensive medication should be started first?

Propranolol

Ramipril

Atenolol

Phenoxybenzamine

Doxazosin

Submit answer

Reference ranges v
Ramipril

Atenolol

Phenoxybenzamine

Doxazosin

PHaeochromocytoma - give PHenoxybenzami ne before beta-


blockers
Importance: 50

Phenoxybenzamine is a non-selective alpha-adrenoceptor ant agonist


and should be started before a beta-blocker is introduced

There is ongoing debate about the optimal medical management of


phaeochromocytoma, with the suggestion that antihypertensive
treatment regimes other than non specific alpha-blockade are just as
effective and safe. There are however no trials to provide an answer to
this question yet

16 11 f tt Discuss (7) Improve

I
Next question )
'

-•- - - - - •- -- - -- - - - - _._ - -- - -
Phaeochromocytoma

Phaeochromocytoma is a rare catecholamine secreting tumour. About


10% are familial and may be associated with MEN type II,
neuroflbromatosls and von Hlppel-Lindau syndrome

Basics
• bilateral in 10%
• malignant in 10%
• extra-adrenal in 10% (most common site = organ of Zuckerkandl,
adjacent to the bifurcation of the aorta)

Features are typically episodic


• hypertension (around 90% of cases, may be sustained)
• headaches
• palpitations
• sweating
• anxiety

Tests
• 24 hr urinary collection of metanephrines (sensitivity 97%*)
• this has replaced a 24 hr urinary collection of catecholamines
(sensitivity 86%)

Surgery is the definitive management. The patient must first however be


stabilized with medical management:
• alpha-blocker (e.g. phenoxybenzamine), given before a
• beta-blocker (e.g. propranolol)

*BMJ 2012; 344 doi: http://dx.doi.org/ 10.1136/ bmj.e1042 (Published


20 February 2012)

Next question )
Q19

A 48-year-old male presents to the clinic. He complains of excessive


flatulence following starting a new agent for management of his type 2
diabetes mellitus. He has no other past medical history.

What drug is most likely implicated?

Metformin

Gliclazide

Acarbose

Sitagliptin

Empagliflozin

Submit answer

Reference ranges v
Acarbose

Sitagliptin

Empagliflozin

Excessive flatulence is an extremely common side effect of


acarbose which is often poorly tolerated
Importance: 50

Acarbose is an inhibitor of intestinal alpha glucosidases, which results


in decreased absorption of starch and sucrose. Since acarbose
prevents the degradation and absorption of complex carbohydrates into
glucose, an increased carbohydrate load will be delivered to the colon.
In the colon, bacteria digest the complex carbohydrates, causing
gastrointestinal side-effects such as flatulence and diarrhoea.
Excessive flatulence is the most commonly reported side effect and is
often the reason for discontinuation of the drug.

Metformin can also cause gastrointestinal side effects, however it is


often diarrhoea rather than excessive flatulence, making acarbose the
better answer.

Gliclazide is a sulphonylurea agent. The sulfonylureas act mainly by


augmenting insulin secretion and consequently are effective only when
some residual pancreatic beta-cell activity is present; during long-term
administration they also have an extrapancreatic action. Side effects
include hypoglycaemia, cholestatic jaundice and diarrhoea.

Sitagliptin is a dipeptidyl peptidase-4 (DPP-4) inhibitor. Side effects


include peripheral oedema and diarrhoea.

Empagliflozin is an inhibitor of the sodium glucose co-transporter-2


(SGLT-2). It cause increased glucose excretion in the urine which
predisposed to recurrent urinary tract infections.
T2DM medications

Drugs used
• sulphonylureas
• metformin
• alpha-glucosidase inhibitors (acarbose)
• glitazones
• insulin

Next question )

B / tj A ..,. ,-
i:: -- ....

Save my notes

Search


Search tex1book ...

Q. Google search on "T2DM medications"


Q20

A 35-year-old female has recently been diagnosed with Addison's


disease due to autoimmune adrenal failure after presenting with a 3-
month history of lethargy, nausea, weight loss and fainting. Which of the
following physical signs may you find in this patient?

Stretch marks on her abdomen

Multiple bruises on her limbs

Frontal balding

Thinning of the axillary hair

Cafe au lait spots

Submit answer
I

Reference ranges v
Multiple bruises on her limbs

Frontal balding

Thinn ing of the axillary hair

Cafe au lait spots

Thinning of pubic and axillary hair is seen in females with


Addison's disease due to reduced production of testosterones
from the adrenal gland
Importance: 50

The correct answer is thinning of the axillary hair. The patient has failure
of the adrenal gland due to autoimmune attack. As well as deficiency of
glucocorticoids and mineralocorticoids, she will have lower levels of
androgens, which are usually produced in females by the zona
reticularis of the adrenal cortex. This leads to thinning of hair grown at
puberty, which is androgen dependent. The scalp hair is unaffected.

Stretch marks, skin thinning, easy bruising and poor wound healing are
found in Cushing's disease or in patients given longterm exogenous
steroids. Cafe au lait spots are seen in neurofibromatosis type I.

NICE CKS. Addison's disease.

[ ,6 •• • Discuss (2) Improve ]

Next question )
Addison's disease

Autoimmune destruction of the adrenal glands is the commonest cause


of primary hypoadrenalism in the UK, accounting for 80% of cases. This
is termed Addison's disease and results in reduced cortisol and
aldosterone being produced.

Features
• lethargy, weakness, anorexia, nausea & vomiting, weight loss, 'salt-
craving'
• hyperpigmentation (especially palmar creases)*, vitiligo, loss of
pubic hair in women, hypotension, hypoglycaemia
• hyponatraemia and hyperkalaemia may be seen
• crisis : collapse, shock, pyrexia

Other causes of hypoadrenalism

Primary causes
• tuberculosis
• metastases ( e.g. bronchial carcinoma)
• meningococcal septicaemia (Waterhouse-Friderichsen syndrome)
• HIV
• antiphospholipid syndrome

Secondary causes
• pituitary disorders (e.g. tumours, irradiation, infiltration)

Exogenous glucocorticoid therapy

* Primary Addison's is associated with hyperpigmentation whereas


secondary adrenal insufficiency is not

Next question )
Q1

Which one of the following regarding the management of thyroid


problems during pregnancy is incorrect?

Maternal free thyroxine levels should be kept in the upper third of


the normal reference range when treating thyrotoxicosls

Increased levels of thyroxine-binding globulin are seen in


pregnancy

Block-and-replace is preferable in pregnancy compared to


antithyroid drug titration

Breast feeding is safe whilst on thyroxine

Untreated thyrotoxicosis increases the risk of premature labour

Submit answer

Reference ranges v
Increased levels of thyroxine-binding globulin are seen in pregnancy

Block-and-replace is preferable in pregnancy compared to


antithyroid drug titration

Breast feeding is safe whilst on thyroxine

Untreated thyrotoxicosis increases the risk of premature labour

[ ,. •• • Discuss (1) Improve ]

Next question )

Pregnancy: thyroid problems

In pregnancy, there is an increase in the levels of thyroxine-binding


globulin (TBG). This causes an increase in the levels of total thyroxine
but does not affect the free thyroxine level.

Thyrotoxicosis,

Untreated thyrotoxicosis increases the risk of fetal loss, maternal heart


failure and premature labour

Graves' disease is the most common cause of thyrotoxicosis in


pregnancy. It is also recognised that activation of the TSH receptor by
HCG may also occur - often termed transient gestational
hyperthyroidism. HCG levels will fall in the second and third trimester
Management
• propylthiouracil has traditionally been the antithyroid drug of
choice
• however, propylthiouracil is associated with an increased risk of
severe hepatic injury
• therefore NICE Clinical Knowledge Summaries advocate the
following : 'Propylthiouracil is used in the first trimester of pregnancy
in place of carbimazole, as the latter drug may be associated with
an increased risk of congenital abnormalities. At the beginning of
the second trimester, the woman should be switched back to
carbimazole'
• maternal free thyroxine levels should be kept in the upper third of
the normal reference range to avoid fetal hypothyroidism
• thyrotrophin receptor stimulating antibodies should be checked at
30-36 weeks gestation - helps to determine the risk of neonatal
thyroid problems
• block-and-replace regimes should not be used in pregnancy
• radioiodine therapy is contraindicated

Hypothyroidism

Key points
• thyroxine is safe during pregnancy
• serum thyroid stimulating hormone measured in each trimester
and 6-8 weeks post-partum
• some women require an increased dose of thyroxine during
pregnancy
• breast feeding is safe whilst on thyroxine

Next question )

("',..., ,,... .................... ,...+,........ I


Q2

A 62-year-old man is investigated for hypertension and proximal


myopathy. On examination he is noted to have abdominal striae. Which
one of the following is most associated with ectopic ACTH secretion?

Carcinoid tumour

Small cell lung cancer

Cardiac myxoma

Squamous cell lung cancer

Adrenal carcinoma

Submit answer

Reference ranges v
Small cell lung cancer

Cardiac myxoma

Squamous cell lung cancer

Adrenal carcinoma

Small cell lung cancer accounts 50-75% of case of ectopic ACTH


Importance: 50

Adrenal carcinoma and cardiac myxoma are causes of ACTH


independent Cushing's syndrome

11• 11f tt Discuss (1) Improve

Next question )

CamScanne, -? Y.¥-> i.b . ~I


WWW I ► DIOCUOO ( I/ 111: p ao ec

Next question )

Cushing's syndrome: causes

It should be noted that exogenous causes of Cushing's syndrome (e.g.


glucocorticoid therapy) are far more common than endogenous ones.

ACTH dependent causes


• Cushing's disease (80%): pituitary tumour secreting ACTH
producing adrenal hyperplasia
• ectopic ACTH production (5-10%): e.g. small cell lung cancer

ACTHindependentcauses
• iatrogenic: steroids
• adrenal adenoma (5-10%)
• adrenal carcinoma (rare)
• Carney complex: syndrome including cardiac myxoma
• micronodular adrenal dysplasia (very rare)

Pseudo-Cushing's
• mimics Cushing's
• often due to alcohol excess or severe depression
• causes false positive dexamethasone suppression test or 24 hr
urinary free cortisol
• insulin stress test may be used to differentiate

Next question )

Save mv notes
Q3

A 78-year-old nursing home resident is admitted to the acute medical


unit after being found collapsed in his room. A carer from the nursing
home is present and reports that he has had regular 'hypos' recently. On
admission he was drowsy and the blood glucose was 1.8 mmol/1.
Following intravenous dextrose the patient's condition significantly
improved.

His medication on admission is as follows:

Metformln 1g bd
Gliclazide 160mg od
Ploglitazone 45mg od
Aspirin 75mg od
Simvastatin 40mg on

What is the most appropriate initial action?

Stop metformin

Stop pioglltazone

Stop gliclazide

Make no changes to the medication

Stop all oral antidiabetic medications

Submit answer

Reference ranges v
His medication on admission is as follows:

Metformin 1g bd
Gliclazide 160mg od
Pioglitazone 45mg od
Aspirin 75mg od
Slmvastatin 40mg on

What is the most appropriate initial action?

rStop metformin

Stop pioglitazone

Stop gliclazide

Make no changes to the medication

Stop all oral antidiabetic medications

Neither metformin nor pioglitazone cause hypoglycaemia . The


gliclazide dose is therefore responsible and should be stopped whilst
deciding upon longer term changes to his medication.

a6 1f tl Discuss (5) Improve ]

Next question )
0 Q4

A 57-year-old woman is referred to urogynaecology with symptoms of


urge incontinence. A trial of bladder retraining is unsuccessful. It is
therefore decided to use a muscarinic antagonist.

Which one of the following medications is an example of a muscarinic


antagonist?

To!terodine

Teriparatide

Toremifene

Finasteride

Tamsulosin

Submit answer

Reference ranges v
I Tolterodine

Teriparatide

Toremifene

Finasteride

Tamsulosin

Other examples of muscarinic antagonists used in urinary incontinence


include oxybutynin and solifenacin. Examples of muscarinic
antagonists used in different conditions include ipratropium (chronic
obstructive pulmonary disease) and procyclidine (Parkinson's disease).

Tamsulosin is an alpha blocker.

•• If • Discuss Improve

Next question )

Urinary incontinence

Urinary incontinence (UI) is a common problem, affecting around 4-5%


of the population. It is more common in elderly females.

Risk factors
• advancing age
• previous pregnancy and childbirth
• high body mass index
• hysterectomy
• family history
Classification
• overactive bladder (OAB)/urge incontinence: due to detrusor
overactivity
• stress incontinence: leaking small amount s when coughing or
laughing
• mixed incontinence: both urge and stress
• overflow incontinence: due to bladder outlet obstruction, e.g. due
to prostate enlargement

Initial investigation
• bladder diaries should be completed for a minimum of 3 days
• vaginal examination to exclude pelvic organ prolapse and ability to
initiate voluntary contraction of pelvic floor muscles ('Kegel'
exercises)
• urine dipstick and culture
• urodynamic studies

Management depends on whether urge or stress UI is the predominant


picture. If urge incontinence is predominant:
• bladder retraining (lasts for a minimum of 6 weeks, the idea is to
gradually increase the intervals between voiding)
• bladder stabilising drugs: antimuscarinics are first-line. NICE
recommend oxybutynin (immediate release), tolterodine
(immediate release) or darifenacin (once daily preparation).
Immediate release oxybutynin should, however, be avoided in 'frail
older women'
• mirabegron (a beta-3 agonist) may be useful if there is concern
about anticholinergic side-effects in frail elderly patients

If stress incontinence is predominant:


• pelvic floor muscle training: NICE recommend at least 8
contractions performed 3 times per day for a minimum of 3
months
• surgical procedures: e.g. retropubic mid-urethral tape procedures
QS

What causes increased sweating in patients with acromegaly?

Increased sodium content in sweat

Raised basal metabolic rate

Episodic hypoglycaemia

Low-grade chronic pyrexia

Sweat gland hypertrophy

Submit answer

Reference ranges v
I Sweat gland hypertrophy

Acromegaly: increased sweating is caused by sweat gland


hypertrophy
Importance: 50

9' •• • Discuss Improve

Next question )

Acromegaly: features

In acromegaly there is excess growth hormone secondary to a pituitary


adenoma in over 95% of cases. A minority of cases are caused by
ectopic GHRH or GH production by tumours e.g. pancreatic.

Features
• coarse facial appearance, spade-like hands, increase in shoe size
• large tongue, prognathism, interdental spaces
• excessive sweating and oily skin: caused by sweat gland
hypertrophy
• features of pituitary tumour: hypopituitarism, headaches,
bitemporal hemianopia
• raised prolactin in 1/ 3 of cases - galactorrhoea
• 6% of patients have MEN-1

Complications
• hypertension
• diabetes (>10%)
• cardiomyopathy
• colorectal cancer
Q6

A 40-year-old woman complains of feeling tired all the t ime and putting
on weight. On examination a diffuse, non-tender goitre is noted. Blood
tests are ordered:

TSH 15_ 1 mU/I

Free T4 7.1 pmol/I

ESR 14 mm/hr

Antl-TSH receptor stimulating antibodies Negative

Anti-thyroid peroxidase antibodies Positive

What is the most likely diagnosis?

Pituitary failure

Primary atrophic hypothyroidism

De Quervain's thyroiditis

Hashimoto's thyroiditis

Grave's disease

Submit answer
De Quervain's thyroiditi s

Hashimoto's thyroiditis

Grave's disease

Hashimoto's thyroiditis =hypothyroidism+ goitre+ anti-TPO


Importance: 50

This patient has Hashimoto's thyroiditis, as evidenced by the


hypothyroidism, goitre and anti-thyroid peroxidase antibodies. De
Quervain's thyroiditis typically causes a painful goitre and a raised ESR.
Around 90% of patients with Grave's disease have anti-TSH receptor
stimulating antibodies.

" •• tll Discuss (1) Improve ]

Next question )

Hashimoto's thyroiditis

Hashimoto's thyroiditis (chronic autoimmune thyroiditis) is an


autoimmune disorder of the thyroid gland . It is typically associated with
1

hypothyroidism although there may be a transient thyrotoxicosis in the


acute phase. It is 1O times more common in women

Features
• features of hypothyroidism
• goitre: firm, non-tender
• anti-thyroid peroxidase and also anti-Tg antibodies
Hypothyroidism Hyperthyrodism

Hashimoto1s
thyroiditis Subacute thyroiditis
{ma) b€- Iii"'} b..c:! II, , .,to•Jt (de Quervain's) Graves' disease
pt,ase II iat,•) (lrutlal 01,,er ll}'pert rolCJ pnase •ouov,eo ~

longer h~potll)'rold pflase)


Amiodarone Toxic multinodular goitre
Riedel's thyroiditis
Postpartum thyroiditis
Iodine deficiency :lit-I ~ ... n, •.er: .,ro,d pl'la:.t! 'OllCnell b-y
longer fl)potnyrold prtas.e)

Lithium

C Passmedlcme corn
Q7

A 24-year-old woman is found to have a blood pressure of 170/100


mmHg during a routine medical check. She is well and clinica l
examination is unremarkable. Blood tests show:

Na+ 140 mmol/I

K+ 2.6 mmol/I

Bicarbonate 31 mmol/I

Urea 3.4 mmol/I

Creatinine 77 µmol/I

Which one of the following investigations is most likely to be


diagnostic?

Renal ultrasound

Overnight dexamethasone suppression test

Renin:aldosterone ratio

MR angiography

21-hydroxylase estimation

Submit answer
Renin :aldosterone ratio

MR angiography

21-hydroxylase estimation

A plasma aldosterone/renin ratio is the first-line investigation in


suspect ed primary hyperaldosteronism
Importance: 50

Conn's syndrome is the likely diagnosis - a renin:aldosterone ratio would


be an appropriate first-line investigation. A normal clinical examination
makes a diagnosis of Cushing's syndrome less likely

9' •• ,- Discuss (4) Improve

Next question )

Primary hyperaldosteronism

Primary hyperaldosteronism was previously thought to be most


commonly caused by an adrenal adenoma, termed Conn's syndrome.
However, recent studies have shown that bilateral idiopathic adrenal
hyperplasia is the cause in up to 70% of cases. Differentiating between
the two is important as this determines treatment. Adrenal carcinoma is
an extremely rare cause of primary hyperaldosteronism.

Features
• hypertension
• hypokalaemia
o e.g. muscle weakness
o this is a classical feature in exams but studies suggest this
is seen in only 10-40% of patients
• alkalosis
Investigations
• the 2016 Endocrine Society recommend that a plasma
aldosterone/renin ratio is the first-line investigation in suspected
primary hyperaldosteronism
o should show high aldosterone levels alongside low renin
levels (negative feedback due to sodium retention from
aldosterone)
• following this a high-resolution CT abdomen and adrenal vein
sampling is used to differentiate between unilateral and bilateral
sources of aldosterone excess
• Adrenal Venous Sampling (AVS) can be done to identify the gland
secreting excess hormone in primary hyperaldosteronism

Management
• adrenal adenoma: surgery
• bilateral adrenocortical hyperplasia: aldosterone antagonist e.g.
spironolactone
Q8

A 60-year-old female, who is a known type 2 diabetic, presents to the GP


for a diabetes review. She is already on metformin and her GP decided
to start her on a sulphonylurea to help gain better control of her blood
sugar. What is the mechanism of action of this medication?

Closes calcium channels on the beta cells

Opens calcium channels on the beta cells

Inhibits sodium transport in the beta cells

Opens potassium-ATP channels on the beta cells

Closes potassium-ATP channels on the beta cells

Submit answer

Reference ranges v
l Closes potassium-ATP channels on the beta cell s

Sulfonylureas - bind to an ATP-dependent K+(KATP) channel on the


cell membrane of pancreatic beta cells
Importance: 50

Sulphonylureas work via mimicking the role of ATP on potassium-ATP


channels from the outside. They act to block these channels causing
membrane depolarisation and thus opening of voltage-gated calcium
channels. This process results in the stimulation of insulin release.

When used acutely they increase insulin secretion and decrease insulin
clearance in the liver. However, due to this stimulation of insulin
secretion they can cause hypoglycaemia, the main side effect, and this
can lead to the serious complication of neuroglycopenia.

This resulting lack of glucose supply to the brain can cause confusion
and possible coma. Treatment of this should be through oral glucose,
intramuscular glucagon or intravenous glucose.

If the tissue is exposed chronically to sulphonylureas there is no acute


increase in insulin release but a decrease in plasma glucose
concentration does remain . Chronic exposure to sulphonylureas does
also lead to down-regulation of their receptors.

[ ,. " • Discuss Improve ]

Next question )
.. I ,,.,,,,,, I J l '-'~UV'-' I I I lt-'I 'VY'-'

Next question )

Sulfonylureas

Sulfonylureas are oral hypoglycaemic drugs used in the management of


type 2 diabetes mellitus. They work by increasing pancreatic insulin
secretion and hence are only effective if functional B-cells are present.
On a molecular level they bind to an ATP-dependent K+(KAr P) channel
on the cell membrane of pancreatic beta cells .

Common adverse effects


• hypoglycaemic episodes (more common with long-acting
preparations such as chlorpropamide)
• weight gain

Rarer adverse effects


• hyponatraemia secondary to syndrome of inappropriate ADH
secretion
• bone marrow suppression
• hepatotoxicity (typically cholestatic)
• peripheral neuropathy

Sulfonylureas shou ld be avoided in breastfeeding and pregnancy.

Next question )

B I l!!J A ...
Q9

A 73-year-old man is seen in incontinence clinic by a speciality doctor.


He has a past medical history of urge incontinence. He has tried
oxybutynin in the past but did not find it helpful. He didn't tolerate
tolterodine either due to his longstanding constipation. The doctor
prescribes a 6- week course of mirabegron.

What is the mechanism of action of mirabegron?

Beta-2 agonist

Beta-1 agonist

Beta-3 agonist

Alpha-1 agonist

Alpha-1 antagonist

Submit answer

Reference ranges v
Beta-1 agonist

Beta-3 agonist

Alpha-1 agonist

Alpha-1 antagonist

Mirabegron is a beta-3 agonist


Importance: 50

Mirabegron is a beta-3 agonist used in the management of urge


incontinence if the other drugs such as oxybutynin fail to work or are
contraindicated.

Salbutamol is a beta-2 receptor agonist and causes bronchial smooth


muscle relaxation . Beta-2 receptors are predominantly found in the
lungs.

Beta-1 agonists, such as dobutamine, are used as inotropic agents in


congestive heart failure.

Doxazosin is an alpha-1 antagonist, used in the management of


hypertension.

Phenylephrine is an alpha-1 agonist. It acts as a vasoconstrictor and is


used as a nasal decongestant.

,6 'f • Discuss Improve ]

Next question )

Ca mSc.an,ner ~ '....,.i;lo-" ~ ~ I
Urinary incontinence

Urinary incontinence (UI) is a common problem, affecting around 4-5%


of the population. It is more common in elderly females.

Risk factors
• advancing age
• previous pregnancy and childbirth
• high body mass index
• hysterectomy
• family history

Classification
• overactive bladder (OAB)/ urge incontinence: due to detrusor
overactivity
• stress incontinence: leaking small amounts when coughing or
laughing
• mixed incontinence: both urge and stress
• overflow incontinence: due to bladder outlet obstruction, e.g. due
to prostate enlargement

Initial investigation
• bladder diaries should be completed for a minimum of 3 days
• vaginal examination to exclude pelvic organ prolapse and ability to
initiate voluntary contraction of pelvic floor muscles ('Kegel'
exercises)
• urine dipstick and culture
• urodynamic studies
Management depends on whether urge or stress UI is the predominant
picture. If urge incontinence is predominant:
• bladder retraining (lasts for a minimum of 6 weeks, the idea is to
gradually increase the intervals between voiding)
• bladder stabilising drugs: antimuscarinics are first-line. NICE
recommend oxybutynin (immediate release), tolterodine
(immediate release) or darifenacin (once daily preparation).
Immediate release oxybutynin should, however, be avoided in 'frail
older women'
• mirabegron (a beta-3 agonist) may be useful if there is concern
about anticholinergic side-effects in frail elderly patients

If stress incontinence is predominant:


• pelvic floor muscle training : NICE recommend at least 8
contractions performed 3 times per day for a minimum of 3
months
• surgical procedures: e.g. retropubic mid-urethral tape procedures

Next question )

B I ~ A ... ·-
1-
i:
- - ...

Save my not es

Search
Q10

A 20-year-old woman with a history of type 1 diabetes since the age of 8


comes to the Emergency department with nausea, vomiting, weight loss
and frequent episodes of hypoglycaemia. She has been treated with a
basal bolus regime of insulin since diagnosis and usually has very
stable diabetes control. On examination, her blood pressure is 105/70
mmHg with a postural drop of 15 mmHg. Her pulse is 74 beats per
minute and regular. Her body mass index is 21 kg/m 2 •

Investigations

Na+ 127 mmol/1

K+ 5.0 mmol/1

Urea 11.2 mmol/1

Creatinine 122 µmol/1

Glucose 4.8 mmol/ 1

TSH 10.2 IU/I

Free thyroxine 7 pmol/1

Which of the following is the most important intervention?

Fluid restriction

IV hydrocortisone

IV normal saline

Oral fludrocortisone

Oral thyroxine
IV hydrocortisone

IV normal saline

Oral fludrocortisone

Oral thyroxine

The hyponatraemia and potassium towards the upper end of the normal
range, coupled with hypoglycaemia, fit well with a diagnosis of
Addison's disease. Although features of hypothyroidism may co-exist
with hypoadrenalism, corticosteroid replacement is the most important
first step in therapy because commencing thyroxine may worsen any
adrenal crisis.

Fluid restriction is not appropriate given signs of volume depletion and


the likelihood of Addison's being the primary diagnosis. Although fluid
replacement with normal saline may be useful in relieving symptoms of
volume depletion, it is unlikely to be effective without commencing
hydrocortisone therapy. Oral fludrocortisone is added to hydrocortisone
in patients who are corticosteroid replete but still suffer from symptoms
of hyponatraemia or volume depletion.

16 11, tit Discuss (4) Improve

Next question )
Addison's disease

Autoimmune destruction of the adrenal glands is the commonest cause


of primary hypoadrenalism in the UK, accounting for 80% of cases. This
is termed Addison's disease and results in reduced cortisol and
aldosterone being produced.

Features
• lethargy, weakness, anorexia, nausea & vomiting, weight loss, 'salt-
craving'
• hyperpigmentation (especially palmar creases)*, vitiligo, loss of
pubic hair in women, hypotension, hypoglycaemia
• hyponatraemia and hyperkalaemia may be seen
• crisis: collapse, shock, pyrexia

Other causes of hypoadrenalism

Primary causes
• tuberculosis
• metastases (e.g. bronchial carcinoma)
• meningococcal septicaemia (Waterhouse-Friderichsen syndrome)
• HIV
• antiphospholipid syndrome

Secondary causes
• pituitary disorders (e.g. tumours, irradiation, infiltration)

Exogenous glucocorticoid therapy

*Primary Addison's is associated with hyperpigmentation whereas


secondary adrenal insufficiency is not

Next question )
Q11

A 21-year-old with type 1 diabetes was admitted with abdominal pain


and vomiting. She had been having dysuria and urine dip showed ++
nitrites and ++ leucocytes. Her heart rate was 90 bpm and blood
pressure was 112/80 mmHg. Capillary glucose was 28 mmol/ I and
capillary ketones were 5.1 mmol/1. A venous gas was obt ained which
showed:

pH 7.25

Bicarbonate 12 mmol/1

Base excess -3.8

Lactate 2.9 mmol/1

Potassium 6.0 mmol/ 1

She was started on IV fluids and fixed-rate IV insulin. Her capillary


glucose and ketones had improved significantly after 24 hours of
treatment, however she gradually started to become confused, irritable
and was slurri ng her words . A repeat venous gas showed:

pH 7.32

Bicarbonate 17 mmol/1

Base excess -2.0

Lactate 2.3 mmol/1

Potassium 3.1 mmol/1


Lactate 2.3 mmol/1

Potassium 3.1 mmol/1

What is the most likely cause of her new neurological symptoms?

Stroke

Encephalopathy

Cerebral oedema

Sepsis

Hypokalaemia

I
Submit answer

Reference ranges v
Cerebral oedema

Sepsis

Hypokalaemia

Cerebral oedema is an important complication of fluid


resuscitation in OKA, especially in young patients
Importance: 50

Children/young adults are particularly vulnerable to cerebral oedema


following fluid resuscitation in OKA and often need 1:1 nursing to
monitor neuro-observations, headache, irritability, visua l disturbance,
focal neurology etc.

It usually occurs 4-12 hours following commencement of treatment but


can be present at any time.

If any suspicion, request a CT head and call senior immediately.

••
11
f • Discuss (4) Improve

Next question >

Diabetic ketoacidosis

Diabetic ketoacidosis (OKA) may be a complication existing type 1


diabetes mellitus or be the first presentation, accounting for around 6%
of cases. Rarely, under conditions of extreme stress, patients with type
2 diabetes mellitus may also develop OKA.

Whilst OKA remains a serious condition mortality rates have decreased


from 8% to under 1% in the past 20 years.
Pathophysiology
• OKA is caused by uncontrolled lipolysis (not proteolysis) which
results in an excess of free fatty acids that are ultimately
converted to ketone bodies

The most common precipitating factors of OKA are infection, missed


insulin doses and myocardial infarction.

Features
• abdominal pain
• polyuria, polydipsia, dehydration
• Kussmaul respiration (deep hyperventilation)
• Acetone-smelling breath ('pear drops' smell)

Diagnostic criteria

American Diabetes Joint British Diabetes Societies


Association (2009) (2013)

Key points Key points


• glucose > 13.8 • glucose > 11 mmol/I or known
mmol/I diabetes mellitus
• pH< 7.30 • pH < 7.3
• serum bicarbonate • bicarbonate< 15 mmol/I
<18 mmol/ I • ketones > 3 mmol/ I or urine
• anion gap> 10 ketones ++ on dipstick
• ketonaemia

Management
• fluid replacement: most patients with OKA are deplete around 5-8
litres. Isotonic saline is used initially. Please see an example fluid
regime below.
• insulin: an intravenous infusion should be started at 0.1
unit/kg/ hour. Once blood glucose is < 15 mmol/ I an infusion of 5%
unit/ kg/ hour. Once blood glucose is< 15 mmol/ I an infusion of 5%
dextrose should be started
• correction of hypokalaemia
• long-acting insulin should be continued, short-acting insulin
should be stopped

JBDS example of fluid replacement regime for patient with a systolic


BP on admission 90mmHg and over

Fluid Volume

0. 9% sodium chloride 1 L 1000ml over 1 st hour

0. 9% sodium chloride 1 L with potassium 1000ml over next 2


chloride hours

0.9% sodium chloride 1 L with potassium 1000ml over next 2


chloride hours

0. 9% sodium chloride 1 L with potassium 1000ml over next 4


chloride hours

0. 9% sodium chloride 1 L with potassium 1000ml over next 4


chloride hours

0. 9% sodium chloride 1 L with potassium 1000ml over next 6


chloride hours

Please note that slower infusion may be indicated in young adults (aged
18-25 years) as they are at greater risk of cerebral oedema.
JBDS potassium guidelines

Potassium level in first 24 Potassium replacement in mmol/L


hours (mmol/L) of infusion solution

Over 5.5 Nil

3.5-5.5 40

Below 3.5 Senior review as additional


potassium needs to be given

Complications of OKA and its treatment


• gastric stasis
• thromboembolism
• arrhythmias secondary to hyperkalaemia/latrogenic hypokalaemia
• iatrogenic due to incorrect fluid therapy: cerebral oedema*,
hypokalaemia, hypoglycaemia
• acute respiratory distress syndrome
• acute kidney injury

* children/young adults are particularly vulnerable to cerebral oedema


following fluid resuscitation in DKA and often need 1:1 nursing to
monitor neuro-observations, headache, irritability, visual disturbance,
focal neurology etc. It usually occurs 4-12 hours following
commencement of treatment but can present at any time. If there is any
suspicion a CT head and senior review should be sought

Next question )

B / ~ A ... ,-
i=: -- ... T I"'
Q12

A 45-year-old female is admitted to the Emergency Department with


abdominal pain associated with vomiting. She has a past medical
history of hypothyroidism and takes thyroxine. On examination she is
pyrexial at 37.6°C. Pulse is 11 O bpm with a blood pressure of 100/64
mmHg. Blood results show the following :

Na+ 131 mmol/1

K+ 4.9 mmol/1

Urea 8.1 mmol/1

Creatinine 11 O µmol/1

Glucose 3.3 mmol/1

What treatment should be given first?

Ceftriaxone + benzylpenicillin

Glucagon

Propranolol

Triiodothyronine

Hydrocortisone

Submit answer
Na+ 131 rnmol/1

K+ 4.9 mmol/1

Urea 8.1 mmol/1

Creat lnlne 11 O µmol/1

Glucose 3 .3 mmol/ 1

What treatment should be given first?

~ eftriaxone + benzylpenicillin

Glucagon

Propranolol

Triiodothyronine

I Hydrocortisone

This is a typical history of Addison's. Patients may have a history of


other autoimmune conditions such as thyroid disorders. Steroids
should be given as soon as possible

[ ,6 If tt Discuss (1 1) Improve ]

Next question )
Q13

Which one of the following drugs is least likely to cause


gynaecomastia?

Sp ironolactone

Sodium va lproate

Digoxin

Cimetidine

Anabolic steroids

Submit answer

Reference ranges v
Sodium valproate

Digoxin

Cimetidine

Anabolic steroids

Whilst sodium valproate may rarely causes gynaecomastia it is much


more common after taking the other listed drugs.

[ ,, If • Discuss (2) Improve ]

Next question )
Gynaecomastia

Gynaecomastia describes an abnormal amount of breast tissue in


males and is usually caused by an increased oestrogen:androgen ratio.
It is important to differentiate the causes of galactorrhoea (due to the
actions of prolactin on breast tissue) from those of gynaecomastia

Causes of gynaecomastia
• physiological: normal in puberty
• syndromes with androgen deficiency: Kallman's, Klinefelter's
• testicular failure: e.g. mumps
• liver disease
• testicular cancer e.g. seminoma secreting hCG
• ectopic tumour secretion
• hyperthyroidism
• haemodialysis
• drugs: see below

Drug causes of gynaecomastia


• spironolactone (most common drug cause)
• cimetidine
• digoxin
• cannabis
• finasteride
• gonadorelin analogues e.g. Goserelin, buserelin
• oestrogens, anabolic steroids

Very rare drug causes of gynaecomastia


• tricyclics
• isoniazid
• calcium channel blockers
• heroin
• busulfan
• methyldopa
Q14

A 34-year-old woman is referred to the endocrinology clinic with


symptoms of anxiety and significant weight loss. She reports feeling
unusually hot all the time. Blood tests are as follows:

TSH 0.03 (0.27 - 4.20 mU/1)

Free T4 37 (10.0 - 23.0 pmol/1)

The endocrinologist arranges a nuclear scintigraphy scan which reveals


patchy uptake.

What is the most likely diagnosis?

Grave's disease

Hashimoto's disease

Iodine deficiency

Solitary adenoma

Toxic multinodular goitre

Submit answer

Ref erence ranaes v


1vrn1 ,e ae11c1e1 ,cy

Solitary adenoma

I Toxic multinodular goitre


-------~

In toxic multinodular goitre, nuclear scintigraphy reveals patchy


uptake
Importance: 50

The combination of hyperthyroid symptoms, blood results, and patchy


uptake on nuclear scintigraphy point towards a diagnosis of toxic
multinodular goitre. This is a common cause of hyperthyroidism in
which there is excess production of thyroid hormones from functionally
autonomous thyroid nodules.

In Graves disease, scintigraphy would show diffuse enlargement of both


thyroid lobes, with uniform uptake throughout. Grave's is an
autoimmune disorder and the most common cause of hyperthyroidism.

Hashimoto's disease typically presents with symptoms of


hypothyroidism as opposed to hyperthyroidism. Hashimoto's is an
autoimmune condition.

Iodine deficiency leads to hypothyroidism, as iodine is used to make


thyroid hormones. This is uncommon in the developed world but may
be seen in developing countries.

A solitary adenoma would present similarly to toxic multinodular goitre,


but you would expect nuclear scintigraphy to show a small focus of
uptake.

9' •t .- Discuss Improve

Next question )
Next question )

Toxic multinodular goitre

Toxic multinodular goitre describes a thyroid gland that contains a


number of autonomously functioning thyroid nodules resulting in
hyperthyroidism.

Nuclear scintigraphy reveals patchy uptake.

The treatment of choice is radioiodine therapy.

Next question )

B I l!j A ,., == 1-
~
=,., T I "'

Save my notes

Search
Q15

A middle-aged man with type 2 diabetes mellitus is reviewed. Despite


weight loss, metformin and gliclazide his HbA 1c is 68 mmol/mol
(8.4%). The patient agrees to start insulin therapy. According to NICE
guidelines which type of insulin should be tried initially?

Basal bolus regime

lsophane (NPH insulin)

Biphasic insulin

Glargine

Detemir

Submit answer

Reference ranges v
lsophane (NPH insulin)

Biphasic insulin

Glargine

Detemir

From NICE Clinical Knowledge Summaries:

The National Institute for Health and Care Excellence (NICE)


recommends that in adults with type 2 diabetes:
• Neutral Protamine Hagedorn (NPH) insulin [also known as
isophane insulin] (injected once or twice daily according to
need) should be offered.
• NPH plus a short-acting insulin should be considered
(particularly if the person's HbA1c is 75 mmol/mol [9.0%] or
higher). This may be administered either separately or as a
pre-mixed (biphasic) human insulin preparation.
• Insulin detemir or insulin glargine should be considered as an
alternative to NPH insulin if:
• - The person needs assistance from a carer or healthcare
professional to inject insulin and the use of insulin detemir or
insulin glargine would reduce the frequency of injections
from twice to once dailY, or
• - The person's lifestyle is restricted by recurrent
symptomatic hypoglycaemic episodes, or
• _, The person would otherwise need twice-daily NPH insulin
injections in combination with oral antidiabetic drugs.
Q16 Jr:::J

One of your patients is diagnosed with having the metabolic syndrome.


Which one of the following is associated with this condition?

Endometriosis

Hypothyroidism

Asymptomatic rise in amylase levels

Elevated albumin levels

Raised uric acid levels

Submit answer

Reference ranges v
l Raised uric acid levels

,, •• • Discuss (4) Improve

Next question )

Metabolic syndrome

Unfortunately there are a number of competing definitions of the


metabolic syndrome around at the present time. It is thought that the
key pathophysiological factor is insulin resistance.

SIGN recommend using criteria similar to those from the American


Heart Association. The similarity of the International Diabetes
Federation criteria should be noted. For a diagnosis of metabolic
syndrome at least 3 of the following should be identified:
• elevated waist circumference: men > 102 cm, women > 88 cm
• elevated triglycerides: > 1.7 mmol/ L
• reduced HDL: < 1.03 mmol/ L in males and< 1.29 mmol/L in
females
• raised blood pressure: > 130/85 mmHg, or active treatment of
hypertension
• raised fasting plasma glucose > 5.6 mmol/L, or previously
diagnosed type 2 diabetes

The International Diabetes Federation produced a consensus set of


diagnostic criteria in 2005, which are now widely in use. These require
the presence of central obesity (defined as waist circumference> 94cm
for Europid men and > 80cm for Europid women, with ethnicity specific
values for other groups) plus any two of the following four factors:
• raised triglycerides level: > 1.7 mmol/ L, or specific treatment for
this lipid abnormality
• reduced HDL cholesterol : < 1.03 mmol/L in males and< 1.29
• reduced HDL cholesterol: < 1.03 mmo l/L in male s and < 1 .29
mmo l/L in females, or speci fic treatm ent for this lipid abno rmali ty
• raised blood pressure: > 130/ 85 mm Hg, or active treatm ent of
hypertension
• raised fastin g plasm a gluco se> 5.6 mmo l/ L, or previously
diagnosed type 2 diabetes

In 1999 the World Health Organization produced diagn ostic criter ia


which required the presence of diabetes mellitus, impaired gluco se
tolerance, impaired fastin g gluco se or insuli n resistance, AND two of
the follow ing:
• blood pressure: > 140/ 90 mmH g
• dyslipidaemia: triglycerides: > 1.695 mmo l/ L and/ or high-dens ity
lipoprotein cholesterol (HDL-C) < 0. 9 mmo l/ L (male), < 1.0 mmo l/L
(female)
• central obesity: waist :hip ratio > 0.90 (male), > 0.85 (fema le),
and/ or body mass index > 30 kg/m 2
• microalbuminuria : urinary albumin excretion ratio > 20 mg/ min or
albumin:creatinine ratio > 30 mg/ g

Other associated features include:


• raised uric acid levels
• non-alcoholic fatty liver disease
• polycystic ovarian syndrome

Next ques tion )

Save my notes

CamScanner ~ ·. .;~ .__;.,..a..:,


Q17

A 62-year-old HGV driver is reviewed. He was diagnosed last year with


type 2 diabetes mellitus. Following weight loss and metformin his
HbA 1c has decreased from 74 mmol/mol (8.9%) to 68 mmol/ mol
(8.4%). What is the most suitable next step in management?

Add exenatide

Make no changes to management

Add gliclazide

Stop metformin for a period to ensure hypoglycaemic awareness


is not lost

Add pioglitazone

Submit answer

Reference ranges v
Make no changes to management

Add gliclazide

Stop metformin for a period to ensure hypoglycaemic awareness is


not lost

l Add pioglitazone

Pioglitazone is the best option here as it would not put him at risk of
hypoglycaemia, which obviously could be dangerous given his job. The
NICE guidelines would also support the use of a DPP-4 inhibitor (e.g.
sitagliptin or vildagliptin) in this situation.

[ ,t •t tt Discuss (8) Improve ]

Next question )
Q18

A 49-year-old woman is investigated for thyrotoxicosis. On examination


she is noted to have a goitre conta ining multiple irregular nodules.
Nuclear scintigraphy with technetium 99m reveals patchy uptake. What
is the treatment of choice?

Corticosteroids

Radioiodine

Block-and-replace regime

Surgery

Anti-thyroid drug titration regime

Submit answer

Reference ranges v
Radioiodine

Block-and-replace regime

surgery

Anti-thyroid drug titration regime

9' •• tt Discuss (5) Improve

Next question >

Toxic multinodular goitre

Toxic multinodular goitre describes a thyroid gland that contains a


number of autonomously functioning thyroid nodules resulting in
hyperthyroidism.

Nuclear scintigraphy reveals patchy uptake.

The treatment of choice is radioiodine therapy.

Next question )

B I A ... ·- 1-
~
=. .

Save my notes
Q19

A 54-year-old man has a routine medical for work. He is asymptomatic


and clinical examination is unremarkable. Which of the following results
establishes a diagnosis of impaired fasting glucose?

Fasting glucose 7.1 mmol/L on one occasion

Fasting glucose 6.8 mmol/L on two occasions

Glycosuria ++

75g oral glucose tolerance test 2 hour value of 8.4 mmol/L

HbA 1 c of 6.7%

Submit answer

Reference ranges v
Fasting glucose 6.8 mmol/L on two occasions

Glycosuria ++

75g oral glucose tolerance test 2 hour value of 8.4 mmol/ L

HbA1 c of 6.7%

A 75g oral glucose tolerance test 2 hour value of 8.4 mmol/L would
imply impaired glucose tolerance rather than impaired fasting glucose

9' •• tt Discuss (1) Improve

Next question )

Diabetes mellitus (type 2): diagnosis

The diagnosis of type 2 diabetes mellitus can be made by either a


plasma glucose or a HbA1 c sample. Diagnostic criteria vary according
to whether the patient is symptomatic (polyuria, polydipsia etc) or not.

If the patient is symptomatic:


• fasting glucose greater than or equal to 7.0 mmol/1
• random glucose greater than or equal to 11.1 mmol/1 (or after 75g
oral glucose tolerance test)

If the patient is asymptomatic the above criteria apply but must be


demonstrated on two separate occasions.

Fasting glucose: -e:: 60mmol1 ~= 7 0 mmol/1

,.,ediabete:s
NonNI ~ I C c--«il Dsabetas---
f-,q:)Alt 114~-H ll"fflOl'ffl()j 18 c-e .I l

°'
"A •r,,g gu:ow 6 1~ P mmo'l
In 20 I I WHO released supplement ary guidance on the use of AbA Icon
the diagnosis of diabetes:
• a HbA 1 c of greater than or equal to 48 mmol/mol (6.5%) is
diagnostic of diabetes mellitus
• a HbAlc value of less than 48 mmoi/mol (6.5%) does not exclude
diabetes (i.e. it is not as sensitive as fasting samples for detecting
diabetes)
• in patients without symptoms, the test must be repeated to
confirm the diagnosis
• it should be remembered that misleading HbA1 c results can be
caused by increased red cell turnover (see below)

Conditions where HbA 1 c may not be used for diagnosis:


• haemoglobinopathies
• haemolytic anaemia
• untreated iron deficiency anaemia
• suspected gestational diabetes
• children
• HIV
• chronic kidney disease
• people taking medication that may cause hyperglycaemia (for
example corticosteroids)

Impaired fasting glucose and impaired glucose tolerance

A fasting glucose greater than or equal to 6.1 but less than 7.0 mmoi/1
implies impaired fasting glucose (IFG)

Impaired glucose tolerance (IGT) is defined as fasting plasma glucose


less than 7.0 mmoi/1and OGTT 2-hour value greater than or equal to 7.8
mmol/1 but less than 11 .1 mmol/1

Diabetes UK suggests:
• 'People with IFG should then be offered an ora l glucose tolerance
test to rule out a diagnosis of diabetes. A result below 11.1
mmoi/1 but above 7.8 mmoi/1 indicates that the person doesn't
Q20

A 53-year-old female with a history of primary atrophic hypothyroidism


is assessed two months following a change in her dose of
levothyroxine. Which one of the following best describes what the TSH
should id eally be?

Between 0.5 to 1.0 mU/ I

Between 0.5 to 2.5 mU/ I

Between 2.5 to 4.5 mU/ I

Between 1.5 to 3.5 mU/ I

Between 3.5 to 5.5 mU/ I

Submit answer

Reference ranges v
Between 0.5 to 2.5 mU/ 1

Between 2.5 to 4 .5 mU/ 1

Between 1.5 to 3.5 mU/ 1

Between 3.5 to 5.5 mU/ 1

A TSH value between 0.5 to 2.5 mU/ 1is now considered preferable.
Dosage changes should of course also take account of symptoms

[ "' I •• t9 Discuss (2) Improve ]

Next question )

Hypothyroidism: management
Hypothyroidism: management

Key points
• initial starting dose of levothyroxine should be lower in elderly
patients and those with ischaemic heart disease. The BNF
recommends that for patients with cardiac disease, severe
hypothyroidism or patients over 50 years the initial starting dose
should be 25mcg od with dose slowly titrated. Other patients
should be started on a dose of 50-1 00mcg od
• following a change in thyroxine dose thyroid function tests should
be checked after 8-1 2 weeks
• the therapeutic goal is 'normalisation' of the thyroid stimulating
hormone (TSH) level. As the majority of unaffected people have a
TSH value 0.5-2.5 mU/ I it is now thought preferable to aim for a
TSH in this range
• women with established hypothyroidism who become pregnant
should have their dose increased 'by at least 25-50 micrograms
levothyroxine'* due to the increased demands of pregnancy. The
TSH should be monitored carefully, aiming for a low-normal value
• there is no evidence to support combination therapy with
levothyroxine and liothyronine

Side-effects of thyroxine therapy


• hyperthyroidism: due to over treatment
• reduced bone mineral density
• worsening of angina
• atrial fibrillation

Interactions
• iron, calcium carbonate
o absorption of levothyroxine reduced, give at least 4 hours
apart
Q1

A 45-year-old female is reviewed in the medical cli nic with a two month
history of lethargy. Blood tests reveal the following:

Na+ 129 mmol/I

K+ 5.1 mmol/1

Urea 5.3 mmol/1

Creatinine 99 µmol/1

Tot al T4 66 nmol/I

Which one of the following investigations is most likely t o reveal the


diagnosis?

Serum glucose

TSH

Free T4

Overnight dexamethasone suppression test

Short synacthen test

Submit answer

Reference ranges v
TSH

Free T4

Overnight dexamethasone suppression test

Short synacthen test

The short synacthen test is the best test to diagnose Addison's


disease
Importance: 50

Hyponatraemia and a high potassium in a patient with lethargy is highly


suggestive of Addison's disease. The thyroxine level is slightly low and
she may indeed have co-existing hypothyroidism but this would not
explain the high potassium

Many labs have an upper reference range for potassium of 5.5 mmol/1,
but in the context of the other results hypoadrenalism shou ld be
suspected

•• •• - Discuss (6) Improve

Next question)
Next question )

Addison's disease: investigations

In a patient with suspected Addison's disease the definite investigation


is an ACTH stimulation test (short Synacthen test). Plasma cortisol is
measured before and 30 minutes after giving Synacthen 250ug IM.
Adrenal autoantibodies such as anti-21-hydroxylase may also be
demonstrated.

If an ACTH stimulation test is not readily available (e.g. in primary care)


then sending a 9 am serum cortisol can be useful:
• > 500 nmol/I makes Addison's very unlikely
• < 100 nmol/I is definitely abnormal
• 100-500 nmol/I should prompt a ACTH stimulation test to be
performed

Associated electrolyte abnormalities are seen in around one-third of


undiagnosed patients:
• hyperkalaemia
• hyponatraemia
• hypoglycaemia
• metabolic acidosis

Next question )

B J ~ A .... == i: =. . T I• IHI .... ~ c-:::>

Save my notes
A 54-year-old-woman with known ovarian cancer presents with
confusion. She has become progressively confused over the last few
days, and prior to that, she had started to become constipated. Her
family describes poor oral intake of fluids and poor urinary output as
well. On further discussion with the family, they mention that she was
seen in oncology clinic two weeks ago with results of a bone scan
which they had been told was normal.

On examination, she appears dehydrated.

Hb 147 g/1

Platelets 321*109/1

WBC 7.8*109/1

Na+ 142 mmol/ 1

K+ 4.7 mmol/ 1

Urea 4.6 mmol/ 1

Creatinine 92 µmol/ 1

Corrected calcium 3.2 mmol/1

Parathyroid hormone Pending

What is the most likely cause of her elevated calcium?

Osteolytic hypercalcaemia

Calcitriol-mediated hypercalcaemia

Ectopic PTH secretion

Parathyroid-hormone-related peptide release

Primary hyperparathyroidism
Calcitriol-mediated hypercalcaemia

Ectopic PTH secretion

Parathyroid-hormone-related peptide release

Primary hyperparathyroidism

The correct answer is parathyroid-hormone-related peptide release.


Whilst this is classically described as secondary to squamous cell lung
cancer, it can occur in many malignancies.The two most common
causes of hypercalcaemia are malignancy and primary
hyperparathyroidism . In malignancy, roughly 80% of cases are due to
parathyroid-hormone-related peptide release. The vast majority of
remaining cases are due to osteolysis, and some due to calcitriol-
mediated hypercalcaemia and ectopic PTH secretion. Primary
hyperparathyroidism is another common cause but is not as likely as
malignant hypercalcaemia given the known diagnosis of ovarian cancer.

Source:

'Hypercalcaemia of Malignancy.' BMJ Best Practice. 14 June 2016.

[ ,6 •t - Discuss (3) Improve ]

Next question >


ex ques 10n

Hypercalcaemia: causes

Two conditions account for 90% of cases of hypercalcaemia:


• 1. Primary hyperparathyroidism: commonest cause in non-
hospitalised patients
• 2. Malignancy: the commonest cause in hospitalised patients.
This may be due to number of processes, including; bone
metastases, myeloma, PTHrP from squamous cell lung cancer

Other causes include


• sarcoidosis*
• vitamin D intoxication
• acromegaly
• thyrotoxicosis
• Milk-alkali syndrome
• drugs: thiazides, calcium containing antacids
• dehydration
• Addison's disease
• Paget's disease of the bone**

*other causes of granulomas may lead to hypercalcaemia e.g.


Tuberculosis and histoplasmosis

**usually normal in this condition but hypercalcaemia may occur with


prolonged immobilisation

Next question )

B I /!j A ..,. T I""


Q1

A 66-year-old lady with a long hist ory of poorly control led type-2
diabetes is started on a new medication. She is told it works by
increasing urinary glucose excretion and the doctor says it is an SGLT-2
inhibitor.

Which of the fo llowing medications is in t his drug class?

Tolbutamide

Dapagliflozin

Exenatide

Linagliptin

Pioglitazone

Submit answer

Reference ranges v
Dapagliflozin

Exenatide

Linagl iptin

Pioglitazone

Gliflozins - SGLT2 inhibitors


lmportahce: 50

Gliflozins are SGLT2 inhibitors.

Tolbutamide is a sulfonylurea.

Exenatide is a GLP-1 receptor agonist.

Linagliptin is a DPP-4 inhibitor.

Pioglitazone is a thiazolidinedione.

,, •• • Discuss (1) Improve

Next question)
Q2

Which one of the following hormones is under continuous inhibition?

Growth hormone

Prolactin

Gonadotropin releasing hormone

Thyroid releasing hormone

Adrenocorticotrophic hormone

Submit answer

Reference ranges v
Prolactin

Gonadotropin releasing hormone

Thyroid releasing hormone

Adrenocorticotrophic hormone

Prolactin - under continuous inhibition


Importance: 50

Prolactin is unique amongst the pituitary hormones in being tonically


inhibited by the hypothalamus

[ ,, •• • Discuss (1) Improve ]

Next question )

CamScanne, -? Y.¥-> i.b.~I


Next ques tion )

Prolactin and galactorrhoea

se being
Prolactin is secreted by the ante rior pitui tary gland with relea
acts as
contr olled by a wide variety of phys iolog ical facto rs. Dopa mine
mine
the prim ary prola ctin releasing inhib itory facto r and henc e dopa
torrh oea.
agon ists such as brom ocrip tine may be used to contr ol galac
to the
It is impo rtant to diffe renti ate the caus es of galac torrh oea (due
astia
actio ns of prola ctin on brea st tissu e) from thos e of gyna ecom

Features of excess prola ctin


• men: impotence, loss of libido, galac torrh oea
• women: amenorrhoea, galac torrh oea

Causes of raised prola ctin


• prola ctino ma
• pregnancy
• oestr ogen s
• physiological: stress, exercise, sleep
• acromegaly: 1/3 of patie nts
• polyc ystic ovarian synd rome
one
• prim ary hypo thyro idism (due to thyro troph in releasing horm
(TRH) stimu lating prola ctin release)

Drug caus es of raised prola ctin


• meto clopr amide, dom perid one
• phenothiazines
• halop erido l
• very rare: SSRl s, opio ids

Next ques tion )


Q3

Which one of the following features is least associated with primary


hyperparathyroidism?

Depression

Polydipsia

Sensory loss

Peptic ulceration

Hypertension

Submit answer

Reference ranges v
Polydipsia

Sensory loss

Peptic ulceration

Hypertension

[ ,6 I •• tll Discuss (2) Improve

Next question)

Primary hyperparathyroidism
Q4

You have been asked to review a 52-year-old woman in the emergency


department with dyspnoea accompanied by nausea and vomiting. She
tells you that she initially attributed this to a flu-like illness but as she
found herself becoming increasingly short of breath she was taken to
hospital by her concerned husband. Her past medical history is
sign ifi cant for type 2 diabetes mellitus, hypertension and obes ity. Her
current medications include metformin 1gram bd, canagliflozin 100mg
od, ramipril 10mg od and amlodipine 5mg od. On examination she has a
respiratory rate of 25 breaths/min, blood pressure 130/ 67 mm Hg, pulse
105 bpm and oxygen saturation of 98% on room air. The only finding
you elicit on physical examination is mild tenderness in the epigastric
area. Her ECG shows a sinus tachycardia. Routine blood results are
shown below:

132 22
Hb 167 g/1 Na+ Bilirubin pH 7.14
mmol/1 µmol/1

410 * K+ 5.5
Platelets ALP 100 u/1 Pa02 12 KPa
109 / 1 mmol/1

11 .2 * 10.4
WBC Urea ALT 55 u/1 PaC02 1.9 KPa
109 / 1 mmol/1

10.0 * 111
Neuts Creatinine yGT 23 u/1 HC03- 11 µmol/1
109 / 1 µmol/1

1.1 * 3.0
Lymphs Amylase 321 U/ 1 A lbumin 33 g/1 Lactat e
109 / 1 mmol/1

0.1 * c1-
101 Ketones
Eosin Urine Glucose 11 mmol/L
109 / 1 mmol/1 3+
Eosin
109 / 1
er mmol/ 1
Urine
3+
Glucose 11 mmo!/ L

What is the most likely diagnosis?

Small bowel obstruction

Metformin induced lactic acidosis

Starvation ketosis

Euglycaemic diabetic ketoacidosis

Pancreatitis

Submit answer

Reference ranges v
Metformin induced lactic acidosis

Starvation ketosis

Euglycaemic diabetic ketoacidosis

Pancreatitis

This lady has euglycaemic diabetic ketoacidosis (EuDKA) secondary to


her sodium-glucose co-transporter 2 (SGLT2) inhibitor, canagliflozin.
EuDKA is an important to recognise side effect of this novel class of
oral hypoglycaemic agents and should be thought of in any patient with
an unexplained raised anion gap acidosis and normal blood sugar level
who is on one of these medications.

Exactly how these agents cause EuDKA has yet to be determined.


However, it is hypothesised that as these agents lower blood sugar
levels by increasing the excretion of glucose the resu lting reduction in
plasma glucose results in reduced insulin secretion from pancreatic
beta-cells and these patients entering a state of relative insulin
deficiency. This leads to a lowering of the antilipolytic activity of insulin,
and the consequent stimulation of the production of free fatty acids,
which are then converted to ketone bodies by beta-oxidation in the liver.
Moreover, insulin stimulates the activity of acetyl-CoA carboxylase,
which produces malonyl-CoA, a potent inhibitor of carnitine
palmitoyltransferase (CPT-1). Given that CPT-1promotes the transport of
fatty acids into mitochondria and hence increases the rate of beta-
oxidation, the decrease in the circulating level of insulin promotes the
production of ketone bodies through activation of CPT-1.

8
"' f • Discuss (8) Improve

Next question )
QS

A 45-year-old who is currently being investigated for lethargy and


reduced libido presets for follow-up. He initially presented after
requesting a testosterone blood test due to his reduced sexual drive.
This was reported as follows:

Testosterone 5.4 nmol/l (>9.0)

Subsequent blood tests show the following:

Prolactin 450 mu (< 400)

Cortisol (8am) 120 nmol/L (130-690)

FSH 0.8 IU/ L (1-8)

What is the most likely diagnosis?

Addison's disease

Testicular cancer with brain metastases

Non-functioning pituitary adenoma

Prolactinoma

Craniopharyngioma

Submit answer

Reference ranges v
Testicular cancer with brain metastases

Non-functioning pituitary adenoma

Prolactinoma

Craniopharyngioma

Non-functioning pituitary tumours present with hypopituitarism


and pressure effects
Importance: 50

Whilst the prolactin level is slightly raised this can be caused by the
pressure effects of the tumour preventing dopamine (which inhibits
prolactin release) from reaching the normal prolactin-producing cells.
Much higher levels would be expected with a prolactinoma.

,6 ,. tt Discuss (7) Improve ]

Next question )

Pituitary adenoma

A pituitary adenoma is a benign tumour of the pituitary gland. They are


common (10% of all people1 ) but in most cases wi ll never be found
(asymptomatic) or are found as an incidental finding . They account for
around 10% of adult brain tumours2 .

Pituitary adenomas can be classified according to:


• size (a microadenoma is <1 cm and a macroadenoma is >1 cm)
• hormonal status (a secretary/functioning adenoma produces and
excess of a particular hormone and a non-secretary/functioning
adenoma does not produce a hormone to excess)

Cam Sc onner ...,, ',&~ .G. ~ I


Prolactinomas are the most common type and they produce an excess
of prolactin. After prolactinomas, non-secreting adenomas are the next
most common, then GH secreting and then ACTH secreting adenomas.

Pituitary adenomas typically cause symptoms by:


• excess of a hormone (e.g. Cushing's disease due to excess ACTH,
acromegaly due to excess GH or amenorrhea and galactorrhea
due to excess prolactin)
• depletion of a hormone(s) (due to compression of the normal
functioning pituitary gland)
o non-functioning tumours, therefore, present with generalised
hypopituitarism
• stretching of the dura within/ around pituitary fossa (causing
headaches)
• compression of the optic chiasm (causing a bitemporal
hemianopia due to crossing nasal fibers)

Alternatively, pituitary adenomas, particularly microadenomas, can be


an incidental finding on neuroimaging and therefore called a 'pituitary
incidentaloma'.

Investigation requires:
• a pituitary blood profile (including: GH, prolactin, ACTH, FH, LSH
and TFTs)
• formal visual field testing
• MRI brain with contrast

Differential diagnoses include:


• pituitary hyperplasia
• craniopharyngioma
• meningioma
• brain metastases
• lymphoma
• hypophysitis
• vascular malformation (e.g. aneurysm)
Treatment may include a combination of:
• hormonal therapy (e.g. bromocriptine is the first line treatment for
prolactinomas)
• surgery (e.g. transsphenoidal transnasal hypophysectomy)
o e.g. if progression in size
• radiotherapy

Next question )

B A ... ·-
== 1-
~
=-...,.

Save my notes

Search

Search textbook...
- - -~ · ■

Q Google search on "Pituitary adenoma"

Links

Patient.info

Pituitary tumours review


0 Q6

A 72-year-old woman presents with polyuria and polydipsia.


Investigations reveal the following:

Fasting glucose 4.5 mmol/1

Calcium 2.88 mmol/1

Phosphate 0.75 mmol/1

Parathyroid hormone 6 pmol/L (normal range= 0.8 - 8.5)

What is the most likely underlying diagnosis?

Myeloma

Sarcoidosis

Primary hyperparathyroidism

Vitamin D excess

Osteomalacia

Submit answer

Reference ranges v
Sarcoidosis

Primary hyperparathyroidism

Vitamin D excess

Osteomalacia

The PTH level in primary hyperparathyroidism may be normal


Importance: 50

Despite a raised calcium level the parathyroid hormone level is


inappropriately normal. This points towards a diagnosis of primary
hyperparathyroidism and the other causes (such as myeloma) would
lead to a suppression of parathyroid hormone

,6 'f - Discuss (10) Improve ]

Next question)
Q7

A 85-year-old male presents to clinic for a review of his overactive


bladder. Non-pharmacologic treatments have so far failed to improve
his symptoms. Which of the following pharmacotherapies represents
the most appropriate initial management step?

Finasteride

Desmopressin

Tamsulosin

Mirabegron

Tolterodine

Submit answer

Reference ranges v
ITolterodine

NICE (Feb 2015) outlines treatment steps for the management of


overactive bladder symptoms in men.

If non-pharmacologic measures fail, an anticholinergic agent is first


line. In older men, tolterodine is preferred to oxybutynin as the latter has
a greater risk of causing confusion.

If anticholinergics fail or are contraindicated, mirabegron may be


trialled . Its mechanism of action is via beta-adrenoreceptor-mediated
relaxation of the bladder wall.

Tamsulosin is an alpha-blocker and indicated if the patient if has


obstructive symptoms, rather than symptoms of overactive bladder.

Finasteride is a 5-alpha reductase inhibitor and indicated if the patient


has obstructive symptoms and an enlarged prostate with a high risk of
progression.

Desmopressin is a synthetic vasopressin analogue that acts in the


collecting duct of the nephron. It is sometimes used off-label for
nocturnal urinary incontinence.

[ ,6 'f • Discuss {8) Improve ]

Next question )
Urinary incontinence
-
Urinary incontinence (UI) is a common problem, affecting around 4-5%
of the population. It is more common in elderly females.

Risk factors
• advancing age
• previous pregnancy and childbirth
• high body mass index
• hysterectomy
• family history

Classification
• overactive bladder (OAB)/urge incontinence: due to detrusor
overactivity
• stress incontinence: leaking small amounts when coughing or
laughing
• mixed incontinence: both urge and stress
• overflow incontinence: due to bladder outlet obstruction, e.g. due
to prostate enlargement

Initial investigation
• bladder diaries should be completed for a minimum of 3 days
• vaginal examination to exclude pelvic organ prolapse and ability to
initiate voluntary contraction of pelvic floor muscles ('Kegel'
exercises)
• urine dipstick and culture
• urodynamic studies
Management depends on whether urge or stress UI is the predominant
picture. If urge incontinence is predominant:
• bladder retraining (lasts for a minimum of 6 weeks, the idea is to
gradually increase the intervals between voiding)
• bladder stabilising drugs: antimuscarinics are first-line. NICE
recommend oxybutynin (immediate release), tolterodine
(immediate release) or darifenacin (once daily preparation).
Immediate release oxybutynin should, however, be avoided in 'frail
older women'
• mirabegron (a beta-3 agonist) may be useful if there is concern
about anticholinergic side-effects in frail elderly patients

If stress incontinence is predominant:


• pelvic floor muscle training : NICE recommend at least 8
contractions performed 3 times per day for a minimum of 3
months
• surgical procedures: e.g. retropubic mid-urethral tape procedures

Next question)

B I ~ A .... ·-
·- 2= =
1-
- .. T I .... EEEI .... ~ c-::,

-
Save my notes

Search

Search textbook ...


-
Q8

A 55-year-old man with type 2 diabetes mellitus has his yearly health
check-up at his GP surgery. His HbA1c is 86mmol/L and his GP is
considering adding empagliflozin to help manage his diabetes.

HbA1c (mmol/L) Interpretation

<42 Normal Glycaemic Control

42-47 Impaired Glucose Tolerance

>48 Diabetes Mellitus

86 Patient's Result

What is the mechanism of action of this medication?

Blocks potassium channels on f3 islet cells in the pancreas

Inhibition of the sodium-glucose transporter in the kidney

Inhibition of dipeptidyl peptidase 4

Inhibition of a glucosidase in the small intestine

Stimulation of peroxisome proliferator-activated receptor

Submit answer
Inhibition of the sodium-glucose transporter in the kidney

Inhibition of dipeptidyl peptidase 4

Inhibition of a glucosidase in the small intestine

Stimulation of peroxisome proliferator-activated receptor

SGLT-2 inhibitors reversibly inhibit sodium-glucose co-transporter


2 (SGLT-2) in the renal proximal convoluted tubule
Importance: 50

Empagliflozin is an SGLT-2 (sodium-glucose transport protein 2)


inhibitor. These act in the proximal convoluted tubule of nephrons to
inhibit glucose re-absorption back into the circulation by blocking SGLT-
2 channels.

Sulphonylureas act via increasing insulin secretion from f3 islet cells in


the pancreas. They do this via blockage of potassium channels, which
causes islet cell depolarisation and release of insulin.

Inhibition of the enzyme DPP-4 (dipeptidyl peptidase 4) prevents the


break down of GLP-1 (glucagon-like peptide), which in turn suppresses
glucagon release and increases insulin release. Sitagliptin is an
example of this class of drug.

Acarbose inhibits a glucosidase and other enzymes in the small


intestine, preventing the breakdown of complex carbohydrates into
glucose. As a result, less glucose is made available for absorption into
the bloodstream.

Thiazolidinediones reduce insulin resistance in the peripheral tissues


and decrease gluconeogenesis in the liver. They achieve this by
stimulation of PPAR-y (peroxisome proliferator-activated receptor-
gamma) which modulates the transcription of genes involved in
glucose metabolism.
Q9

A S6-year-old Muslim man with a history of type 2 diabetes asks for


advice. He is due to start fasting for Ramadan soon and is unsure what
he should do with regards to his diabetes medications. He currently
takes metformin SOOmg tds . What is the most appropriate advice?

Switch to subcutaneous biphasic insulin for the duration of


Ramadan

SOO mg at the predawn meal + 1000 mg at the sunset meal

No change to the metformin dose

1000 mg at the predawn meal + SOO mg at the sunset meal

Stop metformin for the duration of Ramadan

Submit answer

Reference ranges v
500 mg at the predawn meal + 1000 mg at the sunset meal

No change to the metformin dose

1000 mg at the predawn meal + 500 mg at the sunset meal

Stop metformin for the duration of Ramadan

During Ramadan, one-third of the normal metformin dose should


be taken before sunrise and two-thirds should be taken after
sunset
Importance: 50

Please see the Diabetes Care link for more details.

[" •• tit Discuss (4) Improve ]

Next question )
Diabetes mellitus: Ramadan

We know that type 2 diabetes mellitus is more common in people of


Asian ethnicity and a significant proportion of those patients in the UK
will be Muslim. The BMJ published an excellent and comprehensive
review of this issue in 201 O1 .

It is important that we can give appropriate advice to Muslim patients to


allow them safely observe their fast. This is particularly important from
2014 as Ramadan is due to fall in the long days of the summer months
for several years henceforth.

Clearly it is a personal decision whether a patient decides to fast. It may


however be worthwhile exploring the fact that people with chron ic
conditions are exempt from fasting or may be able to delay fasting to
the shorter days of the winter months. It is however known that many
Muslim patients with diabetes do not class themselves as having a
chronic/serious condition which should exempt them from fasting.
Around 79% of Muslim patients with type 2 diabetes mellitus fast
Ramadan2 .There is an excellent patient information leaflet from
Diabetes UK and the Muslim Council of Britain which explores these
options in more detail.

If a patient with type 2 diabetes mellitus does decide to fast


• they should try and and eat a meal containing long-acting
carbohydrates prior to sunrise (Suhoor)
• patients should be given a blood glucose monitor to allow them to
check their glucose levels, particularly if they feel unwell
• for patients taking metformin the expert consensus is that the
dose should be split one-third before sunrise (Suhoor) and two-
thirds after sunset (lftar)
• expert consensus also recommends switching once-daily
sulfonylureas to after sunset. For patients taking twice-daily
preparations such as gliclazide it is recommended that a larger
proportion of the dose is taken after after sunset
• no adjustment is needed for patients taking pioglitazone
Q10

An elderly male of no fixed abode with a history of alcohol dependency


and chron ic liver disease is taken to t he Emergency Department with
reduced consciousness (GCS 5) and a blood glucose of 1.3 mmol/ L.

What is the correct management of his hypoglycaemia?

IM Glucagon STAT

Lucozade

100ml IV Normal Saline

100ml IV Dextrose 5%

100ml IV Glucose 20%

Submit answer

Reference ranges v
Lucoza de

100ml IV Normal Saline

100ml IV Dextros e 5%

I 100ml IV Glucose 20%

Hypogly caemia in patients with alcohol ic liver disease does not


respond to glucago n
Importance: 50

Patients with alcohol ic liver disease have depleted glycogen stores,


therefore, treatme nt with glucago n does not improve blood glucose .

It is not safe to use the oral route when the patient is GCS 5, therefore,
adminis tering Lucozade is not appropriate.

Normal saline does not correct hypoglycaemia .

5% dextros e is a mainten ance flu id and not appropr iate for acute
treatme nt of hypogly caemia .

Therefore, the correct answer is 100ml IV Glucose 20%.

[ ,6 ,. '9 Discuss (6) Improve ]

Next questio n )
Next question )

Hypoglycaemia

Causes
• insulinoma - increased ratio of proinsulin to insulin
• self-administration of insulin/sulphonylureas
• liver failure
• Addison's disease
• alcohol

Other possible causes in children


• nesidioblastosis - beta cell hyperplasia

Physiological response to hypoglycaemia


• hormonal response: the first response of the body is decreased
insulin secretion. This is followed by increased glucagon
secretion. Growth hormone and cortisol are also released but later
• sympathoadrenal response: increased catecholamine-mediated
(adrenergic) and acetylcholine-mediated (cholinergic)
neurotransmission in the peripheral autonomic nervous system
and in the central nervous system

Next question )

B I A .... ·- 21-.: - - ....


T I""

1
Q11

A 53 year man presents as his wife has noticed a change in his


appearance. He has also noticed his hands seem larger. On
examination blood pressure is 170/94 and he is noted to have
bitemporal hemianopia. What is the most appropriate first-line
treatment?

Octreotide

External irradiation

Pegvisomant

Trans-sphenoidal surgery

Bromocriptine

Submit answer

Reference ranges v
External irradiation

Pegvisomant

Trans-sphenoidal surgery

Bromocriptine

Trans-sphenoidal surgery is the treatment of choice in acromegaly.


There is no significant evidence base supporting the use of pre-
operative octreotide

,6 ,. tit Discuss (9) Improve ]

Next question )

a _ __ _ __ _ _ _ I

Next question )

Acromegaly: management

Trans-sphenoidal surgery is the first-line treatment for acromegaly in


the majority of patients

Dopamine agonists
• for example bromocriptine
• the first effective medical treatment for acromegaly, however now
superseded by somatostatin analogues
• effective only in a minority of patients

Somatostatin analogue
• directly inhibits the release of growth hormone
• for example octreotide
• effective in 50-70% of patients
• may be used as an adjunct to surgery

Pegvisomant
• GH receptor antagonist - prevents dimerization of the GH receptor
• once daily s/ c administration
• very effective - decreases IGF-1 levels in 90% of patients to normal
• doesn't reduce tumour volume therefore surgery still needed if
mass effect

External irradiation is sometimes used for older patients or following


failed surgical/medical treatment

Next question )
Q12

A 56-year-old female is admitted to ITU with a severe pneumonia.


Thyroid function tests are most likely to show:

TSH normal; thyroxine high; T3 high

TSH normal / low; thyroxine low; T3 low

TSH high; thyroxine low; T3 low

TSH low; thyroxine high; T3 high

TSH high; thyroxine normal; T3 high

Submit answer

Reference ranges v
TSH normal / low; thyroxine low; T3 low

TSH high; thyroxine low; T3 low

TSH low; thyroxine high; T3 high

TSH high; thyroxine normal; T3 high

8' •• • Discuss (1) Improve

Next question )

Sick euthyroid syndrome

In sick euthyroid syndrome (now referred to as non-thyroidal illness) it is


often said that everything (TSH, thyroxine and T3) is low. In the majority
of cases however the TSH level is within the >normal range
(inappropriately normal given the low thyroxine and T3).

Changes are reversible upon recovery from the systemic illness and
hence no treatment is usually needed.

Next question )

Save my notes
Q13

A 31-year-old woman presents for review. For the past few months she
has been feeling generally tired and has not had a normal period for
around 4 months. Prior to this she had a regular 30 day cycle. A
pregnancy test is negative, pelvic examination is normal and routine
bloods are ordered:

FBC Normal

U&E Normal

TFT Normal

Follicle-stimulating hormone 41 iu/1 ( < 35 iu/1)

Luteinizing hormone 33 mlU/1 (< 20 mlU/1)

Oestradiol 70 pmol/1 ( > 100 pmol/1)

What is the most likely diagnosis?

ovarian cancer

Gonadotropin-producing pituitary adenoma

Turner syndrome

Premature ovarian failure

Aromatase enzyme deficiency


Premature ovarian failure

Aromatase enzyme deficiency

•• •t • Discuss (2) Improve

Next question )

Premature ovarian failure

Premature ovarian failure is defined as the onset of menopausal


symptoms and elevated gonadotrophin levels before the age of 40
years. It occurs in around 1 in 100 women.

Causes
• idiopathic - the most common cause
• chemotherapy
• autoimmune
• radiation

Features are similar to those of the normal climacteric but the actual
presenting problem may differ
• climacteric symptoms: hot flushes, night sweats
• infertility
• secondary amenorrhoea
• raised FSH, LH levels

Next question )
0 Q14

A 61-year-old male is diagnosed with acromegaly after presenting with


headaches, sweaty palms and a change in shoe size. Magnetic
resonance scan has shown a pituitary macroadenoma and he is
planned for transsphenoidal surgery.

What treatment is often used as an adjunct to surgery?

Bromocriptine

Cabergoline

Octreotide

Pegvisomant

Prednisolone

Submit answer

Reference ranges v
Cabergoline

Octreotide

Pegvisomant

Prednisolone

Octreotide can be used as an adjunct to surgery in patients with


acromegaly
Importance: 50

The correct answer is octreotide, a somatostatin analogue which is


often used as an adjunct to surgery resulting in reduced growth
hormone levels and reduction in tumour size.

Bromocriptine is a dopamine agonist used in the medical management


of acromegaly. It is not commonly used as an adjunct to surgery.

Cabergoline is also a dopamine agonist. It is more commonly used in


the medical management of prolactinoma as opposed to acromegaly.

Pegvisomant is a growth hormone receptor antagonist, it is useful for


reducing growth hormone levels but not tumour size. For this reason, it
is not the correct answer.

Prednisolone is not used as an adjunct to surgery. In some instances,


steroids may be used if there were concerns over mass effect or
oedema, but this is not mentioned in this stem.

16 8
f • Discuss Improve

Next question )
0 Q15

A 58-year-old woman presents to the emergency department with


confusion. She is found to have a raised temperature, tachycardia and
is hypotensive. After further investigation, she is found to be in a
thyrotoxic storm.

Given her presentation what is the best first line treatment?

Adrenaline and hydrocortisone

Beta blockers and propylthiouracil

Adrenaline, propylthiouracil and hydrocortisone

Beta blockers, propylthiouracil and hydrocortisone

Propylthiouracil and hydrocorti'sone

Submit answer

Reference ranges v
Beta blockers and propylthiouracil

Adrenaline, propylthiouracil and hydrocortisone

Beta blockers, propylthiouracil and hydrocortisone

Propylthiouracil and hydrocortisone

Thyrotoxic storm is treated with beta blockers, propylthiouracil


and hydrocortisone
Importance: 50

This question is asking about a woman presenting with confusion,


tachycardia, hypotension and a raise temperature who is found to be in
a thyrotoxic storm (also known as a hyperthyroid crisis). You are asked
for the best first-line treatment in this case which is option 4, beta
blockers, propylthiouracil and hydrocortisone.

Beta blockers are used to treat the tachycardia, however, these as


always would be contraindicated in patients suffering from asthma.

Propylthiouracil is used as an anti-thyroid treatment to help reduce the


effect of raised serum thyroid hormones that are causing her
symptoms.

Hydrocortisone is used to treat any underlying adrenal insufficiency


which is more common in patients suffering from hyperthyroidism and
can also help to reduce serum thyroid hormone levels.

116 11
f • Discuss (5) Improve

Next question )
Thyroid storm

Thyroid storm is a rare but life-threatening complication of


thyrotoxicosis. It is typically seen in patients with established
thyrotoxicosis and is rarely seen as the presenting feature. Iatrogenic
thyroxine excess does not usually result in thyroid storm.

Precipitating events:
• thyroid or non-thyroidal surgery
• trauma
• infection
• acute iodine load e.g. CT contrast media

Clinical features include:


• fever > 38.s0 c
• tachycardia
• confusion and agitation
• nausea and vomiting
• hypertension
• heart failure
• abnormal liver function test - jaundice may be seen clinically

Management:
• symptomatic treatment e.g. paracetamol
• treatment of underlying precipitating event
• beta-blockers: typically IV propranolol
• anti-thyroid drugs: e.g. methimazole or propylthiouracil
• Lugol's iodine
• dexamethasone - e.g. 4mg IV qds - blocks the conversion of T 4 to
T3

Next question )
Q16

A 15-year-old girl is investigated for primary amenorrhoea, despite


having developed secondary sexual characteristics at 11 years of age.
On examination she has well developed breasts with scanty pubic hair
and small bilateral groin swellings. What is the most likely diagnosis?

Congenital adrenal hyperplasia

Polycystic ovarian syndrome

Turner's syndrome

Complete androgen insensitivity syndrome

Mullerian duct agenesis

Submit answer

Reference ranges v
Complete androgen insensitivity syndrome

Mullerian duct agenesis

8' 11 f tt Discuss (2) Improve

Next question )

Androgen insensitivity syndrome

Androgen insensitivity syndrome is an X-linked recessive condition due


to end-organ resistance to testosterone causing genotypically male
children (46XY) to have a female phenotype. Complete androgen
insensitivity syndrome is the new term for testicular feminisation
syndrome

Features
• 'primary amennorhoea'
• undescended testes causing groin swellings
• breast development may occur as a result of conversion of
testosterone to oestradiol

Diagnosis
• buccal smear or chromosomal analysis to reveal 46XY genotype

Management
• counselling - raise child as female
• bilateral orchidectomy (increased risk of testicu lar cancer due to
undescended testes)
• oestrogen therapy

Next uestion
Q17

You review a 52-year-old man who is being investigated for weight gain,
impotence and hypertension. On examination you record a blood
pressure of 180/11 O mm Hg and notice purple striae around his
abdomen. He also has some difficulty getting up from a chair and you
observe generalised decreased muscle strength. Routine bloods are
ordered. Given the likely underlying diagnosis, what are the urea and
electrolytes most likely to show?

Hypokalaemic metabolic acidosis

Hyperkalaemic metabolic alkalosis

Hypocalcaemic metabolic acidosis

Hypokalaemic metabolic alkalosis

Hyperkalaemic metabolic acidosis

Submit answer

Reference ranges v
observe generalised decreased muscle strength. Routine bloods are
ordered. Given the likely underlying diagnosis, what are the urea and
electrolytes most likely to show?

Hypokalaemic metabolic acidosis

Hyperkalaemic metabolic alkalosis

Hypocalcaemic metabolic acidosis

Hypokalaemic metabolic alkalosis

Hyperkalaemic metabolic acidosis

Cushing's syndrome - hypokalaemic metabolic alkalosis


Importance: 50

[ ,. 'f • Discuss (2) Improve

Next question )
Q18

A 47-year-old woman is referred to the general medical clinic. She has


gained 1 O kg in weight in the past 3 months but her main problem is
episodic sweating. These episodes of sweating are associated with
double vision and typical ly occur early in the morning. Clinical
examination is unremarkable. What is the most likely diagnosis?

Bronchial carcinoid

Hashimoto's thyroiditis

Menopause

Cushing's syndrome

lnsulinoma

Submit answer

Reference ranges v
I lnsulinoma

This is a typical presentation of insulinoma

9' •• tt Discuss (2) Improve

Next question )

lnsulinoma

An insulinoma is a neuroendocrine tumour deriving mainly from


pancreatic Islets of Langerhans cells

Basics
• most common pancreatic endocrine tumour
• 10% malignant, 10% multiple
• of patients with multiple tumours, 50% have MEN-1

Features
• of hypoglycaemia: typically early in morning or just before meal,
e.g. diplopia, weakness etc
• rapid weight gain may be seen
• high insulin, raised proinsulin:insulin ratio
• high C-peptide

Diagnosis
• supervised, prolonged fasting (up to 72 hours)
• CT pancreas

Management
• surgery
• diazoxide and somatostatin if patients are not candidates for
surgery
Q19

At which point in the menstrual cycle do progesterone levels peak?

Luteal phase

Ovulation

Follicular phase

Levels remain constant throughout cycle

Menstruation

Submit answer

Reference ranges v
Luteal phase

Ovulation

Follicular phase

Levels remain constant throughout cycle

Menstruation

Progesterone is secreted by the corpus luteum following ovulation.

[ ,t •• - Discuss (1) Improve

Next question >

Menstrual cycle

The menstrual cycle may be divided into the following phases:

Days

Menstruation 1-4

Follicular phase (proliferative phase) 5-13

Ovulation 14

Luteal phase (secretory phase) 15-28


Follicular phase
Days 1- 7

Primordial Primary secondary


follicles fomcles folllcles

-+ @ •©>----.
Atresla
r-,. ~ Single, selected
0
-+ \:2, t ~ - - - - - ~~ tertiary follicle
Selection

Constant development Selection of one dominant


of early-stage folllcles secondary follicle begins
(2 months) each new menstrual cycle

ovarian ~cle phases


Folllcular phase LuteaJ phase
5electedtertlary ovulation CorpLS corpus Degradlns
folllcle 1uteur1 alblcans corpus

0 7 14
iJJ 21 28
Day of merstrual cycle

U terlne cycle phases


Pro111e 1a t tve
"-tenses phase Sec.-eto~ p has e

M en.st:ruatt o n

0 7 14 2i 2B
Day cf menstr ual cy -cle

Pituitary ovulation FSH


hormone levels - LH

7 14 21 28

Ovarian - Estrogen
hormone levels - Progesterone

0 7 14 21 28
Day of menstrual cycle
Follicular phase Luteal phase
{proliferative phase) (secretory phase)

Ovarian A number of follicles Corpus luteum


histology develop.

One follicle will become


dominant around the mid-
follicular phase

Endometrial Proliferation of Endometrium


histology endometrium changes to
secretary lining
under influence of
progesterone

Hormones A rise in FSH results in the Progesterone


development of follicles secreted by corpus
which in turn secrete Iuteu m rises
oestradiol through the luteal
phase.
When the egg has matured,
it secretes enough If fertilisation does
oestradiol to trigger the not occur the
acute release of LH. This in corpus luteum will
turn leads to ovulation degenerate and
progesterone levels
fall

Oestradiol levels
also rise again
during the luteal
phase

CamScanne, -? Y.¥-> i.b . ~I


Cervical Following menstruation the Under the influence
mucus mucus is thick and forms a of progesterone it
plug across the external os becomes thick,
scant, and tacky
Just prior to ovulation the
mucus becomes clear,
acellular, low viscosity. It
also becomes 'stretchy' - a
quality termed spinnbarkeit

Basal body Falls prior to ovulation due Rises following


temperature to the influence of oestradiol ovulation in
response to higher
progesterone levels

Next question )

B I l!J

Save my notes

Search

Search textbook ...


----■
Q20

Which one of the following is most likely to be seen in a patient with


multiple endocrine neoplasia (MEN) type I?

Phaeochromocytoma

lnsulinoma

Marfanoid body habitus

Medullary thyroid carcinoma

RET gene

Submit answer

Reference ranges v
lnsulinoma

Marfanoid body habitus

Medullary thyroid carcinoma

RET gene

[ "' 'f • Discuss Improve

Next question )
Q21

A 45-year-old man presents with bitemporal hemianopia and spade-like


hands. What is the most appropriate next test?

Early morning growth hormone

Insulin tolerance test

Pituitary MRI

Insu lin-like growth factor 1 (IGF-1)

Short ACTH test

Submit answer

Reference ranges v
Insulin tolerance test

Pituitary MRI

Insulin-like growth factor 1 (IGF-1 )

Short ACTH test

Serum IGF-1 levels are now the first-line test for acromegaly
Importance: 50

[ ,. 'f ft Discuss (2) Improve ]

Next question )
Acromegaly: investigations

Growth hormone (GH) levels vary during the day and are therefore not
diagnostic.

Serum IGF-1 levels have now overtaken the oral glucose tolerance test
(OGTT) with serial GH measurements as the first-line test. The OGTT
test is recommended to confirm the diagnosis if IGF-1 levels are raised .

The Endocrine Society guidelines suggest the following:

1. 1 We recommend measurement of JGF-1 levels in patients with


typical clinical manifestations of acromegaly, especially those with
acral and facial features.

1. 5 In patients with elevated or equivocal serum IGF-1 levels, we


recommend confirmation of the diagnosis by finding lack of
suppression of GH to < 1 µg/ L following documented
hyperglycemia during an oral glucose load.

Serum IGF-1 may also be used to monitor disease

Oral glucose tolerance test


• in normal patients GH is suppressed to< 2 mu/ L with
hyperglycaemia
• in acromegaly there is no suppression of GH
• may also demonstrate impaired glucose tolerance which is
associated with acromegaly

A pituitary MRI may demonstrate a pituitary tumour.


Q22

A 64-year-old patient is prescribed pegvisomant for the treatment of


acromegaly. What is the mechanism of action of pegvisomant?

IGF-1 receptor antagonist

Growth hormone receptor antagonist

IGF-1 receptor agonist

Growth hormone receptor agonist

Long-acting somatostatin analogue

Submit answer

Reference ranges v
Growth hormone receptor antagonist

IGF-1 receptor agonist

Growth hormone receptor agonist

Long-acting somatostatin analogue

,, f
11 tit Discuss Improve

Next question )
Next question )

Acromegaly: management

Trans-sphenoidal surgery is the first-line treatment for acromegaly in


the majority of patients

Dopamine agonists
• for example bromocriptine
• the first effective medical treatment for acromegaly, however now
superseded by somatostatin analogues
• effective only in a minority of patients

Somatostatin analogue
• directly inhibits the release of growth hormone
• for example octreotide
• effective in 50-70% of patients
• may be used as an adjunct t o surgery

Pegvisomant
• GH receptor antagonist - prevents dimerization of the GH receptor
• once daily s/c administration
• very effective - decreases IGF-1 levels in 90% of patients to normal
• doesn't reduce tumour volume therefore surgery still needed if
mass effect

External irradiation is sometimes used for older patients or following


failed surgical/medical treatment

Next question >


Q1

A 28-year-old woman with polycystic ovarian syndrome consults you as


she is having problems becoming pregnant. She has a past history of
oligomenorrhea and has previously recently stopped taking a combined
oral contraceptive pill. Despite stopping the pill 6 months ago she is still
not having regular periods. Her body mass index is 28 kg/ m"2. Apart
from advising her to lose weight, which one of the following
interventions is most effective in increasing her chances of conceiving?

Metformin

Bromocriptine

Laparoscopic ovarian cautery

Clomifene

Orlistat

Submit answer

Reference ranges v
I Clomifene

Orlistat

Infertility in PCOS - clomifene is superior to metformin


Importance: 50

Whilst metformin has a role in the management of infertility it should be


used second-line to anti-oestrogens such as clomifene. Simi lar
questions to this often appear in which clomifene is not an option, in
this case metformin is clearly the right answer.

,6 •• flt Discuss (1) Improve ]

Next question )

Polycystic ovarian syndrome: management

Polycystic ovarian syndrome (PCOS) is a complex condition of ovarian


dysfunction thought to affect between 5-20% of women of reproductive
age. Management is complicated and problem based partly because
the aetiology of PCOS is not fully understood. Both hyperinsulinaemia
and high levels of luteinizing hormone are seen in PCOS and there
appears to be some overlap with the metabolic syndrome.

General
• weight reduction if appropriate
• if a women requires contraception then a combined oral
contraceptive (COC) pill may help regulate her cycle and induce a
monthly bleed (see below)
Hirsutism and acne
• a coc pill may be used help manage hirsutism. Possible options
include a third generation coc which has fewer androgenic
effects or co-cyprindiol which has an anti-androgen action. Both
of these types of COC may carry an increased risk of venous
thromboembolism
• if doesn't respond to COC then topical eflornithine may be tried
• spironolactone, flutamide and fmasteride may be used under
specialist supervision

Infertility
• weight reduction if appropriate
• the management of infertility in patients with PCOS should be
supervised by a specialist. There is an ongoing debate as to
whether metformin, clomifene or a combination should be used to
stimulate ovulation
• a 2007 trial published in the New England Journal of Medicine
suggested clomifene was the most effective treatment. There is a
potential risk of multiple pregnancies with anti-oestrogen*
therapies such as clomifene. The RCOG published an opinion
paper in 2008 and concluded that on current evidence metformin
is not a first line treatment of choice in the management of PCOS
• metformin is also used, either combined with clomifene or alone,
particularly in patients who are obese
• gonadotrophins

*work by occupying hypothalamic oestrogen receptors without


activating them. This interferes with the binding of oestradiol and thus
prevents negative feedback inhibition of FSH secretion

Next question )
Q2

A 49-year-old female was admitted to the emergency department with


confusion . A history is unobtainable due to this confusion, with a
Glasgow coma scale score of 13/ 1 5. Hospital records note a 2-month
background of recurrent urinary tract infections (UTl 's) and recent
admission for urosepsis. Her past medical history includes: Type 2
diabetes mellitus, hypertension, hypercholesterolaemia and a hiatus
hernia.

On examination the patient is cool peripherally with a cap refill of 3


seconds, dry mucus membranes, heart sounds 1+2+0, vesicular breath
sounds, abdomen was soft, but tender over the suprapubic area. There
was no rigidity or guarding and bowel sounds were present.

Observations: Respiratory rate 16 breaths per minute, saturations 98%


on air, blood pressure 80/ 58mmHg, heart rate 122 beats per minute,
temperature 38.4°C and capillary glucose 16 mmol/L.

Urine dip showed:

Nitrites +++

Leucocyte est erase +++

Blood +

Glucose +++

Ketones

Prot ein
Ketones

Protein

Which medication is likely to contributing to the cause of her


presentations?

Metformin

Tolbutamide

Dapaglifozin

Sitagliptin

Exanetide

Submit answer

Reference ranges v
I Olbutam1de

Dapaglifozin

Sitagliptin

Exanetide

Dapagliflozin is a member of the glofozin anti-diabetic drugs. The


medication works by inhibiting the sodium-glucose transport proteins
(SGLT2), which reabsorbs glucose in the proximal tubule. The drug has
recently been licensed by national institute of clinical excellence (NICE)
for the treatment of type 2 diabetes mellitus. Dapagliflozin can be tried
if blood sugars are poorly controlled following commencement of
metformin and the patient is unable to take a sulfonylurea. Common
side effect s are often secondary to the glycosuria, which include
increased predisposition of urinary tract infection and dehydration.

.. •t tit Discuss (5) Improve

Next question )

Diabetes mellitus: GLP-1 and the new drugs

A number of new drugs to treat diabetes mellitus have become


available in recent years. Much research has focused around the role of
glucagon-like peptide-1 (GLP-1), a hormone released by the small
intestine in response to an oral glucose load

Whilst it is well known that insulin resistance and insufficient 8-cell


compensation occur other effects are also seen in type 2 diabetes
mellitus (T2DM). In normal physiology an oral glucose load results in a
greater release of insulin than if the same load is given intravenously -
this known as the incretin effect. This effect is largely mediated by GLP-
1 and is known to be decreased in T2DM.
Increasing GLP-1 levels, either by the administration of an analogue
(glucagon-like peptide-1, GLP-1 mimetics, e.g. exenatide) or inhibiting
its breakdown (dipeptidyl peptidase-4 ,DPP-4 inhibitors - the gliptins), is
therefore the target of two recent classes of drug.

Glucagon-like peptide-1 (GLP-1) mimetics (e.g. exenatide)

Exenatide is an example of a glucagon-like peptide-1 (GLP-1) mimetic.


These drugs increase insulin secretion and inhibit glucagon secretion .
One of the major advances of GLP-1' mimetics is that they typically
result in weight loss, in contrast to many medications such as insulin,
sulfonylureas and thiazolidinediones. They are sometimes used in
combination with insulin in T2DM to minimise weight gain.

Exenatide must be given by subcutaneous injection within 60 minutes


before the morning and evening meals. It should not be given after a
meal.

Liraglutide is the other GLP-1 mimetic currently available. One the main
advantages of liraglutide over exenatide is that it only needs to be given
once a day.

Both exenatide and liraglutide may be combined with metformin and a


sulfonylurea. Standard release exenatide is also licensed to be used
with basal insulin alone or with metformin. Please see the BNF for a
more complete list of licensed indications.

NICE state the following:

I
Consider adding exenatide to metformin and a sulfonylurea if:
• BMI >= 35 kg/ m 2 in people of European descent and there are
problems associated with high weight, or
• BMI < 35 kg/ m 2 and insulin is unacceptable because of
occupational implications or weight loss would benefit other
comorbidities.

NICE like patients to have achieved a 11 mmol/mol (1%) reduction in


HbA1c and 3% weight loss after 6 months to justify the ongoing
prescription of GLP-1 mimetics.

The major adverse effect of GLP-1 mimetics is nausea and vomiting.


The Medicines and Healthcare products Regulatory Agency has issued
specific warnings on the use of exenatide, reporting that is has been
linked to severe pancreatitis in some patients.

Dipeptidyl peptidase-4 (DPP-4) inhibitors ( e.g. Vildagliptin, sitagliptin)

Key points
• oral preparation
• trials to date show that the drugs are relatively well tolerated with
no increased incidence of hypoglycaemia
• do not cause weight gain

NICE guidelines on DPP-4 inhibitors


• NICE suggest that a DPP-4 inhibitor might be preferable to a
thiazolidinedione if further weight gain would cause significant
problems, a thiazolidinedione is contraindicated or the person has
had a poor response to a thiazolidinedione

Next question )
Q3

An elderly male with T2DM was taken to the Emergency Department


with confuslonw sweating, and ataxia. He is prescribed thiamine,
metformln, gliclazide, and atorvastatin. BM was measured as 1.3
mmol/L with paramedics and, after administration of 100ml IV dextrose
20%, returned to 5.4 mmol/L.

You are called to see the patient in the acute medical unit as his BM has
dropped to 1.8 mmol/ L.

What is the most likely underlying cause?

Low glycogen stores secondary to chronic alcoholism

Metformin associated lactic acidosis

Starvation

Sulfonylurea-induced hypoglycaemia

Wernicke's encephalopathy

Submit answer

Reference ranges v
Metformin associated lactic acidosis

Starvation

Sulfonylurea-induced hypoglycaemia

Werni cke's encephalopathy

In sulphonylurea overdoses, patients are at risk of recurrent


hypoglycaemia
Importance: 50

Sulphonylureas are long-acting anti-glycaemic agents with the potential


to cause recurrent hypoglycaemia.

Glycogen stores are low in chronic alcoholism preventing the use of


glucagon as an effective treatment for hypoglycaemia. Wernicke's
encephalopathy may be precipitated by administration of glucose
before thiamine.

[ ,6 If tll Discuss (6) Improve ]

Next question)
Next question )

Hypoglycaemia

Causes
• insulinoma - increased ratio of proinsulin to insulin
• self-administration of insulin/sulphonylureas
• liver failure
• Addison's disease
• alcohol

Other possible causes in children


• nesidioblastosis - beta cell hyperplasia

Physiological response to hypoglycaemia


• hormonal response: the first response of the body is decreased
insulin secretion. This is followed by increased glucagon
secretion. Growth hormone and cortisol are also released but later
• sympathoadrenal response: increased catecholamine-mediated
(adrenergic) and acetylcholine-mediated (cholinergic)
neurotransmission in the peripheral autonomic nervous system
and in the central nervous system

Next question )

B I c-:,
Q4

A 49-year-old woman with type 2 diabetes mellitus is being considered


for exenatide therapy. Which one of the following is not part of the NICE
criteria for starting or continuing this drug?

BMI > 35 kg/m"2

Greater than 1.0 percentage point HbA 1 c reduction after 6


months

Has failed with insulin therapy

Has type 2 diabetes mellitus

Weight loss > 3% at 6 months

Submit answer

Reference ranges v
Has failed with insulin therapy

Has type 2 d iabetes mellitus

Weight loss > 3% at 6 months

Patients do not need to have been on insulin prior to using exenatide

[ ,6 .. .. Discuss Improve ]

Next question )

Diabetes mellitus: GLP-1 and the new drugs

A number of new drugs to treat diabetes mellitus have become


available in recent years. Much research has focused around the role of
glucagon-like peptide-1 (GLP-1), a hormone released by the small
intestine in response to an oral glucose load

Whilst it is well known that insulin resistance and insufficient B-cell


compensation occur other effects are also seen in type 2 diabetes
mellitus (T2DM). In normal physiology an oral glucose load results in a
greater release of insulin than if the same load is given intravenously -
this known as the incretin effect. This effect is largely mediated by GLP-
1 and is known to be decreased in T2DM.

Increasing GLP-1 levels, either by the administration of an analogue


(glucagon-like peptide-1, GLP-1 mimetics, e.g. exenatide) or inhibiting
its breakdown (dipeptidyl peptidase-4 ,DPP-4 inhibitors - the gliptins), is
therefore the target of two recent classes of drug.

CamSc.an,ner ~ '....,.i;lo-" ~~I


Glucagon-like peptide-1 (GLP-1) mimetics (e.g. exenatide)

Exenatide is an example of a glucagon-like peptide-1 (GLP-1) mimetic.


These drugs increase insulin secretion and inhibit glucagon secretion.
One of the major advances of GLP-1 mimetics is that they typically
result in weight loss, in contrast to many medications such as insulin,
sulfonylureas and thiazolidinediones. They are sometimes used in
combination with insulin in T2DM to minimise weight gain.

Exenatide must be given by subcutaneous injection within 60 minutes


before the morning and evening meals. It should not be given after a
meal.

Liraglutide is the other GLP-1 mimetic currently available. One the main
advantages of liraglutide over exenatide is that it only needs to be given
once a day.

Both exenatide and liraglutide may be combined with metformin and a


sulfonylurea. Standard release exenatide is also licensed to be used
with basal insulin alone or with metformin. Please see the BNF for a
more complete list of licensed indications.

NICE state the following:

Consider adding exenatide to metformin and a sulfonylurea if:


• BMI >= 35 kg/m 2 in people of European descent and there are
problems associated with high weight, or
• BMI < 35 kg/m 2 and insulin is unacceptable because of
occupational implications or weight loss would benefit other
comorbidities.
NICE like patients to have achieved a 11 mmol/mol (1 %) reduction in
HbA1c and 3% weight loss after 6 months to justify the ongoing
prescription of GLP-1 mimetics.

The major adverse effect of GLP-1 mimetics is nausea and vomiting.


The Medicines and Healthcare products Regulatory Agency has issued
specific warnings on the use of exenatide, reporting that is has been
linked to severe pancreatitis in some patients.

Dipeptidyl peptidase-4 (DPP-4) inhibitors (e.g. Vildagliptin, sitagliptin)

Key points
• oral preparation
• trials to date show that the drugs are relatively well tolerated with
no increased incidence of hypoglycaemia
• do not cause weight gain

NICE guidelines on DPP-4 inhibitors


• NICE suggest that a DPP-4 inhibitor might be preferable to a
thiazolidinedione if further weight gain would cause significant
problems, a thiazolidinedione is contraindicated or the person has
had a poor response to a thiazolidinedione

Next question )

B / ~ A ... :: J= = ... T r., ~ ... ~ c-:::,

Save my notes

Search
QS

A 36-year-old woman who presented with a goitre is diagnosed with


autoimmune thyroiditis. Which one of the following types of thyroid
cancer is she predisposed to developing?

Anaplastic

Lymphoma

Medullary

Follicular

Papillary

Submit answer

Reference ranges v
Lymphoma

Medullary

Follicular

Papillary

Hashimoto's thyroiditis is associated with thyroid lymphoma


Importance: 50

• If tt Discuss Improve

Next question >

Thyroid cancer

Features of hyperthyroidism or hypothyroidism are not commonly seen


in patients with thyroid malignancies as they rarely secrete thyroid
hormones

Main points

Type Percentage

Papillary 70% Often young females - excellent


prognosis

Follicular 20%

Medullary 5% Cancer of parafollicular (C) cells,


secrete calcitonin, part of MEN-2
Anaplastic 1% Not responsive to treatment, can
cause pressure symptoms

Lymphoma Rare Associated with Hashimoto's


thyroiditis

Management of papillary and follicular cancer


• total thyroidectomy
• followed by radioiodine (1-131) to kill residual cells
• yearly thyroglobulin levels to detect early recurrent disease

Further information

Type Notes

Papillary • Usually contain a mixture of papillary and


carcinoma colloidal filled follicles
• Histologically tumour has papillary projections
and pale empty nuclei
• Seldom encapsulated
• Lymph node metastasis predominate
• Haematogenous metastasis rare

Follicular • Usually present as a solitary thyroid nodule


adenoma • Malignancy can only be excluded on formal
histological assessment

Follicular • May appear macroscopically encapsulated,


carcinoma microscopically capsular invasion is seen.
Without this finding the lesion is a follicular
adenoma.
• Vascular invasion predominates
• Multifocal disease raree
Medullary • C cells derived from neural crest and not thyroid
carcinoma tissue
• Serum calcitonin levels often raised
• Familial genetic disease accounts for up to 20%
cases
• Both lymphatic and haematogenous metastasis
are recognised, nodal disease is associated with
a very poor prognosis.

Anaplastic • Most common in elderly females


carcinoma • Local invasion is a common feature
• Treatment is by resection where possible,
palliation may be achieved through
isthmusectomy and radiotherapy.
Chemotherapy is ineffective.

Next question )

B I

Save my notes

Search
Q6

Which one of the following processes is responsible for ketone


production during diabetic ketoacidosis?

Glycogenolysis

Exchange with hydrogen ions in the collecting ducts

Gluconeogenesis

Decreased plasma bicarbonate levels

Lipolysis

Submit answer

Reference ranges v
Exchange with hydrogen ions in the colle cting duct s

Gluconeogenesis

Decreased plas ma bicarbonate levels

Lipolysis

the
The low-insulin cond ition s seen in diab etic keto acid osis stim ulate
-
process of lipolysis and the production of the ketone bodies, beta
bolic
hydroxybutyrate and acetoacetate, which can be used as meta
fuel.

[ ,. 'f • Disc uss (1) Improve ]

Nex t ques tion )

CamScon,er ~ ....,...., c..>~ •


Q7

A 27-year-old man is reviewed in a fertility clinic. Semen analysis has


revealed azoospermia. On examination at the previous appointment he
was noted to be 1.83 metres tall with a body mass index of 25 kg / m"2.
A degree of gynaecomastia is noted, testicular volume is around 10ml
bilaterally and his visual fields were normal. Which investigation is likely
to be diagnostic?

FISH analysis of DNA

Prolactin level

Karyotype

MRI pituitary

PCR analysis of DNA

Submit answer

Reference ranges v
Karyotype

MRI pituitary

PCR analysis of DNA

Klinefelter's? - do a karyotype
Importance: 50

116 •• tt Discuss (5) Improve

Next question )

Klinefelter's syndrome

Klinefelter's syndrome is associated with karyotype 47, XXV.

Features
• often taller than average
• lack of secondary sexual characteristics
• small, firm testes
• infertile
• gynaecomastia - increased incidence of breast cancer
• elevated gonadotrophin levels but low testosterone

Diagnosis is by karyotype (chromosomal analysis).

Next question )
Q8

Each one of the following is seen in Klinefelter's syndrome, except:

Small, firm testes

Lack of secondary sexual characteristics

Inferti Iity

Increased incidence of breast cancer

Reduced gonadotrophin levels

Submit answer

Reference ranges v
Increased incidence of breast cancer

Reduced gonadotrophin levels

Klinefelter's syndrome - elevated gonadotrophin levels


Importance: 50

[ ,6 " .- Discuss Improve

Next question )

Klinefelter's syndrome

Klinefelter's syndrome is associated with karyotype 47, XXV.

Features
• often taller than average
• lack of secondary sexual characteristics
• small, firm testes
• infertile
• gynaecomastia - increased incidence of breast cancer
• elevated gonadotrophin levels but low testosterone

Diagnosis is by karyotype (chromosomal analysis).

Next question )
Q9

A 34-year-old woman presents with palpitations, tremor and heat


intolerance. She is diagnosed with Graves' disease and started on
carbimazole. What is the mechanism of action of this drug?

Inhibits 5'-deiodinase reducing production of T3

Increases renal excretion of carbimazole

Blocks uptake of iodine to thyroid gland by reducing levels of


hydrogen peroxide

Blocks thyroid peroxidase from coupling and iodinating the


tyrosine residues on thyroglobulin

Increases rate of thyroxine breakdown by thyroid peroxidase

Submit answer

Reference ranges v
Blocks thyroid peroxidase from coupling and iodinating the tyrosine
residues on thyrog lobulin

Increases rate of thyroxine breakdown by thyroid peroxidase

Carbimazole blocks thyroid peroxidase from coupling and


iodinating the tyrosine residues on thyroglobulin _. reducing
thyroid hormone production
Importance: 50

116 f
11 tt Discuss (1) Improve

Next question )

Carbimazole

Carbimazole is used in the management of thyrotoxicosis. It is typically


given in high doses for 6 weeks until the patient becomes euthyroid
before being reduced.

Mechanism of action
• blocks thyroid peroxidase from coupling and iodinating the
tyrosine residues on thyroglobulin _. reducing thyroid hormone
production
• in contrast propylthiouracil as well as this central mechanism of
action also has a peripheral action by inhibiting 5'-deiodinase
which reduces peripheral conversion of T 4 to T3

Adverse effects
• agranulocytosis
• crosses the placenta, but may be used in low doses during
pregnancy
Q10

A mother brings her 1 5-year-old daughter to see you as she is worried


about her development. She is very tall for her age, has not yet started
her menstrual cycle and has developed dark facial hair. She is referred
to a specialist who diagnoses mild congenital adrenal hyperplasia.

What deficiency is the most common cause of this condition?

17-hydroxylase deficiency

17~-hydroxysteroid dehydrogenase deficiency

5-alpha reductase deficiency

21-hydroxylase deficiency

11-beta hydroxylase deficiency

Submit answer

Reference ranges v
5-alpha reductase deficiency

21 -hydroxylase deficiency

11-beta hydroxylase deficiency

Congenital adrenal hyperplasia is most commonly due to 21-


hydroxylase deficiency
Importance: 50

21-hydroxylase deficiency is the most common cause of congenital


adrenal hyperplasia making option 4 the correct answer.

17-hydroxylase deficiency is a rare cause of congenital adrenal


hyperplasia, making this option incorrect.

17~-hydroxysteroid dehydrogenase deficiency results in a rare condition


of sexual development, resulting in ambiguous male genitalia or female
external genitalia at birth in a 46XY individual. Patients also have
hypothyroidism, cryptorchidism, metabolic disorders and are infertile.

5-alpha reductase deficiency is an autosomal recessive condition that


affects male sexual development resulting in a genetically male
individual with usually either female or ambiguous genitalia. Some
individuals may have micropenis with hypospadias.

11-beta hydroxylase deficiency is responsible for around 5% of cases of


congenital adrenal hyperplasia making this the second most common
cause.

11• •• • Discuss (1) Improve

Next question )

CamScanner -! W~ :.i.;,-~\
Congenital adrenal hyperplasia

Overview
• group of autosomal recessive disorders
• affect adrenal steroid biosynthesis
• in response to resultant low cortis ol levels the anter ior pituita ry
secretes high levels of ACTH
• ACTH stimu lates the produ ction of adrenal androgens that may
virilize a female infant

Cause
• 21-hydroxylase deficiency (90%)
• 11 -beta hydroxylase deficiency (5%)
• 17-hydroxylase deficiency (very rare)

- ,I
Proo:, , _

'
..
B
fi' I

!i v-- ''
D
'

,
p

..•
, /

I .,,,. c....., ._.,n


Ot fflll'fflH
A 24-year-old female experiences general weakness, muscle cramps
and recurrent renal stones. She is found to be hypertensive and her
bloods reveal the following :

Na+ 133 mmol/L (135 - 145)

K+ 2.6 mmol/L (3.5 - 5.0)

Bicarbonate 35 mmol/L (22 - 29)

Urea 7.2 mmol/ L (2.0 - 7.0)

Creatinine 156 µmol/ L (55 -120)

Calcium 2.3 mmol/L (2.1-2.6)

Phosphate 0.8 mmol/L (0.8-1.4)

Magnesium 0.5 mmol/L (0.7-1.0)

She is diagnosed with a salt-wasting nephropathy named Gitelman's


syndrome.

Which channel does the mutation in this condition target?

It targets the sodium-potassium chloride (Na-K-CI) co-transporter


in the loop of Henle

It targets the Na/ K antiporter pump in the OCT

It targets the epithelial Na channel in the OCT

It targets the Na/ Cl channel co-transporter in the OCT

It targets the aquaporin 2 in the collecting duct


A 24-year-old female experiences general weakness, muscle cramps
and recurrent renal stones. She is found to be hypertensive and her
bloods reveal the following:

Na+ 133 mmol/ L (135 - 145)

K+ 2.6 mmol/L (3.5 - 5.0)

Bicarbonate 35mmol/L (22 - 29)

Urea 7.2 mmol/L (2.0 - 7.0)

Creatinine 156 µmol/ L (55-120)

Calcium 2.3 mmol/L (2.1-2.6)

Phosphate 0.8 mmol/L (0.8-1.4)

Magnesium 0.5 mmol/ L (0.7-1.0)

She is diagnosed with a salt-wasting nephropathy named Gitelman's


syndrome.

Which channel does the mutation in this condition target?

It targets the sodium-potassium chloride (Na-K-CI) co-transporter


in the loop of Henle

It targets the Na/K antiporter pump in the DCT

It targets the epithelial Na channel in the DCT

It targets the Na/Cl channel co-transporter in the OCT

It targets the aquaporin 2 in the collecting duct


It targets the Na/ Cl channel co-transporter in the DCT

It targets the aquaporin 2 in the collecting duct

Gitelman's syndrome is due to a reabsorptive defect of the NaCl


symporter in the DCT
Importance: 50

Gitelman syndrome is another rare autosomal recessive disorder which


occurs due to a mutation causing a resorptive defect of the sodium
chloride co-transporter. Its pathogenesis is similar to the mechanism of
a thiazide diuretic. It is characterised by hypokalemia-hypomagnesemia
and alkaline urine.

Bartter syndrome includes a mutation causing a defect in the Na-K-CI


co-transporter.

Liddle's syndrome includes a mutation causing a defect in epithelial


sodium channels located in the DCT.

Aquaporin 2 is controlled by vasopressin. In diabetes insipidus, the


kidneys do not respond to aquaporin 2.

The Na/K antiporter pump serves in regulating electrolytes. Disruption


of this antiporter is loosely associated with cardiovascular disease,
diabetes, neurological disorders and more.

116 if • Discuss (6) Improve

Next question >


[ .. .. • Discuss (6) Improve ]

Next question )

Gitelman's syndrome

Gitelman's syndrome is due to a defect in the thiazide-sensitive Na+ Cl-


transporter in the distal convoluted tubule.

Features
• normotension
• hypokalaemia
• hypocalciuria
• hypomagnesaemia
• metabolic alkalosis

Next question )

B I A ... T I,.. c-::>

Save my notes

Search

Search textbook ...


0 Q12

A 29-year-old woman has just found out she is pregnant for the second
time. Her first pregnancy was complicated by gestational diabetes.
Following her first pregnancy she was told she was no longer diabetic.
What is the most appropriate management?

Check HbA 1 c immediately

Start metformin and ask the woman to self-monitor glucose

Do oral glucose tolerance test as soon as possible after booking

Do oral glucose tolerance test at 16-18 weeks

Do oral glucose tolerance test at 24-28 weeks

Submit answer

Reference ranges v
Do oral glucose tolerance test as soon as possible after booking

Do oral glucose tolerance test at 16-18 weeks

Do oral glucose tolerance test at 24-28 weeks

NICE have recently updated their guidelines. Women who are at risk of
gestational diabetes should have an oral glucose tolerance test as soon
as possible after booking, rather than waiting to 16-18 weeks as was
previously advocated.

[ ,6 •• - Discuss (1) Improve

Next question >

Pregnancy: diabetes mellitus

Diabetes mellitus may be a pre-existing problem or develop during


pregnancy, gestatio nal diabetes. It complic ates up to 1 in 20
pregnancies. NICE estimat e the followin g breakdown:
• 87.5% have gestational diabetes
• 7.5% have type 1 diabetes
• 5% have type 2 diabetes

Risk factors for gestational diabetes


• BMI of > 30 kg/m 2
• previous macros omic baby weighing 4.5 kg or above
• previous gestational diabetes
• first-degree relative with diabetes
• family origin with a high prevalence of diabetes (South Asian,
black Caribbean and Middle Eastern)
Screening for gestational diabetes
• women who've previously had gestational diabetes: oral glucose
tolerance test (OGTT) should be performed as soon as possible
after booking and at 24-28 weeks if the first test is normal. NICE
also recommend that early self-monitoring of blood glucose is an
alternative to the OGTTs
• women with any of the other risk factors should be offered an
OGTT at 24-28 weeks

Diagnostic thresholds for gestational diabetes


• these have recently been updated by NICE, gestational diabetes is
diagnosed if either:
• fasting glucose is >= 5.6 mmol/I
• 2-hour glucose is >= 7.8 mmol/I

Management of gestational diabetes


• newly diagnosed women should be seen in a joint diabetes and
antenatal clinic within a week
• women should be taught about selfmonitoring of blood glucose
• advice about diet (including eating foods with a low glycaemic
index) and exercise should be given
• if the fasting plasma glucose level is < 7 mmol//I a trial of diet and
exercise should be offered
• if glucose targets are not met within 1-2 weeks of altering
diet/exercise metformin should be started
• if glucose targets are still not met insulin should be added to
diet/exercise/metformin
• if at the time of diagnosis the fasting glucose level is >= 7 mmol/I
insulin should be started
• if the plasma glucose level is between 6-6.9 mmol/I, and there is
evidence of complications such as macrosomia or hydramnios,
insulin should be offered
CamScanner -! W~ :.i.;,-~ \
• glibenclamide should only be offered for women who cannot
tolerate metformin or those who fail to meet the glucose targets
with metformin but decline insulin treatment

Management of pre-existing diabetes


• weight loss for women with BMI of > 27 kg/m"2
• stop oral hypoglycaemic agents, apart from metformin, and
commence insulin
• folic acid 5 mg/ day from pre-conception to 12 weeks gestation
• aspirin 75mg/ day from 12 weeks until the birth of the baby, to
reduce the risk of pre-eclampsia
• detailed anomaly scan at 20 weeks including four-chamber view
of the heart and outflow tracts
• tight glycaemic control reduces complication rates
• treat retinopathy as can worsen during pregnancy

Targets for self monitoring of pregnant women (pre-existing and


gestational diabetes)

Time Target

Fasting 5.3 mmol/I

1 hour after meals 7.8 mmol/ I, or:

2 hour after meals 6.4 mmol/I

Next question )

B / ~ A . . . ·-
■- 2,-= -- . . .
l
Q13

An insulin stress test is most usefu l in t he invest igat ion of:

Glucagonoma

lnsulinoma

Addison's disease

Hypopituitarism

Diabetes mellitus

Submit answer

Reference ranges v
Addison's disease

I Hypo pituita rism

Diabetes mellit us

Insulin stress tests are also occasionally used to differentiate Cushing's


from pseudo-Cushing's

[ ,. " tt Discuss (3) Improve ]

Next ques tion>

Insulin stress test

Basics
• used in investigation of hypopituitarism
• IV insulin given, GH and cortis ol levels measured
• with norma l pituitary functi on GH and cortis ol should rise

Contraindications
• epilepsy
• ischa emic heart disease
• adrenal insufficiency

Next question >

B I l!J A ... 21-: =-.


Q14

You review a 70-year-old who has a long past medical history and is on
multiple drugs. He has developed excessive amounts of breast tissue
bilaterally. Which one of the following drugs is most likely to be
responsible?

Tamoxifen

Terbinafine

Amiodarone

Goserelin (Zoladex)

Lymecycline

Submit answer

Reference ranges v

Score: 0%
Goserelin (Zoladex)

Lymecycline

Goserelin is a gonadorelin analogue used in the treatment of advanced


prostate cancer. Tamoxifen may be used to treat gynaecomastia.

[ •• If tt Discuss (2) Improve ]

Next question >

Gynaecomastia

Gynaecomastia describes an abnormal amount of breast tissue in


males and is usually caused by an increased oestrogen:androgen ratio.
It is important to differentiate the causes of galactorrhoea ( due to the
actions of prolactin on breast tissue) from those of gynaecomastia

Causes of gynaecomastia
• physiological : normal in puberty
• syndromes with androgen deficiency: Kallman's, Klinefelter's
• testicular failure: e.g. mumps
• liver disease
• testicular cancer e.g. seminoma secreting hCG
• ectopic tumour secretion
• hyperthyroidism
• haemodialysis
• drugs: see below
Drug causes of gynaecomastia
• spironolactone (most common drug cause)
• cimetidine
• digoxin
• cannabis
• finasteride
• gonadorelin analogues e.g. Goserelin, buserelin
• oestrogens, anabolic steroids

Very rare drug causes of gynaecomastia


• tricycl ics
• isoniazid
• calcium channel blockers
• heroin
• busulfan
• methyldopa

Next question >

B I A .... ·-
·-
12-: == ... c-:>

=
Save my notes

Search


Search textbook...
Q15

A 73-year-old female present with urge incontinence having a significant


impact on her quality of life. She has undergone supervised bladder
training with no improvement in symptoms and is keen to trial
medication. She has a past medical history of atrial fibrillation, well-
controlled hypertension and recurrent urinary retention .

What would be the most appropriate first-line treatment?

Mirabegron

Oxybutynin

Pelvic floor exercises

Tolterodine

Surgical repair

Submit answer

Reference ranges v
I Mirabegron

Oxybutynin

Pelvic floor exercises

Tolterodine

Surgical repair

Antich olinerg ics for urge incont inence are associ ated with
confus ion in elderly people - mirabe gron is a prefera ble alterna tive
Importance: 50

The key to this question is to recognise that antimu scarin ics the usual
treatm ent for urge incont inence are contra indicat ed in patien ts with a
history of urinary retention . As such mirabe gron is the correc t answer.

Oxybutynin is an antimu scarin ic and would be contra indicat ed in this


patient.

Pelvic floor exercises are used in the treatm ent of stress incont inence
and are unlikely to have an effect in patien ts with pure urge
incontinence.

Tolterodine is also an antimu scarin ic and should not be used in a


history of urinary retention.

Surgical repair would be used in patien ts with stress incont inence that
has not improved with pelvic floor exercises and as such is not the
correc t answer.

[ •• I 'f I tt Discuss (3) Improve


CamScanner~ '..J~ ~ j,,-,&a:'I
Q16

What is the mechanism of action of exenatide?

Glucagon inhibitor

Dipeptidyl peptidase-4 (DPP-4) inhibitor

Glucagon-like peptide-1 (GLP-1) mimetic

lncretin inhibitor

Alpha-glucosidase inhibitor

Submit answer

L___ _ _ _ _ _ _ _ _ R_e_fe_re_n_c_e_ra_n_g_es_v
_ _ _ _ _ _ _ _ _ _]
Dipeptidyl peptidase-4 (DPP-4) inhibitor

Glucagon-like peptide-1 (GLP-1) mimetic

lncretin inhibitor

Alpha-glucosidase inhibitor

Exenatide = Glucagon-like peptide-1 (GLP-1) mimetic


Importance: 50

[ 1• 1f tt Discuss (1) Improve

Next question )
Q1 7

A 23-year-old woman is admitted to the intensive care unit following an


episode of diabetic ketoacidosis. On admission her Glasgow coma
scale was 7/ 15. Collateral history revealed long-standing type 1
diabetes mellitus with poor glycaemic control.

Arterial blood gases:

pH 7.12

paC02 3.1 kPa

pa02 12.2 kPa

HC03 3 mmol/1

Capillary Glucose: 33mmol/L

Urine dip:

glucose +++

ketones +++

protein

nitrites

Leucocyte esterase -

The patient was intubated and successfully treated with intravenous


fluids, insulin and venous thromboembolism prophylaxis.

On discharge her GP undertook a routine screen of blood tests.


Which of the following thyroid function tests results would be in
keeping with her presentation?

TSH - high, T 4 - Low, T3 high

TSH - high, T 4 - normal, T3 normal

TSH - low, T 4 - Low, T3 high

TSH - low, T 4 - normal, T3 normal

TSH - normal, T 4 - Low, T3 low

Submit answer

Reference ranges v
I TSH - normal, T4 - Low, T3 low

Sick euthyroid syndrome is a reversible state of abnormal thyroid


function tests due to a non-thyroidal illness, without pre-existing
hypothalamic-pituitary or thyroid gland dysfunction. By definition, after
recovery of the non-thyroidal illness, thyroid function tests should revert
back to normal.

Causes of sick euthyroid include: myocardial infarctions, starvation,


burns, trauma, surgery, malignancy, diabetic ketoacidosis, any organ
failure and inflammatory conditions.

The pathology postulated is the down regulation of type 1 deiodinase,


reducing the peripheral conversion of T 4 to T3 and thus reducing the
basal metabolic rate during periods of stress. Upregulation of type 3
deiodinase to inactive (reverse) T3 also aids to reducing basal
metabolic rate.

116 11, • Discuss (2) Improve

Next question >

Sick euthyroid syndrome

In sick euthyroid syndrome (now referred to as non-thyroidal illness) it is


often said that everything (TSH, thyroxine and T3) is low. In the majority
of cases however the TSH level is within the >normal range
(inappropriately normal given the low thyroxine and T3).

Changes are reversible upon recovery from the systemic illness and
hence no treatment is usually needed.
Q18

A patient with type 2 diabetes is reviewed in clinic. He is currently taking


metformin but his diabetes control remains poor. As he has an erratic
lifestyle consideration is given to starting repagl inide. What is the
mechanism of action of this drug?

Dipeptidyl peptidase-4 (DPP-4) inhibitor

Agonist to the PPAR-gamma receptor

Alpha-glucosidase inhibitor

Glucagon-like peptide-1 (GLP-1) mimetic

Activates an ATP-dependent K+ channel on the cell membrane of


pancreatic beta cells

Submit answer

Reference ranges v
Alpha-glucosidase inhibitor

Glucagon-like peptide-1 (GLP-1) mimetic

Activates an ATP- +channel on the cell membrane of


dependent K pancreatic beta cells

Meglitinides - bind to an ATP-dependent K+(KATP) channel on the


cell membrane of pancreatic beta cells
Importance: 50

[ ,. 'f • Discuss (4) Improve

Next question >

Meglitinides

Meglitinides (e.g. repaglinide, nateglinide)


• increase pancreatic insulin secretion
• like sulfonylureas they bind to .an ATP-dependent K+(KArP) channel
on the cell membrane of pancreatic beta cells
• often used for patients with an erratic lifestyle
• adverse effects include weight gain and hypoglycaemia (less so
than sulfonylureas)

Next question >


Q19

A 31-year-old female with polycystic ovarian syndrome consults you as


she is troubled with excessive facial hair. Switching her combined oral
contraceptive pill to co-cyprindiol has had no effect. On examination
she has hirsutism affecting her moustache, beard, and temple areas.
What is the most appropriate treatment?

Topical salicylic acid

Topical adapalene

Oral clomifene

Topical eflornithine

Topical tazarotene

Submit answer

Reference ranges v
Topical eflornithine

Topical tazarotene

116 11
f • Discuss (1) Improve

Next question )

Polycystic ovarian syndrome: management

Polycystic ovarian syndrome (PCOS) is a complex condition of ovarian


dysfunction thought to affect between 5-20% of women of reproductive
age. Management is complicated and problem based partly because
the aetiology of PCOS is not fully understood. Both hyperinsulinaemia
and high levels of luteinizing hormone are seen in PCOS and there
appears to be some overlap with the metabolic syndrome.

General
• weight reduction if appropriate
• if a women requires contraception then a combined oral
contraceptive (COC) pill may help regulate her cycle and induce a
monthly bleed (see below)

Hirsutism and acne


• a COC pill may be used help manage hirsutism. Possible options
include a third generation COC which has fewer androgenic
effects or co-cyprindiol which has an anti-androgen action. Both
of these types of COC may carry an increased risk of venous
thromboembolism
Cam Sconner ...,, ',&~ .G. ~ I
• if doesn't respond to COC then topical eflornithine may be tried
• spironolactone, flutamide and flnasteride may be used under
specialist supervision

Infertility
• weight reduction if appropriate
• the management of infertility in patients with PCOS should be
supervised by a specialist. There is an ongoing debate as to
whether metformin, clomifene or a combination should be used to
stimulate ovulation
• a 2007 trial published in the New England Journal of Medicine
suggested clomifene was the most effective treatment. There is a
potential risk of multiple pregnancies with anti-oestrogen*
therapies such as clomifene. The RCOG published an opinion
paper in 2008 and concluded that on current evidence metformin
is not a first line treatment of choice in the management of PCOS
• metformin is also used, either combined with clomifene or alone,
particularly in patients who are obese
• gonadotrophins

*work by occupying hypothalamic oestrogen receptors without


activating them. This interferes with the binding of oestradiol and thus
prevents negative feedback inhibition of FSH secretion

Next question )

B / ~

Save my notes
Q20 Jr::"J

Which one of the following statements regarding glucagon-like peptide-


1 (GLP-1) is incorrect?

Secreted in response to an oral glucose load

Increased levels are seen in type 2 diabetes mellitus

Slows gastric emptying

Secreted by the small intestine

Responsible for the incretin effect

Submit answer

Reference ranges v
Increased levels are seen in type 2 diabetes mellitus

Slows gastric emptying

Secreted by the small intestine

Responsible for the incretin effect

Decreased levels of GLP-1 are seen in type 2 diabetes mellitus

[ •• •• • Discuss Improve ]

Next question >


Q1

Which one of the following is least associat ed with hyperca lcaemia?

Sarcoidosis

Primary hyperparathyroidism

Thiazide diuretics

Squamous cell lung cancer

Monoclonal gammopathy of uncertain significance

Submit answer

Reference ranges v
Primary hyperparathyroidism

Thiazide diuretics

Squamous cell lung cancer

Monoclonal gammopathy of uncertain significance

One of the key differentiating features between monoclonal


gammopathy of uncertain significance (MGUS) and myeloma is the
absence of complications such as immune paresis, hypercalcaemia
and bone pain

,6 " - Discuss Improve ]

Next question )
a6 If tt Discuss Improve

Next question )

Hypercalcaemia: causes

Two conditions account for 90% of cases of hypercalcaemia:


• 1. Primary hyperparathyroidism: commonest cause in non-
hospitalised patients
• 2. Malignancy: the commonest cause in hospitalised patients.
This may be due to number of processes, including; bone
metastases, myeloma, PTHrP from squamous cell lung cancer

Other causes include


• sarcoidosis*
• vitamin D intoxication
• acromegaly
• thyrotoxicosis
• Milk-alkali syndrome
• drugs: thiazides, calcium containing antacids
• dehydration
• Addison's disease
• Paget's disease of the bone**

*other causes of granulomas may lead to hypercalcaemia e.g.


Tuberculosis and histoplasmosis

**usually normal in this condition but hypercalcaemia may occur with


prolonged immobilisation

Next question )

B I A ...
Q2

A 46-year-old woman is referred to endocrine with a tender neck


swelling. Blood results are as follows:

TSH <0.1 mU/ 1

T4 188 nmol/1

Hb 14.2 g/ dl

Pit 377 * 109/ 1

WBC 6.4 * 109 / 1

ESR 65 mm/ hr

Technetium thyroid scan shows decreased uptake globally

What is the most likely diagnosis?

Sick thyroid syndrome

Acute bacterial thyroiditis

Hashimoto's thyroiditis

Subacute thyroiditis

Toxic multinodular goitre

Submit answer
Subacute thyroiditis

Toxic multinodular goitre

Subacute thyroiditis is suggested by the tender goitre, hyperthyroidism


and raised ESR. The globally reduced uptake on technetium thyroid
scan is also typical

a6 If • Discuss (3) Improve

Next question )

Subacute (De Quervain's) thyroiditis

Subacute thyroiditis (also known as De Quervain's thyroiditis and


subacute granulomatous thyroiditis) is thought to occur following viral
infection and typically presents with hyperthyroidism.

There are typically 4 phases;


• phase 1 (lasts 3-6 weeks): hyperthyroidism, painful goitre, raised
ESR
• phase 2 (1-3 weeks): euthyroid
• phase 3 (weeks - months): hypothyroidism
• phase 4: thyroid structure and function goes back to normal

Investigations
• thyroid scintigraphy: globally reduced uptake of iodine-131

Management
• usually self-limiting - most patients do not require treatment
• thyroid pain may respond to aspirin or other NSAI Ds
• in more severe cases steroids are used, particularly if
hypothyroidism develops
Q3

A 26-year-old obese female is investigated for menstrual disturbance. A


diagnosis of polycystic ovarian syndrome is made. Which of the
following findings is most consistently seen in polycystic ovarian
syndrome?

Obesity

Hirsutism

Ovarian cysts on ultrasound

Raised LH:FSH ratio

Clitoromegaly

Submit answer

Reference ranges v
Hirsutism

Ovarian cysts on ultrasound

Raised LH :FSH ratio

Clitoromegaly

Polycystic ovarian syndrome - ovarian cysts are the most


consistent feature
Importance: 50

Whilst all of the features listed above may be seen in polycystic ovarian
syndrome, ovarian cysts are the most consistent feature.

116 •• tt Discuss (3) Improve

Next question )
Next question)

Polycystic ovarian syndrome: features and


investigation

Polycystic ovary syndrome (PCOS) is a complex condition of ovarian


dysfunction thought to affect between 5-20% of women of reproductive
age. The aetiology of PCOS is not fully understood. Both
hyperinsulinaemia and high levels of luteinizing hormone are seen in
PCOS and there appears to be some overlap with the metabolic
syndrome.

Features
• subfertility and infertility
• menstrual disturbances: oligomenorrhea and amenorrhoea
• hirsutism, acne (due to hyperandrogenism)
• obesity
• acanthosis nigricans (due to insulin resistance)

Investigations
• pelvic ultrasound: multiple cysts on the ovaries
• FSH, LH, prolactln, TSH, and testosterone are useful
investigations: raised LH:FSH ratio is a 'classical' feature but is no
longer thought to be useful in diagnosis. Prolactin may be normal
or mildly elevated. Testosterone may be normal or mildly elevated
- however, if markedly raised consider other causes
• check for impaired glucose tolerance

Next question )

B I fj A .,,. 2=
1- - - ..,.
T I""
0 Q4

A 45-year-old woman with Graves' disease comes for review. She has
recently been diagnosed with thyroid eye disease and is being
considered for radiotherapy. Over the past three days her right eye has
become red and painful. On examination there is proptosis and
erythema of the right eye. Visua l acuity is 6/ 9 in both eyes. What
complication is she most likely to have developed?

Exposure keratopathy

Optic neuropathy

Carbimazole-related neutropaenia

Central retinal vein occlusion

Sjogren's Syndrome

Submit answer

Reference ranges v
exposure Keratopatny

Optic neuropathy

Carbimazole-related neutropaenia

Central retinal vein occlusion

Sjogren's Syndrome

9' •• tt Discuss (4) Improve

Next question )

Thyroid eye disease

Thyroid eye disease affects between 25-50% of patients with Graves'


disease.

Pathophysiology
• it is thought to be caused by an autoimmune response against an
autoantigen, possibly the TSH receptor - retro-orbital
inflammation
• the inflammation results in glycosaminoglycan and collagen
deposition in the muscles

Prevention
• smoking is the most important modifiable risk factor for the
development of thyroid eye disease
• radioiodine treatment may increase the inflammatory symptoms
seen in thyroid eye disease. In a recent study of patients with
Graves' disease around 15% developed, or had worsening of, eye
disease. Prednisolone mav help reduce the risk
Features
• the patient may be eu-, hypo- or hyperthyroid at the time of
presentation
• exophthalmos
• conjunctiva! oedema
• optic disc swelling
• ophthalmoplegia
• inability to close the eyelids may lead to sore, dry eyes. If severe
and untreated patients can be at risk of exposure keratopathy

Management
• topical lubricants may be needed to help prevent corneal
inflammation caused by exposure
• steroids
• radiotherapy
• surgery

Monitoring patients with established thyroid eye disease

For patients with established thyroid eye disease the following


symptoms/signs should indicate the need for urgent review by an
ophthalmologist (see EUGOGO guidelines):
• unexplained deterioration in vision
• awareness of change in intensity or quality of colour vision in one
or both eyes
• history of eye suddenly 'popping out' (globe subluxation)
• obvious corneal opacity
• cornea still visible when the eyelids are closed
• disc swelling

Next question )
QS

A 35-year-old female who has recently being diagnosed with Grave's


disease presents for review 3 months after starting a 'block and replace'
regime with carbimazole and thyroxine. She Is concerned about
developing thyroid eye disease. What is the best way that her risk of
developing thyroid eye disease can be reduced?

Reduce alcohol intake

A diet rich in omega-3 fatty acids

Regular exercise

Stop smoking

Lose weight

Submit answer

Reference ranges v
A 35-year-old female who has recently being diagnosed with Grave's
disease presents for review 3 months after starting a 'block and replace'
regime with carbimazole and thyroxine. She is concerned about
developing thyiroid eye disease. What is the best way that her risk of
developing thyiroid eye disease can be reduced?

Reduce alcohol intake

A diet rich in omega-3 fatty acids

Regular exercise

Stop smoking

Lose weight

Smoking is the most important modifiable risk factor for the


development of thyroid eye disease

" If - Discuss (1) Improve ]

Next question )
Q6

Which one of the following unwanted effects is most likely to occur in


patients taking gliclazide?

Peripheral neuropathy

Cholestasis

Photosensitivity

Syndrome of inappropriate ADH secretion

Weight gain

Submit answer

Reference ranges v
I Weight gain
---------------

All of the above side-effects may be seen in patients taking


sulfonylureas but weight gain is the most common.

16 11
f • Discuss Improve

Next question )

Sulfonylureas

Sulfonylureas are oral hypoglycaemic drugs used in the management of


type 2 diabetes mellitus. They work by increasing pancreatic insulin
secretion and hence are only effective if functional 8-cells are present.
On a molecular level they bind to an ATP-dependent K+(KATP) channel
on the cell membrane of pancreatic beta cells.

Common adverse effects


• hypoglycaemic episodes (more common with long-acting
preparations such as chlorpropamide)
• weight gain

Rarer adverse effects


• hyponatraemia secondary to syndrome of inappropriate ADH
secretion
• bone marrow suppression
• hepatotoxicity (typically cholestatic)
• peripheral neuropathy

Sulfonylureas should be avoided in breastfeeding and pregnancy.

Next question )
Q7

Which one of t he following statements regard ing the normal menstrual


cycle is incorrect?

A number of follicles develop in the follicular pha se under the


influence of FSH

The luteal phase is al so known as the secretary phase

The follicular phase follows menstruation and occurs around day


5 -13

A surge of FSH causes ovulation

Progesterone levels are low in the follicular phase

Submit answer

Reference ranges v
A surge of FSH causes ovulation

Progesterone levels are low in the follicular phase

LH surge causes ovulation


Importance: 50

" 'f • Discuss (1) Improve ]

Next question )

Menstrual cycle

The menstrual cycle may be divided into t he following phases:

Days

Menstruation 1-4

Follicular phase (proliferative phase) 5-13

Ovulation 14

Luteal phase (secretory phase) 15-28


Folllcular phase
Days 1-7

Prlmordlal Prfmary Secondary


follicles lolllcles folllcles
... •© •® GP
Atresla
Single, selected
© •©>
Selectlon
tertiary folllcle

0
© ~
Atresla
{@

constant development Selectlon of one domtnant


of early-stage folllcles secondary folllcle begins
(2 months) each new menstrual cycle

Ovarian cycle phases


Follicular phase Luteal pflase
Selected tertiary ovuauon

le► =~~;
----f-ol-llc.... \.,:.;~~ 'j
COrpLS corpus DegradJn~

alblcans co,pus

0 7 14 21 28
Day of merstrual cycle

Uterlne cycle phases


Prollt e rat lve
r..tenses phase Sec.-etor>' phase

M enstruatl c,n

0 7 14 21 28
Day of m,enstruaJ cycl e

Pituitary OvulatJon FSH


hormone levels - LH

1~ 0
7 14 21 28

...g
0
ovarian - Estrogen
:c hormone levels - Progesterone

0 7 14 21 28
Day of menstrual cycle
Follicular phase Luteal phase
(proliferative phase) (secretary phase)

Ovarian A number of follicles Corpus luteum


histology develop.

One follicle will become


dominant around the mid-
follicular phase

Endometrial Proliferation of Endometrium


histology endometrium changes to
secretary lining
under influence of
progesterone

Hormones A rise in FSH results in the Progesterone


development of follicles secreted by corpus
which in turn secrete luteum rises
oestradiol through the luteal
phase.
When the egg has matured,
it secretes enough If fertilisation does
oestradiol to trigger the not occur the
acute release of LH. This in corpus luteum will
turn leads to ovulation degenerate and
progesterone levels
fall

Oestradiol levels
also rise again
during the luteal
phase
Cervical Following menstruation the Under the influence
mucus mucus is thick and forms a of progesterone it
plug across the external os becomes thick,
scant, and tacky
Just prior to ovulation the
mucus becomes clear,
acellular, low viscosity. It
also becomes 'stretchy' - a
quality termed spinnbarkeit

Basal body Falls prior to ovulation due Rises following


temperature to the influence of oestradiol ovulation in
response to higher
progesterone levels

Next question )

B J tj

Save my notes

Search


Search textbook...

O. Google search on "Menstrual cycle"


01

An 85-year-old comes for review. She has recently had private health
screening and has been advised to see a doctor regarding her thyroid
function tests (TFTs).

TSH 9.2 mU/L

Free thyroxine 14 pmol/L

She is currently well and asymptomatic. What is the most appropriate


management?

Start levothyroxine

Start ca rbimazole

Order a thyroid ultrasound scan

Start levothyroxine + carbimazole ('block and replace')

Repeat TFTs in a few months time

Submit answer

Ca mSc.an,ner ~ '....,.i;lo-" ~ ~ I
Start carbimazole

Order a thyroid ultrasound scan

Start levothyroxine + carbimazole ('block and replace')

Repeat TFTs in a few months time

This patient has subclinical hypothyroidism. By both the TSH and age
criteria advocated by NICE Clinical Knowledge Summaries she should
be monitored for now.

[ ,. If - Discuss (3) Improve ]

Next question )

Subclinical hypothyroidism

Basics
• TSH raised but T3, T4 normal
• no obvious symptoms
Significance
• risk of progressing to overt hypothyroidism is 2-5% per year
(higher in men)
• risk increased by the presence of thyroid autoantibodies

Management

Not all patients require treatment. NICE Clinical Knowledge Summaries


(CKS) have produced guidelines. Note that not all patients will fall within
the age boundaries given and hence these are guidelines in the broader
sense.

TSH is between 4 - 10mU/L and the free thyroxine level is within the
normal range
• if< 65 years with symptoms suggestive of hypothyroidism, give a
trial of levothyroxine. If there is no improvement in symptoms,
stop levothyroxi ne
• 'in older people (especially those aged over 80 years) follow a
'watch and wait' strategy, generally avoiding hormonal treatment'
• if asymptomatic people, observe and repeat thyroid function in 6
months

TSH is > 10mU/ L and the free thyroxine level is within the normal range
• start treatment (even if asymptomatic) with levothyroxine if<= 70
years
• 'in older people (especially those aged over 80 years) follow a
'watch and wait' strategy, generally avoiding hormonal treatment'

Next question )

B J f!j A •

~ ! l \ / 0 m\t nntoc- I
Q2

A 67-year-old woman presents with lethargy, depression and


constipation. A set of screening blood tests reveals the following:

Calcium 3.05 mmol/ 1

Albumin 41 g/ 1

What is the single most useful test for determining the cause of her
hypercalcaemia?

ESR

Phosphate

Vitamin D level

Parathyroid hormone

ACE level

Submit answer

Reference ranges v
Phosphate

Vitamin D level

Parathyroid hormone

ACE level

Parathyroid hormone levels are useful as malignancy and primary


hyperparathyroidism are the two most common causes of
hypercalcaemia. A parathyroid hormone that is normal or raised
suggests primary hyperparathyroidism.

,6 " tit Discuss (3) Improve ]

Next question )
Q3

A 4-year-old boy is being investigated for failure to thrive and


generalised weakness. His blood pressure is normal. The following
blood results are obtained:

Na+ 137 mmol/1

K+ 3.0 mmol/1

Urea 4.5 mmol/1

Creatlnine 65 µmol/1

Bicarbonate 33 mmol/1

What is the most likely diagnosis?

Conn's syndrome

Bartter's syndrome

Cushing's syndrome

21-hydroxylase deficiency

Liddle's syndrome

Submit answer
I Bartter's syndrome

Cushing's syndrome

21-hydroxylase deficiency

Liddle's syndro me

Bartter's syndrome is associated with normotension


Importance: 50

Bartter's syndro me is the most likely diagnosis. Congenital adrenal


hyperplasia due to 21-hydroxylase deficiency is associated with
precoc ious puberty rather than fai lure to thrive in boys. Both Conn's and
Cushing's are assoc iated with hypertension and are not comm on in this
age group.

Liddle 's syndro me is a rare autosomal domin ant condit ion that causes
hypert ension and hypokalaemic alkalosis. It is though t to be caused by
disordered sodium channels in the distal tubules leading to increased
reabsorption of sodium.

[ ,6 •• • Discuss (2) Improve ]

Next question )
Next question )

Bartter's syndrome

Bartter's syndrome is an inherited cause (usually autosomal recessive)


of severe hypokalaemia due to defective chloride absorption at the Na+
K+ 2CI- cotransporter (NKCC2) in the ascending loop of Henle. It should
be noted that it is associated with normotension (unlike other endocrine
causes of hypokalaemia such as Conn's, Cushing's and Liddle's
syndrome which are associated w ith hypertension) .

Loop diuretics work by inhibiting NKCC2 - think of Bartter's


syndrome as like taking large doses of furosemide

Features
• usually presents in childhood, e.g. Failure to thrive
• polyuria, polydipsia
• hypokalaemia
• normotension
• weakness

Next question )

B / /!J T I""

Save mv notes l
Q4 fc:::J

A 43-year-old man presents to his GP with t iredness, low mood and


unintentional weight gain of 13kg over the past 4 months. Prior to
feeling like this he recalls having a flu-like illness following which he had
a two-week period of feeling very anxious, shaky and energetic. He
wonders if this is connected.

On examination he has a heart rate of 68 bpm, his blood pressure is


147/83 mmHg and his temperature is 37.1 °c. Examination of his
abdomen and chest are unremarkable and he does not have a goitre or
any palpable lymphadenopathy. He has no family history of note and no
past medical history.

Blood tests to look at his thyroid function show the following:

Thyroid stimulating hormone (TSH) 6.1 mu/I (0.5-5.5 mu/I)

Free T 4 6 pmol/1 (9-18 pmol/1)

What is the most likely cause of this man's symptoms?

Grave's disease

Hashimoto's thyroiditis

Papillary cancer of the thyroid

De Quervain's thyroiditis

Toxic multinodular goitre

Submit answer
Papillary cancer of the thyroid

De Quervain's thyroiditis

Toxic multinodular goitre

Subacute thyroiditis causes hyper- then hypothyroidism


Importance: 50

This gentleman has a clinical picture of hypothyroidism with what


appears to be a brief period of hyperthyroidism prior to this. The most
common cause of this is De Quervain's thyroiditis and this would be in
keeping with the history of a viral infection before the initial
hyperthyroid episode. There is a rare form of Hashimoto's in which the
patient has an initial phase of hyperthyroidism before becoming
hypothyroid, however the period of hyperthyroidism is prolonged in
those cases and the clinical picture is often indistinguishable from
Grave's disease. In addition it happens far more commonly in women
than men (around 5 times) and has a strong association with other
auto-immune diseases. The key to this question is what is most likely
and given the relatively brief period of hyperthyroidism (in Hashimoto's
it would be in the order of 6-12 months rather than a few weeks) and the
preceding viral infection, De Quervain's is far more likely.

Grave's disease and toxic multinodular goitre would both present with
hyperthyroidism and papillary thyroid cancer does not produce
thyroxine so would not cause any systemic symptoms.

[ ,6 •• - Discuss (2) Improve ]


Next question ~

Subacute (De Quervain's) thyroiditis

Subacute thyroiditis (also known as De Quervain's thyroiditis and


subacute granulomatous thyroiditis) is thought to occur following viral
infection and typically presents with hyperthyroidism.

There are typically 4 phases;


• phase 1 (lasts 3-6 weeks): hyperthyroidism, painful goitre, raised
ESR
• phase 2 (1-3 weeks) : euthyroid
• phase 3 (weeks - months): hypothyroidism
• phase 4: thyroid structure and function goes back to normal

Investigations
• thyroid scintigraphy: globally reduced uptake of iodine-131

Management
• usually self-limiting - most patients do not require treatment
• thyroid pain may respond to aspirin or other NSAIDs
• in more severe cases steroids are used, particularly if
hypothyroidism develops

Hypothyroidism Hyperthyrodlsm

H shimoco's
th~roic:1•115
Grav s' disease

Atruodarone Toxic mu t nodular goitre


Rlod s lhyrold1ll$
Postpartum tnyroid

l1th111m
Q1

A 32 year-old man presents complaining of persistent headaches. He


was diagnosed with hypertension 4 months ago and started on
perindopril. On examination, heart his rate is 75 beats per minute and
blood pressure is 185/11 SmmHg.

Investigations:

Serum potassium 1.9 mmol/L (3.5-5.0)

Plasma aldosterone (after 30 minutes supine) 700 pmol/L (135-400)

Plasma renin activity (after 30 minutes supine) 0.4 pmol/ml/hr (1 .1-2.7)

What is the most likely cause of his hypertension?

Addison disease

Bilateral renal artery stenosis

Coarctation of the aorta

Phaeochromocytoma

Primary hyperaldosteronism

Submit answer
Bilateral renal artery stenosis

Coarctation of the aorta

Phaeochromocytoma

I Primary hyperaldosteronism

Primary hyperaldosteronism can present with hypertension,


hypernatraemia, and hypokalemia
Importance: 50

Primary hyperaldosteronism is typically caused by an aldosterone


producing adenoma (Conn's syndrome), other causes include: bilateral
adrenocortical hyperplasia and adrenal carcinoma.

Primary hyperaldosteronism and bilateral renal artery stenosis are


associated with hypokalaemia due raised serum aldosterone, which
causes increased sodium reabsorption and potassium excretion.

Aldosterone is elevated in bilateral renal artery stenosis due to reduced


renal perfusion. Aldosterone is high in primary hyperaldosteronism,
however, serum renin is usually low in primary hyperaldosteronism due
to the resulting hypertension causing excessive renal perfusion, which
results in decreased renin production (negative feedback mechanism).
High renin levels are seen in renal artery stenosis as renal perfusion is
permanently reduced, despite hypertension, due to the stenotic renal
arteries.

(Reference: Oxford handbook of cl inical medicine, 8th ed.pg.220)

at •• - Discuss (7) Improve

You might also like